Search And Warrant.docx

  • Uploaded by: Angelique Padilla Ugay
  • 0
  • 0
  • November 2019
  • PDF

This document was uploaded by user and they confirmed that they have the permission to share it. If you are author or own the copyright of this book, please report to us by using this DMCA report form. Report DMCA


Overview

Download & View Search And Warrant.docx as PDF for free.

More details

  • Words: 120,555
  • Pages: 286
G.R. No. L-45358

January 29, 1937

NARCISO ALVAREZ, petitioner, vs. THE COURT OF FIRST INSTANCE OF TAYABAS and THE ANTI-USURY BOARD, respondents. Godofredo Reyes for petitioner. Adolfo N. Feliciano for respondents Anti-Usury Board. No appearance for other respondent. IMPERIAL, J.: The petitioner asks that the warrant of June 3, 1936, issued by the Court of First Instance of Tayabas, ordering the search of his house and the seizure, at any time of the day or night, of certain accounting books, documents and papers belonging to him in his residence situated in Infanta, Province of Tayabas, as well as the order of a later date, authorizing the agents of the Anti-Usury Board to retain the articles seized, be declared illegal and set aside, and prays that all the articles in question be returned to him. On the date above-mentioned, the chief of the secret service of the Anti-Usury Board, of the Department of Justice, presented to Judge Eduardo Gutierrez David then presiding over the Court of First Instance of Tayabas, an affidavit alleging that according to reliable information, the petitioner kept in his house in Infanta, Tayabas, books, documents, receipts, lists, chits and other papers used by him in connection with his activities as a money-lender charging usurious rates of interest in violation of the law. In his oath at the and of the affidavit, the chief of the secret service stated that his answers to the questions were correct to the best of his knowledge and belief. He did not swear to the truth of his statements upon his own knowledge of the facts but upon the information received by him from a reliable person. Upon the affidavit in question the Judge, on said date, issued the warrant which is the subject matter of the petition, ordering the search of the petitioner's house at nay time of the day or night, the seizure of the books and documents above-mentioned and the immediate delivery thereof to him to be disposed of in accordance with the law. With said warrant, several agents of the Anti-Usury Board entered the petitioner's store and residence at seven o'clock on the night of June 4, 1936, and seized and took possession of the following articles: internal revenue licenses for the years 1933 to 1936, one ledger, two journals, two cashbooks, nine order books, four notebooks, four checks stubs, two memorandums, three bankbooks, two contracts, four stubs, forty-eight stubs of purchases of copra, two inventories, two bundles of bills of lading, one bundle of credit receipts, one bundle of stubs of purchases of copra, two packages of correspondence, one receipt book belonging to Luis Fernandez, fourteen bundles of invoices and other papers many documents and loan contracts with security and promissory notes, 504 chits, promissory notes and stubs of used checks of the Hongkong & Shanghai Banking Corporation. The search for and a seizure of said articles were made with the opposition of the petitioner who stated his protest below the inventories on the ground that the agents seized even the originals of the documents. As the articles had not been brought immediately to the judge who issued the search warrant, the petitioner, through his attorney, filed a motion on June 8, 1936, praying that the agent Emilio L. Siongco, or any other agent, be ordered immediately to deposit all the seized articles in the office of the clerk of court and that said agent be declared guilty of contempt for having disobeyed the order of the court. On said date the court issued an order directing Emilio L. Siongco to deposit all the articles seized within twenty-four hours from the receipt of notice thereof and giving him a period of five (5) days within which to show cause why he should not be punished for contempt of court. On June 10th, Attorney Arsenio Rodriguez, representing the Anti-Usury Board, filed a motion praying that the order of the 8th of said month be set aside and that the Anti-Usury Board be authorized to retain the articles seized for a period of thirty (30) days for the necessary investigation.

The attorney for the petitioner, on June 20th, filed another motion alleging that, notwithstanding the order of the 8th of said month, the officials of the Anti-Usury Board had failed to deposit the articles seized by them and praying that a search warrant be issued, that the sheriff be ordered to take all the articles into his custody and deposit of the Anti-Usury Board be punished for contempt of court. Said attorney, on June 24th, filed an ex parte petition alleging that while agent Emilio L. Siongco had deposited some documents and papers in the office of the clerk of court, he had so far failed to file an inventory duly verified by oath of all the documents seized by him, to return the search warrant together with the affidavit it presented in support thereof, or to present the report of the proceedings taken by him; and prayed that said agent be directed to filed the documents in question immediately. On the 25th of said month the court issued an order requiring agent Emilio L. Siongco forthwith to file the search warrant and the affidavit in the court, together with the proceedings taken by him, and to present an inventory duly verified by oath of all the articles seized. On July 2d of said year, the attorney for the petitioner filed another petition alleging that the search warrant issue was illegal and that it had nit yet been returned to date together with the proceedings taken in connection therewith, and praying that said warrant be cancelled, that an order be issued directing the return of all the articles seized to the petitioner, that the agent who seized them be declared guilty of contempt of court, and that charges be filed against him for abuse of authority. On September 10, 1936, the court issued an order holding: that the search warrant was obtained and issued in accordance with the law, that it had been duly complied with and, consequently, should not be cancelled, and that agent Emilio L. Siongco did not commit any contempt of court and must, therefore, be exonerated, and ordering the chief of the Anti-Usury Board in Manila to show case, if any, within the unextendible period of two (2) days from the date of notice of said order, why all the articles seized appearing in the inventory, Exhibit 1, should not be returned to the petitioner. The assistant chief of the Anti-Usury Board of the Department of Justice filed a motion praying, for the reasons stated therein, that the articles seized be ordered retained for the purpose of conducting an investigation of the violation of the Anti-Usury Law committed by the petitioner. In view of the opposition of the attorney for the petitioner, the court, on September 25th, issued an order requiring the Anti-Usury Board to specify the time needed by it to examine the documents and papers seized and which of them should be retained, granting it a period of five (5) days for said purpose. On the 30th of said month the assistant chief of the Anti-Usury Board filed a motion praying that he be granted ten (10) days to comply with the order of September 25th and that the clerk of court be ordered to return to him all the documents and papers together with the inventory thereof. The court, in an order of October 2d of said year, granted him the additional period of ten(10) days and ordered the clerk of court to send him a copy of the inventory. On October 10th, said official again filed another motion alleging that he needed sixty (60) days to examine the documents and papers seized, which are designated on pages 1 to 4 of the inventory by Nos. 5, 1016, 23, 25, 26, 27, 30, 31, 34, 36, 37, 38, 39, 40, 41, 42, 43 and 45, and praying that he be granted said period of sixty (60) days. In an order of October 16th, the court granted him the period of sixty (60) days to investigate said nineteen (19) documents. The petitioner alleges, and it is not denied by the respondents, that these nineteen (19)documents continue in the possession of the court, the rest having been returned to said petitioner. I. A search warrant is an order in writing, issued in the name of the People of the Philippine Islands, signed by a judge or a justice of the peace, and directed to a peace officer, commanding him to search for personal property and bring it before the court (section 95, General Orders. No. 58, as amended by section 6 of Act No. 2886). Of all the rights of a citizen, few are of greater importance or more essential to his peace and happiness than the right of personal security, and that involves the exemption of his private affairs, books, and papers from the inspection and scrutiny of others (In re Pacific Railways Commission, 32 Fed., 241; Interstate Commerce Commission vs Brimson, 38 Law. ed., 1047; Broyd vs. U. S., 29 Law. ed., 746; Caroll vs. U. S., 69 Law. ed., 543, 549). While the power to search and seize is necessary to the public welfare, still it must be exercised and the law enforced without transgressing the constitutional rights or citizen, for the enforcement of no statue is of

sufficient importance to justify indifference to the basis principles of government (People vs. Elias, 147 N. E., 472). II. As the protection of the citizen and the maintenance of his constitutional right is one of the highest duties and privileges of the court, these constitutional guaranties should be given a liberal construction or a strict construction in favor of the individual, to prevent stealthy encroachment upon, or gradual depreciation on, the rights secured by them(State vs. Custer County, 198 Pac., 362; State vs. McDaniel, 231 Pac., 965; 237 Pac., 373). Since the proceeding is a drastic one, it is the general rule that statutes authorizing searches and seizure or search warrants must be strictly construed (Rose vs. St. Clair, 28 Fed., [2d], 189; Leonardvs. U. S., 6 Fed. [2d], 353; Perry vs. U. S. 14 Fed. [2d],88; Cofer vs. State, 118 So., 613). III. The petitioner claims that the search warrant issued by the court is illegal because it has been based upon the affidavit of agent Mariano G. Almeda in whose oath he declared that he had no personal knowledge of the facts which were to serve as a basis for the issuance of the warrant but that he had knowledge thereof through mere information secured from a person whom he considered reliable. To the question "What are your reason for applying for this search warrant", appearing in the affidavit, the agent answered: "It has been reported to me by a person whom I consider to be reliable that there are being kept in said premises, books, documents, receipts, lists, chits, and other papers used by him in connection with his activities as a money-lender, charging a usurious rate of interest, in violation of the law" and in attesting the truth of his statements contained in the affidavit, the said agent states that he found them to be correct and true to the best of his knowledge and belief. Section 1, paragraph 3, of Article III of the Constitution, relative to the bill of rights, provides that "The right of the people to be secure in their persons, houses, papers, and effects against unreasonable searches and seizures shall not be violated, and no warrants shall issue but upon probable cause, to be determined by the judge after examination under oath or affirmation of the complainant and the witnesses he may produce, and particularly describing the place top be searched, and the persons or things to be seized." Section 97 of General Orders, No. 58 provides that "A search warrant shall not issue except for probable cause and upon application supported by oath particularly describing the place to be searched and the person or thing to be seized." It will be noted that both provisions require that there be not only probable cause before the issuance of a search warrant but that the search warrant must be based upon an application supported by oath of the applicant ands the witnesses he may produce. In its broadest sense, an oath includes any form of attestation by which a party signifies that he is bound in conscience to perform an act faithfully and truthfully; and it is sometimes defined asan outward pledge given by the person taking it that his attestation or promise is made under an immediate sense of his responsibility to God (Bouvier's Law Dictionary; State vs. Jackson, 137 N. W., 1034; In re Sage, 24 Oh. Cir. Ct. [N. S.], 7; Pumphery vs.State, 122 N. W., 19; Priest vs. State, 6 N. W., 468; State vs. Jones, 154 Pac., 378; Atwood vs. State, 111 So., 865). The oath required must refer to the truth of the facts within the personal knowledge of the petitioner or his witnesses, because the purpose thereof is to convince the committing magistrate, not the individual making the affidavit and seeking the issuance of the warrant, of the existence of probable cause (U. S. vs. Tureaud, 20 Fed., 621; U. S. vs. Michalski, 265 Fed., 8349; U. S. vs. Pitotto, 267 Fed., 603; U. S.vs. Lai Chew, 298 Fed., 652). The true test of sufficiency of an affidavit to warrant issuance of a search warrant is whether it has been drawn in such a manner that perjury could be charged thereon and affiant be held liable for damages caused (State vs. Roosevelt Country 20th Jud. Dis. Ct., 244 Pac., 280; State vs.Quartier, 236 Pac., 746).

It will likewise be noted that section 1, paragraph 3, of Article III of the Constitution prohibits unreasonable searches and seizure. Unreasonable searches and seizures are a menace against which the constitutional guarantee afford full protection. The term "unreasonable search and seizure" is not defined in the Constitution or in General Orders No. 58, and it is said to have no fixed, absolute or unchangeable meaning, although the term has been defined in general language. All illegal searches and seizure are unreasonable while lawful ones are reasonable. What constitutes a reasonable or unreasonable search or seizure in any particular case is purely a judicial question, determinable from a consideration of the circumstances involved, including the purpose of the search, the presence or absence or probable cause, the manner in which the search and seizure was made, the place or thing searched, and the character of the articles procured (Go-Bart Importing Co. vs. U. S. 75 Law. ed., 374; Peru vs. U. S., 4 Fed., [2d], 881;U. S. vs. Vatune, 292 Fed., 497; Angelo vs. U. S. 70 Law, ed., 145; Lambert vs. U. S. 282 Fed., 413; U. S. vs.Bateman, 278 Fed., 231; Mason vs. Rollins, 16 Fed. Cas. [No. 9252], 2 Biss., 99). In view of the foregoing and under the above-cited authorities, it appears that the affidavit, which served as the exclusive basis of the search warrant, is insufficient and fatally defective by reason of the manner in which the oath was made, and therefore, it is hereby held that the search warrant in question and the subsequent seizure of the books, documents and other papers are illegal and do not in any way warrant the deprivation to which the petitioner was subjected. IV. Another ground alleged by the petitioner in asking that the search warrant be declared illegal and cancelled is that it was not supported by other affidavits aside from that made by the applicant. In other words, it is contended that the search warrant cannot be issued unless it be supported by affidavits made by the applicant and the witnesses to be presented necessity by him. Section 1, paragraph 3, of Article III of the Constitution provides that no warrants shall issue but upon probable cause, to be determined by the judge after examination under oath or affirmation of the complainant and the witnesses he may produce. Section 98 of General Orders, No. 58 provides that the judge or justice must, before issuing the warrant, examine under oath the complainant and any witnesses he may produce and take their depositions in writing. It is the practice in this jurisdiction to attach the affidavit of at least the applicant or complainant to the application. It is admitted that the judge who issued the search warrant in this case, relied exclusively upon the affidavit made by agent Mariano G. Almeda and that he did not require nor take the deposition of any other witness. Neither the Constitution nor General Orders. No. 58 provides that it is of imperative necessity to take the deposition of the witnesses to be presented by the applicant or complainant in addition to the affidavit of the latter. The purpose of both in requiring the presentation of depositions is nothing more than to satisfy the committing magistrate of the existence of probable cause. Therefore, if the affidavit of the applicant or complainant is sufficient, the judge may dispense with that of other witnesses. Inasmuch as the affidavit of the agent in this case was insufficient because his knowledge of the facts was not personal but merely hearsay, it is the duty of the judge to require the affidavit of one or more witnesses for the purpose of determining the existence of probable cause to warrant the issuance of the search warrant. When the affidavit of the applicant of the complaint contains sufficient facts within his personal and direct knowledge, it is sufficient if the judge is satisfied that there exist probable cause; when the applicant's knowledge of the facts is mere hearsay, the affidavit of one or more witnesses having a personal knowledge of the fact is necessary. We conclude, therefore, that the warrant issued is likewise illegal because it was based only on the affidavit of the agent who had no personal knowledge of the facts. V. The petitioner alleged as another ground for the declaration of the illegality of the search warrant and the cancellation thereof, the fact that it authorized its execution at night. Section

101 of General Orders, No. 58 authorizes that the search be made at night when it is positively asserted in the affidavits that the property is on the person or in the place ordered to be searched. As we have declared the affidavits insufficient and the warrant issued exclusively upon it illegal, our conclusion is that the contention is equally well founded and that the search could not legally be made at night. VI. One of the grounds alleged by the petitioner in support of his contention that the warrant was issued illegally is the lack of an adequate description of the books and documents to be seized. Section 1, paragraphs 3, of Article III of the Constitution, and section 97 of General Orders, No. 58 provide that the affidavit to be presented, which shall serve as the basis for determining whether probable cause exist and whether the warrant should be issued, must contain a particular description of the place to be searched and the person or thing to be seized. These provisions are mandatory and must be strictly complied with (Munch vs. U. S., 24 Fed. [2d], 518; U. S. vs. Boyd, 1 Fed. [2d], 1019; U. S. vs. Carlson, 292 Fed., 463; U. S. vs. Borkowski, 268 Fed., 408; In re Tri-State Coal & Coke Co., 253 Fed., 605; People vs. Mayen, 188 Cal., 237; People vs. Kahn, 256 Ill. App., 4125); but where, by the nature of the goods to be seized, their description must be rather generally, it is not required that a technical description be given, as this would mean that no warrant could issue (People vs. Rubio, 57 Phil., 284; People vs. Kahn, supra). The only description of the articles given in the affidavit presented to the judge was as follows: "that there are being kept in said premises books, documents, receipts, lists, chits and other papers used by him in connection with his activities as money-lender, charging a usurious rate of interest, in violation of the law." Taking into consideration the nature of the article so described, it is clear that no other more adequate and detailed description could have been given, particularly because it is difficult to give a particular description of the contents thereof. The description so made substantially complies with the legal provisions because the officer of the law who executed the warrant was thereby placed in a position enabling him to identify the articles, which he did. VII. The last ground alleged by the petitioner, in support of his claim that the search warrant was obtained illegally, is that the articles were seized in order that the Anti-Usury Board might provide itself with evidence to be used by it in the criminal case or cases which might be filed against him for violation of the Anti-usury Law. At the hearing of the incidents of the case raised before the court it clearly appeared that the books and documents had really been seized to enable the Anti-Usury Board to conduct an investigation and later use all or some of the articles in question as evidence against the petitioner in the criminal cases that may be filed against him. The seizure of books and documents by means of a search warrant, for the purpose of using them as evidence in a criminal case against the person in whose possession they were found, is unconstitutional because it makes the warrant unreasonable, and it is equivalent to a violation of the constitutional provision prohibiting the compulsion of an accused to testify against himself (Uy Kheytin vs.Villareal, 42 Phil,, 886; Brady vs. U. S., 266 U. S., 620; Temperani vs. U. S., 299 Fed., 365; U. S. vs.Madden, 297 Fed., 679; Boyd vs. U. S.,116 U. S., 116; Caroll vs. U. S., 267 U. S., 132). Therefore, it appearing that at least nineteen of the documents in question were seized for the purpose of using them as evidence against the petitioner in the criminal proceeding or proceedings for violation against him, we hold that the search warrant issued is illegal and that the documents should be returned to him. The Anti-Usury Board insinuates in its answer that the petitioner cannot now question the validity of the search warrant or the proceedings had subsequent to the issuance thereof, because he has waived his constitutional rights in proposing a compromise whereby he agreed to pay a fine of P200 for the purpose of evading the criminal proceeding or proceedings. We are of the opinion that there was no such waiver, first, because the petitioner has emphatically denied the offer of compromise

and, second, because if there was a compromise it reffered but to the institution of criminal proceedings fro violation of the Anti-Usury Law. The waiver would have been a good defense for the respondents had the petitioner voluntarily consented to the search and seizure of the articles in question, but such was not the case because the petitioner protested from the beginning and stated his protest in writing in the insufficient inventory furnished him by the agents. Said board alleges as another defense that the remedy sought by the petitioner does not lie because he can appeal from the orders which prejudiced him and are the subject matter of his petition. Section 222 of the Code of Civil Procedure in fact provides that mandamus will not issue when there is another plain, speedy and adequate remedy in the ordinary course of law. We are of the opinion, however, that an appeal from said orders would have to lapse before he recovers possession of the documents and before the rights, of which he has been unlawfully deprived, are restored to him (Fajardo vs. Llorente, 6 Phil., 426; Manotoc vs. McMicking and Trinidad, 10 Phil., 119; Cruz Herrera de Lukban vs. McMicking, 14 Phil., 641; Lamb vs. Phipps, 22 Phil., 456). Summarizing the foregoing conclusions, we hold: 1. That the provisions of the Constitution and General Orders, No. 58, relative to search and seizure, should be given a liberal construction in favor of the individual in order to maintain the constitutional guaranties whole and in their full force; 2. That since the provisions in question are drastic in their form and fundamentally restrict the enjoyment of the ownership, possession and use of the personal property of the individual, they should be strictly construed; 3. That the search and seizure made are illegal for the following reasons: (a) Because the warrant was based solely upon the affidavit of the petitioner who had no personal knowledge of the facts of probable cause, and (b) because the warrant was issued for the sole purpose of seizing evidence which would later be used in the criminal proceedings that might be instituted against the petitioner, for violation of the Anti-Usury Law; 4. That as the warrant had been issued unreasonably, and as it does not appear positively in the affidavit that the articles were in the possession of the petitioner and in the place indicated, neither could the search and seizure be made at night; 5. That although it is not mandatory to present affidavits of witnesses to corroborate the applicant or a complainant in cases where the latter has personal knowledge of the facts, when the applicant's or complainant's knowledge of the facts is merely hearsay, it is the duty of the judge to require affidavits of other witnesses so that he may determine whether probable cause exists; 6. That a detailed description of the person and place to be searched and the articles to be seized is necessary, but whereby, by the nature of the articles to be seized, their description must be rather general, but is not required that a technical description be given, as this would mean that no warrant could issue; 7. That the petitioner did not waive his constitutional rights because the offer of compromise or settlement attributed to him, does not mean, if so made, that he voluntarily tolerated the search and seizure; and

8. That an appeal from the orders questioned by the petitioner, if taken by him, would not be an effective, speedy or adequate remedy in the ordinary course of law, and, consequently, the petition for mandamusfiled by him, lies. For the foregoing considerations, the search warrant and the seizure of June 3, 1936, and the orders of the respondent court authorizing the relation of the books and documents, are declared illegal and are set aside, and it is ordered that the judge presiding over the Court of First Instance of Tayabas direct the immediate return to the petitioner of the nineteen (19) documents designated on pages 1 to 4 of the inventory by Nos. 5, 10, 16, 23, 25,26, 27, 30, 31, 34, 36, 37, 38, 39, 40, 41, 42, 43 and 45, without special pronouncement as to costs. So ordered. G.R. No. L-64261 December 26, 1984 JOSE BURGOS, SR., JOSE BURGOS, JR., BAYANI SORIANO and J. BURGOS MEDIA SERVICES, INC.,petitioners, vs. THE CHIEF OF STAFF, ARMED FORCES OF THE PHILIPPINES, THE CHIEF, PHILIPPINE CONSTABULARY, THE CHIEF LEGAL OFFICER, PRESIDENTIAL SECURITY COMMAND, THE JUDGE ADVOCATE GENERAL, ET AL., respondents. Lorenzo M. Tañada, Wigberto E. Tañada, Martiniano Vivo, Augusto Sanchez, Joker P. Arroyo, Jejomar Binay and Rene Saguisag for petitioners. The Solicitor General for respondents.

ESCOLIN, J.: Assailed in this petition for certiorari prohibition and mandamus with preliminary mandatory and prohibitory injunction is the validity of two [2] search warrants issued on December 7, 1982 by respondent Judge Ernani Cruz-Pano, Executive Judge of the then Court of First Instance of Rizal [Quezon City], under which the premises known as No. 19, Road 3, Project 6, Quezon City, and 784 Units C & D, RMS Building, Quezon Avenue, Quezon City, business addresses of the "Metropolitan Mail" and "We Forum" newspapers, respectively, were searched, and office and printing machines, equipment, paraphernalia, motor vehicles and other articles used in the printing, publication and distribution of the said newspapers, as well as numerous papers, documents, books and other written literature alleged to be in the possession and control of petitioner Jose Burgos, Jr. publishereditor of the "We Forum" newspaper, were seized. Petitioners further pray that a writ of preliminary mandatory and prohibitory injunction be issued for the return of the seized articles, and that respondents, "particularly the Chief Legal Officer, Presidential Security Command, the Judge Advocate General, AFP, the City Fiscal of Quezon City, their representatives, assistants, subalterns, subordinates, substitute or successors" be enjoined from using the articles thus seized as evidence against petitioner Jose Burgos, Jr. and the other accused in Criminal Case No. Q- 022782 of the Regional Trial Court of Quezon City, entitled People v. Jose Burgos, Jr. et al. 1 In our Resolution dated June 21, 1983, respondents were required to answer the petition. The plea for preliminary mandatory and prohibitory injunction was set for hearing on June 28, 1983, later reset to July 7, 1983, on motion of the Solicitor General in behalf of respondents.

At the hearing on July 7, 1983, the Solicitor General, while opposing petitioners' prayer for a writ of preliminary mandatory injunction, manifested that respondents "will not use the aforementioned articles as evidence in the aforementioned case until final resolution of the legality of the seizure of the aforementioned articles. ..." 2 With this manifestation, the prayer for preliminary prohibitory injunction was rendered moot and academic. Respondents would have this Court dismiss the petition on the ground that petitioners had come to this Court without having previously sought the quashal of the search warrants before respondent judge. Indeed, petitioners, before impugning the validity of the warrants before this Court, should have filed a motion to quash said warrants in the court that issued them. 3 But this procedural flaw notwithstanding, we take cognizance of this petition in view of the seriousness and urgency of the constitutional issues raised not to mention the public interest generated by the search of the "We Forum" offices, which was televised in Channel 7 and widely publicized in all metropolitan dailies. The existence of this special circumstance justifies this Court to exercise its inherent power to suspend its rules. In the words of the revered Mr. Justice Abad Santos in the case of C. Vda. de Ordoveza v. Raymundo, 4 "it is always in the power of the court [Supreme Court] to suspend its rules or to except a particular case from its operation, whenever the purposes of justice require it...". Respondents likewise urge dismissal of the petition on ground of laches. Considerable stress is laid on the fact that while said search warrants were issued on December 7, 1982, the instant petition impugning the same was filed only on June 16, 1983 or after the lapse of a period of more than six [6] months. Laches is failure or negligence for an unreasonable and unexplained length of time to do that which, by exercising due diligence, could or should have been done earlier. It is negligence or omission to assert a right within a reasonable time, warranting a presumption that the party entitled to assert it either has abandoned it or declined to assert it. 5 Petitioners, in their Consolidated Reply, explained the reason for the delay in the filing of the petition thus: Respondents should not find fault, as they now do [p. 1, Answer, p. 3, Manifestation] with the fact that the Petition was filed on June 16, 1983, more than half a year after the petitioners' premises had been raided. The climate of the times has given petitioners no other choice. If they had waited this long to bring their case to court, it was because they tried at first to exhaust other remedies. The events of the past eleven fill years had taught them that everything in this country, from release of public funds to release of detained persons from custody, has become a matter of executive benevolence or largesse Hence, as soon as they could, petitioners, upon suggestion of persons close to the President, like Fiscal Flaminiano, sent a letter to President Marcos, through counsel Antonio Coronet asking the return at least of the printing equipment and vehicles. And after such a letter had been sent, through Col. Balbino V. Diego, Chief Intelligence and Legal Officer of the Presidential Security Command, they were further encouraged to hope that the latter would yield the desired results. After waiting in vain for five [5] months, petitioners finally decided to come to Court. [pp. 123-124, Rollo]

Although the reason given by petitioners may not be flattering to our judicial system, We find no ground to punish or chastise them for an error in judgment. On the contrary, the extrajudicial efforts exerted by petitioners quite evidently negate the presumption that they had abandoned their right to the possession of the seized property, thereby refuting the charge of laches against them. Respondents also submit the theory that since petitioner Jose Burgos, Jr. had used and marked as evidence some of the seized documents in Criminal Case No. Q- 022872, he is now estopped from challenging the validity of the search warrants. We do not follow the logic of respondents. These documents lawfully belong to petitioner Jose Burgos, Jr. and he can do whatever he pleases with them, within legal bounds. The fact that he has used them as evidence does not and cannot in any way affect the validity or invalidity of the search warrants assailed in this petition. Several and diverse reasons have been advanced by petitioners to nullify the search warrants in question. 1. Petitioners fault respondent judge for his alleged failure to conduct an examination under oath or affirmation of the applicant and his witnesses, as mandated by the above-quoted constitutional provision as wen as Sec. 4, Rule 126 of the Rules of Court .6 This objection, however, may properly be considered moot and academic, as petitioners themselves conceded during the hearing on August 9, 1983, that an examination had indeed been conducted by respondent judge of Col. Abadilla and his witnesses. 2. Search Warrants No. 20-82[a] and No. 20- 82[b] were used to search two distinct places: No. 19, Road 3, Project 6, Quezon City and 784 Units C & D, RMS Building, Quezon Avenue, Quezon City, respectively. Objection is interposed to the execution of Search Warrant No. 20-82[b] at the latter address on the ground that the two search warrants pinpointed only one place where petitioner Jose Burgos, Jr. was allegedly keeping and concealing the articles listed therein, i.e., No. 19, Road 3, Project 6, Quezon City. This assertion is based on that portion of Search Warrant No. 20- 82[b] which states: Which have been used, and are being used as instruments and means of committing the crime of subversion penalized under P.D. 885 as amended and he is keeping and concealing the same at 19 Road 3, Project 6, Quezon City. The defect pointed out is obviously a typographical error. Precisely, two search warrants were applied for and issued because the purpose and intent were to search two distinct premises. It would be quite absurd and illogical for respondent judge to have issued two warrants intended for one and the same place. Besides, the addresses of the places sought to be searched were specifically set forth in the application, and since it was Col. Abadilla himself who headed the team which executed the search warrants, the ambiguity that might have arisen by reason of the typographical error is more apparent than real. The fact is that the place for which Search Warrant No. 20- 82[b] was applied for was 728 Units C & D, RMS Building, Quezon Avenue, Quezon City, which address appeared in the opening paragraph of the said warrant. 7 Obviously this is the same place that respondent judge had in mind when he issued Warrant No. 20-82 [b]. In the determination of whether a search warrant describes the premises to be searched with sufficient particularity, it has been held "that the executing officer's prior knowledge as to the place intended in the warrant is relevant. This would seem to be especially true where the executing officer is the affiant on whose affidavit the warrant had issued, and when he knows that the judge who issued the warrant intended the building described in the affidavit, And it has also been said that the executing officer may look to the affidavit in the official court file to resolve an ambiguity in the warrant as to the place to be searched." 8

3. Another ground relied upon to annul the search warrants is the fact that although the warrants were directed against Jose Burgos, Jr. alone, articles b belonging to his co-petitioners Jose Burgos, Sr., Bayani Soriano and the J. Burgos Media Services, Inc. were seized. Section 2, Rule 126 of the Rules of Court, enumerates the personal properties that may be seized under a search warrant, to wit: Sec. 2. Personal Property to be seized. — A search warrant may be issued for the search and seizure of the following personal property: [a] Property subject of the offense; [b] Property stolen or embezzled and other proceeds or fruits of the offense; and [c] Property used or intended to be used as the means of committing an offense. The above rule does not require that the property to be seized should be owned by the person against whom the search warrant is directed. It may or may not be owned by him. In fact, under subsection [b] of the above-quoted Section 2, one of the properties that may be seized is stolen property. Necessarily, stolen property must be owned by one other than the person in whose possession it may be at the time of the search and seizure. Ownership, therefore, is of no consequence, and it is sufficient that the person against whom the warrant is directed has control or possession of the property sought to be seized, as petitioner Jose Burgos, Jr. was alleged to have in relation to the articles and property seized under the warrants. 4. Neither is there merit in petitioners' assertion that real properties were seized under the disputed warrants. Under Article 415[5] of the Civil Code of the Philippines, "machinery, receptables, instruments or implements intended by the owner of the tenement for an industry or works which may be carried on in a building or on a piece of land and which tend directly to meet the needs of the said industry or works" are considered immovable property. In Davao Sawmill Co. v. Castillo9 where this legal provision was invoked, this Court ruled that machinery which is movable by nature becomes immobilized when placed by the owner of the tenement, property or plant, but not so when placed by a tenant, usufructuary, or any other person having only a temporary right, unless such person acted as the agent of the owner. In the case at bar, petitioners do not claim to be the owners of the land and/or building on which the machineries were placed. This being the case, the machineries in question, while in fact bolted to the ground remain movable property susceptible to seizure under a search warrant. 5. The questioned search warrants were issued by respondent judge upon application of Col. Rolando N. Abadilla Intelligence Officer of the P.C. Metrocom.10 The application was accompanied by the Joint Affidavit of Alejandro M. Gutierrez and Pedro U. Tango, 11 members of the Metrocom Intelligence and Security Group under Col. Abadilla which conducted a surveillance of the premises prior to the filing of the application for the search warrants on December 7, 1982.

It is contended by petitioners, however, that the abovementioned documents could not have provided sufficient basis for the finding of a probable cause upon which a warrant may validly issue in accordance with Section 3, Article IV of the 1973 Constitution which provides:

SEC. 3. ... and no search warrant or warrant of arrest shall issue except upon probable cause to be determined by the judge, or such other responsible officer as may be authorized by law, after examination under oath or affirmation of the complainant and the witnesses he may produce, and particularly describing the place to be searched and the persons or things to be seized. We find petitioners' thesis impressed with merit. Probable cause for a search is defined as such facts and circumstances which would lead a reasonably discreet and prudent man to believe that an offense has been committed and that the objects sought in connection with the offense are in the place sought to be searched. And when the search warrant applied for is directed against a newspaper publisher or editor in connection with the publication of subversive materials, as in the case at bar, the application and/or its supporting affidavits must contain a specification, stating with particularity the alleged subversive material he has published or is intending to publish. Mere generalization will not suffice. Thus, the broad statement in Col. Abadilla's application that petitioner "is in possession or has in his control printing equipment and other paraphernalia, news publications and other documents which were used and are all continuously being used as a means of committing the offense of subversion punishable under Presidential Decree 885, as amended ..." 12 is a mere conclusion of law and does not satisfy the requirements of probable cause. Bereft of such particulars as would justify a finding of the existence of probable cause, said allegation cannot serve as basis for the issuance of a search warrant and it was a grave error for respondent judge to have done so.

Equally insufficient as basis for the determination of probable cause is the statement contained in the joint affidavit of Alejandro M. Gutierrez and Pedro U. Tango, "that the evidence gathered and collated by our unit clearly shows that the premises above- mentioned and the articles and things above-described were used and are continuously being used for subversive activities in conspiracy with, and to promote the objective of, illegal organizations such as the Light-a-Fire Movement, Movement for Free Philippines, and April 6 Movement." 13 In mandating that "no warrant shall issue except upon probable cause to be determined by the judge, ... after examination under oath or affirmation of the complainant and the witnesses he may produce; 14 the Constitution requires no less than personal knowledge by the complainant or his witnesses of the facts upon which the issuance of a search warrant may be justified. In Alvarez v. Court of First Instance, 15 this Court ruled that "the oath required must refer to the truth of the facts within the personal knowledge of the petitioner or his witnesses, because the purpose thereof is to convince the committing magistrate, not the individual making the affidavit and seeking the issuance of the warrant, of the existence of probable cause." As couched, the quoted averment in said joint affidavit filed before respondent judge hardly meets the test of sufficiency established by this Court in Alvarez case.

Another factor which makes the search warrants under consideration constitutionally objectionable is that they are in the nature of general warrants. The search warrants describe the articles sought to be seized in this wise: 1] All printing equipment, paraphernalia, paper, ink, photo (equipment, typewriters, cabinets, tables, communications/recording equipment, tape recorders, dictaphone and the like used and/or connected in the printing of the "WE FORUM" newspaper and any and all documents communication, letters and facsimile of prints related to the "WE FORUM" newspaper. 2] Subversive documents, pamphlets, leaflets, books, and other publication to promote the objectives and piurposes of the subversive organization known as Movement for Free Philippines, Light-a-Fire Movement and April 6 Movement; and, 3] Motor vehicles used in the distribution/circulation of the "WE FORUM" and other subversive materials and propaganda, more particularly,

1] Toyota-Corolla, colored yellow with Plate No. NKA 892; 2] DATSUN pick-up colored white with Plate No. NKV 969 3] A delivery truck with Plate No. NBS 524; 4] TOYOTA-TAMARAW, colored white with Plate No. PBP 665; and, 5] TOYOTA Hi-Lux, pick-up truck with Plate No. NGV 427 with marking "Bagong Silang." In Stanford v. State of Texas 16 the search warrant which authorized the search for "books, records, pamphlets, cards, receipts, lists, memoranda, pictures, recordings and other written instruments concerning the Communist Party in Texas," was declared void by the U.S. Supreme Court for being too general. In like manner, directions to "seize any evidence in connectionwith the violation of SDC 13-3703 or otherwise" have been held too general, and that portion of a search warrant which authorized the seizure of any "paraphernalia which could be used to violate Sec. 54-197 of the Connecticut General Statutes [the statute dealing with the crime of conspiracy]" was held to be a general warrant, and therefore invalid. 17 The description of the articles sought to be seized under the search warrants in question cannot be characterized differently.

In the Stanford case, the U.S. Supreme Courts calls to mind a notable chapter in English history: the era of disaccord between the Tudor Government and the English Press, when "Officers of the Crown were given roving commissions to search where they pleased in order to suppress and destroy the literature of dissent both Catholic and Puritan Reference herein to such historical episode would not be relevant for it is not the policy of our government to suppress any newspaper or publication that speaks with "the voice of non-conformity" but poses no clear and imminent danger to state security. As heretofore stated, the premises searched were the business and printing offices of the "Metropolitan Mail" and the "We Forum newspapers. As a consequence of the search and seizure, these premises were padlocked and sealed, with the further result that the printing and publication of said newspapers were discontinued. Such closure is in the nature of previous restraint or censorship abhorrent to the freedom of the press guaranteed under the fundamental law, 18 and constitutes a virtual denial of petitioners' freedom to express themselves in print. This state of being is patently anathematic to a democratic framework where a free, alert and even militant press is essential for the political enlightenment and growth of the citizenry.

Respondents would justify the continued sealing of the printing machines on the ground that they have been sequestered under Section 8 of Presidential Decree No. 885, as amended, which authorizes "the sequestration of the property of any person, natural or artificial, engaged in subversive activities against the government and its duly constituted authorities ... in accordance with implementing rules and regulations as may be issued by the Secretary of National Defense." It is doubtful however, if sequestration could validly be effected in view of the absence of any implementing rules and regulations promulgated by the Minister of National Defense. Besides, in the December 10, 1982 issue of the Daily Express, it was reported that no less than President Marcos himself denied the request of the military authorities to sequester the property seized from petitioners on December 7, 1982. Thus: The President denied a request flied by government prosecutors for sequestration of the WE FORUM newspaper and its printing presses, according to Information Minister Gregorio S. Cendana.

On the basis of court orders, government agents went to the We Forum offices in Quezon City and took a detailed inventory of the equipment and all materials in the premises. Cendaña said that because of the denial the newspaper and its equipment remain at the disposal of the owners, subject to the discretion of the court. 19 That the property seized on December 7, 1982 had not been sequestered is further confirmed by the reply of then Foreign Minister Carlos P. Romulo to the letter dated February 10, 1983 of U.S. Congressman Tony P. Hall addressed to President Marcos, expressing alarm over the "WE FORUM " case. 20 In this reply dated February 11, 1983, Minister Romulo stated:

2. Contrary to reports, President Marcos turned down the recommendation of our authorities to close the paper's printing facilities and confiscate the equipment and materials it uses. 21 IN VIEW OF THE FOREGOING, Search Warrants Nos. 20-82[a] and 20-82[b] issued by respondent judge on December 7, 1982 are hereby declared null and void and are accordingly set aside. The prayer for a writ of mandatory injunction for the return of the seized articles is hereby granted and all articles seized thereunder are hereby ordered released to petitioners. No costs.

[G.R. No. 113930. March 5, 1996]

PAUL G. ROBERTS, JR., RODOLFO C. SALAZAR, LUIS LORENZO, SR., LUIS LORENZO, JR., AMAURY R. GUTIERREZ, BAYANI N. FABIC, JOSE YULO, JR., ESTEBAN B. PALANNUAYAN, and WONG FONG FUI, petitioners, vs. THE COURT OF APPEALS, THE HON. MAXIMIANO ASUNCION, in his capacity as the Presiding Judge of the Regional Trial Court, Quezon City, Branch 104, HON. APOLINARIO G. EXEVEA, HON. HENRICK F. GINGOYON, and HON. PHILIP A. AGUINALDO, in their capacities as Members of the Department of Judge 349 Committee, and the CITY PROSECUTOR OF QUEZON CITY, respondents. ROBERTO DELGADO, petitioner-intervenor. DECISION DAVIDE, JR., J.:

We are urged in this petition to set aside (a) the decision of the Court of Appeals of 28 September 1993 in CA-G.R. SP No. 31226, which dismissed the petition therein on the ground that it has been mooted with the release by the Department of Justice of its decision x x x dismissing petitioners petition for review; (b) the resolution of the said court of 9 February 1994 denying the [1]

[2]

petitioners motion to reconsider the decision; (c) the order of 17 May 1993 of respondent Judge Maximiano C. Asuncion of Branch 104 of the Regional Trial Court (RTC) of Quezon City in Criminal Case No. Q-93-43198 denying petitioners motion to suspend proceedings and to hold in abeyance the issuance of the warrants of arrest and the public prosecutors motion to defer arraignment; and (d) the resolution of 23 July 1993 and 3 February 1994 of the Department of Justice, (DOJ) dismissing petitioners petition for the review of the Joint Resolution of the Assistant City Prosecutor of Quezon City and denying the motion to reconsider the dismissal, respectively. [3]

[4]

The petitioners rely on the following grounds for the grant of the reliefs prayed for in this petition: I

Respondent Judge acted with grave abuse of discretion when he ordered the arrest of the petitioners without examining the record of the preliminary investigation and in determining for himself on the basis thereof the existence of probable cause. II

The Department of Justice 349 Committee acted with grave abuse of discretion when it refused to review the City Prosecutors Joint Resolution and dismissed petitioners appeal therefrom. III

The Court of Appeals acted with grave abuse of discretion when it upheld the subject order directing the issuance of the warrants of arrest without assessing for itself whether based on such records there is probable cause against petitioners. IV

The facts on record do not establish prima facie probable cause and Criminal Case No. Q-93-43198 should have been dismissed. [5]

The antecedents of this petition are not disputed. Several thousand holders of 349 Pepsi crowns in connection with the Pepsi Cola Products Phils., Inc.s (PEPSIs) Number Fever Promotion filed with the Office of the City Prosecutor of Quezon City complaints against the petitioners in their respective capacities as Presidents or Chief Executive Officers, Chairman of the Board, Vice-Chairman of the Board, and Directors of PEPSI, and also against other officials of PEPSI. The complaints respectively accuse the petitioners and the other PEPSI officials of the following crimes: [6]

[7]

(a) estafa; (b) violation of R.A. No. 7394, otherwise known as the Consumer Act of the Philippines; (c) violation of E.O. No. 913; and (d) violation of Act No. 2333, entitled An Act Relative to Untrue, Deceptive and Misleading Advertisements, as amended by Act No. 3740. [8]

[9]

After appropriate proceedings, the investigating prosecutor, Ramon M. Gerona, released on 23 March 1993 a Joint Resolution where he recommended the filing of an information against the petitioners and others for the violation of Article 3 18 of the Revised Penal Code and the dismissal of the complaints for the violation of Article 315, 2(d) of the Revised Penal Code; R.A. No. 7394; Act No. 2333, as amended by Act No. 3740; and E.O. No. 913. The dispositive portion thereof reads as follows: [10]

In view of all the foregoing, it is recommended that: 1. The attached information be filed against respondents Paul G. Roberts, Jr., Rodolfo C. Salazar, Rosemarie R. Vera, Luis F. Lorenzo, Sr., Luis P. Lorenzo, Jr., J. Roberto Delgado, Amaury R. Gutierrez, Bayani N. Fabic, Jose Yulo, Jr., Esteban B. Pacannuayan, Jr., Wong Fong Fui, Quintin J. Gomez, Jr. and Chito V. Gutierrez for estafa under Article 318, Revised Penal Code, while the complaint for violation of Article 315, 2(d), Revised Penal Code against same respondents Juanito R. Ignacio, R. Sobong, R.O. Sinsuan, M.P. Zarsadias, L.G. Dabao, Jr., R.L. Domingo, N.N. Bacsal, Jesus M. Manalastas, Janette P. Pio de Roda, Joaquin W. Sampaico, Winefreda 0. Madarang, Jack Gravey, Les G. Ham, Corazon Pineda, Edward S. Serapio, Alex 0. Caballes, Sandy Sytangco, Jorge W. Drysdale, Richard Blossom, Pablo de Borja, Edmundo L. Tan, Joseph T. Cohen, Delfin Dator, Zosimo B. San Juan, Joaquin Franco, Primitivo S. Javier, Jr., Luisito Guevarra, Asif H. Adil, Eugenio Muniosguren, James Ditkoff and Timothy Lane be dismissed; 2. The complaints against all respondents for violation of R.A. 7394 otherwise known as the Consumer Act of the Philippines and violation of Act 2333 as amended by Act 3740 and E 0. 913 be also dismissed for insufficiency of evidence, and 3. I.S. Nos. 92-7833; 92-8710 and 92-P-1065 involving Crowns Nos. 173; 401; and 117, 425, 703 and 373, respectively, alleged to be likewise winning ones be further investigated to afford respondents a chance to submit their counter-evidence.[11]

On 6 April 1993, City Prosecutor Candido V. Rivera approved the recommendation with the modification that Rosemarie Vera, Quintin Gomez, Jr., and Chito Gonzales be excluded from the charge on the ground of insufficiency of evidence. [12]

The information for estafa attached to the Joint Resolution was approved (on 7 April 1993) by Ismael P. Casabar, Chief of the Prosecution Division, upon authority of the City Prosecutor of Quezon City, and was filed with the RTC of Quezon City on 12 April 1993. It was docketed as Criminal Case No. Q-93-43198. The information reads as follows: [13]

The undersigned 1st Assistant City Prosecutor accuses PAUL G. ROBERTS, JR. RODOLFO C. SALAZAR, LUIS F. LORENZO, SR., LUIS P. LORENZO, JR., J. ROBERTO DELGADO, AMAURY R. GUTIERREZ, BAYANI N. FABIC, JOSE YULO, JR., ESTEBAN B. PACANNUAYAN, JR. and WONG FONG FUI, of the crime of ESTAFA, committed as follows: That in the month of February, 1992, in Quezon City, Philippines and for sometime prior and subsequent thereto, the above-named accused Paul G. Roberts, Jr. ) being then the Presidents Rodolfo G. Salazar and Executive Officers Luis F. Lorenzo, Sr. ) being then the Chairman of the Board of Directors Luis P. Lorenzo, Jr. ) being then the Vice Chairman of the Board J. Roberto Delgado ) being then Members of the Board Amaury R. Gutierrez ) Bayani N. Fabic ) Jose Yulo, Jr. ) Esteban B. Pacannuayan, Jr. and Wong Fong Fui ) OF THE PEPSI COLA PRODUCTS PHILIPPINES, INC., CONSPIRING with one another, with intent of gain, by means of deceit, fraudulent acts or false pretenses, executed prior to or simultaneously with the commission of the fraud, did then and there willfully, unlawfully and feloniously defraud the private complainants whose names with their prizes claimed appear in the attached lists marked as Annexes A to A-46; B to -33; C to C-281; D to D-238; E to E-3O and F to F-244 in the following manner: on the date and in the place aforementioned, said accused pursuant to their conspiracy, launched the Pepsi Cola Products Philippines, Inc. Number Fever Promotion from February 17 to May 8, 1992 later extended to May 11-June 12, 1992 and announced and advertised in the media that all holders of crowns and/or caps of Pepsi, Mirinda, Mountain Dew and Seven-Up bearing the winning 3-digit number will win the full amount of the prize printed on the crowns/caps which are marked with a seven-digit security code as a measure against tampering or faking of crowns and each and every number has its own unique matching security code, enticing the public to buy Pepsi softdrinks with aforestated alluring and attractive advertisements to become millionaires, and by virtue of such representations made by the accused, the said complainants bought Pepsi softdrinks, but, the said accused after their TV announcement on May 25, 1992 that the winning number for the next day was 349, in violation of their aforecited mechanics, refused as they still refuse to redeem/pay the

said Pepsi crowns and/or caps presented to them by the complainants, who, among others, were able to buy Pepsi softdrinks with crowns/caps bearing number 349 with security codes L-2560-FQ and L-3560-FQ, despite repeated demands made by the complainants, to their damage and prejudice to the extent of the amount of the prizes respectively due them from their winning 349 crowns/caps, together with such other amounts they spent ingoing to and from the Office of Pepsi to claim their prizes and such other amounts used in buying Pepsi softdrinks which the complainants normally would not have done were it not for the false, fraudulent and deceitful posters of Pepsi Cola Products, Inc. CONTRARY TO LAW. On 14 April 1993, the petitioners filed with the Office of the City Prosecutor a motion for the reconsideration of the Joint Resolution alleging therein that (a) there was neither fraud in the Number Fever Promotion nor deviation from or modification of the promotional rules approved by the Department of Trade and industry (DTI), for from the start of the promotion, it had always been clearly explained to the public that for one to be entitled to the cash prize his crown must bear both the winning number and the correct security code as they appear in the DTI list; (b) the complainants failed to allege, much less prove with prima facie evidence, the specific overt criminal acts or ommissions purportedly committed by each of the petitioners; (c) the compromise agreement entered into by PEPSI is not an admission of guilt; and (d) the evidence establishes that the promo was carried out with utmost good faith and without malicious intent. [14]

On 15 April 1993, the petitioners filed with the DOJ a Petition for Review wherein, for the same grounds adduced in the aforementioned motion for reconsideration, they prayed that the Joint Resolution be reversed and the complaints dismissed. They further stated that the approval of the Joint Resolution by the City prosecutor was not the result of a careful scrutiny and independent evaluation of the relevant facts and the applicable law but of the grave threats, intimidation, and actual violence which the complainants had inflicted on him and his assistant prosecutors. [15]

On that same date, the petitioners filed in Criminal Case No. Q-93-43198 Motions to Suspend Proceedings and to Hold in Abeyance Issuance of Warrants of Arrest on the ground that they had filed the aforesaid Petition for Review. [16]

On 21 April 1993, acting on the Petition for Review, Chief State Prosecutor Zenon L. De Guia issued a 1st Indorsement, directing the City Prosecutor of Quezon City to inform the DOJ whether the petitioners have already been [17]

arraigned, and if not, to move in court for the deferment of further proceedings in the case and to elevate to the DOJ the entire records of the case, for the case is being treated as an exception pursuant to Section 4 of Department Circular No. 7 dated 25 January 1990. On 22 April 1993, Criminal Case no. Q-93-41398 was raffled to Branch 104 of the RTC of Quezon City. [18]

In the morning of 27 April 1993, private prosecutor Julio Contreras filed an Ex-Parte Motion for Issuance of Warrants of Arrest. [19]

In the afternoon of that same day, petitioner Paul Roberts, Jr., filed a Supplemental Urgent Motion to hold in Abeyance Issuance of Warrant of Arrest and to Suspend Proceedings. He stressed that the DOJ had taken cognizance of the Petition for Review by directing the City Prosecutor to elevate the records of I.S. No. P-4401 and its related cases and asserted that the petition for review was an essential part of the petitioners right to a preliminary investigation. [20]

The next day, respondent Judge Asuncion, Presiding Judge of Branch 104 of the RTC of Quezon City, issued an order advising the parties that his court would be guided by the doctrine laid down by the Supreme Court in the case of Crespo vs. Mogul, 151 SCRA 462 and not by the resolution of the Department of Justice on the petition for review undertaken by the accused. [21]

On 30 April 1993, Assistant City Prosecutor Tirso M. Gavero filed with the trial court a Motion to Defer Arraignment wherein he also prayed that further proceedings be held in abeyance pending final disposition by the Department of Justice. [22]

On 4 May 1993, Gavero filed an Amended Information, accompanied by a corresponding motion to admit it. The amendments merely consist in the statement that the complainants therein were only among others who were defrauded by the accused and that the damage or prejudice caused amounted to several billions of pesos, representing the amounts due them from their winning 349 crowns/caps. The trial court admitted the amended information on the same date. [23]

[24]

[25]

Later, the attorneys for the different private complainants filed, respectively, an Opposition to Motion to Defer Arraignment, and Objection and Opposition to Motion to Suspend Proceedings and to Hold in Abeyance the Issuance of Warrants of Arrest. [26]

[27]

On 14 May 1993, the petitioners filed a Memorandum in support of their Motion to Suspend Proceedings and to Hold in Abeyance the Issuance of the Warrants of Arrest. [28]

On 17 May 1993, respondent Judge Asuncion issued the challenged order (1) denying the petitioners Motion to Suspend Proceedings and to Hold In Abeyance Issuance of Warrants of Arrest and the public prosecutors Motion to Defer Arraignment and (2) directing the issuance of the warrants of arrest after 21 June 1993 and setting the arraignment on 28 June 1993. Pertinent portions of the order read as follows: [29]

In the Motion filed by the accused, it is alleged that on April 15, 1993, they filed a petition for review seeking the reversal of the resolution of the City Prosecutor of Quezon City approving the filing of the case against the accused, claiming that: 1. The resolution constituting [sic] force and duress; 2. There was no fraud or deceit therefore there can be no estafa; 3. No criminal overt acts by respondents were proved; 4. Pepsi nor the accused herein made no admission of guilt before the Department of Trade and Industry; 5. The evidence presented clearly showed no malicious intent on the part of the accused.

Trial Prosecutor Tirso M. Gavero in his Motion to Defer Arraignment averred that there is a pending petition for review with the Department of Justice filed by the accused and the Office of the City Prosecutor was directed, among other things, to cause for the deferment of further proceedings pending final disposition of said petition by the Department of Justice. The motions filed by the accused and the Trial Prosecutor are hereby DENIED. This case is already pending in this Court for trial. To follow whatever opinion the Secretary of Justice may have on the matter would undermine the independence and integrity of this Court. This Court is still capable of administering justice. The Supreme Court in the case of Crespo vs. Mogul (SCRA 151, pp. 471472) stated as follows: In order therefor to avoid such a situation whereby the opinion of the Secretary of Justice who reviewed the action of the fiscal may be disregarded by the trial court, the Secretary of Justice should, as far as practicable, refrain from entertaining a petition for review or appeal from the action of the fiscal, when the complaint or information has already been filed in Court. The matter should be left entirely for the determination of the Court.

WHEREFORE, let warrant of arrest be issued after June 21, 1993, and arraignment be set on June 28, 1993, at 9:30 in the morning. On 7 June 1993, the petitioners filed with the Court of Appeals a special civil action for certiorari and prohibition with application for a temporary restraining order, which was docketed as CA-G.R. SP No. 31226. They contended therein that respondent Judge Asuncion had acted without or in excess of jurisdiction or with grave abuse of discretion in issuing the aforementioned order of 17 May 1993 because [30]

I. RESPONDENT JUDGE FAILED TO EXAMINE THE RECORD OF PRELIMINARY INVESTIGATION BEFORE ORDERING THE ARREST OF PETITIONERS. II. THERE IS NO PROBABLE CAUSE TO HOLD PETITIONERS CRIMINALLY LIABLE FOR ESTAFA, OTHER DECEITS, OR ANY OTHER OFFENSE. III. THE PROCEEDINGS BELOW SHOULD HAVE BEEN SUSPENDED TO AWAIT THE SECRETARY OF JUSTICES RESOLUTION OF PETITIONERS APPEAL, AND IV. THERE IS NO OTHER PLAIN, SPEEDY AND ADEQUATE REMEDY IN THE ORDINARY COURSE OF LAW.

On 15 June 1993, the Court of Appeals issued a temporary restraining order to maintain the status quo. In view thereof, respondent Judge Asuncion issued an order on 28 June 1993 postponing indefinitely the arraignment of the petitioners which was earlier scheduled on that date. [31]

[32]

On 28 June 1993, the Court of Appeals heard the petitioners application for a writ of preliminary injunction, granted the motion for leave to intervene filed by J. Roberto Delgado, and directed the Branch Clerk of Court of the RTC of Quezon City to elevate the original records of Criminal Case No. Q93-43198 [33]

Upon receipt of the original records of the criminal case, the Court of Appeals found that a copy of the Joint Resolution had in fact been forwarded to, and received by, the trial court on 22 April 1993, which fact belied the petitioners claim that the respondent Judge had not the slightest basis at all for determining probable cause when he ordered the issuance of warrants of arrest. It ruled that the Joint Resolution was sufficient in itself to have been relied upon by respondent Judge in convincing himself that probable cause indeed exists for the purpose of issuing the corresponding warrants of arrest; and that the mere silence of the records or the absence of any express declaration in the questioned order as to the basis of such finding does not give rise to an adverse inference, for the respondent Judge enjoys in his favor the presumption of regularity in the performance of his official duty. The Court

of Appeals then issued a resolution denying the application for a writ of preliminary injunction. [34]

On 8 June 1993, the petitioners filed a motion to reconsider the aforesaid resolution. The Court of Appeals required the respondents therein to comment on the said motion. [35]

[36]

On 3 August 1993, the counsel for the private complainants filed in CAG.R. SP No. 31226 a Manifestation informing the court that the petitioners petition for review filed with the DOJ was dismissed in a resolution dated 23 July 1993. A copy of the resolution was attached to the Manifestation. [37]

[38]

On 21 September 1993, the public respondents filed in CA-G.R. SP No. 31226 a motion to dismiss the petition on the ground that it has become moot and academic in view of the dismissal by the DOJ of the petitioners petition to review the Joint Resolution. The dismissal by the DOJ is founded on the following exposition: [39]

You questioned the said order of the RTC before the Court of Appeals and prayed for the issuance of a writ of preliminary injunction to restrain the Trial Judge from issuing any warrant of arrest and from proceeding with the arraignment of the accused. The appellate court in a resolution dated July 1, 1993, denied your petition. In view of the said developments, it would be an exercise in futility to continue reviewing the instant cases for any further action on the part of the Department would depend on the sound discretion of the Trial Court. The denial by the said court of the motion to defer arraignment filed at our instance was clearly an exercise of its discretion. With the issuance of the order dated May 17, 1993, the Trial Court was in effect sending a signal to this Department that the determination of the case is within its exclusive jurisdiction and competence. The rule is that x x x once a complaint or information is filed in Court, any disposition of the case as to dismissal or the conviction or acquittal of the accused rests in the sound discretion of the Court. Although the fiscal retains the direction and control of the prosecution of criminal cases even while the case is already in Court, he cannot impose his opinion on the trial court. The court is the best and sole judge on what to do with the case before it. x x x (Crespo vs. Mogul, 151 SCRA 462). [40]

On 28 September 1993, the Court of Appeals promulgated a decision dismissing the petition because it had been mooted with the release by the Department of Justice of its decision x x x dismissing petitioners petition for review by inerrantly upholding the criminal courts exclusive and unsupplantable authority to control the entire course of the case brought [41]

against petitioners, reiterating with approval the dictum laid down in the Crespo case. The petitioners filed a motion to reconsider the DOJs dismissal of the petition citing therein its resolutions in other similar cases which were favorable to the petitioners and adverse to other 349 Pepsi crowns holders. In its resolution of 3 February 1994, the DOJ, through its 349 Committee, denied the motion and stated: The instant petition is different from the other petitions resolved by this Department in similar cases from the provinces. In the latter petitions, the complaints against herein respondents [sic] were dismissed inasmuch as the informations have not yet been filed or even if already filed in court, the proceedings have been suspended by the courts to await the outcome of the appeal with this Department. [42]

[43]

The petitioners likewise filed a motion to reconsider the aforesaid Court of Appeals decision, which the said court denied in its resolution of 9 February 1994. Hence, the instant petition. [44]

[45]

The First Division of this Court denied due course to this petition in its resolution of 19 September 1994. [46]

On 7 October 1994, the petitioners filed a motion for the reconsideration of the aforesaid resolution. Acting thereon, the First Division required the respondents to comment thereon. [47]

Later, the petitioners filed a supplemental motion for reconsideration and a motion to refer this case to the Court en banc. In its resolution of 14 November 1994, the First Division granted the latter motion and required the respondents to comment on the supplemental motion for reconsideration [48]

[49]

[50]

In the resolution of 24 November 1994, the Court en banc accepted the referral. On 10 October 1995, after deliberating on the motion for reconsideration and the subsequent pleadings in relation thereto, the Court en banc granted the motion for reconsideration; reconsidered and set aside the resolution of 19 September 1994; and reinstated the petition. It then considered the case submitted for decision, since the parties have exhaustively discussed the issues in their pleadings, the original records of Criminal Case No. Q-9343198 and of CA-G.R. SP No. 31226 had been elevated to this Court, and both the petitioners and the Office of the Solicitor General pray, in effect, that this Court resolve the issue of probable cause On the basis thereof. The pleadings of the parties suggest for this Courts resolution the following key issues:

1. Whether public respondent Judge Asuncion committed grave abuse of discretion in denying, on the basis of Crespo vs. Mogul, the motions to suspend proceedings and hold in abeyance the issuance of warrants of arrest and to defer arraignment until after the petition for review filed with the DOJ shall have been resolved. 2. Whether public respondent Judge Asuncion committed grave abuse of discretion in ordering the issuance of warrants of arrest without examining the records of the preliminary investigation. 3. Whether the DOJ, through its 349 Committee, gravely abused its discretion in dismissing the petition for review on the following bases: (a) the resolution of public respondent Court of Appeals denying the application for a writ of preliminary injunction and (b) of public respondent Asuncions denial of the abovementioned motions. 4. Whether public respondent Court of Appeals committed grave abuse of discretion (a) in denying the motion for a writ of preliminary injunction solely on the ground that public respondent Asuncion had already before him the Joint Resolution of the investigating prosecutor when he ordered the issuance of the warrants of arrest, and (b) in ultimately dismissing the petition on the ground of mootness since the DOJ has dismissed the petition for review. 5. Whether this Court may determine in this proceedings the existence of probable cause either for the issuance of warrants of arrest against the petitioners or for their prosecution for the crime of estafa.

We resolve the first four issues in the affirmative and the fifth, in the negative. I. There is nothing in Crespo vs. Mogul which bars the DOJ from taking cognizance of an appeal, by way of a petition for review, by an accused in a criminal case from an unfavorable ruling of the investigating prosecutor. It merely advised the DOJ to, as far as practicable, refrain from entertaining a petition for review or appeal from the action of the fiscal, when the complaint or information has already been filed in Court. More specifically, it stated: [51]

In order therefore to avoid such a situation whereby the opinion of the Secretary of Justice who reviewed the action of the fiscal may be disregarded by the trial court, the Secretary of Justice should, as far as practicable, refrain from entertaining a petition for review or appeal from the action of the fiscal, when the complaint or information has already been filed in Court. The matter should be left entirely for the determination of the Court. [52]

In Marcelo vs. Court of Appeals, this Court explicitly declared: [53]

Nothing in the said ruling forecloses the power or authority of the Secretary of Justice to review resolutions of his subordinates in criminal cases. The Secretary of Justice is only enjoined to refrain as far as practicable from entertaining a petition for review or appeal from the action of the prosecutor once a complaint or information is filed in court. In any case, the grant of a motion to dismiss, which the prosecution may file after the Secretary of Justice reverses an appealed resolution, is subject to the discretion of the court. Crespo could not have intended otherwise without doing violence to, or repealing, the last paragraph of Section 4, Rule 112 of the Rules of Court which recognizes the authority of the Secretary of Justice to reverse the resolution of the provincial or city prosecutor or chief state prosecutor upon petition by a proper party. [54]

Pursuant to the said provision, the Secretary of Justice had promulgated the rules on appeals from resolutions in preliminary investigation. At the time the petitioners filed their petition for the review of the Joint Resolution of the investigating prosecutor, the governing rule was Circular No. 7, dated 25 January 1990. Section 2 thereof provided that only resolutions dismissing a criminal complaint may be appealed to the Secretary of Justice. Its Section 4, however, provided an exception, thus allowing, upon a showing of manifest error or grave abuse of discretion, appeals from resolutions finding probable cause, provided that the accused has not been arraigned. [55]

The DOJ gave due course to the petitioners petition for review as an exception pursuant to Section 4 of Circular No. 7. Meanwhile, the DOJ promulgated on 30 June 1993 Department Order No. 223 which superseded Circular No. 7. This Order, however, retained the provisions of Section 1 of the Circular on appealable cases and Section 4 on the non-appealable cases and the exceptions thereto. [56]

There is nothing in Department Order No. 223 which would warrant a recall of the previous action of the DOJ giving due course to the petitioners petition for review. But whether the DOJ would affirm or reverse the challenged Joint Resolution is still a matter of guesswork. Accordingly, it was premature for respondent Judge Asuncion to deny the motions to suspend proceedings and to defer arraignment on the following grounds: This case is already pending in this Court for trial. To follow whatever opinion the Secretary of Justice may have on the matter would undermine the independence and integrity of this Court. This Court is still capable of administering justice.

The real and ultimate test of the independence and integrity of this court is not the filing of the aforementioned motions at that stage of the proceedings but the filing of a motion to dismiss or to withdraw the information on the basis of a resolution of the petition for review reversing the Joint Resolution of the investigating prosecutor. Before that time, the following pronouncement in Crespo did not yet truly become relevant or applicable: The rule therefore in this jurisdiction is that once a complaint or information is filed in Court any disposition of the case as its dismissal or the conviction or acquittal of the accused rests in the sound discretion of the court. Although the fiscal retains the direction and control of the prosecution of criminal cases even while the case is already in court he cannot impose his opinion on the trial court. The court is the best and sole judge on what to do with the case before it. The determination of the case is within its exclusive jurisdiction and competence. A motion to dismiss the case filed by the fiscal should be addressed to the Court who has the option to grant or deny the same. It does not matter if this is done before or after the arraignment of the accused or that the motion was filed after a reinvestigation or upon instructions of the Secretary of Justice who reviewed the records of the investigation. [57]

However, once a motion to dismiss or withdraw the information is filed the trial judge may grant or deny it, not out of subservience to the Secretary of Justice, but in faithful exercise of judicial prerogative. This Court pertinently stated so in Martinez vs. Court of Appeals: [58]

Whether to approve or disapprove the stand taken by the prosecution is not the exercise of discretion required in cases like this. The trial judge must himself be convinced that there was indeed no sufficient evidence against the accused, and this conclusion can be arrived at only after an assessment of the evidence in the possession of the prosecution. What was imperatively required was the trial judges own assessment of such evidence, it not being sufficient for the valid and proper exercise of judicial discretion merely to accept the prosecutions word for its supposed insufficiency. As aptly observed the Office of the Solicitor General, in failing to make an independent finding of the merits of the case and merely anchoring the dismissal on the revised position of the prosecution, the trial judge relinquished the discretion he was duty bound to exercise. In effect, it was the prosecution, through the Department of Justice which decided what to do and not the court which was reduced to a mere rubber stamp in violation of the ruling in Crespo vs. Mogul.

II. Section 2, Article III of the present Constitution provides that no search warrant or warrant of arrest shall issue except upon probable cause to be determined personally by the judge after examination under oath or affirmation of the complainant and the witnesses he may produce. Under existing laws, warrants of arrest may be issued (1) by the Metropolitan Trial Courts (MeTCs) except those in the National Capital Region, Municipal Trial Courts (MTCs), and Municipal Circuit Trial Courts (MCTCs) in cases falling within their exclusive original jurisdiction; in cases covered by the rule on summary procedure where the accused fails to appear when required; and in cases filed with them which are cognizable by the Regional Trial Courts (RTCs); and (2) by the Metropolitan Trial Courts in the National Capital Region (MeTCs-NCR) and the RTCs in cases filed with them after appropriate preliminary investigations conducted by officers authorized to do so other than judges of MeTCs, MTCs and MCTCs. [59]

[60]

[61]

[62]

As to the first, a warrant can issue only if the judge is satisfied after an examination in writing and under oath of the complainant and the witnesses, in the form of searching questions and answers, that a probable cause exists and that there is a necessity of placing the respondent under immediate custody in order not to frustrate the ends of justice. As to the second, this Court held in Soliven vs. Makasiar that the judge is not required to personally examine the complainant and the witnesses, but [63]

[f]ollowing established doctrine and procedure, he shall: (1) personally evaluate the report and supporting documents submitted by the fiscal regarding the existence of probable cause and, on the basis thereof, issue a warrant of arrest; or (2) if on the basis thereof he finds no probable cause, he may disregard the fiscals report and require the submission of supporting affidavits of witnesses to aid him in arriving at a conclusion as to the existence of probable cause. [64]

Sound policy supports this procedure, otherwise judges would be unduly laden with the preliminary examination and investigation of criminal complaints instead of concentrating on hearing and deciding cases filed before their courts. It must be emphasized that judges must not rely solely on the report or resolution of the fiscal (now prosecutor); they must evaluate the report and the supporting documents. In this sense, the aforementioned requirement has modified paragraph 4(a) of Circular No. 12 issued by this Court on 30 June 1987 prescribing the Guidelines on Issuance of Warrants of

Arrest under Section 2, Article III of the 1987 Constitution, which provided in part as follows: 4. In satisfying himself of the existence of a probable cause for the issuance of a warrant of arrest, the judge, following established doctrine and procedure, may either: (a) Rely upon the fiscals certification of the existence of probable cause whether or not the case is cognizable only by the Regional Trial Court and on the basis thereof, issue a warrant of arrest. x x x

This requirement of evaluation not only of the report or certification of the fiscal but also of the supporting documents was further explained in People vs. Inting, where this Court specified what the documents may consist of, viz., the affidavits, the transcripts of stenographic notes (if any), and all other supporting documents behind the Prosecutors certification which are material in assisting the Judge to make his determination of probable cause. Thus: [65]

We emphasize the important features of the constitutional mandate that x x x no search warrant or warrant of arrest shall issue except upon probable cause to be determined personally by the judge x x x (Article III, Section 2, Constitution). First, the determination of probable cause is a function of the Judge. It is not for the Provincial Fiscal or Prosecutor nor the Election Supervisor to ascertain. Only the Judge and the Judge alone makes this determination. Second, the preliminary inquiry made by a Prosecutor does not bind the Judge. It merely assists him to make the determination of probable cause. The Judge does not have to follow what the Prosecutor presents to him. By itself, the Prosecutors certification of probable cause is ineffectual. It is the report, the affidavits, the transcripts of stenographic notes (if any), and all other supporting documents behind the Prosecutors certification which are material in assisting the Judge to make his determination. In adverting to a statement in People vs. Delgado that the judge may rely on the resolution of the Commission on Elections (COMELEC) to file the information by the same token that it may rely on the certification made by the prosecutor who conducted the preliminary investigation in the issuance of the warrant of arrest, this Court stressed in Lim vs. Felix that [66]

[67]

Reliance on the COMELEC resolution or the Prosecutors certification presupposes that the records of either the COMELEC or the Prosecutor have been submitted to the Judge and he relies on the certification or resolution because the records of the

investigation sustain the recommendation. The warrant issues not on the strength of the certification standing alone but because of the records which sustain it. And noting that judges still suffer from the inertia of decisions and practice under the 1935 and 1973 Constitutions, this Court found it necessary to restate the rule in greater detail and hopefully clearer terms. It then proceeded to do so, thus: We reiterate the ruling in Soliven vs. Makasiar that the Judge does not have to personally examine the complainant and his witnesses. The Prosecutor can perform the same functions as a commissioner for the taking of the evidence. However, there should be a report and necessary documents supporting the Fiscals bare certification. All of these should be before the Judge. The extent of the Judges personal examination of the report and its annexes depends on the circumstances of each case. We cannot determine beforehand how cursory or exhaustive the Judges examination should be. The Judge has to exercise sound discretion for, after all, the personal determination is vested in the Judge by the Constitution. It can be as brief as or detailed as the circumstances of each case require. To be sure, the Judge must go beyond the Prosecutors certification and investigation report whenever, necessary. He should call for the complainant and witnesses themselves to answer the courts probing questions when the circumstances of the case so require. This Court then set aside for being null and void the challenged order of respondent Judge Felix directing the issuance of the warrants of arrest against petitioners Lim, et al., solely on the basis of the prosecutors certification in the informations that there existed probable cause without having before him any other basis for his personal determination of the existence of a probable cause. In Allado vs. Diokno, this Court also ruled that before issuing a warrant of arrest, the judge must satisfy himself that based on the evidence submitted there is sufficient proof that a crime has been committed and that the person to be arrested is probably guilty thereof. [68]

In the recent case of Webb vs. De Leon, this Court rejected the thesis of the petitioners of absence probable cause and sustained the investigating panels and the respondent Judges findings of probable cause. After quoting extensively from Soliven vs. Makasiar, this Court explicitly pointed out: [69]

[70]

Clearly then, the Constitution, the Rules of Court, and our case law repudiate the submission of petitioners that respondent judges should have conducted searching examination of witnesses before issuing warrants of arrest against them. They also reject petitioners contention that a judge must first issue an order of arrest before issuing a warrant of arrest. There is no law or rule requiring the issuance of an Order of Arrest prior to a warrant of arrest. In the case at bar, the DOJ Panel submitted to the trial court its 26-page report, the two (2) sworn statements of Alfaro and the sworn statements of Carlos Cristobal and Lolita Birrer as well as the counter- affidavits of the petitioners. Apparently, the painstaking recital and analysis of the parties evidence made in the DOJ Panel Report satisfied both judges that there is probable cause to issue warrants of arrest against petitioners. Again, we stress that before issuing warrants of arrest, judges merely determine personally the probability, not the certainty of the guilt of an accused. In doing so, judges do not conduct a de novo hearing to determine the existence of probable cause. They just personally review the initial determination of the prosecutor finding a probable cause to see if it is supported by substantial evidence. The sufficiency of the review process cannot be measured by merely counting minutes and hours. The fact that it took the respondent judges a few hours to review and affirm the Probable cause determination of the DOJ Panel does not mean they made no personal evaluation of the evidence attached to the records of the case. (italics supplied) The teachings then of Soliven, Inting, Lim, Allado, and Webb reject the proposition that the investigating prosecutors certification in an information or his resolution which is made the basis for the filing of the information, or both, would suffice in the judicial determination of probable cause for the issuance of a warrant of arrest. In Webb, this Court assumed that since the respondent Judges had before them not only the 26-page resolution of the investigating panel but also the affidavits of the prosecution witnesses and even the counter-affidavits of the respondents, they (judges) made personal evaluation of the evidence attached to the records of the case. Unfortunately, in Criminal Case No. Q-93-43198, nothing accompanied the information upon its filing on 12 April 1993 with the trial court. As found by the Court of Appeals in its resolution of 1 July 1993, a copy of the Joint Resolution was forwarded to, and received by, the trial court only on 22 April 1993. And as revealed by the certification of Branch Clerk of Court Gibson Araula, Jr., no affidavits of the witnesses, transcripts of stenographic notes of the proceedings during the preliminary investigation, or other documents submitted in the course thereof were found in the records of Criminal Case [71]

No. Q-93-43198 as of 19 May 1993. Clearly, when respondent Judge Asuncion issued the assailed order of 17 May 1993 directing, among other things, the issuance of warrants of arrest, he had only the information, amended information, and Joint Resolution as bases thereof. He did not have the records or evidence supporting the prosecutors finding of probable cause. And strangely enough, he made no specific finding of probable cause; he merely directed the issuance of warrants of arrest after June 21, 1993. It may, however, be argued that the directive presupposes a finding of probable cause. But then compliance with a constitutional requirement for the protection of individual liberty cannot be left to presupposition, conjecture, or even convincing logic. III. As earlier stated, per its 1st Indorsement of 21 April 1993, the DOJ gave due course to the petitioners petition for review pursuant to the exception provided for in Section 4 of Circular No. 7, and directed the Office of the City Prosecutor of Quezon City to forward to the Department the records of the cases and to file in court a motion for the deferment of the proceedings. At the time it issued the indorsement, the DOJ already knew that the information had been filed in court, for which reason it directed the City Prosecutor to inform the Department whether the accused have already been arraigned and if not yet arraigned, to move to defer further proceedings. It must have been fully aware that, pursuant to Crespo vs. Mogul, a motion to dismiss a case filed by the prosecution either as a consequence of a reinvestigation or upon instructions of the Secretary of Justice after a review of the records of the investigation is addressed to the trial court, which has the option to grant or to deny it. Also, it must have been still fresh in its mind that a few months back it had dismissed for lack of probable cause other similar complaints of holders of 349 Pepsi crowns. Thus, its decision to give due course to the petition must have been prompted by nothing less than an honest conviction that a review of the Joint Resolution was necessary in the highest interest of justice in the light of the special circumstances of the case. That decision was permissible within the as far as practicable criterion in Crespo. [72]

Hence, the DOJ committed grave abuse of discretion when it executed on 23 July 1993 a unilateral volte-face, which was even unprovoked by a formal pleading to accomplish the same end, by dismissing the petition for review. It dismissed the petition simply because it thought that a review of the Joint Resolution would be an exercise in futility in that any further action on the part of the Department would depend on the sound discretion of the trial

court, and that the latters denial of the motion to defer arraignment filed at the instance of the DOJ was clearly an exercise of that discretion or was, in effect, a signal to the Department that the determination of the case is within the courts exclusive jurisdiction and competence. This infirmity becomes more pronounced because the reason adduced by the respondent Judge for his denial of the motions to suspend proceedings and hold in abeyance issuance of warrants of arrest and to defer arraignment finds, as yet, no support in Crespo. IV. If the only issue before the Court of Appeals were the denial of the petitioners Motion to Suspend Proceedings and to Hold in Abeyance Issuance of Warrants of Arrest and the public prosecutors Motion to Defer Arraignment, which were both based on the pendency before the DOJ of the petition for the review of the Joint Resolution, the dismissal of CA-G.R. SP No. 31226 on the basis of the dismissal by the DOJ of the petition for review might have been correct. However, the petition likewise involved the issue of whether respondent Judge Asuncion gravely abused his discretion in ordering the issuance of warrants of arrest despite want of basis. The DOJs dismissal of the petition for review did not render moot and academic the latter issue. In denying in its resolution of 1 July 1993 the petitioners application for a writ of preliminary injunction to restrain respondent Judge Asuncion from issuing warrants of arrest, the Court of Appeals ,justified its action in this wise: The Joint Resolution was sufficient in itself to have been relied upon by respondent Judge in convincing himself that probable cause indeed exists for the purpose of issuing the corresponding warrants of arrest. The mere silence of the records or the absence of any express declaration in the questioned Order of May 17, 1993 as to where the respondent Judge based his finding of probable cause does not give rise to any adverse inference on his part. The fact remains that the Joint Resolution was at respondent Judges disposal at the time he issued the Order for the issuance of the warrants of arrest. After all, respondent Judge enjoys in his favor the presumption of regularity in the performance of official actuations. And this presumption prevails until it is overcome by clear and convincing evidence to the contrary. Every reasonable intendment will be made in support of the presumption, and in case of doubt as to an officers act being lawful or unlawful it should be construed to be lawful. (31 C.J.S., 808-810. See also Mahilum, et al. vs. Court of Appeals, 17 SCRA 482; People vs. Cortez, 21 SCRA 1228; Government of the P.I. vs.Galarosa, 36 Phil. 338).

We are unable to agree with this disquisition, for it merely assumes at least two things: (1) that respondent Judge Asuncion had read and relied on the Joint Resolution and (2) he was convinced that probable cause exists for the issuance of the warrants of arrest against the petitioners. Nothing in the records provides reasonable basis for these assumptions. In his assailed order, the respondent Judge made no mention of the Joint Resolution, which was attached to the records of Criminal Case No. Q-93-43198 on 22 April 1993. Neither did he state that he found probable cause for the issuance of warrants of arrest. And, for an undivinable reason, he directed the issuance of warrants of arrest only after June 21, 1993. If he did read the Joint Resolution and, in so reading, found probable cause, there was absolutely no reason at all to delay for more than one month the issuance of warrants of arrest. The most probable explanation for such delay could be that the respondent Judge had actually wanted to wait for a little while for the DOJ to resolve the petition for review. It is, nevertheless, contended in the dissenting opinion of Mr. Justice Reynato S. Puno that whatever doubts may have lingered on the issue of probable cause was dissolved when no less than the Court of Appeals sustained the finding of probable cause made by the respondent Judge after an evaluation of the Joint Resolution. We are not persuaded with that opinion. It is anchored on erroneous premises. In its 1 July 1993 resolution, the Court of Appeals does not at all state that it either sustained respondent Judge Asuncions finding of probable cause, or found by itself probable cause. As discussed above, it merely presumed that Judge Asuncion might have read the Joint Resolution and found probable cause from a reading thereof. Then too, that statement in the dissenting opinion erroneously assumes that the Joint Resolution can validly serve as sufficient basis for determining probable cause. As stated above, it is not. V. In criminal prosecutions, the determination of probable cause may either be an executive or a judicial prerogative. In People vs. Inting, this Court aptly stated: [73]

And third, Judges and Prosecutors alike should distinguish the preliminary inquiry which determines probable cause for the issuance of a warrant of arrest from a preliminary investigation proper which ascertains whether the offender should be held for trial or released. Even if the two inquiries are conducted in the course of one and the same proceeding, there should be no confusion about the objectives. The

determination of probable cause for the warrant of arrest is made by the Judge. The preliminary investigation proper - whether or not there is reasonable ground to believe that the accused is guilty of the offense charged and, therefore, whether or not he should be subjected to the expense, rigors and embarrassment of trial- is the function of the Prosecutor. xxx xxx xxx We reiterate that preliminary investigation should be distinguished as to whether it is an investigation for the determination of a sufficient ground for the filing of the information or it is an investigation for the determination of a probable cause for the issuance of a warrant of arrest. The first kind of preliminary investigation is executive in nature. It is part of the prosecutions job. The second kind of preliminary investigation which is more properly called preliminary examination is judicial in nature and is lodged with the judge x x x. Ordinarily, the determination of probable cause is not lodged with this Court. Its duty in an appropriate case is confined to the issue of whether the executive or judicial determination, as the case may be, of probable cause was done without or in excess of jurisdiction or with grave abuse of discretion amounting to want of jurisdiction. This is consistent with the general rule that criminal prosecutions may not be restrained or stayed by injunction, preliminary or final. There are, however, exceptions to this rule. Among the exceptions are enumerated in Brocka vs. Enrile as follows: [74]

a. To afford adequate protection to the constitutional rights of the accused (Hernandez vs. Albano, et al., L-19272, January 25, 1967, 19 SCRA 95); b. When necessary for the orderly administration of justice or to avoid oppression or multiplicity of actions (Dimayuga, et al. vs. Fernandez, 43 Phil. 304; Hernandez vs. Albano, supra; Fortun vs. Labang, et al., L-38383, May 27, 1981, 104 SCRA 607); c. When there is a pre-judicial question which is sub judice (De Leon vs. Mabanag, 70 Phil. 202); d. When the acts of the officer are without or in excess of authority (Planas vs. Gil, 67 Phil. 62); e. Where the prosecution is under an invalid law, ordinance or regulation (Young vs. Rafferty, 33 Phil. 556; Yu Cong Eng vs. Trinidad, 47 Phil. 385, 389); f. When double jeopardy is clearly apparent (Sangalang vs. People and Avendia, 109 Phil. 1140); g. Where the court has no jurisdiction over the offense (Lopez vs. City Judge, L25795, October 29, 1966, 18 SCRA 616);

h. Where it is a case of persecution rather than prosecution (Rustia vs. Ocampo, CAG.R. No. 4760, March 25, 1960); i. Where the charges are manifestly false and motivated by the lust for vengeance (Recto vs. Castelo, 18 L.J., [1953], cited in Raoa vs. Alvendia, CA-G.R. No. 30720R, October 8, 1962; Cf. Guingona, et al. vs. City Fiscal, L-60033, April 4, 1984, 128 SCRA 577); and j. When there is clearly no prima facie case against the accused and a motion to quash on that ground has been denied (Salonga vs. Pao, et al., L-59524, February 18, 1985, 134 SCRA 438). 7. Preliminary injunction has been issued by the Supreme Court to prevent the threatened unlawful arrest of petitioners (Rodriguez vs. Castelo, L-6374, August 1, 1953). (cited in Regalado, Remedial Law Compendium, p. 188, 1988 Ed.)

In these exceptional cases, this Court may ultimately resolve the existence or non-existence of probable cause by examining the records of the preliminary investigation, as it did in Salonga vs. Pao, Allado,and Webb. [75]

There can be no doubt that, in light of the several thousand private complainants in Criminal Case No. Q-93-43198 and several thousands more in different parts of the country who are similarly situated as the former for being holders of 349 Pepsi crowns, any affirmative holding of probable cause in the said case may cause or provoke, as justly feared by the petitioners, the filing of several thousand cases in various courts throughout the country. Inevitably, the petitioners would be exposed to the harassments of warrants of arrest issued by such courts and to huge expenditures for premiums on bailbonds and for travels from one court to another throughout the length and breadth of the archipelago for their arraignments and trials in such cases. Worse, the filing of these staggering number of cases would necessarily affect the trial calendar of our overburdened judges and take much of their attention, time, and energy, which they could devote to other equally, if not more, important cases. Such a frightful scenario would seriously affect the orderly administration of justice, or cause oppression or multiplicity of actions - a situation already long conceded by this Court to be an exception to the general rule that criminal prosecutions may not be restrained or stayed by injunction. [76]

We shall not, however, reevaluate the evidence to determine if indeed there is probable cause for the issuance of warrants of arrest in Criminal Case No. Q-93-43298. For, as earlier stated, the respondent Judge did not, in fact, find that probable cause exists, and if he did he did not have the basis therefor as mandated by Soliven, Inting, Lim, Allado, and even Webb. Moreover, the records of the preliminary investigation in Criminal Case No. Q-93-43198 are not with this Court. They were forwarded by the Office of the City Prosecutor of Quezon City to the DOJ in compliance with the latters 1st Indorsement

of 21 April 1993. The trial court and the DOJ must be required to perform their duty. WHEREFORE, the instant petition is granted and the following are hereby SET ASIDE: (a) Decision of 28 September 1993 and Resolution of 9 February 1994 of respondent Court of Appeals in CA-G.R. SP No. 31226; (b) The Resolution of the 349 Committee of the Department of Justice of 23 July 1993 dismissing the petitioners petition for review and of 3 February 1994 denying the motion to reconsider the dismissal; and (c) The Order of respondent Judge Maximiano C. Asuncion of 17 May 1993 in Criminal Case No. Q-93-43198.

The Department of Justice is DIRECTED to resolve on the merits, within sixty (60) days from notice of this decision, the petitioners petition for the review of the Joint Resolution of Investigating Prosecutor Ramon Gerona and thereafter to file the appropriate motion or pleading in Criminal Case No. Q93-43198, which respondent Judge Asuncion shall then resolve in light of Crespo vs. Mogul, Soliven vs. Makasiar, People vs. Inting, Lim vs. Felix, Allado vs. Diokno, and Webb vs. De Leon. In the meantime, respondent Judge Asuncion is DIRECTED to cease and desist from further proceeding with Criminal Case No. Q-93-43198 and to defer the issuance of warrants of arrest against the petitioners.

[G.R. No. 128222. June 17, 1999]

PEOPLE OF THE PHILIPPINES, plaintiff-appellee, vs. CHUA HO SAN @ TSAY HO SAN, accused-appellant. DECISION DAVIDE, JR., C.J.:

Chua Ho San @ Tsay Ho San (hereafter CHUA) prays for his acquittal and the reversal of the judgment of 10 February 1997 of the Regional Trial Court (RTC) of San Fernando, La Union, Branch 66, finding him guilty of transporting, without appropriate legal authority, the regulated substance methamphetamine hydrochloride, in violation of Section 15,[1] Article III of Republic Act No. 6425, otherwise known as the Dangerous Drugs Act of 1972 as further amended by R.A. No. 7659,[2] and sentencing him to "die by lethal injection." In view thereof, the judgment was brought to this Court for automatic review pursuant to Article 47 of the Revised Penal Code, as amended by Section 11 of R.A. No. 7659.

In response to reports of rampant smuggling of firearms and other contraband, Jim Lagasca Cid (hereafter CID), as Chief of Police of the Bacnotan Police Station, of La Union began patrolling the Bacnotan coastline with his officers. While monitoring the coastal area of Barangay Bulala on 29 March 1995, he intercepted a radio call at around 12:45 p.m. from Barangay Captain Juan Almoite (hereafter ALMOITE) of Barangay Tammocalao requesting police assistance regarding an unfamiliar speedboat the latter had spotted. According to ALMOITE, the vessel looked different from the boats ordinarily used by fisherfolk of the area and was poised to dock at Tammocalao shores.CID and six of his men led by his Chief Investigator, SPO1 Reynoso Badua (hereafter BADUA), proceeded forthwith to Tammocalao beach and there conferred with ALMOITE. CID then observed that the speedboat ferried a lone male passenger. As it was routine for CID to deploy his men in strategic places when dealing with similar situations, he ordered his men to take up positions thirty meters from the coastline. When the speedboat landed, the male passenger alighted, and using both hands, carried what appeared a multicolored strawbag. He then walked towards the road. By this time, ALMOITE, CID and BADUA, the latter two conspicuous in their uniform and issued side-arms, became suspicious of the man as he suddenly changed direction and broke into a run upon seeing the approaching officers. BADUA, however, prevented the man from fleeing by holding on to his right arm. Although CID introduced themselves as police officers, the man appeared impassive. Speaking in English, CID then requested the man to open his bag, but he seemed not to understand. CID thus tried speaking Tagalog, then Ilocano, but still to no avail. CID then resorted to what he termed sign language; he motioned with his hands for the man to open the bag. This time, the man apparently understood and acceded to the request. A search of the bag yielded several transparent plastic packets containing yellowish crystalline substances. CID then gestured to the man to close the bag, which he did. As CID wished to proceed to the police station, he signaled the man to follow, but the latter did not to comprehend. Hence, CID placed his arm around the shoulders of the man and escorted the latter to the police headquarters. At the police station, CID surmised, after having observed the facial features of the man, that he was probably Taiwanese. CID then "recited and informed the man of his constitutional rights" to remain silent, to have the assistance of a counsel, etc. Eliciting no response from the man, CID ordered his men to find a resident of the area who spoke Chinese to act as an interpreter. In the meantime, BADUA opened the bag and counted twenty-nine (29) plastic packets containing yellowish crystalline substances which he and CID suspected was shabu. The interpreter, Mr. Go Ping Guan, finally arrived, through whom the man was "apprised of his constitutional rights." The police authorities were satisfied that the man and the interpreter perfectly understood each other despite their uncertainty as to what language was spoken. But when the policemen asked the man several questions, he retreated to his obstinate reticence and merely showed his I.D. with the name Chua Ho San printed thereon. CHUA's bag and its contents were sent to the PNP Crime Laboratory at Camp Diego Silang, Carlatan, San Fernando, La Union for laboratory examination. In the meantime, CHUA was detained at the Bacnotan Police Station. Later that same day, Police Chief Inspector and Forensic Chemist Theresa Ann Bugayong Cid of the Philippine National Police, Region I, received a letter request [3] from CID incidentally her husband to conduct a laboratory examination of twenty-nine (29) plastic packets placed inside a multicolored strawbag. In her Chemistry Report No. D-025-95,[4] she stated that her

qualitative examination established the contents of the plastic packets, weighing 28.7 kilos, to be positive of methamphetamine hydrochloride or shabu, a regulated drug. CHUA was initially charged with illegal possession of methamphetamine hydrochloride before the RTC which docketed the case as Criminal Case No. 4037. However, pursuant to the recommendation of the Office of the Provincial Prosecutor of San Fernando, La Union, that the facts of the case could support an indictment for illegal transport of a regulated drug, the information was subsequently amended to allege that CHUA "willfully, unlawfully and feloniously transpor(ted) 28.7 kilos of [m]ethamphetamine [h]ydrochloride (shabu) without the necessary permit or authority to transport the same" in violation of Section 15, Article III of R.A. 6425 as amended by R.A. 7659. At his arraignment on 31 July 1995, CHUA entered a plea of not guilty. The RTC was satisfied that CHUA understood the amended information read to him in Fukien by the Fukienspeaking interpreter, Thelma Sales Go. Thereafter, the RTC exerted all efforts to obtain the services of a Taiwanese Interpreter through the auspices of the Department of Foreign Affairs. However, it was only after directing the request to the Taipei Economic and Cultural Office in the Philippines that interpreters were assigned to CHUA. Trial finally ensued. The State presented evidence tending to establish the above narration of facts which were culled chiefly from the testimony of CID, its first witness, and whose testimony, in turn, was substantially corroborated by witnesses BADUA and ALMOITE. Expert witness Theresa Ann Cid, confirmed the entries of her chemistry report in that the contents of the 29 plastic packets weighing 28.7 kilos sent to her for chemical analysis were pure, unadulterated methamphetamine hydrochloride or shabu. She also explained that they were unwashed, hence they appeared yellowish. For the defense, CHUA testified in his own behalf through interpreter Steven Yu. He disclosed that he hails from Taiwan and was employed in a shipbuilding and repairing company. On 21 March 1995, he was instructed by his employer Cho Chu Rong (hereafter RONG) to board the latters 35-tonner ship which would embark for Nan Au Port, Mainland China where they would buy fish. Upon arrival at their destination, RONG left the ship, came back without the fish, but with two bags, the contents of which he never divulged to CHUA. RONG then showed to CHUA a document purportedly granting them authority to fish on Philippine waters. So they sailed towards the Philippines and reached Dagupan, Pangasinan on 29 March 1995. At around 10:30 a.m., they disembarked on a small speedboat with the two bags RONG brought with him from China. While sailing, RONG made several phone calls using his mobile phone. CHUA heard RONG asked the person on the other side of the line if he could see the speedboat they were riding. Apparently, the person on shore could not see them so they cruised over the waters for about five hours more when finally, low on fuel and telephone battery, they decided to dock. CHUA anchored the boat while RONG carried the bags to shore. The tasks completed, RONG left to look for a telephone while CHUA rested and sat one and half (1 1/2) meters away from one bag. A child thereafter pointed out to him that one bag was missing much to RONGs dismay when he learned of it. When a crowd started to mill around them, the police arrived. CHUA then realized that RONG was nowhere to be found. The police immediately approached CHUA, and with nary any spoken word, only gestures and hand

movements, they escorted him to the precinct where he was handcuffed and tied to a chair. Later, the police, led by an officer who CHUA guessed as the Chief of Police arrived with the motor engine of the speedboat and a bag. They presented the bag to him, opened it, inspected and weighed the contents, then proclaimed them as methamphetamine hydrochloride. CHUA denounced the prosecutions story as a distortion of the truth. He denied he was ever favored with an interpreter or informed of his "constitutional rights," particularly of his right to counsel. Consequently, his arrest was tainted with illegality and the methamphetamine hydrochloride found in the bag should have been regarded inadmissible as evidence. He also maintained that CID never graced the occasion of his setting foot for the first time at Tammocalao beach. BADUA certainly never prevented him from running away, as such thought failed to make an impression in his mind. Most significantly, he denied ownership and knowledge of the contents of the bag, emphasizing that RONG alone exercised dominion over the same. Elmer Parong, (hereafter PARONG) a Sangguniang Bayan member, recalled that on the date in question, he arrived at the beach with the police. He saw CHUA standing with a bag beside him. He also remembered hearing from the people congregating at the beach that CHUA arrived with a companion and a certain policeman Anneb had chased the latters car. He additionally claimed that when the crowd became unruly, the police decided to bring CHUA to police headquarters. There, the mayor took charge of the situation -- he opened CHUA's bag with the assistance of the police, he called for a forensic chemist surnamed CID to take a sample of the contents of the bag, and he ordered his officials to find an interpreter. Throughout the proceedings, photographers were busy taking pictures to document the event. Last to testify was Arsenio CRAIG, a farmer and resident of Tammocalao who narrated that he was standing with CHUA on the beach when two men and a lady arrived. They were about to get a bag situated near CHUA when they detected the arrival of the local police. They quickly disappeared. CRAIG then noticed ALMOITE and PARONG at the beach but not CID. In a decision promulgated on 10 February 1997, the RTC found that the prosecution successfully discharged its burden of proving that CHUA transported 28.7 kilos of methamphetamine hydrochloride without legal authority to do so. Invoking People v. Tagliben[5] as authority, the RTC characterized the search as incidental to a valid in flagrante delicto arrest, hence it allowed the admission of the methamphetamine hydrochloride as corpus delicti. The RTC also noted the futility of informing CHUA of his constitutional rights to remain silent, and to have competent and independent counsel preferably of his own choice, considering the language barrier and the observation that such irregularity was rectified when accused was duly arraigned and (afterwards) participated in the trial of this case. The RTC then disregarded the inconsistencies and contradictions in the testimonies of the prosecution witnesses as these referred to minor details which did not impair the credibility of the witnesses or tarnish the credence conferred on the testimonies thus delivered. The RTC also believed that CHUA conspired not only with his alleged employer RONG and the Captain of the 35-tonner vessel in the illegal trade of prohibited drugs on Philippine shores, but with several other members of an organized syndicate bent on perpetrating said illicit traffic. Such predilection was plainly evident in the dispositive portion, to wit:

WHEREFORE, and in view of all the foregoing, as proven and established by convincing and satisfactory evidence that the accused had conspired and acted in concert with one Cho Chu Rong, not to mention Chen Ho Fa, the Skipper of the 35tonner ship they used in coming to the Country from China and Taiwan, this Court finds the accused Chua Ho San @ Tsay Ho San guilty beyond reasonable doubt of the offense of Violation of Sec. 15, Art. III of R.A. No. 6425, as amended by R.A. No. 7659 as charged in the Information, and considering the provisions of Sec. 20 of R.A. No. 7659 that the maximum penalty shall be imposed if the quantity sold/possessed/transported is 200 grams or more in the case of Shabu, and considering, further that the quantity involved in this case is 28.7 kilograms which is far beyond the weight ceiling specified in said Act, coupled with the findings of conspiracy or that accused is a member of an organized syndicated crime group, this Court, having no other recourse but to impose the maximum penalty to accused, this Court hereby sentences the said accused Chua Ho San @ Tsay Ho San to die by lethal injection; to pay a fine of Ten Million Pesos (P10,000,000.00); and to pay the costs. The Court hereby orders Director Ricareido [sic] Sarmiento of the Philippine National Police to immediately form an investigating Committee to be composed by [sic] men of unimpeachable integrity, who will conduct an exhaustive investigation regarding this case to determine whether there was negligence or conspiracy in the escape of Cho Chu Rong and the two (2) or three (3) persons who approached the accused in the seashore of Tammocalao, Bacnotan, La Union, and attempted to take the remaining bag from accused, as well as the whereabouts of the other bag; and to furnish this Court a copy of the report/result of the said investigation in order to show compliance herewith sixty (60) days from receipt hereof. The confiscated 28.7 kilograms of Methamphetamine Hydrochloride or Shabu is ordered turned over immediately to the Dangerous Drugs Board for destruction in accordance with the law. The fiberglass boat with its motor engine is hereby ordered confiscated in favor of the government and to be turned over to the Philippine National Police, La Union Command, for use in their Bantay-Dagat operations against all illegal seaborne activities. SO ORDERED.[6] Before this Court, CHUA posits that the RTC erred in (1) admitting as competent evidence the 29 plastic packets of methamphetamine hydrochloride since they were indubitably "forbidden fruits;" (2) granting weight and credence to the testimonies of prosecution witnesses despite glaring inconsistencies on material points; and in (3) appreciating conspiracy between him and

an organized syndicate in the illicit commerce of prohibited drugs since this was not alleged in the information. The Solicitor General traverses CHUA's contentions by asserting that: (1) the search was licitly conducted despite the absence of search and seizure warrants as circumstances immediately preceding to and contemporaneous with the search necessitated and validated the police action; and (2) that there was an effective and valid waiver of CHUA's right against unreasonable searches and seizures since he consented to the search. We reverse the RTC. Enshrined in the Constitution is the inviolable right to privacy of home and person. It explicitly ordains that people have the right to be secure in their persons, houses, papers and effects against unreasonable searches and seizures of whatever nature and for any purpose.[7] Inseparable, and not merely corollary or incidental to said right and equally hallowed in and by the Constitution, is the exclusionary principle which decrees that any evidence obtained in violation of said right is inadmissible for any purpose in any proceeding.[8] The Constitutional proscription against unreasonable searches and seizures does not, of course, forestall reasonable searches and seizure. What constitutes a reasonable or even an unreasonable search in any particular case is purely a judicial question, determinable from a consideration of the circumstances involved.[9] Verily, the rule is, the Constitution bars State intrusions to a person's body, personal effects or residence except if conducted by virtue of a valid search warrant issued in compliance with the procedure outlined in the Constitution and reiterated in the Rules of Court; otherwise such search and seizure become unreasonable within the meaning of the aforementioned constitutional provision.[10] This interdiction against warrantless searches and seizures, however, is not absolute and such warrantless searches and seizures have long been deemed permissible by jurisprudence[11] in instances of (1) search of moving vehicles, (2) seizure in plain view, (3) customs searches, (4) waiver or consent searches, (5) stop and frisk situations (Terry search),[12] and (6) search incidental to a lawful arrest. The last includes a valid warrantless search and seizure pursuant to an equally valid warrantless arrest, for, while as a rule, an arrest is considered legitimate if effected with a valid warrant of arrest, the Rules of Court recognize permissible warrantless arrests, to wit: (1) arrests in flagrante delicto, (2) arrests effected in hot pursuit, and (3) arrests of escaped prisoners.[13] This Court is therefore tasked to determine whether the warrantless arrest, search and seizure conducted under the facts of the case at bar constitute a valid exemption from the warrant requirement. Expectedly and quite understandably, the prosecution and the defense painted extremely divergent versions of the incident. But this Court is certain that CHUA was arrested and his bag searched without the benefit of a warrant. In cases of in flagrante delicto arrests, a peace officer or a private person may without a warrant, arrest a person, when, in his presence, the person to be arrested has committed, is actually committing, or is attempting to commit an offense. The arresting officer, therefore, must have personal knowledge of such fact[14] or as recent case law[15] adverts to, personal knowledge of facts or circumstances convincingly indicative or constitutive of probable cause.The term probable cause had been understood to mean a reasonable ground of suspicion supported by circumstances sufficiently strong in themselves to warrant a cautious mans belief that the person accused is guilty of the offense with which he is charged.[16] Specifically with respect to arrests, it

is such facts and circumstances which would lead a reasonably discreet and prudent man to believe that an offense has been committed by the person sought to be arrested.[17] In People v. Montilla,[18] the Court acknowledged that the evidentiary measure for the propriety of filing criminal charges, and correlatively, for effecting warrantless arrest, has been reduced and liberalized. Noting that the previous statutory and jurisprudential evidentiary standard was "prima facie evidence" and that it had been dubiously equated with probable cause, the Court explained:

[F]elicitously, those problems and confusing concepts (referring to prima facie evidence and probable cause) were clarified and set aright, at least on the issue under discussion, by the 1985 amendment of the Rules of Court which provides in Rule 112 thereof that the quantum of evidence required in preliminary investigation is such evidence as suffices to engender as well founded belief as to the fact of the commission of the crime and the respondents probable guilt thereof. It has the same meaning as the related phraseology used in other parts of the same Rule, that is, that the investigating fiscal finds cause to hold the respondent for trial, or where a probable cause exists. It should, therefore, be in that sense, wherein the right to effect a warrantless arrest should be considered as legally authorized. (emphasis supplied)[19] Guided by these principles, this Court finds that there are no facts on record reasonably suggestive or demonstrative of CHUAs participation in an ongoing criminal enterprise that could have spurred police officers from conducting the obtrusive search. The RTC never took the pains of pointing to such facts, but predicated mainly its decision on the finding that "accused was caught red-handed carrying the bagful of [s]habu when apprehended. In short, there is no probable cause. At least in People v. Tangliben, the Court agreed with the lower court's finding that compelling reasons (e.g., accused was acting suspiciously, on the spot identification by an informant that accused was transporting prohibitive drug, and the urgency of the situation) constitutive of probable cause impelled police officers from effecting an in flagrante delicto arrest. In the case at bar, the Solicitor General proposes that the following details are suggestive of probable cause -- persistent reports of rampant smuggling of firearm and other contraband articles, CHUA's watercraft differing in appearance from the usual fishing boats that commonly cruise over the Bacnotan seas, CHUAs illegal entry into the Philippines (he lacked the necessary travel documents or visa), CHUAs suspicious behavior, i.e. he attempted to flee when he saw the police authorities, and the apparent ease by which CHUA can return to and navigate his speedboat with immediate dispatch towards the high seas, beyond the reach of Philippine laws. This Court, however, finds that these do not constitute probable cause. None of the telltale clues, e.g., bag or package emanating the pungent odor of marijuana or other prohibited drug,[20] confidential report and/or positive identification by informers of courier(s) of prohibited drug and/or the time and place where they will transport/deliver the same,[21] suspicious demeanor or behavior[22] and suspicious bulge in the waist[23]-- accepted by this Court as sufficient to justify a warrantless arrest exists in this case. There was no classified information that a foreigner would disembark at Tammocalao beach bearing prohibited drug on the date in question. CHUA was not identified as a drug courier by a police informer or agent. The fact that

the vessel that ferried him to shore bore no resemblance to the fishing boats of the area did not automatically mark him as in the process of perpetrating an offense. And despite claims by CID and BADUA that CHUA attempted to flee, ALMOITE testified that the latter was merely walking and oblivious to any attempt at conversation when the officers approached him. This cast serious doubt on the truthfulness of the claim, thus: Q How far were you when the accused put the bag on his shoulder? A We were then very near him about three meters away from the male person carrying the bag. Q To what direction was he facing when he put the bag on his shoulder? A To the east direction. Q In relation to you, where were you. A With the company of Sgt. Reynoso and Maj. Cid we approached the accused and when Maj. Cid went near him, he spoke in Tagalog, English and Ilocano which accused did not understand because he did not respond. Q When Maj. Cid was talking, what was the accused doing at that time? A He was walking. Q To what direction he was walking? A He was walking to the east direction. (sic) Q He was walking away from you or going near you? A He was going away from us. That is why Sgt. Reynoso held the right arm of the accused. Q Was Sgt. Badua able to hold the right arm of the accused? A Yes sir and he stopped.[24]

True, CHUA entered Philippine territory without a visa. This was not obvious to the police. But gossamer to the officers sense perception and view were CHUA disembarking from a speedboat, CHUA walking casually towards the road, and CHUA carrying a multicolored strawbag. These acts did not convey any impression that he illegally entered Philippine shores. Neither were these overt manifestations of an ongoing felonious activity nor of CHUAs criminal behavior as clearly established in CIDs testimony, thus: Q Was the accused committing a crime when you introduced yourselves: A No, sir. Q No, so there was no reason for you to approach the accused because he was not doing anything wrong? A No, sir, that is our objective, to approach the person and if ever or whatever assistance that we can give we will give.[25]

The search cannot therefore be denominated as incidental to an arrest. While a contemporaneous search of a person arrested may be effected to deliver dangerous weapons or proofs or implements used in the commission of the crime and which search may extend to the area within his immediate control where he might gain possession of a weapon or evidence he can destroy,[26] a valid arrest must precede the search. The process cannot be reversed.

In a search incidental to a lawful arrest, as the precedent arrest determines the validity of the incidental search, the legality of the arrest is questioned in a large majority of these cases, e.g., whether an arrest was merely used as a pretext for conducting a search. In this instance, the law requires that there be first a lawful arrest before a search can be made - the process cannot be reversed.[27] To reiterate, the search was not incidental to an arrest. There was no warrant of arrest and the warrantless arrest did not fall under the exemptions allowed by the Rules of Court[28] as already shown. From all indications, the search was nothing but a fishing expedition. It is worth mentioning here that after introducing themselves, the police officers immediately inquired about the contents of the bag. What else could have impelled the officers from displaying such inordinate interest in the bag but to ferret out evidence and discover if a felony had indeed been committed by CHUA -- in effect to "retroactively establish probable cause and validate an illegal search and seizure." The State then attempted to persuade this Court that there was a consented search, a legitimate waiver of the constitutional guarantee against obtrusive searches. It is fundamental, however, that to constitute a waiver, it must first appear that the right exists; secondly, that the person involved had knowledge, actual or constructive, of the existence of such a right; and lastly, that said person had an actual intention to relinquish the right.[29] CHUA never exhibited that he knew, actually or constructively of his right against unreasonable searches or that he intentionally conceded the same. This can be inferred from the manner by which the search was performed, thus: Q Together with your Chief Investigator, what was the first thing that you did when you approached him (CHUA)? A We introduced ourselves as police officers, sir. Q Okey, in the first place why did you introduce yourselves? A That is normal practice in our part, sir.

*** Q If it is possible . Okey (sic) now, after introducing yourselves what did you do? A He did not answer me and he did not utter any word, Q When he did not utter any word. What else did he do? A I asked again a question that if he can open his bag sir. Q And did he understand your question when you requested him to open his bag? A No, sir, there is no answer. Q No answer? A Yes, sir, no answer. Q And when there was no answer what did you do next? A I used sign language sir.

Q Will you demonstrate to this Honorable Court how you demonstrated that sign language of opening the bag mr. (sic) witness? A I pointed to the zipper of the bag and then made an action like this sir.

*** SHERIFF: The witness demonstrating (sic) by pointing to the straw bag and then manifesting a sign to open the zipper of the straw bag moving his right hand from left to right or from the opening to the end of the zipper. COURT: From the start of the zipper where you open it up to the end of the zipper. Witness: Yes, sir, and then I made a motion like this. (The witness repeating the motion described on record.) COURT: Did you open that personally? WITNESS: A No, your honor. Q Now, mr. (sic) witness, why did you request the accused to open the bag? A Because it is our duty also to inspect his belongings sir. Q Why, why was it - no, I reform my question your honor. Is it normal procedure for you to examine anybody or to request anybody to open his bag? A The fact that he was a foreigner, sir, it is also our duty to inspect the baggage, it is our routine duty of a police (sic), sir. Q Is that the normal duty of a police officer to request a person to open his bag? A yes, sir. Q Okey, (sic) you did not ask the accused, mr. (sic) witness, to open his bag? A No, sir. Q But you simply requested him to open the nag? A Yes, sir.[30]

CHUA obviously failed to understand the events that overran and overwhelmed him. The police officers already introduced themselves to CHUA in three languages, but he remained completely deadpan. The police hence concluded that CHUA failed to comprehend the three languages. When CHUA failed to respond again to the polices request to open the bag, they resorted to what they called sign language. They claimed that CHUA finally understood their hand motions and gestures. This Court disagrees. If CHUA could not understand what was orally articulated to him, how could he understand the polices sign language. More importantly, it cannot logically be inferred from his alleged cognizance of the sign language that he deliberately, intelligently, and consciously waived his right against such an intrusive search. This Court is not unmindful of cases upholding the validity of consented warrantless searches and seizure. But in these cases, the police officers' request to search personnel effects was orally articulated to the accused and in such language that left no room for doubt that the latter fully

understood what was requested. In some instances, the accused even verbally replied to the request demonstrating that he also understood the nature and consequences of such request.[31] It was eventually discovered that the bag contained the regulated substance. But this is a trifling matter. If evidence obtained during an illegal search even if tending to confirm or actually confirming initial information or suspicion of felonious activity is absolutely considered inadmissible for any purpose in any proceeding, the same being the fruit of a poisonous tree[32] how much more of "forbidden fruits" which did not confirm any initial suspicion of criminal enterprise as in this case - because the police admitted that they never harbored any initial suspicion. Casting aside the regulated substance as evidence, the remaining evidence on record are insufficient, feeble and ineffectual to sustain CHUAs conviction. Indeed, the likelihood of CHUA having actually transported methamphetamine hydrochloride cannot be quickly dispelled. But the constitutional guarantee against unreasonable searches and seizures cannot be so carelessly disregarded as overzealous police officers are sometimes wont to do. Fealty to the Constitution and the rights it guarantees should be paramount in their minds, otherwise their good intentions will remain as such simply because they have blundered. "There are those who say that 'the criminal is to go free because the constable has blundered.' In some cases this will undoubtedly be the result. But 'there is another consideration -- the imperative of judicial integrity.'The criminal goes free, if he must, but it is the law that sets him free. Nothing can destroy a government more quickly than its failure to observe its own laws, or worse, its disregard of the charter of its own existence."[33] As to the averred glaring inconsistencies in the testimonies of the prosecution witnesses, this Court considers them trivial as they refer to insignificant details which will not affect the outcome of the case. On a passing note, this Court calls the attention of the trial court regarding its erroneous appreciation of conspiracy. This aggravating circumstance is without question unsupported by the records. Conspiracy was not included in the indictment nor raised in the pleadings or proceedings of the trial court. It is also fundamental that conspiracy must be proven just like any other criminal accusation, that is, independently and beyond reasonable doubt.[34] WHEREFORE, for all the foregoing, the decision of the Regional Trial Court, Branch 66, San Fernando, La Union in Criminal Case No. 4037 is hereby REVERSED and SET ASIDE and accused-appellant CHUA HO SAN @TSAY HO SAN is hereby ACQUITTED of the crime charged, the evidence not being sufficient to establish his guilt beyond reasonable doubt.

[G.R. No. 133917. February 19, 2001]

PEOPLE OF THE PHILIPPINES, plaintiff-appellee, vs. NASARIO MOLINA y MANAMAT @ BOBONG and GREGORIO MULA y MALAGURA @ BOBOY, accused-appellants. DECISION YNARES-SANTIAGO, J.:

To sanction disrespect and disregard for the Constitution in the name of protecting the society from lawbreakers is to make the government itself lawless and to subvert those values upon which our ultimate freedom and liberty depend.[1] For automatic review is the Decision[2] of the Regional Trial Court of Davao City, Branch 17, in Criminal Case No. 37,264-96, finding accused-appellants Nasario Molina y Manamat alias Bobong and Gregorio Mula y Malagura alias Boboy, guilty beyond reasonable doubt of violation of Section 8, [3] of the Dangerous Drugs Act of 1972 (Republic Act No. 6425), as amended by Republic Act No. 7659,[4] and sentencing them to suffer the supreme penalty of death. The information against accused-appellants reads: That on or about August 8, 1996, in the City of Davao, Philippines, and within the jurisdiction of this Honorable Court, the above-named accused, in conspiracy with each other, did then and there willfully, unlawfully and feloniously was found in their possession 946.9 grams of dried marijuana which are prohibited. CONTRARY TO LAW.[5] Upon arraignment on September 4, 1996, accused-appellants pleaded not guilty to the accusation against them.[6] Trial ensued, wherein the prosecution presented Police Superintendent Eriel Mallorca, SPO1 Leonardo Y. Pamplona, Jr., and SPO1 Marino S. Paguidopon, Jr. as witnesses. The antecedent facts are as follows: Sometime in June 1996, SPO1 Marino Paguidopon, then a member of the Philippine National Police detailed at Precinct No. 3, Matina, Davao City, received an information regarding the presence of an alleged marijuana pusher in Davao City.[7] The first time he came to see the said marijuana pusher in person was during the first week of July 1996. SPO1 Paguidopon was then with his informer when a motorcycle passed by. His informer pointed to the motorcycle driver, accusedappellant Mula, as the pusher. As to accused-appellant Molina, SPO1 Paguidopon had no occasion to see him before the arrest. Moreover, the names and addresses of the accused-appellants came to the knowledge of SPO1 Paguidopon only after they were arrested.[8] At about 7:30 in the morning of August 8, 1996, SPO1 Paguidopon received an information that the alleged pusher will be passing at NHA, Ma-a, Davao City any time that morning.[9] Consequently, at around 8:00 A.M. of the same day, he called for assistance at the PNP, Precinct No. 3, Matina, Davao City, which immediately dispatched the team of SPO4 Dionisio Cloribel (team leader), SPO2 Paguidopon (brother of SPO1 Marino Paguidopon), and SPO1 Pamplona, to proceed to the house

of SPO1 Marino Paguidopon where they would wait for the alleged pusher to pass by.[10] At around 9:30 in the morning of August 8, 1996, while the team were positioned in the house of SPO1 Paguidopon, a trisikad carrying the accused-appellants passed by. At that instance, SPO1 Paguidopon pointed to the accused-appellants as the pushers. Thereupon, the team boarded their vehicle and overtook the trisikad.[11] SPO1 Paguidopon was left in his house, thirty meters from where the accused-appellants were accosted.[12] The police officers then ordered the trisikad to stop. At that point, accusedappellant Mula who was holding a black bag handed the same to accused-appellant Molina. Subsequently, SPO1 Pamplona introduced himself as a police officer and asked accused-appellant Molina to open the bag.[13] Molina replied, Boss, if possible we will settle this.[14] SPO1 Pamplona insisted on opening the bag, which revealed dried marijuana leaves inside. Thereafter, accused-appellants Mula and Molina were handcuffed by the police officers.[15] On December 6, 1996, accused-appellants, through counsel, jointly filed a Demurrer to Evidence, contending that the marijuana allegedly seized from them is inadmissible as evidence for having been obtained in violation of their constitutional right against unreasonable searches and seizures.[16] The demurrer was denied by the trial court.[17] A motion for reconsideration was filed by accused-appellants, but this was likewise denied. Accused-appellants waived presentation of evidence and opted to file a joint memorandum. On April 25, 1997, the trial court rendered the assailed decision, [18] the decretal portion of which reads: WHEREFORE, finding the evidence of the prosecution alone without any evidence from both accused who waived presentation of their own evidence through their counsels, more than sufficient to prove the guilt of both accused of the offense charged beyond reasonable doubt, pursuant to Sec. 20, sub. par. 5 of Republic Act 7659, accused NASARIO MOLINA and GREGORIO MULA, are sentenced to suffer a SUPREME PENALTY OF DEATH through lethal injection under Republic Act 8176, to be effected and implemented as therein provided for by law, in relation to Sec. 24 of Rep. Act 7659. The Branch Clerk of Court of this court, is ordered to immediately elevate the entire records of this case with the Clerk of Court of the Supreme Court, Manila, for the automatic review of their case by the Supreme Court and its appropriate action as the case may be. SO ORDERED.[19]

Pursuant to Article 47 of the Revised Penal Code and Rule 122, Section 10 of the Rules of Court, the case was elevated to this Court on automatic review. Accusedappellants contend: I.

THAT THE MARIJUANA IS INADMISSIBLE IN EVIDENCE FOR HAVING BEEN SEIZED IN VIOLATION OF APPELLANTS CONSTITUTIONAL RIGHTS AGAINST UNREASONABLE SEARCHES AND SEIZURES; II.

THAT ASSUMING IT IS ADMISSIBLE IN EVIDENCE, THE GOVERNMENT HAS NOT OTHERWISE PROVED THEIR GUILT BEYOND REASONABLE DOUBT; AND III.

THAT, FINALLY, ASSUMING THEIR GUILT HAS BEEN PROVED BEYOND REASONABLE DOUBT, THE IMPOSABLE PENALTY FOR VIOLATION OF SEC. 8 OF RA No. 7659 (sic), IN THE ABSENCE OF ANY AGGRAVATING CIRCUMSTANCE, IS LIFE IMPRISONMENT, NOT DEATH.[20] The Solicitor General filed a Manifestation and Motion (In Lieu of Brief), wherein he prayed for the acquittal of both accused-appellants. The fundamental law of the land mandates that searches and seizures be carried out in a reasonable fashion, that is, by virtue or on the strength of a search warrant predicated upon the existence of a probable cause. The pertinent provision of the Constitution provides: SEC. 2. The right of the people to be secure in their persons, houses, papers, and effects against unreasonable searches and seizures of whatever nature and for any purpose shall be inviolable, and no search warrant or warrant of arrest shall issue except upon probable cause to be determined personally by the judge after examination under oath or affirmation of the complainant and the witnesses he may produce, and particularly describing the place to be searched and the persons or things to be seized.[21] Complementary to the foregoing provision is the exclusionary rule enshrined under Article III, Section 3, paragraph 2, which bolsters and solidifies the protection against unreasonable searches and seizures.[22] Thus:

Any evidence obtained in violation of this or the preceding section shall be inadmissible for any purpose in any proceeding. Without this rule, the right to privacy would be a form of words, valueless and undeserving of mention in a perpetual charter of inestimable human liberties; so too, without this rule, the freedom from state invasions of privacy would be so ephemeral and so neatly severed from its conceptual nexus with the freedom from all brutish means of coercing evidence as not to merit this Courts high regard as a freedom implicit in the concept of ordered liberty.[23] The foregoing constitutional proscription, however, is not without exceptions. Search and seizure may be made without a warrant and the evidence obtained therefrom may be admissible in the following instances: (1) search incident to a lawful arrest; (2) search of a moving motor vehicle; (3) search in violation of customs laws; (4) seizure of evidence in plain view; (5) when the accused himself waives his right against unreasonable searches and seizures;[24] and (6) stop and frisk situations (Terry search).[25] The first exception (search incidental to a lawful arrest) includes a valid warrantless search and seizure pursuant to an equally valid warrantless arrest which must precede the search. In this instance, the law requires that there be first a lawful arrest before a search can be made --- the process cannot be reversed.[26] As a rule, an arrest is considered legitimate if effected with a valid warrant of arrest. The Rules of Court, however, recognizes permissible warrantless arrests. Thus, a peace officer or a private person may, without warrant, arrest a person: (a) when, in his presence, the person to be arrested has committed, is actually committing, or is attempting to commit an offense (arrest in flagrante delicto); (b) when an offense has just been committed and he has probable cause to believe based on personal knowledge of facts or circumstances that the person to be arrested has committed it (arrest effected in hot pursuit); and (c) when the person to be arrested is a prisoner who has escaped from a penal establishment or a place where he is serving final judgment or is temporarily confined while his case is pending, or has escaped while being transferred from one confinement to another (arrest of escaped prisoners).[27] In the case at bar, the court a quo anchored its judgment of conviction on a finding that the warrantless arrest of accused-appellants, and the subsequent search conducted by the peace officers, are valid because accused-appellants were caught in flagrante delicto in possession of prohibited drugs.[28] This brings us to the issue of whether or not the warrantless arrest, search and seizure in the present case fall within the recognized exceptions to the warrant requirement. In People v. Chua Ho San,[29] the Court held that in cases of in flagrante delicto arrests, a peace officer or a private person may, without a warrant, arrest a

person when, in his presence, the person to be arrested has committed, is actually committing, or is attempting to commit an offense. The arresting officer, therefore, must have personal knowledge of such fact or, as recent case law adverts to, personal knowledge of facts or circumstances convincingly indicative or constitutive of probable cause. As discussed in People v. Doria,[30] probable cause means an actual belief or reasonable grounds of suspicion. The grounds of suspicion are reasonable when, in the absence of actual belief of the arresting officers, the suspicion that the person to be arrested is probably guilty of committing the offense, is based on actual facts, i.e., supported by circumstances sufficiently strong in themselves to create the probable cause of guilt of the person to be arrested. A reasonable suspicion therefore must be founded on probable cause, coupled with good faith on the part of the peace officers making the arrest. As applied to in flagrante delicto arrests, it is settled that reliable information alone, absent any overt act indicative of a felonious enterprise in the presence and within the view of the arresting officers, are not sufficient to constitute probable cause that would justify an in flagrante delicto arrest. Thus, in People v. Aminnudin,[31] it was held that the accused-appellant was not, at the moment of his arrest, committing a crime nor was it shown that he was about to do so or that he had just done so. What he was doing was descending the gangplank of the M/V Wilcon 9 and there was no outward indication that called for his arrest. To all appearances, he was like any of the other passengers innocently disembarking from the vessel. It was only when the informer pointed to him as the carrier of the marijuana that he suddenly became suspect and so subject to apprehension. Likewise, in People v. Mengote,[32] the Court did not consider eyes... darting from side to side ... [while] holding ... [ones] abdomen, in a crowded street at 11:30 in the morning, as overt acts and circumstances sufficient to arouse suspicion and indicative of probable cause. According to the Court, [b]y no stretch of the imagination could it have been inferred from these acts that an offense had just been committed, or was actually being committed, or was at least being attempted in [the arresting officers] presence. So also, in People v. Encinada,[33] the Court ruled that no probable cause is gleanable from the act of riding a motorela while holding two plastic baby chairs. Then, too, in Malacat v. Court of Appeals,[34] the trial court concluded that petitioner was attempting to commit a crime as he was standing at the corner of Plaza Miranda and Quezon Boulevard with his eyes moving very fast and looking at every person that come (sic) nearer (sic) to them.[35] In declaring the warrantless arrest therein illegal, the Court said: Here, there could have been no valid in flagrante delicto ... arrest preceding the search in light of the lack of personal knowledge on the part of Yu, the arresting officer, or

an overt physical act, on the part of petitioner, indicating that a crime had just been committed, was being committed or was going to be committed.[36] It went on to state that Second, there was nothing in petitioners behavior or conduct which could have reasonably elicited even mere suspicion other than that his eyes were moving very fast - an observation which leaves us incredulous since Yu and his teammates were nowhere near petitioner and it was already 6:30 p.m., thus presumably dusk. Petitioner and his companions were merely standing at the corner and were not creating any commotion or trouble... Third, there was at all no ground, probable or otherwise, to believe that petitioner was armed with a deadly weapon. None was visible to Yu, for as he admitted, the alleged grenade was discovered inside the front waistline of petitioner, and from all indications as to the distance between Yu and petitioner, any telltale bulge, assuming that petitioner was indeed hiding a grenade, could not have been visible to Yu.[37] Clearly, to constitute a valid in flagrante delicto arrest, two requisites must concur: (1) the person to be arrested must execute an overt act indicating that he has just committed, is actually committing, or is attempting to commit a crime; and (2) such overt act is done in the presence or within the view of the arresting officer. [38] In the case at bar, accused-appellants manifested no outward indication that would justify their arrest. In holding a bag on board a trisikad, accused-appellants could not be said to be committing, attempting to commit or have committed a crime. It matters not that accused-appellant Molina responded Boss, if possible we will settle this to the request of SPO1 Pamplona to open the bag. Such response which allegedly reinforced the suspicion of the arresting officers that accused-appellants were committing a crime, is an equivocal statement which standing alone will not constitute probable cause to effect an inflagrante delicto arrest. Note that were it not for SPO1 Marino Paguidopon (who did not participate in the arrest but merely pointed accusedappellants to the arresting officers), accused-appellants could not be the subject of any suspicion, reasonable or otherwise. While SPO1 Paguidopon claimed that he and his informer conducted a surveillance of accused-appellant Mula, SPO1 Paguidopon, however, admitted that he only learned Mulas name and address after the arrest. What is more, it is doubtful if SPO1 Paguidopon indeed recognized accused-appellant Mula. It is worthy to note that, before the arrest, he was able to see Mula in person only once, pinpointed to him by his informer while they were on the side of the road. These circumstances could not have afforded SPO1 Paguidopon a closer look at accused-appellant Mula, considering that the latter was then driving a motorcycle when SPO1 Paguidopon

caught a glimpse of him. With respect to accused-appellant Molina, SPO1 Paguidopon admitted that he had never seen him before the arrest. This belies the claim of SPO1 Pamplona that he knew the name of accusedappellants even before the arrest, to wit Q- When you said that certain Mula handed a black bag to another person and how did you know that it was Mula who handed the black bag to another person? A- Because I have already information from Paguidopon, regarding Mula and Molina, when they pass by through the street near the residence of Paguidopon. He told that the one who is big one that is Gregorio Mula and the thin one is Nazario Molina[39]

The aforecited testimony of SPO1 Pamplona, therefore, is entirely baseless. SPO1 Pamplona could not have learned the name of accused-appellants from SPO1 Paguipodon because Paguipodon himself, who allegedly conducted the surveillance, was not even aware of accused-appellants name and address prior to the arrest. Evidently, SPO1 Paguidopon, who acted as informer of the arresting officers, more so the arresting officers themselves, could not have been certain of accusedappellants identity, and were, from all indications, merely fishing for evidence at the time of the arrest. Compared to People v. Encinada, the arresting officer in the said case knew appellant Encinada even before the arrest because of the latters illegal gambling activities, thus, lending at least a semblance of validity on the arrest effected by the peace officers. Nevertheless, the Court declared in said case that the warrantless arrest and the consequent search were illegal, holding that [t]he prosecutions evidence did not show any suspicious behavior when the appellant disembarked from the ship or while he rode the motorela. No act or fact demonstrating a felonious enterprise could be ascribed to appellant under such bare circumstances.[40] Moreover, it could not be said that accused-appellants waived their right against unreasonable searches and seizure. Implied acquiescence to the search, if there was any, could not have been more than mere passive conformity given under intimidating or coercive circumstances and is thus considered no consent at all within the purview of the constitutional guarantee.[41] Withal, the Court holds that the arrest of accused-appellants does not fall under the exceptions allowed by the rules. Hence, the search conducted on their person was likewise illegal. Consequently, the marijuana seized by the peace officers could not be admitted as evidence against accused-appellants, and the Court is thus, left with no choice but to find in favor of accused-appellants. While the Court strongly supports the campaign of the government against drug addiction and commends the efforts of our law-enforcement officers towards this drive, all efforts for the achievement of a drug-free society must not encroach on the

fundamental rights and liberties of individuals as guaranteed in the Bill of Rights, which protection extends even to the basest of criminals. WHEREFORE, the Decision of the Regional Trial Court of Davao City, Branch 17, in Criminal Case No. 37, 264-96, is REVERSED and SET ASIDE. For lack of evidence to establish their guilt beyond reasonable doubt, accused-appellants Nasario Molina y Manamat alias Bobong and Gregorio Mula y Malagura alias Boboy, are ACQUITTED and ordered RELEASED from confinement unless they are validly detained for other offenses. No costs. SO ORDERED.

[G.R. Nos. 133254-55. April 19, 2001]

THE PEOPLE OF THE PHILIPPINES, plaintiff-appellee, vs. ROBERTO SALANGUIT y KO, accused-appellant. DECISION MENDOZA, J.:

This is an appeal from the decision,[1] dated January 27, 1998, of the Regional Trial Court, Branch 96, Quezon City, finding accused-appellant Roberto Salanguit y Ko guilty of violation of 16 of Republic Act No. 6425, as amended, and sentencing him accordingly to suffer imprisonment ranging from six (6) months of arresto mayor, as minimum, to four (4) years and two (2) months of prision correccional, as maximum, and of 8 of the same law and sentencing him for such violation to suffer the penalty of reclusion perpetua and to pay a fine of P700,000.00. Charges against accused-appellant for violations of R.A. No. 6425 were filed on December 28, 1995. In Criminal Case No. Q-95-64357, the information alleged: That on or about the 26th day of December 1995, in Quezon City, Philippines, the said accused, did then and there willfully, unlawfully and knowingly possess and/or use 11.14 grams of Methamphetamine Hydrochloride (Shabu) a regulated drug, without the necessary license and/or prescription therefor, in violation of said law. CONTRARY TO LAW.[2] In Criminal Case No. Q-95-64358, the information charged:

That on or about the 26th day of December 1995, in Quezon City, Philippines, the said accused not being authorized by law to possess or use any prohibited drug, did, then and there willfully, unlawfully and knowingly have in his possession and under his custody and control 1,254 grams of Marijuana, a prohibited drug. CONTRARY TO LAW.[3] When arraigned on May guilty,[4] whereupon he was tried.

21,

1996,

accused-appellant

pleaded

not

Three witnesses were presented by the prosecution: P/Insp. Sonia S. Ludovico, forensic chemist and chief of the Physical Science Branch of the Philippine National Police Crime Laboratory, Senior Inspector Rodolfo Aguilar of the Narcotics Command, Camp Crame, Quezon City, and PO3 Rolando Duazo of Station 10, Kamuning, Quezon City, a field operative. The prosecution evidence established the following: On December 26, 1995, Sr. Insp. Aguilar applied for a warrant [5] in the Regional Trial Court, Branch 90, Dasmarias, Cavite, to search the residence of accusedappellant Robert Salanguit y Ko on Binhagan St., Novaliches, Quezon City. He presented as his witness SPO1 Edmund Badua, who testified that as a poseur-buyer, he was able to purchase 2.12 grams of shabu from accused-appellant. The sale took place in accused-appellants room, and Badua saw that the shabu was taken by accused-appellant from a cabinet inside his room. The application was granted, and a search warrant was later issued by Presiding Judge Dolores L. Espaol. At about 10:30 p.m. of December 26, 1995, a group of about 10 policemen, along with one civilian informer, went to the residence of accused-appellant to serve the warrant.[6] The police operatives knocked on accused-appellants door, but nobody opened it. They heard people inside the house, apparently panicking. The police operatives then forced the door open and entered the house.[7] After showing the search warrant to the occupants of the house, Lt. Cortes and his group started searching the house.[8] They found 12 small heat-sealed transparent plastic bags containing a white crystalline substance, a paper clip box also containing a white crystalline substance, and two bricks of dried leaves which appeared to be marijuana wrapped in newsprint[9] having a total weight of approximately 1,255 grams.[10] A receipt of the items seized was prepared, but the accused-appellant refused to sign it.[11] After the search, the police operatives took accused-appellant with them to Station 10, EDSA, Kamuning, Quezon City, along with the items they had seized.[12]

PO3 Duazo requested a laboratory examination of the confiscated evidence. [13] The white crystalline substance with a total weight of 2.77 grams and those contained in a small box with a total weight of 8.37 grams were found to be positive for methamphetamine hydrochloride. On the other hand, the two bricks of dried leaves, one weighing 425 grams and the other 850 grams, were found to be marijuana.[14] For the defense, accused-appellant testified in his own behalf. His testimony was corroborated by his mother-in-law, Soledad Arcano. Accused-appellant testified that on the night of December 26, 1995, as they were about to leave their house, they heard a commotion at the gate and on the roof of their house. Suddenly, about 20 men in civilian attire, brandishing long firearms, climbed over the gate and descended through an opening in the roof.[15] When accused-appellant demanded to be shown a search warrant, a piece of paper inside a folder was waved in front of him. As accused-appellant fumbled for his glasses, however, the paper was withdrawn and he had no chance to read it.[16] Accused-appellant claimed that he was ordered to stay in one place of the house while the policemen conducted a search, forcibly opening cabinets and taking his bag containing money, a licensed .45 caliber firearm, jewelry, and canned goods.[17] The policemen left at around 12:30 a.m. of December 27, 1995, and, after putting handcuffs on accused-appellant, took him with them to the NARCOM on EDSA, Quezon City, where accused-appellant was detained.[18] Accused-appellants mother-in law, Soledad Arcano, corroborated his testimony. Arcano testified that the policemen ransacked their house, ate their food, and took away canned goods and other valuables.[19] After hearing, the trial court rendered its decision, the dispositive portion of which reads: WHEREFORE, judgment is hereby rendered: 1. In Criminal Case No. Q-95-64357, for violation of Sec. 16, Republic Act No. 6425, as amended, finding the accused ROBERTO SALANGUIT y KO guilty beyond reasonable doubt of the crime charged and he is hereby accordingly sentenced to suffer an indeterminate sentence with a minimum of six (6) months of arresto mayor and a maximum of four (4) years and two (2) months of prision correccional; and, 2. In Criminal Case No. Q-95-64358, for violation of Sec. 8, Republic Act No. 6425, as amended, finding the accused ROBERTO SALANGUIT y KO guilty beyond

reasonable doubt of the crime charged and he is hereby accordingly sentenced to suffer reclusion perpetua and to pay a fine of P700,000.00. The accused shall further pay the costs of suit. The 11.14 grams of methamphetamine hydrochloride and the 1,254 grams of marijuana bricks are hereby confiscated and condemned for disposition according to law. The evidence custodian of this Court is hereby directed to turn such substances over to the National Bureau of Investigation pursuant to law. SO ORDERED.[20] Hence this appeal. Accused-appellant contends that THE COURT A QUO GRAVELY ERRED IN DECLARING THE SEARCH WARRANT VALID THE COURT A QUO ERRED IN CONVICTING ACCUSED-APPELLANT FOR ILLEGAL POSSESSION OF METHAMPHETAMINE HYDROCHLORIDE (SHABU) THE COURT A QUO GRAVELY ERRED IN CONVICTING ACCUSEDAPPELLANT FOR VIOLATION 8, R.A. NO. 6425 THE COURT A QUO ERRED IN ADMITTING IN EVIDENCE THE TWO (2) BRICKS OF MARIJUANA THE COURT A QUO ERRED IN NOT FINDING THAT THE POLICEMEN USED EXCESSIVE FORCE IN ENFORCING THE SEARCH WARRANT. Accused-appellant is contesting his conviction on three grounds. First, the admissibility of the shabu allegedly recovered from his residence as evidence against him on the ground that the warrant used in obtaining it was invalid.Second, the admissibility in evidence of the marijuana allegedly seized from accused-appellant pursuant to the plain view doctrine. Third, the employment of unnecessary force by the police in the execution of the warrant. First. Rule 126, 4 of the Revised Rules on Criminal Procedure[21] provides that a search warrant shall not issue except upon probable cause in connection with one specific offense to be determined personally by the judge after examination under oath or affirmation of the complainant and the witnesses he may produce, and particularly describing the place to be searched and the things to be seized which may be anywhere in the Philippines.

In issuing a search warrant, judges must comply strictly with the requirements of the Constitution and the Rules of Criminal Procedure. No presumption of regularity can be invoked in aid of the process when an officer undertakes to justify its issuance.[22] Nothing can justify the issuance of the search warrant unless all the legal requisites are fulfilled. In this case, the search warrant issued against accused-appellant reads: SEARCH WARRANT NO. 160 For: Violation of RA 6425 SEARCH WARRANT TO ANY PEACE OFFICER: GREETINGS: It appearing to the satisfaction of the undersigned after examining under oath SR. INSP. RODOLFO V. AGUILAR, PNP and his witness SPO1 EDMUND M. BADUA, PNP that there is probable cause to believe that ROBERT SALANGUIT has in his possession and control in his premises Binhagan St., San Jose, Quezon City as shown in Annex A, the properties to wit: UNDETERMINED QUANTITY OF SHABU AND DRUG PARAPHERNALIA which should be seized and brought to the undersigned. You are hereby commanded to make an immediate search anytime of the day/night of the premises above-described and forthwith seize and take possession of the abovestated properties and bring said properties to the undersigned to be dealt with as the law directs. GIVEN UNDER MY HAND this 26th day of December 1995 at Imus, Cavite, Philippines. (SGD.) DOLORES L. ESPAOL Judge Accused-appellant assails the validity of the warrant on three grounds: (1) that there was no probable cause to search for drug paraphernalia; (2) that the search warrant was issued for more than one specific offense; and (3) that the place to be searched was not described with sufficient particularity.

Existence of Probable Cause

The warrant authorized the seizure of undetermined quantity of shabu and drug paraphernalia. Evidence was presented showing probable cause of the existence of methamphetamine hydrochloride or shabu. Accused-appellant contends, however, that the search warrant issued is void because no evidence was presented showing the existence of drug paraphernalia and the same should not have been ordered to be seized by the trial court.[23] The contention has no merit. To be sure, SPO1 Edmund Badua, the intelligence officer who acted as a poseur-buyer, did not testify in the proceedings for the issuance of a search warrant on anything about drug paraphernalia. He stated: Q - Being a member of the Intelligence and Operation Section, NMDU, NARCOM, do you remember if you were assigned into a monitoring or surveillance work? A - Yes, sir. Q - Of what particular assignment or area were you assigned for monitoring or surveillance? A - Its within the Quezon City area particularly a house without a number located at Binhagan St., San Jose, Quezon City, sir. Q - Do you know the person who occupies the specific place? A - Yes, sir, he is ROBERT SALANGUIT @ Robert. Q - Are you familiar with that place? A - Yes, sir, as part of my surveillance, I was able to penetrate inside the area and established contract with ROBERT SALANGUIT alias Robert through my friend who introduced me to the former. Q - In what particular occasion did you meet ROBERT SALANGUIT alias Robert? A - When I was introduced by my friend as a good buyer and drug pusher of shabu, sir. Q - Were you able to buy at that time? A - Yes, sir. Q - How much if you can still remember the amount involved? A - I was able to buy two point twelve (2.12) grams of shabu in the amount of Two Thousand Seven Hundred Fifty (P2,750.00) pesos, sir. Q - Having established contact with ROBERT SALANGUIT @ Robert, do you know where the stuff (shabu) were being kept? A - Yes, sir, inside a cabinet inside his room. Q - How were you able to know the place where he kept the stuff? A - When I first bought the 2.12 grams of shabu from him, it was done inside his room and I saw that the shabu was taken by him inside his cabinet. Q - Do you know who is in control of the premises?

A - Yes, sir, it was ROBERT SALANGUIT @ Robert. Q - How sure are you, that the shabu that you bought from ROBERT SALANGUIT @ Robert is genuine shabu? A - After I left the house of ROBERT SALANGUIT @ Robert, I proceeded back to our office and reported the progress of my mission to our Chief and presented to him the 2.12 grams of shabu I bought from the subject. Then afterwards, our Chief formally requested the Chief PNP Central Crime Laboratory Services, NPDC, for Technical Analysis which yielded positive result for shabu, a regulated drug as shown in the attached certification of PNP CLS result No. D-414-95 dated 19 Dec. 95. Q - Do you have anything more to add or retract from your statement? A - Yes, sir, I was offered by him (ROBERT SALANGUIT @ Robert) that anything I wish to buy bigger quantity of shabu, he is willing to transact to me on cash basis at his price of One Thousand Seven Hundred Fifty (P1,750.00) pesos per gram. Q - Are you willing to sign your statement freely and voluntarily? A - Yes, sir.[24]

However, the fact that there was no probable cause to support the application for the seizure of drug paraphernalia does not warrant the conclusion that the search warrant is void. This fact would be material only if drug paraphernalia was in fact seized by the police. The fact is that none was taken by virtue of the search warrant issued. If at all, therefore, the search warrant is void only insofar as it authorized the seizure of drug paraphernalia, but it is valid as to the seizure of methamphetamine hydrochloride as to which evidence was presented showing probable cause as to its existence. Thus, in Aday v. Superior Court,[25] the warrant properly described two obscene books but improperly described other articles. It was held: Although the warrant was defective in the respects noted, it does not follow that it was invalid as a whole. Such a conclusion would mean that the seizure of certain articles, even though proper if viewed separately, must be condemned merely because the warrant was defective with respect to other articles. The invalid portions of the warrant are severable from the authorization relating to the named books, which formed the principal basis of the charge of obscenity. The search for and seizure of these books, if otherwise valid, were not rendered illegal by the defects concerning other articles. . . . In so holding we do not mean to suggest that invalid portions of a warrant will be treated as severable under all circumstances. We recognize the danger that warrants might be obtained which are essentially general in character but as to minor items meet the requirement of particularity, and that wholesale seizures might be made under them, in the expectation that the seizure would in any event be upheld as to the property specified. Such an abuse of the warrant procedure, of course, could not be tolerated. It would be a drastic remedy indeed if a warrant, which was issued on probable cause and particularly describing the items to be seized on the basis thereof, is to be

invalidated in toto because the judge erred in authorizing a search for other items not supported by the evidence.[26] Accordingly, we hold that the first part of the search warrant, authorizing the search of accused-appellants house for an undetermined quantity of shabu, is valid, even though the second part, with respect to the search for drug paraphernalia, is not. Specificity of the Offense Charged

Accused-appellant contends that the warrant was issued for more than one specific offense because possession or use of methamphetamine hydrochloride and possession of drug paraphernalia are punished under two different provisions of R.A. No. 6425.[27] It will suffice to quote what this Court said in a similar case to dispose of this contention: While it is true that the caption of the search warrant states that it is in connection with Violation of R.A. 6425, otherwise known as the Dangerous Drugs Act of 1972, it is clearly recited in the text thereof that There is probable cause to believe that Adolfo Olaes alias Debie and alias Baby of No. 628 Comia St., Filtration, Sta. Rita, Olongapo City, has in their session and control and custody of marijuana dried stalks/leaves/seeds/cigarettes and other regulated/prohibited and exempt narcotics preparations which is the subject of the offense stated above. Although the specific section of the Dangerous Drugs Act is not pinpointed, there is no question at all of the specific offense alleged to have been committed as a basis for the finding of probable cause. The search warrant also satisfies the requirement in the Bill of Rights of the particularity of the description to be made of the place to be searched and the persons or things to be seized. [28] Indeed, in People v. Dichoso[29] the search warrant was also for Violation of R.A. 6425, without specifying what provisions of the law were violated, and it authorized the search and seizure of dried marijuana leaves and methamphetamine hydrochloride (shabu) and sets of paraphernalias (sic). This Court, however, upheld the validity of the warrant: Appellants contention that the search warrant in question was issued for more than (1) offense, hence, in violation of Section 3, Rule 126 of the Rules of Court, is unpersuasive. He engages in semantic juggling by suggesting that since illegal possession of shabu, illegal possession of marijuana and illegal possession of paraphernalia are covered by different articles and sections of the Dangerous Drugs Act of 1972, the search warrant is clearly for more than one (1) specific offense. In short, following this theory, there should have been three (3) separate search warrants,

one for illegal possession of shabu, the second for illegal possession of marijuana and the third for illegal possession of paraphernalia. This argument is pedantic. The Dangerous Drugs Act of 1972 is a special law that deals specifically with dangerous drugs which are subsumed into prohibited and regulated drugs and defines and penalizes categories of offenses which are closely related or which belong to the same class or species. Accordingly, one (1) search warrant may thus be validly issued for the said violations of the Dangerous Drugs Act.[30] Similarly, in another case,[31] the search warrant was captioned: For Violation of P.D. No. 1866 (Illegal Possession of Firearms, etc.). The validity of the warrant was questioned on the ground that it was issued without reference to any particular provision in P.D. No. 1866, which punished several offenses. We held, however, that while illegal possession of firearms is penalized under 1 of P.D. No. 1866 and illegal possession of explosives is penalized under 3 thereof, the decree is a codification of the various laws on illegal possession of firearms, ammunitions, and explosives which offenses are so related as to be subsumed within the category of illegal possession of firearms, etc. under P.D. No. 1866. Thus, only one warrant was necessary to cover the violations under the various provisions of the said law. Particularity of the Place

Accused-appellant contends that the search warrant failed to indicate the place to be searched with sufficient particularity. This contention is without merit. As the Solicitor General states: . . . While the address stated in the warrant is merely Binhagan St., San Jose, Quezon City, the trial court took note of the fact that the records of Search Warrant Case No. 160 contained several documents which identified the premises to be searched, to wit: 1) the application for search warrant which stated that the premises to be searched was located in between No. 7 and 11 at Binhagan Street, San Jose, Quezon City; 2) the deposition of witness which described the premises as a house without a number located at Binhagan St., San Jose, Quezon City; and 3) the pencil sketch of the location of the premises to be searched. In fact, the police officers who raided appellants house under the leadership of Police Senior Inspector Rodolfo Aguilar could not have been mistaken as Inspector Aguilar resides in the same neighborhood in Binhagan where appellant lives and in fact Aguilars place is at the end of appellants place in Binhagan. Moreover, the house raided by Aguilars team is undeniably appellants house and it was really appellant who was the target. The raiding team even first ascertained through their informant that appellant was inside his residence before they actually started their operation.[32]

The rule is that a description of the place to be searched is sufficient if the officer with the warrant can, with reasonable effort, ascertain and identify the place intended to be searched.[33] For example, a search warrant authorized a search of Apartment Number 3 of a building at 83 Pleasant Street, Malborough, Massachusetts. As it turned out, there were five apartments in the basement and six apartments on both the ground and top floors and that there was an Apartment Number 3 on each floor. However, the description was made determinate by a reference to the affidavit supporting the warrant that the apartment was occupied by the accused Morris Ferrante of 83 Pleasant Street, Malboro Mass.[34] In this case, the location of accusedappellants house being indicated by the evidence on record, there can be no doubt that the warrant described the place to be searched with sufficient particularity. In sum, we hold that with respect to the seizure of shabu from accused-appellants residence, Search Warrant No. 160 was properly issued, such warrant being founded on probable cause personally determined by the judge under oath or affirmation of the deposing witness and particularly describing the place to be searched and the things to be seized. Second. The search warrant authorized the seizure of methamphetamine hydrochloride or shabu but not marijuana. However, seizure of the latter drug is being justified on the ground that the drug was seized within the plain view of the searching party. This is contested by accused-appellant. Under the plain view doctrine, unlawful objects within the plain view of an officer who has the right to be in the position to have that view are subject to seizure and may be presented in evidence.[35] For this doctrine to apply, there must be: (a) prior justification; (b) inadvertent discovery of the evidence; and (c) immediate apparent illegality of the evidence before the police.[36] The question is whether these requisites were complied with by the authorities in seizing the marijuana in this case. Prior Justification and Discovery by Inadvertence

Because the location of the shabu was indicated in the warrant and thus known to the police operatives, it is reasonable to assume that the police found the packets of the shabu first. Once the valid portion of the search warrant has been executed, the plain view doctrine can no longer provide any basis for admitting the other items subsequently found. As has been explained: What the plain view cases have in common is that the police officer in each of them had a prior justification for an intrusion in the course of which he came inadvertently across a piece of evidence incriminating the accused. The doctrine serves to supplement the prior justification whether it be a warrant for another object, hot

pursuit, search incident to lawful arrest, or some other legitimate reason for being present unconnected with a search directed against the accused and permits the warrantless seizure. Of course, the extension of the original justification is legitimate only where it is immediately apparent to the police that they have evidence before them; the plain view doctrine may not be used to extend a general exploratory search from one object to another until something incriminating at last emerges.[37] The only other possible justification for an intrusion by the police is the conduct of a search pursuant to accused-appellants lawful arrest for possession of shabu. However, a search incident to a lawful arrest is limited to the person of the one arrested and the premises within his immediate control.[38] The rationale for permitting such a search is to prevent the person arrested from obtaining a weapon to commit violence, or to reach for incriminatory evidence and destroy it. The police failed to allege in this case the time when the marijuana was found, i.e., whether prior to, or contemporaneous with, the shabu subject of the warrant, or whether it was recovered on accused-appellants person or in an area within his immediate control. Its recovery, therefore, presumably during the search conducted after the shabu had been recovered from the cabinet, as attested to by SPO1 Badua in his depostion, was invalid. Apparent Illegality of the Evidence

The marijuana bricks were wrapped in newsprint. There was no apparent illegality to justify their seizure. This case is similar to People. v. Musa[39] in which we declared inadmissible the marijuana recovered by NARCOM agents because the said drugs were contained in a plastic bag which gave no indication of its contents. We explained: Moreover, when the NARCOM agents saw the plastic bag hanging in one corner of the kitchen, they had no clue as to its contents. They had to ask the appellant what the bag contained. When the appellant refused to respond, they opened it and found the marijuana. Unlike Ker v. California, where the marijuana was visible to the police officers eyes, the NARCOM agents in this case could not have discovered the inculpatory nature of the contents of the bag had they not forcibly opened it. Even assuming then, that the NARCOM agents inadvertently came across the plastic bag because it was within their plain view, what may be said to be the object in their plain view was just the plastic bag and not the marijuana. The incriminating nature of the contents of the plastic bag was not immediately apparent from the plain view of said object. It cannot be claimed that the plastic bag clearly betrayed its contents, whether

by its distinctive configuration, is transparency, or otherwise, that its contents are obvious to an observer.[40] No presumption of regularity may be invoked by an officer in aid of the process when he undertakes to justify an encroachment of rights secured by the Constitution.[41] In this case, the marijuana allegedly found in the possession of accused-appellant was in the form of two bricks wrapped in newsprint. Not being in a transparent container, the contents wrapped in newsprint could not have been readily discernible as marijuana. Nor was there mention of the time or manner these items were discovered. Accordingly, for failure of the prosecution to prove that the seizure of the marijuana without a warrant was conducted in accordance with the plain view doctrine, we hold that the marijuana is inadmissible in evidence against accusedappellant. However, the confiscation of the drug must be upheld. Third. Accused-appellant claims that undue and unnecessary force was employed by the searching party in effecting the raid. Rule 126, 7 of the Revised Rules on Criminal Procedure[42] provides: Right to break door or window to effect search. The officer, if refused admittance to the place of directed search after giving notice of his purpose and authority, may break open any outer or inner door or window of a house or any part of a house or anything therein to execute the warrant or liberate himself or any person lawfully aiding him when unlawfully detained therein. Accused-appellants claim that the policemen had clambered up the roof of his house to gain entry and had broken doors and windows in the process is unsupported by reliable and competent proof. No affidavit or sworn statement of disinterested persons, like the barangay officials or neighbors, has been presented by accusedappellant to attest to the truth of his claim. In contrast, Aguilar and Duanos claim that they had to use some force in order to gain entry cannot be doubted. The occupants of the house, especially accusedappellant, refused to open the door despite the fact that the searching party knocked on the door several times. Furthermore, the agents saw the suspicious movements of the people inside the house. These circumstances justified the searching partys forcible entry into the house, founded as it is on the apprehension that the execution of their mission would be frustrated unless they do so. WHEREFORE, in Criminal Case No. Q-95-64357, the decision of the Regional Trial Court, Branch 96, Quezon City, finding accused-appellant Roberto Salanguit y Ko guilty of possession of illegal drugs under 16 of R.A. No. 6425, otherwise known as the Dangerous Drugs Act, as amended, and sentencing him to suffer a prison term ranging from six (6) months of arresto mayor, as minimum, and four (4) years and

two (2) months of prision correccional, as maximum, and ordering the confiscation of 11.14 grams of methamphetamine hydrochloride is AFFIRMED. In Criminal Case No. Q-95-64358, the decision of the same court finding accusedappellant Roberto Salanguit y Ko guilty of possession of prohibited drugs under 8 of R.A. No. 6425, as amended, and sentencing him to suffer the penalty of reclusion perpetua and to pay a fine of P700,000.00 is hereby REVERSED and SET ASIDE and accused-appellant is ACQUITTED of the crime charged. However, the confiscation of the 1,254 grams of marijuana, as well as the 11.14 grams of methamphetamine hydrochloride, and its disposition as ordered by the trial court is AFFIRMED.

G.R. No. 140946. September 13, 2004]

MICROSOFT CORPORATION and LOTUS DEVELOPMENT CORPORATION, petitioners, vs. MAXICORP, INC., respondent. DECISION CARPIO, J.:

The Case This petition for review on certiorari seeks to reverse the Court of Appeals Decision dated 23 December 1998 and its Resolution dated 29 November 1999 in CA-G.R. SP No. 44777. The Court of Appeals reversed the Order of the Regional Trial Court, Branch 23, Manila (RTC), denying respondent Maxicorp, Inc.s (Maxicorp) motion to quash the search warrant that the RTC issued against Maxicorp. Petitioners are the private complainants against Maxicorp for copyright infringement under Section 29 of Presidential Decree No. 49 (Section 29 of PD 49) and for unfair competition under Article 189 of the Revised Penal Code (RPC). [1]

[2]

[3]

[4]

[5]

Antecedent Facts On 25 July 1996, National Bureau of Investigation (NBI) Agent Dominador Samiano, Jr. (NBI Agent Samiano) filed several applications for search

warrants in the RTC against Maxicorp for alleged violation of Section 29 of PD 49 and Article 189 of the RPC. After conducting a preliminary examination of the applicant and his witnesses, Judge William M. Bayhon issued Search Warrants Nos. 96-451, 96-452, 96-453 and 96-454, all dated 25 July 1996, against Maxicorp. Armed with the search warrants, NBI agents conducted on 25 July 1996 a search of Maxicorps premises and seized property fitting the description stated in the search warrants. On 2 September 1996, Maxicorp filed a motion to quash the search warrants alleging that there was no probable cause for their issuance and that the warrants are in the form of general warrants. The RTC denied Maxicorps motion on 22 January 1997. The RTC also denied Maxicorps motion for reconsideration. The RTC found probable cause to issue the search warrants after examining NBI Agent Samiano, John Benedict Sacriz (Sacriz), and computer technician Felixberto Pante (Pante). The three testified on what they discovered during their respective visits to Maxicorp. NBI Agent Samiano also presented certifications from petitioners that they have not authorized Maxicorp to perform the witnessed activities using petitioners products. On 24 July 1997, Maxicorp filed a petition for certiorari with the Court of Appeals seeking to set aside the RTCs order. On 23 December 1998, the Court of Appeals reversed the RTCs order denying Maxicorps motion to quash the search warrants. Petitioners moved for reconsideration. The Court of Appeals denied petitioners motion on 29 November 1999. The Court of Appeals held that NBI Agent Samiano failed to present during the preliminary examination conclusive evidence that Maxicorp produced or sold the counterfeit products. The Court of Appeals pointed out that the sales receipt NBI Agent Samiano presented as evidence that he bought the products from Maxicorp was in the name of a certain Joel Diaz. Hence, this petition. The Issues Petitioners seek a reversal and raise the following issues for resolution: 1. WHETHER THE PETITION RAISES QUESTIONS OF LAW;

2. WHETHER PETITIONERS HAVE LEGAL PERSONALITY TO FILE THE PETITION; 3. WHETHER THERE WAS PROBABLE CAUSE TO ISSUE THE SEARCH WARRANTS; 4. WHETHER THE SEARCH WARRANTS ARE GENERAL WARRANTS. The Ruling of the Court The petition has merit. On Whether the Petition Raises Questions of Law Maxicorp assails this petition as defective since it failed to raise questions of law. Maxicorp insists that the arguments petitioners presented are questions of fact, which this Court should not consider in a Rule 45 petition for review. Petitioners counter that all the issues they presented in this petition involve questions of law. Petitioners point out that the facts are not in dispute. A petition for review under Rule 45 of the Rules of Court should cover questions of law. Questions of fact are not reviewable. As a rule, the findings of fact of the Court of Appeals are final and conclusive and this Court will not review them on appeal, subject to exceptions as when the findings of the appellate court conflict with the findings of the trial court. [6]

[7]

[8]

The distinction between questions of law and questions of fact is settled. A question of law exists when the doubt or difference centers on what the law is on a certain state of facts. A question of fact exists if the doubt centers on the truth or falsity of the alleged facts. Though this delineation seems simple, determining the true nature and extent of the distinction is sometimes problematic. For example, it is incorrect to presume that all cases where the facts are not in dispute automatically involve purely questions of law. There is a question of law if the issue raised is capable of being resolved without need of reviewing the probative value of the evidence. The resolution of the issue must rest solely on what the law provides on the given set of circumstances. Once it is clear that the issue invites a review of the evidence presented, the question posed is one of fact. If the query requires a reevaluation of the credibility of witnesses, or the existence or relevance of surrounding circumstances and their relation to each other, the issue in that query is factual. Our ruling in Paterno v. Paterno is illustrative on this point: [9]

[10]

[11]

[12]

Such questions as whether certain items of evidence should be accorded probative value or weight, or rejected as feeble or spurious, or whether or not the proofs on one side or the other are clear and convincing and adequate to establish a proposition in issue, are without doubt questions of fact. Whether or not the body of proofs presented by a party, weighed and analyzed in relation to contrary evidence submitted by adverse party, may be said to be strong, clear and convincing; whether or not certain documents presented by one side should be accorded full faith and credit in the face of protests as to their spurious character by the other side; whether or not inconsistencies in the body of proofs of a party are of such gravity as to justify refusing to give said proofs weight all these are issues of fact. It is true that Maxicorp did not contest the facts alleged by petitioners. But this situation does not automatically transform all issues raised in the petition into questions of law. The issues must meet the tests outlined in Paterno. Of the three main issues raised in this petition the legal personality of the petitioners, the nature of the warrants issued and the presence of probable cause only the first two qualify as questions of law. The pivotal issue of whether there was probable cause to issue the search warrants is a question of fact. At first glance, this issue appears to involve a question of law since it does not concern itself with the truth or falsity of certain facts. Still, the resolution of this issue would require this Court to inquire into the probative value of the evidence presented before the RTC. For a question to be one of law, it must not involve an examination of the probative value of the evidence presented by the litigants or any of them. [13]

Yet, this is precisely what the petitioners ask us to do by raising arguments requiring an examination of the TSNs and the documentary evidence presented during the search warrant proceedings. In short, petitioners would have us substitute our own judgment to that of the RTC and the Court of Appeals by conducting our own evaluation of the evidence. This is exactly the situation which Section 1, Rule 45 of the Rules of Court prohibits by requiring the petition to raise only questions of law. This Court is not a trier of facts. It is not the function of this court to analyze or weigh evidence. When we give due course to such situations, it is solely by way of exception. Such exceptions apply only in the presence of extremely meritorious circumstances. [14]

[15]

Indeed, this case falls under one of the exceptions because the findings of the Court of Appeals conflict with the findings of the RTC. Since petitioners properly raised the conflicting findings of the lower courts, it is proper for this Court to resolve such contradiction. [16]

On Whether Petitioners have the Legal Personality to File this Petition Maxicorp argues that petitioners have no legal personality to file this petition since the proper party to do so in a criminal case is the Office of the Solicitor General as representative of the People of the Philippines. Maxicorp states the general rule but the exception governs this case. We ruled in Columbia Pictures Entertainment, Inc. v. Court of Appeals that the petitioner-complainant in a petition for review under Rule 45 could argue its case before this Court in lieu of the Solicitor General if there is grave error committed by the lower court or lack of due process. This avoids a situation where a complainant who actively participated in the prosecution of a case would suddenly find itself powerless to pursue a remedy due to circumstances beyond its control. The circumstances in Columbia Pictures Entertainmentare sufficiently similar to the present case to warrant the application of this doctrine. [17]

[18]

On Whether there was Probable Cause to Issue the Search Warrants Petitioners argue that the Court of Appeals erred in reversing the RTC based on the fact that the sales receipt was not in the name of NBI Agent Samiano. Petitioners point out that the Court of Appeals disregarded the overwhelming evidence that the RTC considered in determining the existence of probable cause. Maxicorp counters that the Court of Appeals did not err in reversing the RTC. Maxicorp maintains that the entire preliminary examination that the RTC conducted was defective. The Court of Appeals based its reversal on two factual findings of the RTC. First, the fact that the sales receipt presented by NBI Agent Samiano as proof that he bought counterfeit goods from Maxicorp was in the name of a certain Joel Diaz. Second, the fact that petitioners other witness, John Benedict Sacriz, admitted that he did not buy counterfeit goods from Maxicorp. We rule that the Court of Appeals erred in reversing the RTCs findings. Probable cause means such reasons, supported by facts and circumstances as will warrant a cautious man in the belief that his action and the means taken in prosecuting it are legally just and proper. Thus, probable cause for a search warrant requires such facts and circumstances that would lead a reasonably prudent man to believe that an offense has been committed and the objects sought in connection with that offense are in the place to be searched. [19]

[20]

The judge determining probable cause must do so only after personally examining under oath the complainant and his witnesses. The oath required

must refer to the truth of the facts within the personal knowledge of the petitioner or his witnesses, because the purpose thereof is to convince the committing magistrate, not the individual making the affidavit and seeking the issuance of the warrant, of the existence of probable cause. The applicant must have personal knowledge of the circumstances. Reliable information is insufficient. Mere affidavits are not enough, and the judge must depose in writing the complainant and his witnesses. [21]

[22]

[23]

The Court of Appeals reversal of the findings of the RTC centers on the fact that the two witnesses for petitioners during the preliminary examination failed to prove conclusively that they bought counterfeit software from Maxicorp. The Court of Appeals ruled that this amounted to a failure to prove the existence of a connection between the offense charged and the place searched. The offense charged against Maxicorp is copyright infringement under Section 29 of PD 49 and unfair competition under Article 189 of the RPC. To support these charges, petitioners presented the testimonies of NBI Agent Samiano, computer technician Pante, and Sacriz, a civilian. The offenses that petitioners charged Maxicorp contemplate several overt acts. The sale of counterfeit products is but one of these acts. Both NBI Agent Samiano and Sacriz related to the RTC how they personally saw Maxicorp commit acts of infringement and unfair competition. During the preliminary examination, the RTC subjected the testimonies of the witnesses to the requisite examination. NBI Agent Samiano testified that he saw Maxicorp display and offer for sale counterfeit software in its premises. He also saw how the counterfeit software were produced and packaged within Maxicorps premises. NBI Agent Samiano categorically stated that he was certain the products were counterfeit because Maxicorp sold them to its customers without giving the accompanying ownership manuals, license agreements and certificates of authenticity. Sacriz testified that during his visits to Maxicorp, he witnessed several instances when Maxicorp installed petitioners software into computers it had assembled. Sacriz also testified that he saw the sale of petitioners software within Maxicorps premises. Petitioners never authorized Maxicorp to install or sell their software. The testimonies of these two witnesses, coupled with the object and documentary evidence they presented, are sufficient to establish the existence of probable cause. From what they have witnessed, there is reason to believe that Maxicorp engaged in copyright infringement and unfair competition to the prejudice of petitioners. Both NBI Agent Samiano and

Sacriz were clear and insistent that the counterfeit software were not only displayed and sold within Maxicorps premises, they were also produced, packaged and in some cases, installed there. The determination of probable cause does not call for the application of rules and standards of proof that a judgment of conviction requires after trial on the merits. As implied by the words themselves, probable cause is concerned with probability, not absolute or even moral certainty. The prosecution need not present at this stage proof beyond reasonable doubt. The standards of judgment are those of a reasonably prudent man, not the exacting calibrations of a judge after a full-blown trial. [24]

No law or rule states that probable cause requires a specific kind of evidence. No formula or fixed rule for its determination exists. Probable cause is determined in the light of conditions obtaining in a given situation. Thus, it was improper for the Court of Appeals to reverse the RTCs findings simply because the sales receipt evidencing NBI Agent Samianos purchase of counterfeit goods is not in his name. [25]

[26]

For purposes of determining probable cause, the sales receipt is not the only proof that the sale of petitioners software occurred. During the search warrant application proceedings, NBI Agent Samiano presented to the judge the computer unit that he purchased from Maxicorp, in which computer unit Maxicorp had pre-installed petitioners software. Sacriz, who was present when NBI Agent Samiano purchased the computer unit, affirmed that NBI Agent Samiano purchased the computer unit. Pante, the computer technician, demonstrated to the judge the presence of petitioners software on the same computer unit. There was a comparison between petitioners genuine software and Maxicorps software pre-installed in the computer unit that NBI Agent Sambiano purchased. Even if we disregard the sales receipt issued in the name of Joel Diaz, which petitioners explained was the alias NBI Agent Samiano used in the operation, there still remains more than sufficient evidence to establish probable cause for the issuance of the search warrants. [27]

[28]

[29]

[30]

This also applies to the Court of Appeals ruling on Sacrizs testimony. The fact that Sacriz did not actually purchase counterfeit software from Maxicorp does not eliminate the existence of probable cause. Copyright infringement and unfair competition are not limited to the act of selling counterfeit goods. They cover a whole range of acts, from copying, assembling, packaging to marketing, including the mere offering for sale of the counterfeit goods. The clear and firm testimonies of petitioners witnesses on such other acts stand untarnished. The Constitution and the Rules of Court only require that the judge examine personally and thoroughly the applicant for the warrant and his

witnesses to determine probable cause. The RTC complied adequately with the requirement of the Constitution and the Rules of Court. Probable cause is dependent largely on the opinion and findings of the judge who conducted the examination and who had the opportunity to question the applicant and his witnesses. For this reason, the findings of the judge deserve great weight. The reviewing court should overturn such findings only upon proof that the judge disregarded the facts before him or ignored the clear dictates of reason. Nothing in the records of the preliminary examination proceedings reveal any impropriety on the part of the judge in this case. As one can readily see, here the judge examined thoroughly the applicant and his witnesses. To demand a higher degree of proof is unnecessary and untimely. The prosecution would be placed in a compromising situation if it were required to present all its evidence at such preliminary stage. Proof beyond reasonable doubt is best left for trial. [31]

[32]

On Whether the Search Warrants are in the Nature of General Warrants A search warrant must state particularly the place to be searched and the objects to be seized. The evident purpose for this requirement is to limit the articles to be seized only to those particularly described in the search warrant. This is a protection against potential abuse. It is necessary to leave the officers of the law with no discretion regarding what articles they shall seize, to the end that no unreasonable searches and seizures be committed. [33]

In addition, under Section 4, Rule 126 of the Rules of Criminal Procedure, a search warrant shall issue in connection with one specific offense. The articles described must bear a direct relation to the offense for which the warrant is issued. Thus, this rule requires that the warrant must state that the articles subject of the search and seizure are used or intended for use in the commission of a specific offense. [34]

Maxicorp argues that the warrants issued against it are too broad in scope and lack the specificity required with respect to the objects to be seized. After examining the wording of the warrants issued, the Court of Appeals ruled in favor of Maxicorp and reversed the RTCs Order thus: Under the foregoing language, almost any item in the petitioners store can be seized on the ground that it is used or intended to be used in the illegal or unauthorized copying or reproduction of the private respondents software and their manuals. [35]

The Court of Appeals based its reversal on its perceived infirmity of paragraph (e) of the search warrants the RTC issued. The appellate court found that similarly worded warrants, all of which noticeably employ the phrase used or

intended to be used, were previously held void by this Court. The disputed text of the search warrants in this case states: [36]

a) Complete or partially complete reproductions or copies of Microsoft software bearing the Microsoft copyrights and/or trademarks owned by MICROSOFT CORPORATION contained in CD-ROMs, diskettes and hard disks; b) Complete or partially complete reproductions or copies of Microsoft instruction manuals and/or literature bearing the Microsoft copyrights and/or trademarks owned by MICROSOFT CORPORATION; c) Sundry items such as labels, boxes, prints, packages, wrappers, receptacles, advertisements and other paraphernalia bearing the copyrights and/or trademarks owned by MICROSOFT CORPORATION; d) Sales invoices, delivery receipts, official receipts, ledgers, journals, purchase orders and all other books of accounts and documents used in the recording of the reproduction and/or assembly, distribution and sales, and other transactions in connection with fake or counterfeit products bearing the Microsoft copyrights and/or trademarks owned by MICROSOFT CORPORATION; e) Computer hardware, including central processing units including hard disks, CD-ROM drives, keyboards, monitor screens and diskettes, photocopying machines and other equipment or paraphernalia used or intended to be used in the illegal and unauthorized copying or reproduction of Microsoft software and their manuals, or which contain, display or otherwise exhibit, without the authority of MICROSOFT CORPORATION, any and all Microsoft trademarks and copyrights; and f) Documents relating to any passwords or protocols in order to access all computer hard drives, data bases and other information storage devices containing unauthorized Microsoft software. (Emphasis supplied) [37]

It is only required that a search warrant be specific as far as the circumstances will ordinarily allow. The description of the property to be seized need not be technically accurate or precise. The nature of the description should vary according to whether the identity of the property or its character is a matter of concern. Measured against this standard we find that paragraph (e) is not a general warrant. The articles to be seized were not only sufficiently identified physically, they were also specifically identified by stating [38]

[39]

their relation to the offense charged. Paragraph (e) specifically refers to those articles used or intended for use in the illegal and unauthorized copying of petitioners software. This language meets the test of specificity. [40]

The cases cited by the Court of Appeals are inapplicable. In those cases, the Court found the warrants too broad because of particular circumstances, not because of the mere use of the phrase used or intended to be used. In Columbia Pictures, Inc. v. Flores, the warrants ordering the seizure of television sets, video cassette recorders, rewinders and tape cleaners x x x were found too broad since the defendant there was a licensed distributor of video tapes. The mere presence of counterfeit video tapes in the defendants store does not mean that the machines were used to produce the counterfeit tapes. The situation in this case is different. Maxicorp is not a licensed distributor of petitioners. In Bache & Co. (Phil.), Inc., et al. v. Judge Ruiz, et al., the Court voided the warrants because they authorized the seizure of records pertaining to all business transactions of the defendant. And in 20th Century Fox Film Corp. v. Court of Appeals, the Court quashed the warrant because it merely gave a list of articles to be seized, aggravated by the fact that such appliances are generally connected with the legitimate business of renting out betamax tapes. [41]

[42]

[43]

However, we find paragraph (c) of the search warrants lacking in particularity. Paragraph (c) states: c) Sundry items such as labels, boxes, prints, packages, wrappers, receptacles, advertisements and other paraphernalia bearing the copyrights and/or trademarks owned by MICROSOFT CORPORATION; The scope of this description is all-embracing since it covers property used for personal or other purposes not related to copyright infringement or unfair competition. Moreover, the description covers property that Maxicorp may have bought legitimately from Microsoft or its licensed distributors. Paragraph (c) simply calls for the seizure of all items bearing the Microsoft logo, whether legitimately possessed or not. Neither does it limit the seizure to products used in copyright infringement or unfair competition. Still, no provision of law exists which requires that a warrant, partially defective in specifying some items sought to be seized yet particular with respect to the other items, should be nullified as a whole. A partially defective warrant remains valid as to the items specifically described in the warrant. A search warrant is severable, the items not sufficiently described may be cut off without destroying the whole warrant. The exclusionary rule found in Section 3(2) of Article III of the Constitution renders inadmissible in any proceeding all [44]

[45]

evidence obtained through unreasonable searches and seizure. Thus, all items seized under paragraph (c) of the search warrants, not falling under paragraphs a, b, d, e or f, should be returned to Maxicorp. WHEREFORE, we PARTIALLY GRANT the instant petition. The Decision of the Court of Appeals dated 23 December 1998 and its Resolution dated 29 November 1999 in CA-G.R. SP No. 44777 are REVERSED and SET ASIDE except with respect to articles seized under paragraph (c) of Search Warrants Nos. 96-451, 96-452, 96-453 and 96-454. All articles seized under paragraph (c) of the search warrants, not falling under paragraphs a, b, d, e or f, are ordered returned to Maxicorp, Inc. immediately. G.R. No. 82585 November 14, 1988 MAXIMO V. SOLIVEN, ANTONIO V. ROCES, FREDERICK K. AGCAOLI, and GODOFREDO L. MANZANAS,petitioners, vs. THE HON. RAMON P. MAKASIAR, Presiding Judge of the Regional Trial Court of Manila, Branch 35, UNDERSECRETARY SILVESTRE BELLO III, of the Department of Justice, LUIS C. VICTOR, THE CITY FISCAL OF MANILA and PRESIDENT CORAZON C. AQUINO, respondents. G.R. No. 82827 November 14, 1988 LUIS D. BELTRAN, petitioner, vs. THE HON. RAMON P. MAKASIAR, Presiding Judge of Branch 35 of the Regional Trial Court, at Manila, THE HON. LUIS VICTOR, CITY FISCAL OF MANILA, PEOPLE OF THE PHILIPPINES, SUPERINTENDENT OF THE WESTERN POLICE DISTRICT, and THE MEMBERS OF THE PROCESS SERVING UNIT AT THE REGIONAL TRIAL COURT OF MANILA, respondents. G.R. No. 83979 November 14, 1988. LUIS D. BELTRAN, petitioner, vs. EXECUTIVE SECRETARY CATALINO MACARAIG, SECRETARY OF JUSTICE SEDFREY ORDOÑEZ, UNDERSECRETARY OF JUSTICE SILVESTRE BELLO III, THE CITY FISCAL OF MANILA JESUS F. GUERRERO, and JUDGE RAMON P. MAKASIAR, Presiding Judge of Branch 35 of the Regional Trial Court, at Manila, respondents. Angara, Abello, Concepcion, Regala and Cruz for petitioners in G.R. No. 82585. Perfecto V. Fernandez, Jose P. Fernandez and Cristobal P. Fernandez for petitioner in G.R. Nos. 82827 and 83979. RESOLUTION

PER CURIAM:

In these consolidated cases, three principal issues were raised: (1) whether or not petitioners were denied due process when informations for libel were filed against them although the finding of the existence of a prima faciecase was still under review by the Secretary of Justice and, subsequently, by the President; (2) whether or not the constitutional rights of Beltran were violated when respondent RTC judge issued a warrant for his arrest without personally examining the complainant and the witnesses, if any, to determine probable cause; and (3) whether or not the President of the Philippines, under the Constitution, may initiate criminal proceedings against the petitioners through the filing of a complaint-affidavit. Subsequent events have rendered the first issue moot and academic. On March 30, 1988, the Secretary of Justice denied petitioners' motion for reconsideration and upheld the resolution of the Undersecretary of Justice sustaining the City Fiscal's finding of a prima facie case against petitioners. A second motion for reconsideration filed by petitioner Beltran was denied by the Secretary of Justice on April 7, 1988. On appeal, the President, through the Executive Secretary, affirmed the resolution of the Secretary of Justice on May 2, 1988. The motion for reconsideration was denied by the Executive Secretary on May 16, 1988. With these developments, petitioners' contention that they have been denied the administrative remedies available under the law has lost factual support. It may also be added that with respect to petitioner Beltran, the allegation of denial of due process of law in the preliminary investigation is negated by the fact that instead of submitting his counteraffidavits, he filed a "Motion to Declare Proceedings Closed," in effect waiving his right to refute the complaint by filing counter-affidavits. Due process of law does not require that the respondent in a criminal case actually file his counter-affidavits before the preliminary investigation is deemed completed. All that is required is that the respondent be given the opportunity to submit counteraffidavits if he is so minded. The second issue, raised by petitioner Beltran, calls for an interpretation of the constitutional provision on the issuance of warrants of arrest. The pertinent provision reads: Art. III, Sec. 2. The right of the people to be secure in their persons, houses, papers and effects against unreasonable searches and seizures of whatever nature and for any purpose shall be inviolable, and no search warrant or warrant of arrest shall issue except upon probable cause to be determined personally by the judge after examination nder oath or affirmation of the complainant and the witnesses he may produce, and particularly describing the place to be searched and the persons or things to be seized. The addition of the word "personally" after the word "determined" and the deletion of the grant of authority by the 1973 Constitution to issue warrants to "other responsible officers as may be authorized by law," has apparently convinced petitioner Beltran that the Constitution now requires the judge to personally examine the complainant and his witnesses in his determination of probable cause for the issuance of warrants of arrest. This is not an accurate interpretation. What the Constitution underscores is the exclusive and personal responsibility of the issuing judge to satisfy himself of the existence of probable cause. In satisfying himself of the existence of probable cause for the issuance of a warrant of arrest, the judge is not required to personally examine the complainant and his witnesses. Following established doctrine and procedure, he shall: (1) personally evaluate the report and the supporting documents submitted by the fiscal regarding the existence of probable cause and, on the basis thereof, issue a warrant of arrest; or (2) if on the basis thereof he finds no probable cause, he may disregard the fiscal's report and require the submission

of supporting affidavits of witnesses to aid him in arriving at a conclusion as to the existence of probable cause. Sound policy dictates this procedure, otherwise judges would be unduly laden with the preliminary examination and investigation of criminal complaints instead of concentrating on hearing and deciding cases filed before their courts. On June 30, 1987, the Supreme Court unanimously adopted Circular No. 12, setting down guidelines for the issuance of warrants of arrest. The procedure therein provided is reiterated and clarified in this resolution. It has not been shown that respondent judge has deviated from the prescribed procedure. Thus, with regard to the issuance of the warrants of arrest, a finding of grave abuse of discretion amounting to lack or excess of jurisdiction cannot be sustained. Anent the third issue, petitioner Beltran argues that "the reasons which necessitate presidential immunity from suit impose a correlative disability to file suit." He contends that if criminal proceedings ensue by virtue of the President's filing of her complaint-affidavit, she may subsequently have to be a witness for the prosecution, bringing her under the trial court's jurisdiction. This, continues Beltran, would in an indirect way defeat her privilege of immunity from suit, as by testifying on the witness stand, she would be exposing herself to possible contempt of court or perjury. The rationale for the grant to the President of the privilege of immunity from suit is to assure the exercise of Presidential duties and functions free from any hindrance or distraction, considering that being the Chief Executive of the Government is a job that, aside from requiring all of the office holder's time, also demands undivided attention. But this privilege of immunity from suit, pertains to the President by virtue of the office and may be invoked only by the holder of the office; not by any other person in the President's behalf. Thus, an accused in a criminal case in which the President is complainant cannot raise the presidential privilege as a defense to prevent the case from proceeding against such accused. Moreover, there is nothing in our laws that would prevent the President from waiving the privilege. Thus, if so minded the President may shed the protection afforded by the privilege and submit to the court's jurisdiction. The choice of whether to exercise the privilege or to waive it is solely the President's prerogative. It is a decision that cannot be assumed and imposed by any other person. As regards the contention of petitioner Beltran that he could not be held liable for libel because of the privileged character or the publication, the Court reiterates that it is not a trier of facts and that such a defense is best left to the trial court to appreciate after receiving the evidence of the parties. As to petitioner Beltran's claim that to allow the libel case to proceed would produce a "chilling effect" on press freedom, the Court finds no basis at this stage to rule on the point. The petitions fail to establish that public respondents, through their separate acts, gravely abused their discretion as to amount to lack of jurisdiction. Hence, the writs of certiorari and prohibition prayed for cannot issue. WHEREFORE, finding no grave abuse of discretion amounting to excess or lack of jurisdiction on the part of the public respondents, the Court Resolved to DISMISS the petitions in G. R. Nos. 82585,

82827 and 83979. The Order to maintain the status quo contained in the Resolution of the Court en banc dated April 7, 1988 and reiterated in the Resolution dated April 26, 1988 is LIFTED. G.R. Nos. 94054-57

February 19, 1991

VICENTE LIM, SR. and MAYOR SUSANA LIM, petitioners, vs. HON. NEMESIO S. FELIX and HON. ANTONIO ALFANE, respondents. G.R. Nos. 94266-69

February 19, 1991

JOLLY T. FERNANDEZ, FLORENCIO T. FERNANDEZ, JR., NONILON A. BAGALIHOG, MAYOR NESTOR C. LIM and MAYOR ANTONIO KHO, petitioners, vs. HON. NEMESIO S. FELIX and PROSECUTOR ANTONIO C. ALFANE, respondents. Francisco R. Llamas for petitioners in G.R. Nos. 94054-57. Jolly T. Fernandez, Elenito Bagalihog, Orlando M. Danao and Hechanova, Ballicid & Associates for petitioners in G.R. Nos. 94266-69.

GUTIERREZ, JR., J.: May a Judge without ascertaining the facts through his own personal determination and relying solely on the certification or recommendation of a prosecutor that a probable cause exists issue a warrant of arrest? On March 17, 1989, at about 7:30 o'clock in the morning, at the vicinity of the airport road of the Masbate Domestic Airport, located at the municipality of Masbate province of Masbate, Congressman Moises Espinosa, Sr. and his security escorts, namely Provincial Guards Antonio Cortes, Gaspar Amaro, and Artemio Fuentes were attacked and killed by a lone assassin. Dante Siblante another security escort of Congressman Espinosa, Sr. survived the assassination plot, although, he himself suffered a gunshot wound. An investigation of the incident then followed. Thereafter, and for the purpose of preliminary investigation, the designated investigator, Harry O. Tantiado, TSg, of the PC Criminal Investigation Service at Camp Bagong Ibalon Legazpi City filed an amended complaint with the Municipal Trial Court of Masbate accusing, among others, Vicente Lim, Sr., Mayor Susana Lim of Masbate (petitioners in G.R. Nos. 9405457), Jolly T. Fernandez, Florencio T. Fernandez, Jr., Nonilon A. Bagalihog, Mayor Nestor C. Lim and Mayor Antonio Kho (petitioners in G.R. Nos. 94266-69) of the crime of multiple murder and frustrated murder in connection with the airport incident. The case was docketed as Criminal Case No. 9211. After conducting the preliminary investigation, the court issued an order dated July 31, 1989 stating therein that:

. . . after weighing the affidavits and answers given by the witnesses for the prosecution during the preliminary examination in searching questions and answers, concludes that a probable cause has been established for the issuance of a warrant of arrest of named accused in the amended complaint, namely, Jimmy Cabarles, Ronnie Fernandez, Nonilon Bagalihog, Jolly Fernandez, Florencio Fernandez, Jr., Vicente Lim, Sr., Susana Lim, Nestor Lim, Antonio Kho, Jaime Liwanag, Zaldy Dumalag and Rene Tualla aliasTidoy. (Rollo, p. 58, G.R. Nos. 94054-57) xxx

xxx

xxx

In the same Order, the court ordered the arrest of the petitioners and recommended the amount of P200,000.00 as bail for the provisional liberty of each of the accused. Petitioners Jolly Fernandez and Nonilon Bagalihog filed a motion for the reduction of bail which was granted by the court and they were allowed to post bail in the amount of P150,000.00 each. Except for Jimmy Cabarles, all the rest of the accused posted bail at P200,000.00 each. On August 29, 1989, the entire records of the case consisting of two hundred sixty one (261) pages were transmitted to the Provincial Prosecutor of Masbate. Respondent Acting Fiscal Antonio C. Alfane was designated to review the case. On September 22, 1989, Fiscal Alfane issued a Resolution which affirmed the finding of a prima facie case against the petitioners but differed in the designation of the crime in that the ruled that ". . . all of the accused should not only be charged with Multiple Murder With Frustrated Murder" but for a case of MURDER for each of the killing of the four victims and a physical injuries case for inflicting gunshot wound on the buttocks of Dante Siblante." (Annex "H", Comment of Fiscal Alfane, p. 186, Rollo, G.R. Nos. 94054-57) A motion to reconsider the Resolution filed by petitioners Vicente Lim, Sr. and Mayor Susana Lim was denied. On October 30, 1989, Fiscal Alfane filed with the Regional Trial Court of Masbate, four (4) separate informations of murder against the twelve (12) accused with a recommendation of no bail. On November 21, 1989, petitioners Vicente Lim, Sr. and Susana Lim filed with us a verified petition for change of venue. (Case No. A.M. No. 89-11-1270-MTC, formerly, G.R. Nos. 90587-90) On December 14, 1989, we issued an en banc Resolution authorizing the change of venue from the Regional Trial Court of Masbate to the Regional Trial Court of Makati to avoid a miscarriage of justice, to wit: Acting on the petition for change of venue of the trial of Criminal Cases Nos. 5811, 5812, 5813, and 5814 from the Regional Trial Court, Masbate, Masbate to any of the Regional Trial Courts at Quezon City or Makati, the Court Resolved to (a) GRANT the aforesaid petition for transfer of venue in order to avoid miscarriage of justice (Article VIII, Section 5(4) of the Philippine Constitution); (b) DIRECT the Clerk of Court, Regional Trial Court, Masbate, Masbate to transmit the records of the aforesaid cases to the Executive Judge, Regional Trial Court, Makati, for raffling among the other branches of the court; and (c) ORDER the Regional Trial Court of Masbate, Masbate to desist from further taking cognizance of the said cases until such time that the petition is finally resolved. The cases were raffled to Branch 56 presided by respondent Judge Nemesio S. Felix.

Petitioners Vicente Lim, Sr. and Susana Lim filed with the respondent court several motions and manifestations which in substance prayed for the following: 1. An order be issued requiring the transmittal of the initial records of the preliminary inquiry or investigation conducted by the Municipal Judge Barsaga of Masbate for the best enlightenment of this Honorable Court in its personal determination of the existence of a probable cause or prima facie evidence as well as its determination of the existence of guilt, pursuant to the mandatory mandate of the constitution that no warrant shall issue unless the issuing magistrate shall have himself been personally convinced of such probable cause. 2. Movants be given ample opportunity to file their motion for preliminary investigation as a matter of right; and 3. In the event that this court may later be convinced of the existence of a probable cause, to be allowed to file a motion for reduction of bail or for admission of bail. (p. 17, Rollo, G.R. Nos. 94054-57) In another manifestation, the Lims reiterated that the court conduct a hearing to determine if there really exists aprima facie case against them in the light of documents which are recantations of some witnesses in the preliminary investigation. The motions and manifestations were opposed by the prosecution. On July 5, 1990, the respondent court issued an order denying for lack of merit the motions and manifestations and issued warrants of arrest against the accused including the petitioners herein. The respondent Judge said: In the instant cases, the preliminary investigation was conducted by the Municipal Trial Court of Masbate, Masbate which found the existence of probable cause that the offense of multiple murder was committed and that all the accused are probably guilty thereof, which was affirmed upon review by the Provincial Prosecutor who properly filed with the Regional Trial Court four separate informations for murder. Considering that both the two competent officers to whom such duty was entrusted by law have declared the existence of probable cause, each information is complete in form and substance, and there is no visible defect on its face, this Court finds it just and proper to rely on the prosecutor's certification in each information which reads: (pp. 19-20, Rollo, G.R Nos. 94054-57; Emphasis supplied) xxx

xxx

xxx

The petitioners then filed these consolidated petitions questioning the July 5, 1990 Order. In a Resolution dated July 17, 1990 in G.R. Nos. 94054-57, we issued ". . . a TEMPORARY RESTRAINING ORDER, effective immediately and continuing until further orders from this Court, ordering the respondent judge or his duly authorized representatives or agents to CEASE and DESIST from enforcing or implementing the warrant of arrest without bail issued against the petitioners in his Order dated July 5, 1990 in Criminal Cases Nos. 5811-14. In another Resolution dated July 31, 1990 in G.R. Nos. 94266-69, we resolved: xxx

xxx

xxx

. . . To ISSUE writs of (1) PRELIMINARY MANDATORY INJUNCTION, ordering and directing the respondent judge to recall/set aside and/or annul the legal effects of the warrants of arrest without bail issued against and served upon herein petitioners Jolly T. Fernandez, Florencio T. Fernandez, Jr. and Nonilon Bagalihog and release them from confinement at PC-CIS Detention Center, Camp Crame, Quezon City; and (2) TEMPORARY RESTRAINING ORDER, effective immediately and continuing until further orders from this Court, ordering the respondent judge or his duly authorized representatives or agents, to CEASE AND DESIST from enforcing or implementing the warrants of arrest without bail issued against petitioners Mayors Nestor C. Lim and Antonio T. Kho. The primary issue in these consolidated petitions centers on whether or not a judge may issue a warrant of arrest without bail by simply relying on the prosecution's certification and recommendation that a probable cause exists. This is not a novel question. In the case of Placer v. Villanueva (126 SCRA 463 [1983]), we ruled that a judge may rely upon the fiscal's certification of the existence of probable cause and, on the basis thereof, issue a warrant of arrest. However, the certification does not bind the judge to come out with the warrant of arrest. This decision interpreted the "search and seizure" provision of the 1973 Constitution which provides: . . . no search warrant or warrant of arrest shall issue except upon probable cause to be determined by the judge, or such other responsible officer as may be authorized by law, after examination under oath or affirmation of the complainant and the witnesses he may produce ... We ruled: . . . The issuance of a warrant is not a mere ministerial function; it calls for the exercise of judicial discretion on the part of the issuing magistrate. This is clear from the following provisions of Section 6, Rule 112 of the Rules of Court. Warrant of arrest, when issued. — If the judge be satisfied from the preliminary examination conducted by him or by the investigating officer that the offense complained of has been committed and that there is reasonable ground to believe that the accused has committed it, he must issue a warrant or order for his arrest. Under this section, the judge must satisfy himself of the existence of probable cause before issuing a warrant or order of arrest. If on the face of the information the judge finds no probable cause, he may disregard the fiscal's certification and require the submission of the affidavits of witnesses to aid him in arriving at a conclusion as to the existence of a probable cause. This has been the rule since U.S. v. Ocampo (18 Phil. 1) and Amarga v. Abbas (98 Phil. 739). And this evidently is the reason for the issuance by respondent of the questioned orders of April 13, 15, 16, 19, 1982 and July 13, 1982. Without the affidavits of the prosecution witnesses and other evidence which, as a matter of long-standing practice had been attached to the information filed in his sala, respondent found the informations inadequate bases for the determination of probable cause. For as the ensuing events would show, after petitioners had submitted the required affidavits, respondent wasted no time in issuing the warrants of arrest in the case where he was satisfied that probable cause existed. The case of Soliven v. Makasiar (167 SCRA 393 [19881) was decided after the effectivity of the 1987 Constitution. We stated:

The second issue, raised by petitioner Beltran, calls for an interpretation of the constitutional provision on the issuance of warrants of arrest. The pertinent provision reads: Art. III, Sec. 2. The right of the people to be secure in their persons, houses, papers and effects against unreasonable searches and seizures of whatever nature and for any purpose shall be inviolable, and no search warrant or warrant of arrest shall issue except upon probable cause to be determined personally by the judge after examination under oath or affirmation of the complainant and the witnesses he may produce, and particularly describing the place to be searched and the persons or things to be seized. The addition of the word "personally" after the word "determined" and the deletion of the grant of authority by the 1973 Constitution to issue warrants to "other respondent officers as may be authorized by law", has apparently convinced petitioner Beltran that the Constitution now requires the judge to personally examine the complainant and his witnesses in his determination of probable cause for the issuance of arrest. This is not an accurate interpretation. What the Constitution underscores is the exclusive and personal responsibility of the issuing judge to satisfy himself of the existence of probable cause. In satisfying himself of the existence of probable cause for the issuance of a warrant of arrest, the judge is not required to personally examine the complainant and his witnesses. Following established doctrine and procedures, he shall: (1) personally evaluate the report and the supporting documents submitted by the fiscal regarding the existence of probable cause and, on the basis thereof, issue a warrant of arrest; or (2) if on the basis thereof he finds no probable cause, he may disregard the fiscal's report and require the submission of supporting affidavits of witnesses to aid him in arriving at a conclusion as to the existence of probable cause. Sound policy dictates this procedure, otherwise judges would be unduly laden with the preliminary examinations and investigation of criminal complaints instead of concentrating on hearing and deciding cases filed before their courts. The decision in People v. Honorable Enrique B. Inting, et al. (G.R. No. 88919, July 25, 1990), reiterated the above interpretation of "personal" determination by the Judge: We emphasize important features of the constitutional mandate that ". . . no search warrant or warrant of arrest shall issue except upon probable cause to be determined personally by the judge . . ." (Article III, Section 2, Constitution) First, the determination of probable cause is a function of the Judge. It is not for the Provincial Fiscal or Prosecutor nor for the Election Supervisor to ascertain. Only the Judge and the Judge alone makes this determination. Second, the preliminary inquiry made by a Prosecutor does not bind the Judge. It merely assists him to make the determination of probable cause. The Judge does not have to follow what the Prosecutor presents to him. By itself, the Prosecutor's certification of probable cause is ineffectual. It is the report, the affidavits, the transcripts of stenographic notes (if any), and all other supporting documents behind the Prosecutor's certification which are material in assisting the Judge to make his determination. And third, Judges and Prosecutors alike should distinguish the preliminary inquiry which determines probable cause for the issuance of a warrant of arrest from the preliminary investigation proper which ascertains whether the offender should be held for trial or

released. Even if the two inquiries are conducted in the course of one and the same proceeding, there should be no confusion about the objectives. The determination of probable cause for the warrant of arrest is made by the Judge. The preliminary investigation proper –– whether or not there is reasonable ground to believe that the accused is guilty of the offense charged and, therefore, whether or not he should be subjected to the expense, rigors and embarrassment of trial –– is the function of the Prosecutor. The Court made this clear in the case of Castillo v. Villaluz (171 SCRA 39 [19891): Judges of Regional Trial Courts (formerly Courts of First Instance) no longer have authority to conduct preliminary investigations. That authority, at one time reposed in them under Sections 13, 14 and 16, Rule 112 of the Rules of Court of 1964, (See Sec. 4, Rule 108, Rules of Court of 1940; People v. Solon, 47 Phil. 443, cited in Moran, Comments on the Rules, 1980 ed., Vol. 4, pp. 115-116) was removed from them by the 1985 Rules on Criminal Procedure, effective on January 1, 1985, (Promulgated on November 11, 1984) which deleted all provisions granting that power to said Judges. We had occasion to point tills out in Salta v. Court of Appeals, 143 SCRA 228, and to stress as well certain other basic propositions, namely: (1) that the conduct of a preliminary investigation is "not a judicial function . . . (but) part of the prosecution's job, a function of the executive," (2) that whenever "there are enough his or prosecutors to conduct preliminary investigations, courts are counseled to leave this job which is essentially executive to them," and the fact "that a certain power is granted does not necessary mean that it should be indiscriminately exercised. The 1988 Amendments to the 1985 Rules on Criminal Procedure, declared effective on October 1, 1988, (The 1988 Amendments were published in the issue of Bulletin Today of October 29, 1988) did not restore that authority to Judges of Regional Trial Courts; said amendments did not in fact deal at all with the officers or courts having authority to conduct preliminary investigations. This is not to say, however, that somewhere along the line RTC Judges also lost the power to make a preliminary examination for the purpose of determining whether probable cause exists to justify the issuance of a warrant of arrest (or search warrant). Such a power –– indeed, it is as much a duty as it is a power –– has been and remains vested in every judge by the provisions in the Bill of Rights in the 1935, the 1973 and the present [1987] Constitutions securing the people against unreasonable searches and seizures, thereby placing it beyond the competence of mere Court Rule or Statute to revoke. The distinction must, therefore, be made clear while an RTC Judge may no longer conduct preliminary investigations to ascertain whether there is sufficient ground for the filing of a criminal complaint or information, he retains the authority, when such a pleading is filed with his court, to determine whether there is probable cause justifying the issuance of a warrant of arrest. It might be added that this distinction accords, rather than conflicts, with the rationale of Salta because both law and rule, in restricting to judges the authority to order arrest, recognize the function to be judicial in nature. We reiterate that preliminary investigation should be distinguished as to whether it is an investigation for the determination of a sufficient ground for the filing of the information or it is an investigation for the determination of a probable cause for the issuance of a warrant of arrest. The first kind of preliminary investigation is executive in nature. It is part of the

prosecution's job. The second kind of preliminary investigation which is more properly called preliminary examination is judicial in nature and is lodged with the Judge. . . . Finally in the recent case of People v. Delgado, et al. (G.R. Nos. 93419-32, September 18, 1990) there is a statement that the judge may rely on the resolution of COMELEC to file the information by the same token that it may rely on the certification made by the prosecutor who conducted the preliminary investigation in the issuance of the warrant of arrest. We, however, also reiterated that ". . . the court may require that the record of the preliminary investigation be submitted to it to satisfy itself that there is probable cause which will warrant the issuance of a warrant of arrest." (Section 2, Article III, Constitution). Reliance on the COMELEC resolution or the Prosecutor's certification presupposes that the records of either the COMELEC or the Prosecutor have been submitted to the Judge and he relies on the certification or resolution because the records of the investigation sustain the recommendation. The warrant issues not on the strength of the certification standing alone but because of the records which sustain it. It is obvious from the present petition that notwithstanding the above decisions, some Judges are still bound by the inertia of decisions and practice under the 1935 and 1973 Constitutions and are sadly confused or hesitant. Prosecutors are also interested in a clear cut ruling. We will, therefore, restate the rule in greater detail and hopefully clearer terms. There is no problem with search warrants which are relatively fewer and far between and where there is no duplication of work between the Judge and the Prosecutor. The problem lies with warrants of arrest especially in metropolitan or highly urban areas. If a Judge has to personally question each complainant and witness or go over the records of the Prosecutor's investigation page by page and word for word before he acts on each of a big pile of applications for arrest warrants on his desk, he or she may have no more time for his or her more important judicial functions. At the same time, the Judge cannot ignore the clear words of the 1987 Constitution which requires ". . . probable cause to be personally determined by the judge . . .", not by any other officer or person. If a Judge relies solely on the certification of the Prosecutor as in this case where all the records of the investigation are in Masbate, he or she has not personally determined probable cause. The determination is made by the Provincial Prosecutor. The constitutional requirement has not been satisfied. The Judge commits a grave abuse of discretion. The records of the preliminary investigation conducted by the Municipal Court of Masbate and reviewed by the respondent Fiscal were still in Masbate when the respondent Fiscal issued the warrants of arrest against the petitioners. There was no basis for the respondent Judge to make his own personal determination regarding the existence of a probable cause for the issuance of a warrant of arrest as mandated by the Constitution. He could not possibly have known what transpired in Masbate as he had nothing but a certification. Significantly, the respondent Judge denied the petitioners' motion for the transmittal of the records on the ground that the mere certification and recommendation of the respondent Fiscal that a probable cause exists is sufficient for him to issue a warrant of arrest. We reiterate the ruling in Soliven v. Makasiar that the Judge does not have to personally examine the complainant and his witnesses. The Prosecutor can perform the same functions as a commissioner for the taking of the evidence. However, there should be a report and necessary documents supporting the Fiscal's bare certification. All of these should be before the Judge. The extent of the Judge's personal examination of the report and its annexes depends on the circumstances of each case. We cannot determine beforehand how cursory or exhaustive the 1âw phi 1

Judge's examination should be. The Judge has to exercise sound discretion for, after all, the personal determination is vested in the Judge by the Constitution. It can be as brief or as detailed as the circumstances of each case require. To be sure, the Judge must go beyond the Prosecutor's certification and investigation report whenever necessary. He should call for the complainant and witnesses themselves to answer the court's probing questions when the circumstances of the case so require. It is worthy to note that petitioners Vicente Lim, Sr. and Susana Lim presented to the respondent Judge documents of recantation of witnesses whose testimonies were used to establish a prima facie case against them. Although, the general rule is that recantations are not given much weight in the determination of a case and in the granting of a new trial (Tan Ang Bun v. Court of Appeals, et al. G.R. No. L-47747, February 15, 1990, People v. Lao Wan Sing, 46 SCRA 298 [1972]) the respondent Judge before issuing his own warrants of arrest should, at the very least, have gone over the records of the preliminary examination conducted earlier in the light of the evidence now presented by the concerned witnesses in view of the "political undertones" prevailing in the cases. Even the Solicitor General recognized the significance of the recantations of some witnesses when he recommends a reinvestigation of the cases, to wit: It must be pointed out, however, that among the documents attached to this Petition are affidavits of recantation subsequently executed by Jimmy Cabarles and Danilo Lozano and an affidavit executed by one, Camilo Sanano, father of the complainant's witnesses, Renato and Romeo Sanano. It was precisely on the strength of these earlier written statements of these witnesses that the Municipal Trial Court of Masbate found the existence of a prima facie case against petitioners and accordingly recommended the filing of a Criminal Information. Evidently, the same written statements were also the very basis of the "Fiscal's Certification", since the attached affidavits of recantation were not yet then available. Since the credibility of the prosecution witnesses is now assailed and put in issue and, since the petitioners have not yet been arraigned, it would be to the broader interest of justice and fair play if a reinvestigation of this case be had to secure the petitioners against hasty prosecution and to protect them from an open and public accusation of crime, from the trouble, expense and anxiety of a public trial, and also to protect the State from useless and expensive trials (Salonga v. Paño G.R. No. 59524, February 18,1985). (Rollo of G.R. Nos. 94054-56, pp. 200-201) We reiterate that in making the required personal determination, a Judge is not precluded from relying on the evidence earlier gathered by responsible officers. The extent of the reliance depends on the circumstances of each case and is subject to the Judge's sound discretion. However, the Judge abuses that discretion when having no evidence before him, he issues a warrant of arrest. Indubitably, the respondent Judge committed a grave error when he relied solely on the Prosecutor's certification and issued the questioned Order dated July 5, 1990 without having before him any other basis for his personal determination of the existence of a probable cause. WHEREFORE, the instant petitions are hereby GRANTED. The questioned Order of respondent Judge Nemesio S. Felix of Branch 56, Regional Trial Court of Makati dated July 5, 1990 is declared NULL and VOID and SET ASIDE. The Temporary Restraining Orders and Preliminary Mandatory Injunction issued in the instant Petitions are made PERMANENT. G.R. No. 96080

April 19, 1991

ATTY. MIGUEL P. PADERANGA petitioner, vs.

HON. FRANKLIN M. DRILON, HON. SILVESTRE H. BELLO III, ATTY. HENRICK F. GINGOYON, HELEN B. CANOY and REBECCA B. TAN, respondent Concordio C. Diel, Constantino G. Jaraula for petitioner. Benjamin G. Guimong for private respondents.

REGALADO, J.: p

In this special civil action for mandamus and prohibition with prayer for a writ of preliminary injunction/restraining order, petitioner seeks to enjoin herein public respondents from including the former as an accused in Criminal Case No. 86-39 for multiple murder, through a second amended information, and to restrain them from prosecuting him. The records disclose that on October 16, 1986, an information for multiple murder was filed in the Regional Trial Court, Gingoog City, against Felipe Galarion, Manuel Sabit, Cesar Sabit, Julito Ampo, Eddie Torion, John Doe, Peter Doe and Richard Doe, for the deaths on May 1, 1984 of Renato Bucag, his wife Melchora Bucag, and their son Renato Bucag II. Venue was, however, transferred to Cagayan de Oro City per Administrative Matter No. 87-2-244. Only Felipe Galarion was tried and found guilty as charged. The rest of the accused remained at large. Felipe Galarion, however, escaped from detention and has not been apprehended since then. In an amended information filed on October 6, 1988, Felizardo Roxas, alias "Ely Roxas," "Fely Roxas" and "Lolong Roxas," was included as a co-accused. Roxas retained petitioner Paderanga as his counsel. As counsel for Roxas, petitioner filed, among others, an Omnibus Motion to dismiss, to Quash the Warrant of Arrest and to Nullify the Arraignment on October 14, 1988. The trial court in an order dated January 9, 1989, denied this omnibus motion but directed the City Prosecutor "to conduct another preliminary investigation or reinvestigation in order to grant the accused all the opportunity to adduce whatever evidence he has in support of his defense." In the course of the preliminary investigation, through a signed affidavit, Felizardo Roxas implicated herein petitioner in the commission of the crime charged. The City Prosecutor of Cagayan de Oro City inhibited himself from further conducting the preliminary investigation against petitioner at the instance of the latter's counsel, per his resolution dated July 7, 1989. In his first indorsement to the Department of Justice, dated July 24, 1989, said city prosecutor requested the Department of Justice to designate a state prosecutor to continue the preliminary investigation against herein petitioner. In a resolution dated September 6, 1989,1 respondent State Prosecutor Henrick F. Gingoyon, who was designated to continue with the conduct of the preliminary investigation against petitioner, directed the amendment of the previously amended information to include and implead herein petitioner as one of the accused therein. Petitioner moved for reconsideration,2 contending that the preliminary investigation was not yet completed when said resolution was promulgated, and that he

was deprived of his right to present a corresponding counter-affidavit and additional evidence crucial to the determination of his alleged "linkage" to the crime charged. The motion was, however, denied by respondent Gingoyon in his order dated January 29, 1990.3 From the aforesaid resolution and order, petitioner filed a Petition for Review4 with the Department of Justice. Thereafter, he submitted a Supplemental Petition with Memorandum,5 and then a Supplemental Memorandum with Additional Exculpatory/Exonerating Evidence Annexed,6 attaching thereto an affidavit of Roxas dated June 20, 1990 and purporting to be a retraction of his affidavit of March 30, 1990 wherein he implicated herein petitioner. On August 10, 1990, the Department of Justice, through respondent Undersecretary Silvestre H. Bello III, issued Resolution No. 6487 dismissing the said petition for review. His motion for reconsideration having been likewise denied, petitioner then flied the instant petition for mandamus and prohibition. Petitioner raises two basic issues, namely: (1) that the preliminary investigation as to him was not complete; and (2) that there exists no prima facie evidence or probable cause to justify his inclusion in the second amended information. Preliminary investigation is generally inquisitorial, and it is often the only means of discovering the persons who may be reasonably charged with a crime, to enable the fiscal to prepare his complaint or information. It is not a trial of the case on the merits and has no purpose except that of determining whether a crime has been committed and whether there is probable cause to believe that the accused is guilty thereof, and it does not place the person against whom it is taken in jeopardy.8 The institution of a criminal action depends upon the sound discretion of the fiscal. He has the quasijudicial discretion to determine whether or not a criminal case should be filed in court.9 Hence, the general rule is that an injunction will not be granted to restrain a criminal prosecution.10 The case of Brocka, et al. vs. Enrile, et al.11 cites several exceptions to the rule, to wit: a. To afford adequate protection to the constitutional rights of the accused; b. When necessary for the orderly administration of justice or to avoid oppression or multiplicity of actions; c. When there is a pre-judicial question which is sub judice; d. When the acts of the officer are without or in excess of authority; e. Where the prosecution is under an invalid law, ordinance or regulation; f. When double jeopardy is clearly apparent; g. Where the court has no jurisdiction over the offense; h. Where it is a case of persecution rather than prosecution; i. Where the charges are manifestly false and motivated by the lust for vengeance; and

j. When there is clearly no prima facie case against the accused and a motion to quash on that ground has been denied. A careful analysis of the circumstances obtaining in the present case, however, will readily show that the same does not fall under any of the aforesaid exceptions. Hence, the petition at bar must be dismissed. 1. Petitioner avers that he was deprived of a full preliminary investigation by reason of the fact that at the time the resolution of September 6, 1989 was issued, there were still several incidents pending resolution such as the validity of the testimonies and affidavits of Felizardo Roxas and Rogelio Hanopol as bases for preliminary investigation, the polygraph test of Roxas which he failed, and the clarificatory questions which were supposed to be propounded by petitioner's counsel to Roxas and Hanopol. Petitioner likwise claims that he was deprived of the opportunity to file his counter-affidavit to the subpoena of April 25, 1989. These contentions are without merit. Firstly, it will be noted that petitioner had already filed his counter-affidavit, pursuant to the subpoena issued to him on April 17, 1989, wherein he controverted the charge against him and dismissed it as a malicious design of his political opponents and enemies to link him to the crime. We hold that this is sufficient compliance with the procedural requirement of the Rules of Court, specifically Section 3(b) of Rule 112 thereof. Besides, petitioner failed to show that the subpoena issued on April 25, 1989 involved a separate complaint charging an offense different and distinct from that charged in the complaint attached to the first subpoena issued to him earlier. Secondly, the veracity and credibility of the witnesses and their testimonies are matters of defense best addressed to the trial court for its appreciation and evaluation. Thirdly, the right of petitioner to ask clarificatory questions is not absolute. The fiscal has the discretion to determine whether or not he will propound these questions to the parties or witnesses concerned. As clearly provided for under Section 3(e), Rule 112 of the Rules of Court.: 1âwphi 1

(e) If the investigating officer believes that there are matters to be clarified, he may set a hearing to propound clarificatory questions to the parties or their witnesses, during which the parties shall be afforded an opportunity to be present but without the right to examine or cross-examine. If the parties so desire, they may submit questions to the to the investigating officer which the latter may propound to the parties or witnesses concerned. Lastly, it has been held that "the proper forum before which absence of preliminary investigation should be ventilated is the Court of First Instance of a preliminary investigation does not go to the jurisdiction of the court but merely to the regularity of the proceedings. It could even be waived. Indeed, it is frequently waived. These are matters to be inquired into by the trail court not an appellate court."12 2. Petitioner further submits that there is no prima facie evidence, or probable cause, or sufficient justification to hold him to a tedious and prolonged public trial, on the basis of the following grounds: the questioned resolution of respondent Gingoyon is full of factual misrepresentations or misapprehensions; respondent's reliance on the decision of the Regional Trial Court against Felipe Galarion suffers from constitutional and procedural infirmities considering that petitioner was not a party thereto, much less was he given any opportunity to comment on or rebut the prosecution evidence; reliance on Rogelio Hanopol's testimony is likewise "contemptible," it being merely hearsay in addition to the fact that petitioner was never given the opportunity to cross-examine Hanopol at the time he testified in court; and the affidavit of Roxas dated March 30, 1989, which is

the only evidence against petitioner, has been rendered nugatory by his affidavit of retraction dated June 20, 1990. A preliminary investigation is defined as an inquiry or proceeding for the purpose of determining whether there is sufficient ground to engender a well founded belief that a crime cognizable by the Regional Trial Court has been committed and that the respondent is probably guilty thereof, and should be held for trial.13 The quantum of evidence now required in preliminary investigation is such evidence sufficient to "engender a well founded belief as to the fact of the commission of a crime and the respondent's probable guilt thereof. A preliminary investigation is not the occasion for the full and exhaustive display of the parties' evidence; it is for the presentation of such evidence only as may engender a wen grounded belief that an offense has been committed and that the accused is probably guilty thereof.14 We are in accord with the state prosecutor's findings in the case at bar that there exists prima facie evidence of petitioner's involvement in the commission of the crime, it being sufficiently supported by the evidence presented and the facts obtaining therein. Likewise devoid of cogency is petitioner's argument that the testimonies of Galarion and Hanopol are inadmissible as to him since he was not granted the opportunity of cross-examination. It is a fundamental principle that the accused in a preliminary investigation has no right to crossexamine the witnesses which the complainant may present. Section 3, Rule 112 of the Rules of Court expressly provides that the respondent shall only have the right to submit a counter-affidavit, to examine all other evidence submitted by the complainant and, where the fiscal sets a hearing to propound clarificatory questions to the parties or their witnesses, to be afforded an opportunity to be present but without the right to examine or cross-examine. Thus, even if petitioner was not given the opportunity to cross-examine Galarion and Hanopol at the time they were presented to testify during the separate trial of the case against Galarion and Roxas, he cannot assert any legal right to crossexamine them at the preliminary investigation precisely because such right was never available to him. The admissibility or inadmissibility of said testimonies should be ventilated before the trial court during the trial proper and not in the preliminary investigation. Furthermore, the technical rules on evidence are not binding on the fiscal who has jurisdiction and control over the conduct of a preliminary investigation. If by its very nature a preliminary investigation could be waived by the accused, we find no compelling justification for a strict application of the evidentiary rules. In addition, considering that under Section 8, Rule 112 of the Rules of Court, the record of the preliminary investigation does not form part of the record of the case in the Regional Trial Court, then the testimonies of Galarion and Hanopol may not be admitted by the trial court if not presented in evidence by the prosecuting fiscal. And, even if the prosecution does present such testimonies, petitioner can always object thereto and the trial court can rule on the admissibility thereof; or the petitioner can, during the trial, petition said court to compel the presentation of Galarion and Hanopol for purposes of cross-examination. WHEREFORE, the instant petition is hereby DISMISSED for lack of merit.

[G.R. No. 118821. February 18, 2000] MAYOR BAI UNGGIE D. ABDULA and ODIN ABDULA, petitioners, vs. HON. JAPAL M. GUIANI, in his capacity as Presiding Judge, of Branch 14 of the Regional Trial Court of Cotabato City, respondent. DECISION

GONZAGA_REYES, J.: At bench is a petition for certiorari and prohibition to set aside the warrant of arrest issued by herein respondent Japal M. Guiani, then presiding judge of Branch 14 of the Regional Trial Court of Cotabato City, ordering the arrest of petitioners without bail in Criminal Case No. 2376 for murder. The antecedent facts of the case are as follows: On 24 June 1994, a complaint for murder, docketed as I.S. No. 94-1361, was filed before the Criminal Investigation Service Command, ARMM Regional Office XII against herein petitioners and six (6) other persons in connection with the death of a certain Abdul Dimalen, the former COMELEC Registrar of Kabuntalan, Maguindanao. The complaint alleged that herein petitioners paid the six other respondents the total amount of P200,000.00 for the death of Abdul Dimalen. [1]

[2]

[3]

Acting on this complaint, the Provincial Prosecutor of Maguindanao, Salick U. Panda, in a Resolution dated 22 August 1994 , dismissed the charges of murder against herein petitioners and five other respondents on a finding that there was no prima facie case for murder against them. Prosecutor Panda, however, recommended the filing of an information for murder against one of the respondents, a certain Kasan Mama. Pursuant to this Resolution, an information for murder was thereafter filed against Kasan Mama before the sala of respondent Judge. [4]

In an Order dated 13 September 1994 , respondent Judge ordered that the case, now docketed as Criminal Case No. 2332, be returned to the Provincial Prosecutor for further investigation. In this Order, respondent judge noted that although there were eight (8) respondents in the murder case, the information filed with the court "charged only one (1) of the eight (8) respondents in the name of Kasan Mama without the necessary resolution required under Section 4, Rule 112 of the Revised Rules of Court to show how the investigating prosecutor arrived at such a conclusion." As such, the respondent judge reasons, the trial court cannot issue the warrant of arrest against Kasan Mama. [5]

Upon the return of the records of the case to the Office of the Provincial Prosecutor for Maguindanao, it was assigned to 2nd Assistant Prosecutor Enok T. Dimaraw for further investigation. In addition to the evidence presented during the initial investigation of the murder charge, two new affidavits of witnesses were submitted to support the charge of murder against

herein petitioners and the other respondents in the murder complaint. Thus, Prosecutor Dimaraw treated the same as a refiling of the murder charge and pursuant to law, issued subpoena to the respondents named therein. On December 6, 1994, herein petitioners submitted and filed their joint counteraffidavits. [6]

After evaluation of the evidence, Prosecutor Dimaraw, in a Resolution dated 28 December 1994, found a prima facie case for murder against herein petitioners and three (3) other respondents. He thus recommended the filing of charges against herein petitioners Bai Unggie Abdula and Odin Abdula, as principals by inducement, and against the three (3) others, as principals by direct participation. [7]

[8]

Likewise in this 28 December 1994 Resolution, Provincial Prosecutor Salick U. Panda, who conducted the earlier preliminary investigation of the murder charge, added a notation stating that he was inhibiting himself from the case and authorizing the investigating prosecutor to dispose of the case without his approval. The reasons he cited were that the case was previously handled by him and that the victim was the father-in-law of his son. [9]

On 2 January 1995, an information for murder dated 28 December 1994 was filed against the petitioner spouses and Kasan Mama, Cuenco Usman and Jun Mama before Branch 14 of the Regional Trial Court of Cotabato City, then the sala of respondent judge. This information was signed by investigating prosecutor Enok T. Dimaraw. A notation was likewise made on the information by Provincial Prosecutor Panda, which explained the reason for his inhibition. [10]

[11]

The following day, or on 3 January 1995, the respondent judge issued a warrant for the arrest of petitioners. Upon learning of the issuance of the said warrant, petitioners filed on 4 January 1995 an Urgent Ex-parte Motion for the setting aside of the warrant of arrest on 4 January 1995. In this motion, petitioners argued that the enforcement of the warrant of arrest should be held in abeyance considering that the information was prematurely filed and that the petitioners intended to file a petition for review with the Department of Justice. [12]

[13]

A petition for review was filed by the petitioners with the Department of Justice on 11 January 1995. Despite said filing, respondent judge did not act upon petitioners pending Motion to Set Aside the Warrant of Arrest. [14]

[15]

Hence, this Petition for Certiorari and Prohibition wherein petitioners pray for the following: "1. upon filing of this petition, a temporary restraining order be issued enjoining the implementation and execution of the order of arrest dated January 3, 1995 and enjoining the respondent judge from further proceeding with Criminal Case No. 2376 entitled People of the Philippines vs. Bai Unggie D. Abdula, et al. upon such bond as may be required by the Honorable Court; 2. this petition be given due course and the respondent be required to answer; 3. after due hearing, the order of arrest dated January 3, 1995 be set aside and declared void ab initio and the respondent judge be disqualified from hearing Criminal Case No. 2376 entitled People of the Philippines vs. Bai Unggie D. Abdula, et al." [16]

In a Resolution dated 20 February 1995, this Court resolved to require respondent judge to submit a comment to the petition. The Court further resolved to issue a temporary restraining order enjoining the respondent judge from implementing and executing the Order of Arrest dated 3 January 1995 and from further proceeding with Criminal Case No. 2376. [17]

[18]

At the onset, it must be noted that petitions for certiorari and prohibition require that there be no appeal, nor any plain, speedy and adequate remedy in the ordinary course of law available to the petitioner. In the instant case, it cannot be said that petitioners have no other remedy available to them as there is pending before the lower court an Urgent Motion praying for the lifting and setting aside of the warrant of arrest. Ordinarily, we would have dismissed the petition on this ground and let the trial court decide the pending motion. However, due to the length of time that the issues raised in the petition have been pending before the courts, it behooves us to decide the case on the merits in order to preclude any further delay in the resolution of the case. [19]

[20]

Respondent Japal M. Guiani retired from the judiciary on 16 April 1996. For this reason, respondent is no longer the presiding judge of the Regional Trial Court Branch 14 of Cotabato City; and the prayer of petitioner as to respondents disqualification from hearing Criminal Case No. 2376 is now moot and academic. Thus, there remain two issues left for the determination of the Court: first, the legality of the second information for murder filed before

respondents court; and second, the validity of the warrant of arrest issued against petitioners. With respect to the first issue, petitioners aver that it is the respondent judge himself who is orchestrating the filing of the alleged murder charge against them. In support, petitioners cite five (5) instances wherein respondent judge allegedly issued illegal orders in a mandamus case pending in respondents sala filed against petitioner Mayor Bai Unggie Abdula. These allegedly illegal orders formed the basis for a criminal complaint which they filed on 6 October 1994 against respondent and ten (10) others before the Office of the Ombudsman for Mindanao. In this complaint, herein petitioners alleged that the respondent judge illegally ordered the release of the total amount of P1,119,125.00 from the municipal funds of Kabuntalan, Mindanao to a certain Bayoraiz Saripada, a purported niece of respondent judge. The Office of the Ombudsman for Mindanao, in an Order dated 12 December 1994, found "sufficient basis to proceed with the preliminary investigation of the case" and directed the respondents therein to file their respective counteraffidavits and controverting evidence. From these facts, petitioners argue, it is clear that it is the respondent judge himself who is orchestrating and manipulating the charges against the petitioner. [21]

[22]

Petitioners further state that respondent judge exhibited extreme hostility towards them after the filing of the said complaint before the Ombudsman. Petitioners claim that immediately after the issuance of the Order of the Ombudsman requiring respondent judge to file his counter-affidavit, respondent allegedly berated petitioner Bai Unggie Abdula in open court when she appeared before him in another case Allegedly, in full view of the lawyers and litigants, respondent judge uttered the following words in the Maguindanaoan dialect: "If I cannot put you in jail within your term, I will cut my neck. As long as I am a judge here, what I want will be followed." [23]

Respondent judge, in compliance with the Order of this Court, filed a Comment dated 3 March 1995. In this Comment, he argues that petitioners enumeration of "incontrovertible facts" is actually a list of misleading facts which they are attempting to weave into Criminal Case No. 2376 for the purpose of picturing respondent as a partial judge who abused his discretion to favor petitioners accuser. He claims that the anti-graft charge filed by petitioners against him is a harassment suit concocted by them when they failed to lay their hands on the amount of P1,119,125.00 of municipal funds which respondent had previously ruled as rightfully belonging to the municipal [24]

[25]

councilors of Kabuntalan, Maguindanao. Respondent vehemently denies having personally profited from the release of the municipal funds. Moreover, respondent points out that the allegations in the complaint seem to imply that the Vice Mayor of Kabuntalan, Bayoraiz Sarupada, was a party to the mandamus case filed with respondents court when in truth, there was no case filed by the vice mayor pending in his court. Finally, respondent denies berating petitioner Bai Unggie Abdula and uttering the words attributed to him in the petition. According to respondent, the last time petitioner Bai Unggie Abdula appeared in his sala on December 28, 1994, in connection with the lifting of an order for her apprehension in another case, he neither berated nor scolded her and in fact, he even lifted the said order of arrest. In its Comment with Urgent Motion for the Lifting of the Temporary Restraining Order dated 5 June 1995, the Office of the Solicitor-General states that petitioners allegation that the respondent judge was biased and prejudiced was pure speculation as no proof was presented that respondent assumed the role of prosecutor. Moreover, the OSG argued that the fact that the respondent judge and petitioners had pending cases against each other did not necessarily result in the respondents bias and prejudice. [26]

An analysis of these arguments shows that these should have been properly raised in a motion for the disqualification or inhibition of respondent judge. As previously stated however, the issue as to whether respondent should be disqualified from proceeding with the case has been rendered moot and academic as he is no longer hearing the case against petitioners. As such, there is no need for a prolonged discussion on this issue. It is sufficient to say that in order to disqualify a judge on the ground of bias and prejudice, petitioner must prove the same by clear and convincing evidence. This is a heavy burden which petitioners have failed to discharge. This Court has to be shown acts or conduct of the judge clearly indicative of arbitrariness or prejudice before the latter can be branded the stigma of being biased and partial. [27]

[28]

Petitioners next argue that the act of respondent in motu proprio ordering a reinvestigation of the murder charge against them is another indication of the latters bias and prejudice. They claim that the filing of their complaint against respondent motivated the latters Order of 13 September 1994 which ordered the return of the records of the murder case to the provincial prosecutor. Furthermore, they posit that the latter had no authority to order the reinvestigation considering that same had already been dismissed as against them by the provincial prosecutor in his Resolution dated 22 August 1994. [29]

A review of the pertinent dates in the petition however show that respondent could not have been motivated by the Ombudsmans complaint when he issued the 13 September 1994 Order. Petitioner Bai Unggie Abdula filed the complaint before the Ombudsman of Cotabato City on October 6, 1994 or about a month after the issuance of the 13 September 1994 Order. As such, when respondent issued the said Order, the same could not have been a retaliatory act considering that at that time, there was as yet no complaint against him. [30]

With respect to the allegation that the respondent had no legal authority to order a reinvestigation of the criminal charge considering that the said charge had been previously dismissed as against them, we hold that respondent did not abuse his discretion in doing so. [31]

It is true that under the circumstances, the respondent judge, upon seeing that there were no records attached to the complaint, could have simply ordered the office of the provincial prosecutor to forward the same. Upon receipt of these records, respondent judge would then have sufficient basis to determine whether a warrant of arrest should issue. However, from the bare terms of the questioned order alone, we fail to see any illegal reason that could have motivated the judge in issuing the same. The order merely stated that the records of the case should be returned to the Office of the Provincial Prosecutor for further investigation or reinvestigation. He did not unduly interfere with the prosecutors duty to conduct a preliminary investigation by ordering the latter to file an information against any of the respondents or by choosing the fiscal who should conduct the reinvestigation which are acts certainly beyond the power of the court to do. It was still the prosecutor who had the final say as to whom to include in the information. [32]

[33]

As pointed out by the Office of the Solicitor General, petitioners only imputed bias against the respondent judge and not against the investigating prosecutor. Consequently, this imputation is of no moment as the discretion to file an information is under the exclusive control and supervision of the prosecutor and not of respondent judge. Furthermore, petitioners cannot claim that they were denied due process in the reinvestigation of the charges against them as they actively participated therein by submitting their joint counter-affidavit. [34]

Petitioners likewise allege that the information charging petitioners with murder is null and void because it was filed without the authority of the Provincial Prosecutor. They note that in the Resolution dated 28 December 1994 and in the corresponding information, it clearly appears that the same

were not approved by the Provincial Prosecutor as it was signed only by the investigating prosecutor, Anok T. Dimaraw. Petitioners contention is not well-taken. The pertinent portion of the Rules of Court on this matter state that "(n)o complaint or information shall be filed or dismissed by an investigating fiscal without the prior written authority or approval of the provincial or city fiscal or chief state prosecutor (underscoring ours)." In other words, a complaint or information can only be filed if it is approved or authorized by the provincial or city fiscal or chief state prosecutor. In the case at bench, while the Resolution and the Information were not approved by Provincial Prosecutor Salick U. Panda, the filing of the same even without his approval was authorized. Both the Resolution and information contain the following notation: *

"The herein Provincial Prosecutor is inhibiting himself from this case and Investigating Prosecutor Enok Dimaraw may dispose of the case without his approval on the following ground: That this case has been previously handled by him, and whose findings differ from the findings of Investigating Prosecutor Dimaraw; and the victim is a relative by affinity, he being a fatherin-law of his son. (Signed) Salick U. Panda Provincial Prosecutor It must be stressed that the Rules of Court speak of authority or approval by the provincial, city, or chief state prosecutor. The notation made by Prosecutor Panda clearly shows that Investigating Prosecutor Dimaraw was authorized to "dispose of the case without his approval." In issuing the resolution and in filing the information, the investigating prosecutor was acting well within the authority granted to him by the provincial prosecutor. Thus, this resolution is sufficient compliance with the aforecited provision of the Rules of Court. Having thus ruled on the validity of the information filed against the respondents, we now address the issue as to the legality of the warrant of arrest issued by respondent judge by virtue of the said information.

On this issue, petitioners, citing the case of Allado vs. Diokno argue that the warrant for his arrest should be recalled considering that the respondent judge "did not personally examine the evidence nor did he call the complainant and his witnesses in the face of their incredible accounts." As proof, he points to the fact that the information was filed at around 4:00 p.m. of the January 2, 1995 and the order of arrest was immediately issued the following day or on January 3, 1995. Moreover, petitioner argues, respondent judge did not even issue an order stating that there is probable cause for the issuance of the warrant of arrest, a clear violation of the guidelines set forth in the Allado case. [35]

Respondent, in his Comment, denies any irregularity in the issuance of the warrant of arrest. He argues as follows: "Written authority having been granted by the Provincial Prosecutor, as required by the third paragraph of Section 4, Rule 112 of (the) Rules on Criminal Procedure, and there having been no reason for the respondent to doubt the validity of the certification made by the Assistant Prosecutor that a preliminary investigation was conducted and that probable cause was found to exist as against those charged in the Information filed, and recognizing the prosecutions legal authority to initiate and control criminal prosecution (Rule 110, Section 5) and considering that the court cannot interfere in said prosecutions authority (People vs. Moll, 68 Phil. 626), the respondent issued the warrant for the arrest of the accused pursuant to paragraph (a), section 6, Rule 112;" [36]

The OSG, in defending the act of respondent judge, argues that the allegation that respondent did not personally examine the evidence is not supported by current jurisprudence. In support, the OSG invokes the pronouncement in Soliven vs. Makasiar that "(I)n satisfying himself of the existence of probable cause, the judge is not required to personally examine the complainant and his witnesses." Moreover, the OSG points out that the judge enjoys a wide degree of latitude in the determination of probable cause for the issuance of warrants of arrest depending on the circumstances of each case. [37]

[38]

The OSG further argues that the case of Allado vs. Diokno, relied upon by petitioners, has no application in the case at bar considering that in the cited case, the documents submitted before the court failed to establish any probable cause as they were conflicting and contradictory. Significantly, the OSG continues, petitioners could not point out a single flaw in the evidence

presented by the prosecutor to negate the existence of probable cause. Finally, the OSG points out that petitioners unfounded allegations cannot prevail over the well-settled rule that official duty is presumed to be regularly performed. [39]

After a careful analysis of these arguments, we find merit in the contention of petitioners. The pertinent provision of the Constitution reads: "Section 2 [Article III]. The right of the people to be secure in their persons, houses, papers, and effects against unreasonable searches and seizures of whatever nature and for any purpose shall be inviolable, and no search warrant or warrant of arrest shall issue except upon probable cause to be determined personally by the judge after examination under oath or affirmation of the complainant and the witnesses he may produce and particularly describing the place to be searched and the persons or things to be seized." (Undersoring supplied.) It must be stressed that the 1987 Constitution requires the judge to determine probable cause "personally," a requirement which does not appear in the corresponding provisions of our previous constitutions. This emphasis evinces the intent of the framers to place a greater degree of responsibility upon trial judges than that imposed under previous Constitutions. [40]

In Soliven vs. Makasiar, this Court pronounced: "What the Constitution underscores is the exclusive and personal responsibility of the issuing judge to satisfy himself of the existence of probable cause. In satisfying himself of the existence of probable cause for the issuance of a warrant of arrest, the judge is not required to personally examine the complainant and his witnesses. Following established doctrine and procedure, he shall: (1) personally evaluate the report and the supporting documents submitted by the fiscal regarding the existence of probable cause and, on the basis thereof, issue a warrant of arrest; or (2) if on the basis thereof he finds no probable cause, he may disregard the fiscals report and require the submission of supporting affidavits of witnesses to aid him in arriving at a conclusion as to the existence of probable cause."

Ho vs. People summarizes existing jurisprudence on the matter as follows: [41]

"Lest we be too repetitive, we only wish to emphasize three vital matters once more: First, as held in Inting, the determination of probable cause by the prosecutor is for a purpose different from that which is to be made by the judge. Whether there is reasonable ground to believe that the accused is guilty of the offense charged and should be held for trial is what the prosecutor passes upon. The judge, on the other hand, determines whether a warrant of arrest should be issued against the accused, i.e., whether there is a necessity for placing him under immediate custody in order not to frustrate the ends of justice. Thus, even if both should base their findings on one and the same proceeding or evidence, there should be no confusion as to their distinct objectives. Second, since their objectives are different, the judge cannot rely solely on the report of the prosecutor in finding probable cause to justify the issuance of a warrant of arrest. Obviously and understandably, the contents of the prosecutors report will support his own conclusion that there is reason to charge the accused for an offense and hold him for trial. However, the judge must decide independently. Hence, he must have supporting evidence, other than the prosecutors bare report, upon which to legally sustain his own findings on the existence (or nonexistence) of probable cause to issue an arrest order. This responsibility of determining personally and independently the existence or nonexistence of probable cause is lodged in him by no less than the most basic law of the land. Parenthetically, the prosecutor could ease the burden of the judge and speed up the litigation process by forwarding to the latter not only the information and his bare resolution finding probable cause, but also so much of the records and the evidence on hand as to enable the His Honor to make his personal and separate judicial finding on whether to issue a warrant of arrest. Lastly, it is not required that the complete or entire records of the case during the preliminary investigation be submitted to and examined by the judge. We do not intend to unduly burden trial courts by obliging them to examine the complete records of every case all the time simply for the purpose of ordering the arrest of an accused. What is required, rather, is that the judge must

havesufficient supporting documents (such as the complaint, affidavits, counter-affidavits, sworn statements of witnesses or transcript of stenographic notes, if any) upon which to make his independent judgment or, at the very least, upon which to verify the findings of the prosecutor as to the existence of probable cause. The point is: he cannot rely solely and entirely on the prosecutors recommendation, as Respondent Court did in this case. Although the prosecutor enjoys the legal presumption of regularity in the performance of his official duties and functions, which in turn gives his report the presumption of accuracy, the Constitution, we repeat, commands the judge to personally determine probable cause in the issuance of warrants of arrest. This Court has consistently held that a judge fails in his bounden duty if he relies merely on the certification or the report of the investigating officer." (citations omitted) In the case at bench, respondent admits that he issued the questioned warrant as there was "no reason for (him) to doubt the validity of the certification made by the Assistant Prosecutor that a preliminary investigation was conducted and that probable cause was found to exist as against those charged in the information filed." The statement is an admission that respondent relied solely and completely on the certification made by the fiscal that probable cause exists as against those charged in the information and issued the challenged warrant of arrest on the sole basis of the prosecutors findings and recommendations. He adopted the judgment of the prosecutor regarding the existence of probable cause as his own. Although the prosecutor enjoys the legal presumption of regularity in the performance of his official duties, which in turn gives his report the presumption of accuracy, nothing less than the fundamental law of the land commands the judge to personally determine probable cause in the issuance of warrants of arrest. A judge fails in this constitutionally mandated duty if he relies merely on the certification or report of the investigating officer. To be sure, we cannot determine beforehand how cursory or exhaustive the respondents examination of the records should be. The extent of the judges examination depends on the exercise of his sound discretion as the circumstances of the case require. In the case at bench, the respondent had before him two different informations and resolutions charging two different sets of suspects. In the face of these conflicting resolutions, it behooves him not to take the certification of the investigating prosecutor at face value. The circumstances thus require that respondent look beyond the bare certification [42]

of the investigating prosecutor and examine the documents supporting the prosecutors determination of probable cause. The inordinate haste that attended the issuance of the warrant of arrest and respondents own admission are circumstances that tend to belie any pretense of the fulfillment of this duty. Clearly, respondent judge, by merely stating that he had no reason to doubt the validity of the certification made by the investigating prosecutor has abdicated his duty under the Constitution to determine on his own the issue of probable cause before issuing a warrant of arrest. Consequently, the warrant of arrest should be declared null and void. WHEREFORE, premises considered, the petition for certiorari and prohibition is GRANTED. The temporary restraining order we issued on 20 February 1995 in favor of petitioners insofar as it enjoins the implementation and execution of the order of arrest dated 3 January 1995 is made permanent. Criminal Case No. 2376 is REMANDED to Branch 14 of the Regional Trial Court of Cotabato City for a proper determination of whether a warrant of arrest should be issued and for further proceedings.

[G.R. No. 147607. January 22, 2004]

PEOPLE OF THE MAMARIL, appellant.

PHILIPPINES, appellee, vs.

BENHUR

DECISION AZCUNA, J.:

Before us is a petition for review on certiorari of the decision of the Regional Trial Court of Lingayen, Pangasinan, Branch 39, in Criminal Case No. L-5963, finding appellant Benhur Mamaril guilty beyond reasonable doubt of violation of Section 8 of Republic Act (RA) No. 6425, as amended by RA No. 7659. [1]

The Information filed against appellant reads: That on or about the 1st day of February, 1999 and sometime prior thereto, in the municipality of Lingayen, province of Pangasinan, Philippines, and within the jurisdiction of this Honorable Court, the above-named accused, willfully, unlawfully

and criminally [did] keep and possess crushed marijuana leaves contained in seventy eight (78) sachets with a total weight of two hundred thirty six and eighty three hundredth (236.83) grams and two (2) bricks of marijuana fruiting tops weighing one thousand six hundred grams, each brick weighing eight hundred (800) grams, with a total weight of one thousand eight hundred thirty six and eighty three hundredth (1,836.83) grams, a prohibited drug, without authority to possess the same. CONTRARY to Sec. 8 of R.A. 6425, as amended by R.A. 7659, otherwise known as the Dangerous Drugs Act of 1972. [2]

When arraigned on October 8, 1999, appellant pleaded not guilty. At the pre-trial conference held on October 18, 1999, the parties admitted the following facts: [3]

1. That the search was made in the house and premises of the parents of the accused where he (accused) also lives, at Ramos St., Lingayen, Pangasinan, on February 1, 1999 at about 2:30 o clock in the afternoon; 2. That the search was conducted by the elements of the PNP particularly SPO4 Faustino Ferrer, SPO1 Alfredo Rico and others; 3. That the policemen brought along with them a camera; 4. That the accused was in the balcony of the house when it was searched; 5. The existence of the report of physical science report No. (DT-077-99) issued by the PNP Crime Laboratory through Chemist Theresa Ann Bugayong Cid; 6. That accused was subjected to urine sample laboratory on February 2, 1999. [4]

Thereafter, trial ensued. The Prosecutions Evidence On January 25, 1999, the Intelligence Section PNCO of the Lingayen Police Station, represented by SPO2 Chito S. Esmenda, applied before the Regional Trial Court of Lingayen, Pangasinan, Branch 39, for a search warrant authorizing the search for marijuana, a prohibited drug, at the family residence of appellant Benhur Mamaril, situated at Ramos Street, Poblacion, [5]

Lingayen, Pangasinan. On said date, then presiding Executive Judge Eugenio G. Ramos (now retired) issued Search Warrant No. 99-51. [6]

On February 1, 1999, at about 2:30 p.m., the Chief of Police of the Lingayen Police Station, SPO3 Alfredo Rico, SPO4 Faustino Ferrer, Jr. and other police officers went to the residence of appellant and implemented Search Warrant No. 99-51. When they arrived at appellants house, they saw appellants mother under the house. They asked her where appellant was, and she told them that appellant was in the house, upstairs. When they went upstairs, they saw appellant coming out of the room. Upon seeing the policemen, appellant turned back and tried to run towards the back door. SPO3 Rico told appellant to stop, which appellant did. SPO3 Rico informed appellant that they had a search warrant to search the house premises. They showed appellant and his mother the search warrant. Appellant looked at the search warrant and did not say anything. Thereafter, the policemen searched the house. The search was witnessed by two members of the barangay council in said area, namely, Barangay Kagawad Leonardo Ramos and Barangay Tanod Valentino Quintos, whom the police brought with them. [7]

The searching team confiscated the following: (1) fifty-five (55) heatsealed plastic sachets containing suspected marijuana leaves, which were found in a buri bag (bayong) under appellants house; (2) three heat-sealed plastic sachets containing suspected marijuana leaves and seeds contained in an eye-glass case; (3) twenty-two (22) heat-sealed plastic sachets containing suspected marijuana leaves and seeds taken under a pillow placed on a monobloc chair; and (4) two (2) bricks of suspected marijuana contained inside a white and gray bag found inside the closet of appellants room. SPO3 Alfredo Rico took pictures of the confiscated items and prepared a receipt of the property seized. SPO4 Faustino Ferrer, Jr. prepared a certification that the house was properly searched, which was signed by appellant and the barangay officials who witnessed the search. After the search, the police officers brought appellant and the confiscated articles to the Lingayen Police Station and turned them over to the desk officer. [8]

[9]

[10]

[11]

The next day, on February 2, 1999, police officers Alfredo Rico, Alberto Santiago and Rodolfo Madrid brought the confiscated articles to the Crime Laboratory at Camp Florendo, San Fernando, La Union for examination. Appellant was also brought there for a drug test. [12]

Police Superintendent Ma. Theresa Ann Bugayong Cid, forensic chemist and head of the PNP Crime Laboratory, Regional Office I, Camp Florendo, Parian, San Fernando City, La Union, testified that on February 2, 1999, she

received from the Chief of Police of Lingayen, Pangasinan, a request for a drug test on the person of appellant Benhur Mamaril and a laboratory examination of the confiscated specimens. After weighing the specimens and testing the same, Police Superintendent Cid issued a report finding the specimens to be POSITIVE to the test for the presence of marijuana x x x. [13]

[14]

[15]

[16]

[17]

Moreover, Police Superintendent Cid affirmed the findings in her report that the examination conducted on the urine sample of appellant was positive for the presence of methamphetamine hydrochloride known as shabu. [18]

[19]

After the prosecution formally offered its testimonial and documentary exhibits on March 5, 2000, appellant, through his counsel, filed a motion with memorandum contending that: (1) the exhibits of the prosecution are inadmissible in evidence under Section 2 and Section 3 (2) of Article III (Bill of Rights) of the 1987 Constitution as the search warrant, by virtue of which said exhibits were seized, was illegally issued, considering that the judges examination of the complainant and his two witnesses was not in writing; and (2) said search warrant was illegally or improperly implemented. Appellant prayed that all the exhibits of the prosecution be excluded as evidence or in the alternative, that the resolution of the admissibility of the same be deferred until such time that he has completed the presentation of his evidence in chief. On August 25, 2000, the prosecution opposed the motion, and the trial court denied appellants motion. [20]

[21]

The Defenses Evidence Appellant Benhur Mamaril, 31, single, laborer, denied that he was residing at his parents house at Ramos Street, Lingayen, Pangasinan since he has been residing at a rented house at Barangay Matic-matic, Sta. Barabara, Pangasinan since December 18, 1998. Appellant declared that on February 1, 1999, it was his brother and the latters family who were residing with his mother at Ramos Street, but on said day, his brother and family were not in the house since they were at the fishpond. [22]

Appellant testified that on February 1, 1999, he was at his parents house at Ramos Street, Lingayen, Pangasinan, because he and his live-in partner visited his mother on said day and arrived there at 10:00 a.m. At about 2:00 p.m. of February 1, 1999, while appellant was at the back of his parents house, about seven to nine policemen, in civilian clothes, arrived. The policemen asked appellant to go upstairs and they immediately handcuffed him and brought him to the balcony of the house. He stayed at the balcony

until the search was finished after more than 30 minutes. Thereafter, he was brought to the clinic of one Dr. Felix and a medical examination was conducted on him. Then he was brought to the municipal hall. [23]

Appellant testified that he saw the buri bag, the eye-glass case, and the gray and white bag containing suspected marijuana for the first time on the day of the search when he was at the balcony of their house. He also testified that he saw the Receipt of Property Seized for the first time while he was testifying in court. He admitted that the signature on the certification that the house was properly searched was his. [24]

Moreover, appellant testified that in the early morning of February 2, 1999, he was brought to the PNP Crime Laboratory in San Fernando, La Union where he gave his urine sample. Appellant insinuated that the confiscated items were only planted because he had a misunderstanding with some policemen in Lingayen. However, he admitted that the policemen who searched his parents house did not threaten or harm him in any way and he had no misunderstanding with SPO3 Alfredo Rico. [25]

Atty. Enrico O. Castillo, Branch Clerk of Court, RTC-Branch 39, Lingayen, Pangasinan, was requested to testify on the available records regarding Search Warrant No. 99-51 on file in the trial court and to identify said documents. Atty. Castillo testified that he only had with him the application for search warrant, the supporting affidavits of PO3 Alberto Santiago and Diosdado Fernandez and the return of the search warrant. [26]

Atty. Enrico declared that before he assumed office as Branch Clerk of Court, the person supposed to be in custody of any transcript of the searching questions and answers made by Executive Judge Eugenio G. Ramos in connection with the application for Search Warrant No. 99-51 was Mrs. Liberata Ariston, who was then a legal researcher and at the same time OICBranch Clerk of Court. However, during the trial of this case, Mrs. Liberata Ariston was in the United States of America. Atty. Enrico averred that he asked Mrs. Liberata Aristons daughter, Catherine Ramirez, who is a court stenographer, about said transcript, but it has not been found. Atty. Enrico testified that based on the records, there is no stenographic notes. He added that they tried their best to locate the subject transcript, but they could not find it. [27]

The Trial Courts Decision

On January 23, 2001, the trial court rendered a decision, the dispositive portion of which reads: WHEREFORE, the prosecution having established beyond reasonable doubt the guilt of the accused of the crime of possession of marijuana defined and penalized under Section 8 of RA 6425, as amended, this Court in the absence of any modifying circumstances, hereby sentences said accused to suffer the penalty of RECLUSION PERPETUA and to pay a fine of Five Hundred Thousand Pesos (P500,000), plus costs of this suit. The period of preventive imprisonment suffered by the accused shall be credited in full in service of his sentence in accordance with Article 29 of the Revised Penal Code. SO ORDERED.

[28]

The Appeal Appellant contends that the trial court made the following errors: I

THE TRIAL COURT ERRED IN NOT DECLARING AS INADMISSIBLE IN EVIDENCE THE ARTICLES ALLEGEDLY SEIZED FROM ACCUSEDAPPELLANT CONSIDERING THAT SEARCH WARRANT NO. 99-51 WAS ILLEGALLY ISSUED. II

THE TRIAL COURT LIKEWISE ERRED IN NOT DECLARING AS TOTALLY INADMISSIBLE THE INVENTORIED ARTICLES IN THE RECEIPT OF SEIZED PROPERTY AND THE CORRESPONDING CERTIFICATION ISSUED THERETO (EXHS. J AND I) SINCE THE ACCUSED-APPELLANT WAS NOT ASSISTED BY COUNSEL WHEN HE SIGNED THE SAME. III

THE TRIAL COURT ERRED IN CONVICTING ACCUSED-APPELLANT OF THE CRIME CHARGED DESPITE THE FACT THAT HIS GUILT WAS NOT PROVEN BEYOND REASONABLE DOUBT. [29]

Appellant prays for his acquittal on the ground that Search Warrant No. 99-51 was illegally issued considering that there was no evidence showing that the required searching questions and answers were made anent the application for said search warrant. Appellant pointed out that Branch Clerk of Court Enrico O. Castillo testified that based on the records, there was no transcript of stenographic notes of the proceedings in connection with the application for said search warrant. Appellant thus asserts that it cannot be said that the judge made searching questions upon the alleged applicant and his witnesses, which is in violation of Section 2, Article III of the Constitution and Section 5, Rule 126 of the Rules of Court. Our Ruling Appellants contention is meritorious. The right against unreasonable searches and seizures is guaranteed under Article III, Section 2, of the Constitution, thus: Sec. 2. The right of the people to be secure in their persons, houses, papers and effects against unreasonable searches and seizures of whatever nature and for any purpose shall be inviolable, and no search warrant or warrant of arrest shall issue except upon probable cause to be determined personally by the judge after examination under oath or affirmation of the complainant and the witnesses he may produce, and particularly describing the place to be searched and the persons or things to be seized. Said Constitutional provision is implemented under Rule 126 of the Rules of Court, thus: Sec. 4. Requisites for issuing search warrant. A search warrant shall not issue except upon probable cause in connection with one specific offense to be determined personally by the judge after examination under oath or affirmation of the complainant and the witnesses he may produce, and particularly describing the place to be searched and the things to be seized which may be anywhere in the Philippines. Sec. 5. Examination of complainant; record. -- The judge must, before issuing the warrant, personally examine in the form of searching questions and answers, in writing and under oath, the complainant and the witnesses he may produce on facts personally known to them and attach to the record their sworn statements, together with the affidavits submitted.

Under the above provisions, the issuance of a search warrant is justified only upon a finding of probable cause. Probable cause for a search has been defined as such facts and circumstances which wouldlead a reasonably discreet and prudent man to believe that an offense has been committed and that the objects sought in connection with the offense are in the place sought to be searched. In determining the existence of probable cause, it is required that: (1) the judge must examine the complainant and his witnesses personally; (2) the examination must be under oath; and (3) the examination must be reduced in writing in the form of searching questions and answers. [30]

[31]

Atty. Enrico O. Castillo, Branch Clerk of Court, RTC-Branch 39 of Lingayen, Pangasinan, who was requested to testify on the available records kept in their office regarding Search Warrant No. 99-51, presented before the court only the application for search warrant and the supporting affidavits of PO3 Alberto Santiago and Diosdado Fernandez. Atty. Castillo could not produce the sworn statements of the complainant and his witnesses showing that the judge examined them in the form of searching questions and answers in writing as required by law. Atty. Castillo testified, thus: [32]

[33]

xxx xxx xxx Q Would you admit that from the records available there is no transcript of the proceedings of a searching questions and answers made by the Executive Judge upon the complainant as well as the two (2) witnesses not only in connection with application for Search Warrant 99-51 but in all of those application covered by that record namely, 99-49, 99-50, 99-51, 99-52, 99-53 and 99-54? A Sir, based on the records there is no transcript of [s]tenographic notes. Q Did you not ask Catherine Ramirez, the daughter of then OIC-Mrs. Liberata Ariston about said transcript? A I asked her for several times, sir, and in fact I asked her again yesterday and she told me that she will try to find on (sic) the said transcript. Q But until now there is no transcript yet? A Yes, sir. Q Because according to the rules the transcript must be attached to the records of this case together with the application for search warrant as well as the supporting affidavit of the said application, but there is no records available to have it with you and there is no proof with you? A Because during the time I assumed the office, sir, the records in the store room which they placed is topsy turvy and all the records are scattered. So, we are having a hard time in scanning the records, sir. Q But did you not try your very best assisted by the Court personnel to locate said transcript, Mr. Witness?

A Sir, we tried our best but based on the transcript I can not just read the said transcript. Q You mean to say you were able to [find] the stenographic notes? A No, sir. There are stenographic notes but they are not yet transcribed, sir. Q That is by a machine steno? A Yes, sir. Q Did you not ask the assistance of the co-stenographers in your sala who are using the machine steno to identify what cases does that stenographic notes (sic)? A Sir, I was assisted by some stenographers but we can (sic) not find the transcript of stenographic notes concerning Search Warrant No. 99-49 to 9954.[34] (Underscoring ours)

Based on the above testimony and the other evidence on record, the prosecution failed to prove that Executive Judge Eugenio G. Ramos put into writing his examination of the applicant and his witnesses in the form of searching questions and answers before issuance of the search warrant. The records only show the existence of an application for a search warrant and the affidavits of the complainants witnesses. In Mata v. Bayona, we held: [35]

[36]

[37]

Mere affidavits of the complainant and his witnesses are thus not sufficient. The examining Judge has to take depositions in writing of the complainant and the witnesses he may produce and to attach them to the record. Such written deposition is necessary in order that the Judge may be able to properly determine the existence or non-existence of the probable cause, to hold liable for perjury the person giving it if it will be found later that his declarations are false. We, therefore, hold that the search warrant is tainted with illegality by the failure of the Judge to conform with the essential requisites of taking the depositions in writing and attaching them to the record, rendering the search warrant invalid. We cannot give credit to the argument of the Solicitor General that the issuing judge examined under oath, in the form of searching questions and answers, the applicant SPO2 Chito S. Esmenda and his witnesses on January 25, 1999 as it is so stated in Search Warrant No. 99-51. Although it is possible that Judge Ramos examined the complainant and his witnesses in the form of searching questions and answers, the fact remains that there is no evidence that the examination was put into writing as required by law. Otherwise, the depositions in writing of the complainant and his witnesses would have been attached to the record, together with the affidavits that the witnesses submitted, as required by Section 5, Rule 126 of the Rules of Court. Consequently, we find untenable the assertion of the Solicitor General that the

subject stenographic notes could not be found at the time Branch Clerk of Court Enrico Castillo testified before the trial court because of the confused state of the records in the latters branch when he assumed office. The Solicitor General also argues that appellant is deemed to have waived his right to question the legality of the search because he did not protest against it, and even admitted during his testimony that he was neither threatened nor maltreated by the policemen who searched their residence. We disagree. The cases cited by the Solicitor General involved a warrantless search. In this case, the police authorities presented a search warrant to appellant before his residence was searched. At that time, appellant could not determine if the search warrant was issued in accordance with the law. It was only during the trial of this case that appellant, through his counsel, had reason to believe that the search warrant was illegally issued causing appellant to file a motion with memorandum objecting to the admissibility of the evidence formally offered by the prosecution. In People v. Burgos, we ruled: [38]

[39]

Neither can it be presumed that there was a waiver, or that consent was given by the accused to be searched simply because he failed to object. To constitute a waiver, it must appear first that the right exists; secondly, that the person involved had knowledge, actual or constructive, of the existence of such a right; and lastly, that said person had an actual intention to relinquish the right. (Pasion Vda. de Garcia v. Locsin, 65 Phil. 689). The fact that the accused failed to object to the entry into his house does not amount to a permission to make a search therein (Magoncia v. Palacio, 80 Phil. 770). As pointed out by Justice Laurel in the case of Pasion Vda. de Garcia v. Locsin (supra): xxxxxxxxx x x x As the constitutional guaranty is not dependent upon any affirmative act of the citizen, the courts do not place the citizen in the position of either contesting an officers authority by force, or waiving his constitutional rights; but instead they hold that a peaceful submission to a search or seizure is not a consent, or an invitation thereto, but is merely a demonstration of regard for the supremacy of the law. (56 C.J., pp.1180, 1181). We apply the rule that: courts indulge every reasonable presumption against waiver of fundamental constitutional rights and that we do not presume acquiescence in the loss of fundamental rights. (Johnson v. Zerbst, 304 U.S. 458).

In this case, we construe the silence of appellant at the time the policemen showed him the search warrant as a demonstration of regard for the supremacy of the law. Moreover, appellant seasonably objected on constitutional grounds to the admissibility of the evidence seized pursuant to said warrant during the trial of the case, after the prosecution formally offered its evidence. Under the circumstances, no intent to waive his rights can reasonably be inferred from his conduct before or during the trial. [40]

[41]

[42]

No matter how incriminating the articles taken from the appellant may be, their seizure cannot validate an invalid warrant. In Mata v. Bayona, we ruled: [43]

[44]

.[N]othing can justify the issuance of the search warrant but the fulfillment of the legal requisites. It might be well to point out what has been said in Asian Surety & Insurance Co., Inc. vs. Herrera: It has been said that of all the rights of a citizen, few are of greater importance or more essential to his peace and happiness than the right of personal security, and that involves the exemption of his private affairs, books and papers from inspection and scrutiny of others. While the power to search and seize is necessary to the public welfare, still it must be exercised and the law enforced without transgressing the constitutional rights of the citizens, for the enforcement of no statute is of sufficient importance to justify indifference to the basic principles of government. Thus, in issuing a search warrant the Judge must strictly comply with the requirements of the Constitution and the statutory provisions. A liberal construction should be given in favor of the individual to prevent stealthy encroachment upon, or gradual depreciation of the rights secured by the Constitution. No presumption of regularity are to be invoked in aid of the process when an officer undertakes to justify it. We, therefore, find that the requirement mandated by the law that the examination of the complainant and his witnesses must be under oath and reduced to writing in the form of searching questions and answers was not complied with, rendering the search warrant invalid. Consequently, the evidence seized pursuant to said illegal search warrant cannot be used in evidence against appellant in accordance with Section 3 (2), Article III of the Constitution. [45]

It is unnecessary to discuss the other issues raised by appellant in seeking to exclude the evidence seized pursuant to said illegal search warrant. Without the aforesaid illegally obtained evidence, there is no sufficient basis to sustain the conviction of appellant.

WHEREFORE, the decision of the Regional Trial Court of Lingayen, Pangasinan, Branch 39, in Criminal Case No. L-5963, is REVERSED and SET ASIDE. Judgment is hereby rendered declaring Search Warrant No. 99-51 NULL and VOID and the search and seizure made at appellants residence illegal. For lack of evidence to establish appellants guilt beyond reasonable doubt, appellant BENHUR MAMARIL is hereby ACQUITTED and ordered RELEASED from confinement unless he is being held for some other legal grounds. The Director of the Bureau of Corrections is ORDERED to IMPLEMENT without delay this Decision and to INFORM this Court, within ten (10) days from receipt hereof, of the date appellant was actually released from confinement. The confiscated marijuana is ORDERED forfeited in favor of the State and the trial court is hereby directed to deliver or cause its delivery to the Dangerous Drugs Board for proper disposition.

PEOPLE OF THE PHILIPPINES,

G.R. No. 174774

Plaintiff-Appellee, Present: CORONA, C.J., - versus -

Chairperson, LEONARDO-DE CASTRO, BERSAMIN, DEL CASTILLO, and

ROLANDO S. DELOS REYES, alias Botong, and RAYMUNDO G. REYES, alias Mac-Mac,

VILLARAMA, JR., JJ.

Promulgated:

Accused-Appellants. August 31, 2011

x- - - - - - - - - - - - - - - - - - - - - - - - - - - - - - - - - - - - - - - - - - - - - - - - - - -x

DECISION

LEONARDO-DE CASTRO, J.:

On appeal is the Decision[1] dated July 12, 2006 of the Court of Appeals in CA-G.R. CR.-H.C. No. 01733, which affirmed with modification the Decision[2] dated September 23, 2003 of Branch 214 of the Regional Trial Court (RTC) of Mandaluyong City in Criminal Case No. MC-00-2375-D. The Court of Appeals found accused-appellants Rolando S. delos Reyes and Raymundo G. Reyes (Reyes) guilty beyond reasonable doubt of violation of Section 21 of Article IV, in relation to Section 16 of Article III, of Republic Act No. 6425, otherwise known as the Dangerous Drugs Act of 1972, and imposing upon them the penalty of reclusion perpetua. The following antecedent facts are culled from the records:

On February 17, 2000, accused-appellants Rolando S. delos Reyes and Raymundo G. Reyes, Emmanuel de Claro, and Mary Jane Lantion-Tom (LantionTom) were all arrested for illegal possession, sale, delivery, distribution, and/or transportation of Methamphetamine Hydrochloride, a regulated drug commonly known as shabu. The Office of the City Prosecutor of Mandaluyong City, in its Resolution dated March 3, 2000, found probable cause to indict accusedappellants, together with Emmanuel de Claro, for violation of Republic Act No. 6425, and resolved to continue the preliminary investigation in so far as Lantion-

Tom was concerned. The criminal information against accused-appellants and Emmanuel de Claro, filed with the RTC, reads: The undersigned 2nd Asst. City Prosecutor accuses ROLANDO DELOS REYES y SANTOS @ BOTONG, RAYMUNDO REYES y GUINZON @ MACMAC and EMMANUEL DE CLARO y ENRIQUEZ @ COCOY of the crime of VIOLATION OF SEC. 21 ART. IV IN REL. TO SEC. 16 ART. III OF R.A. 6425 AS AMENDED, committed in the manner herein narrated as follows:

That on or about the 17th day of February, 2000, in the City of Mandaluyong, Philippines, and within the jurisdiction of this Honorable Court, the above-named accused, not being lawfully authorized to possess any regulated drug, conspiring and confederating together and mutually helping and aiding one another, commit to sell, deliver, distribute and/or transport a carton of ten (10) heat-sealed transparent plastic bags containing white crystalline substance with the following grams, to wit: 99.2, 94.9, 99.6, 93.5, 98.3, 99.5, 99.6, 99.5, 98.4 and 98.4 grams or a total of 980.9 grams, which substance when submitted for drug examination, were found positive to the test for Methamphetamine Hydrochloride, commonly known as shabu, a regulated drug, without the corresponding license and prescription.[3]

On March 7, 2000, accused-appellant Rolando delos Reyes, Emmanuel de Claro, and Lantion-Tom, insisting on their innocence, moved for a reinvestigation of their case before the RTC, which said trial court granted in an Order[4] dated March 15, 2000.

After the reinvestigation, the Office of the City Prosecutor issued a Resolution dated April 3, 2000, recommending that the RTC proceed with the indictment of accused-appellant Reyes and Emmanuel de Claro, and dismiss the charges against accused-appellant Rolando delos Reyes and Lantion-Tom. The Office of the City Prosecutor considered the different versions of events presented by the parties during the preliminary investigation and reinvestigation (except accusedappellant Reyes who did not participate in the proceedings), which it summarized as follows:

In their Joint Affidavit of Arrest, the arresting officers, members of the Intelligence and Investigation of the Regional Mobile Group (RMG) of the National Capital Region Police Office (NCRPO) claims that on 17 February 2000 a confidential informant called up relative to a narcotics drug deal to commence at the vicinity of the parking area of Shangrila Plaza Hotel, Mandaluyong City; that they were dispatched to verify the reports and conduct police operations; that about 2:00 p.m. after meeting with the confidential agent, they strategically positioned themselves at the vicinity parking area of said hotel; that about 10:00 p.m., accused/respondent Reyes a.k.a. Mac-Mac, on board a white Toyota Corolla, and accused/respondent [Rolando] delos Reyes, a.k.a. Botong, on board a red Toyota Corolla, arrived with accused/respondent Reyes subsequently proceeding inside Whistletop Bar and Restaurant, and accused/respondent [Rolando] delos Reyes calling accused/respondent [Emmanuel] de Claro through his cellular phone; that accused/respondent [Rolando] delos Reyes and [Emmanuel] de Claro then proceeded to the latters parked Mazda car where respondent Lantion-Tom was waiting; from the parked car, a box in transparent plastic bag was taken, which accused/respondent [Emmanuel] de Claro handed-over to accused/respondent [Rolando] delos Reyes; accused/respondent [Rolando] delos Reyes in turn handed the box in a plastic bag to accused/respondent Reyes; that the arresting officers accosted the accused/respondents who according to the arresting officers admitted having in their possession illegal drugs; that

the recovered items containing ten (10) pcs. of heat sealed transparent plastic bags of white crystalline substance with a total weight of 980.9 grams turned positive to the test for methylamphetamine hydrochloride or shabu, a regulated drug.

In his Sinumpaang Kontra-Salaysay, accused/respondent [Rolando] delos Reyes claims that on 17 February 2000, he went to Buenas Market, Manggahan, Pasig City, together with a neighbor, one Marlon David, to talk to Raymundo Reyes who was to pay his indebtedness; that while looking for a parking space, several men with firearms suddenly appeared, with one shouting, buksan mo ang pintuan ng sasakyan at kung hindi babasagin ko ito; that he and Marlon David were forced out of their vehicle with one of the armed men bringing out a plastic shopping bag of Shoe Mart, asking where the said bag allegedly containing shabu came from; that accused/respondent [Rolando] delos Reyes answered hindi ko alam, that he and Marlon David were blindfolded when forcibly taken to the groups vehicle and continuously asked who the source of the shabu was, with respondent/accused [Rolando] delos Reyes replying, hindi ko alam at wala akong kinalaman diyan; that Marlon David was separated from accused/respondent [Rolando] delos Reyes and later released on 18 February 2000; that when accused/respondent [Rolando] delos Reyes blindfold was removed, he found himself at Camp Bagong Diwa, Bicutan, Taguig, Metro Manila.

xxxx

To confirm respondent/accused [Rolando] delos Reyes claim, that he was arrested in Brgy. Manggahan, Pasig City, and not in the vicinity of Whistletop Bar and Restaurant in Mandaluyong City, respondent/accused [Emmanuel] de Claros spouse submitted a certified

true xerox copy of barangay blotter of Barangay Manggahan, Pasig City, reflecting the entry on 19 February 2000 made by Mrs. Delos Reyes, on the incident reported to by Marlon David thus:

BLOTTER

Dumulog po rito sa himpilan ng Punong Barangay si Gng. Virginia Delos Reyes, upang ipagbigay alam ang pagkawala ng kanyang asawa na si Mr. Rolando delos Reyes, nuong petsa 17 ng Pebrero taong dalawang libo (2000) na ayon sa batang pamangkin na si Marlon David, ay hinuli ng mga hindi kilalang lalaki sa Buenas Market, Manggahan, Pasig City nais niyang alamin kung ang nasabing insidente ay coordinated dito sa himpilan o tanggapan ng Barangay.

(Sgd) Virginia delos Reyes Nagpapahayag

The blotter was apparently made after Marlon David informed Mrs. [Virginia] Delos Reyes of the incident upon his release on 18 February 2000. Another witness, one Joel Navarro, claims having seen the actual incident confirming the events as narrated to by accused/respondent [Rolando] delos Reyes and Marlon David.

Accused/respondent [Emmanuel] de Claro and his common law wife, respondent Lantion-Tom, submitted their separate Counter-Affidavits jointly denying the charges and claiming that they were at the Whistlestop Bar and Restaurant to talk to respondent Lantion-Toms

accountant Ms. Daisy Milan regarding the Mayors Permit, Business Location Clearance issued by the Office of the Barangay Captain, insurance documents, BIR Certificate of Registration of her business; that they were with accused/respondent [Emmanuel] de Claros brother, Roberto and a friend, James, with the two remaining outside the restaurant; that respondent Lantion-Tom went to accompany Ms. Milan, while accused/respondent [Emmanuel] de Claro was left inside; that after Ms. Milan left, respondent Lantion-Tom was suddenly surrounded by men who introduced themselves as police officers and were arresting them for being the source of shabu in a drug deal; that all of them, accused/respondent [Emmanuel] de Claro, Roberto and James were likewise arrested and continuously questioned on their complicity in the drug deal; that they were taken to Camp Bagong Diwa, Taguig, Metro Manila and subjected to further investigation; that Roberto and James were released the following day. Both respondents maintain that the allegations of the arresting officers as to the circumstances on the alleged drug deal leading to their arrest are unfounded and purely fabricated.

During the preliminary investigation proceedings on 21 March 2000, the arresting officers manifested that they are going to submit replyaffidavit on 29 March 2000. However, no such reply-affidavit was submitted.[5]

The Office of the City Prosecutor pointed out that the arresting police officers failed to refute accused-appellant Rolando delos Reyes counter-allegation that he was not arrested at Shangri-La Plaza in Mandaluyong City, but he was illegally arrested without warrant at Buenas Market in Cainta, Rizal, as corroborated by Marlon David and Joel Navarro (Navarro) in their respective sworn statements (Sinumpaang Salaysay) dated March 14, 2000. The Office of the

City Prosecutor also observed that Lantion-Tom was merely in the company of the other respondents without performing any overt act showing her to be part of the illicit transaction and her drug test revealed negative results. On the other hand, it considered the conflicting claims of Emmanuel de Claro (i.e., that he was illegally arrested and that the drug deal was a mere fabrication) and the arresting officers (i.e., that Emmanuel de Claro was the seller/pusher in the drug deal and the shabu was seized from his vehicle) would be best ventilated during the trial on the merits.

In accordance with the foregoing resolution, the prosecution filed with the RTC a motion with leave of court to admit amended information.

In its Order[6] dated April 4, 2000, the RTC denied the prosecutions motion. Contrary to the finding of the Office of the City Prosecutor, the RTC adjudged that probable cause exists not only against accused-appellant Reyes and Emmanuel de Claro, but accused-appellant Rolando delos Reyes as well.

Accused-appellants were arraigned on May 23, 2000,[7] while Emmanuel de Claro was arraigned on July 12, 2000.[8] All three pleaded not guilty. After the pre-trial conference, trial ensued. The prosecution presented in evidence the testimonies of Police Officer (PO) 3 Virgilio Santiago,[9] Senior Police Officer (SPO) 1 Eraldo Lectura,[10] PO3 Angel Yumul,[11] and SPO1 Benjamin David,[12] members of the Regional Mobile Group (RMG) of the Philippine National Police (PNP) National Capital Regional Police Office (NCRPO) who apprehended and/or investigated the case against accusedappellants, Emmanuel de Claro, and Lantion-Tom; and P/Insp. Benjamin Cruto, Jr.[13] (Cruto), the forensic chemist of the PNP Crime Laboratory.

PO3 Santiago was one of the police officers who arrested Emmanuel de Claro and Lantion-Tom on February 17. 2000. He testified that at around 10:30 a.m., their operation chief, Major Arnold Aguilar, received information from a

confidential informant regarding an illegal drug deal that would take place between Botong and Mac-Mac at the parking lot of Shangri-La Plaza in Madaluyong City. Botong and Mac-Mac were identified during the investigation as accused-appellants Rolando delos Reyes and Reyes, respectively.

As narrated by PO3 Santiago, a team to bust the illegal drug deal was organized by Major Aguilar, composed of PO3 Santiago himself, SPO1 Lectura, and PO3 Yumul, along with PO3 Elmer Corbe, PO3 Marcelo Arcancia, Jr., PO3 Randy Fuentes, PO3 Dennis Padpad, and PO3 Edwin dela Cruz. At around 1:00 p.m. of the same day, the police team was dispatched, using four vehicles, to the location of the drug deal and upon arrival, they waited for the confidential informant to arrive. When the confidential informant arrived at around 3:30 p.m., he told the police team that the drug deal would possibly take place between 6:00 p.m. and 11:00 p.m., and that the suspects would utilize a red Toyota Corolla with plate number TRP-868 and a white Toyota Corolla with plate number ULF706. The police team then positioned their cars strategically in such a way that they could see the vehicles coming from St. Francis Street and EDSA.

PO3 Santiago further recounted that at around 10:00 p.m., the suspected vehicles arrived, both stopping along the driveway of Shangri-La Plaza. The drivers of the vehicles alighted and talked to each other. The confidential informant recognized the driver of the white Toyota car as Mac-Mac and the driver of the red Toyota car as Botong. After a few minutes, Botong made a call on his cellular phone and then proceeded inside Whistle Stop Restaurant, leaving MacMac behind. Inside the restaurant, Botong talked to another person, who was identified during the investigation as Emmanuel de Claro alias Cocoy. PO3 Santiago was about three to five meters away. Thereafter, Botong and Cocoy went out of the restaurant and approached a car parked right outside. The person at the back seat of the car, later on identified as Lantion-Tom, handed to Cocoy a white plastic bag containing a box. Cocoy gave the bag to Botong, who, in turn, handed the same bag to Mac-Mac. In the meantime, Cocoy went back inside the restaurant.

PO3 Santiago related that their team leader sensed that the drug deal had already been consummated, so the police team immediately effected the arrest of the suspected drug dealers. PO3 Santiago and PO3 Yumul arrested Cocoy and Lantion-Tom, while SPO1 Lectura and the remaining police team members arrested Botong and Mac-Mac. The plastic bag containing the box was seized from Mac-Mac. The arrested suspects were brought to the police office for investigation. The plastic bag, the box, and the 10 heat-sealed sachets of white crystalline substance inside the box, were marked for identification and physical examination at the police office.

According to PO3 Santiago, the physical examination of the contents of each of the 10 heat-sealed sachets yielded positive test results for methamphetamine hydrochloride or shabu. PO3 Santiago then signed a Joint Affidavit of Arrest dated February 18, 2000 together with the other arresting police officers, namely, SPO1 Lectura, PO3 Corbe, PO3 Arcancia, PO3 Fuentes, and PO3 Nelson Gene Javier.

On cross-examination, PO3 Santiago admitted that he did not actually see what was inside the plastic bag and that he did not even see Botong hand over such plastic bag to Mac-Mac. From PO3 Santiagos position, he could not conclude that the suspects were committing an illegal drug deal as he had no prior knowledge of the contents of the plastic bag, and that he and the other arresting officers just relied on the information relayed by the confidential informant. Also, the police team did not recover any money from the arrested suspects. The confidential informant merely informed the police the following morning that the money for the illegal drugs was already deposited in the bank. The police, however, failed to make further queries from the confidential informant about the bank.

SPO1 Lectura related that their office received a telephone call from a confidential informant about an illegal drug deal involving Cocoy, Botong, and Mac-Mac in the vicinity of Shangri-La Plaza in Mandaluyong City on February

17, 2000. SPO1 Lectura was designated as the leader of the team that will bust said illegal drug deal. After the briefing, SPO1 Lecturas team proceeded to the subject location.

The confidential informant arrived and met SPO1 Lecturas team at around 3:30 p.m. SPO1 Lectura conducted a short briefing then positioned his team strategically within the vicinity. The confidential informant told the police team that the drug deal would take place between 6:00 p.m. and 11:00 p.m. At around 10:00 p.m., the confidential informant identified the suspected drug dealers Botong and Mac-Mac, who were arriving in two cars. After conversing for a moment with Mac-Mac, Botong went inside Whistle Stop Restaurant to talk to Cocoy. Botong and Cocoy then went outside the restaurant and approached another car. Cocoy took a white plastic bag from the car, which he handed to Botong. Thereafter, Cocoy went back inside the restaurant, while [Botong] proceeded to his car near [Mac-Mac]. SPO1 Lectura was positioned at the other lane of the road, approximately 10 to 15 meters away from the suspects. At that moment, SPO1 Lectura sensed that the drug deal had been consummated, so he decided to already arrest the suspects. SPO1 Lectura arrested Mac-Mac, from whom he seized the white plastic bag. PO3 Yumul and PO3 Padpad arrestedBotong; and PO3 Santiago apprehended Cocoy. The police team brought the arrested suspects to the police office for investigation.

SPO1 Lectura submitted to SPO1 David the white plastic bag containing a box with 10 heat-sealed plastic sachets inside. In front of SPO1 Lectura, SPO1 David marked the said articles with his initials. After physical and chemical examinations revealed that the contents of the sachets were shabu, SPO1 Lectura signed the Joint Affidavit of Arrest dated February 18, 2000.

During cross-examination, SPO1 Lectura initially denied that Marlon David was with Botong when the latter was arrested, but he later admitted that the police also arrested Marlon David.Marlon David was brought to Camp Bagong Diwa, Taguig, together with the other arrested suspects, for verification, and was

released the following day. SPO1 Lectura also admitted that during the preliminary investigation, he and PO3 Corbe, PO3 Arcancia, and PO3 Javier, answered that it was PO3 Santiago who seized the shabu from Mac-Mac; but SPO1 Lectura explained that what the investigating prosecutor actually asked during preliminary investigation was who saw where the shabu came from and that he signed the minutes of the preliminary investigation without reading the same. SPO1 Lectura maintained that it was he who recovered the shabu from Mac-Mac. Lastly, SPO1 Lectura acknowledged that his team heavily relied on the information given by the confidential informant in identifying the suspects in the illegal drug deal, who were eventually arrested.

PO3 Yumul substantially narrated the same version of events as that of PO3 Santiago and SPO1 Lectura. On February 17, 2000, he was assigned at the Intelligence Investigation Division of the RMG based in Camp Bagong Diwa, Bicutan, Taguig. He was with SPO1 Lectura, PO3 Santiago, PO3 Fuentes, PO3 Padpad, and several other police officers at the vicinity of Shangri-La Plaza in Mandaluyong City, conducting surveillance operation regarding the tipped-off illegal drug deal. He was with SPO1 Lectura and PO3 Padpad in the car parked in front of Shangri-La Plaza, while PO3 Fuentes, PO3 Dela Cruz, and their confidential informant were in another car also parked along the driveway of Shangri-La Plaza. PO3 Santiago, PO3 Arcancia, and PO3 Corbe were in the car stationed in front of Whistle Stop Restaurant. PO3 Yumul could not recall where the other members of the team were located.

At around 10:00 p.m., the suspects Botong and Mac-Mac arrived in separate cars, stopping in front of Shangri-La Plaza. Botong and Mac-Mac alighted from their cars and talked to each other.At that time, PO3 Yumul was about five meters away from the two suspects. Moments later, Botong called someone on his cellular phone, and then went inside Whistle Stop Restaurant, leavingMacMac behind. PO3 Yumul followed Botong inside the restaurant and saw the latter talking to Cocoy. PO3 Yumul though did not hear the conversation between Botong and Cocoy. Afterwards,Botong and Cocoy went out of the restaurant and approached a parked car. From his position about three meters

away, PO3 Yumul saw the passenger at the back seat of the car, Lantion-Tom, opening the window and handing over a white plastic bag with carton inside to Cocoy, who, in turn, gave the plastic bag to Botong. Cocoy then returned inside the restaurant and [Botong] went back to [Mac-Mac]. PO3 Yumul followed Cocoy inside the restaurant. A few minutes later, PO3 Santiago also went inside the restaurant informing PO3 Yumul that they would be arresting Cocoy, and that Botong and Mac-Mac were already arrested outside the restaurant. PO3 Santiago, assisted by PO3 Yumul, approached Cocoy and arrested him. The police team proceeded to the police office with all the arrested suspects for further investigation. PO3 Yumul, however, failed to join the other arresting officers in signing the Joint Affidavit of Arrest dated February 18, 2000.

SPO1 David was an investigator at the Intelligence and Investigation Section of the RMG at Camp Bagong Diwa, Bicutan, Taguig, assigned to the instant case following the arrests of accused-appellants, Emmanuel de Claro and LantionTom. He also referred the case for inquest to the Office of the City Prosecutor.

SPO1 David testified that on February 17, 2000, he received from SPO1 Lectura a plastic bag containing a box with 10 heat-sealed sachets of suspected shabu inside. SPO1 Lectura told SPO1 David that the articles were seized from the suspected drug dealers. SPO1 David marked his initials BSD on the confiscated articles, then prepared a request to the PNP Crime Laboratory for examination of the specimens. SPO1 David disclosed that he prepared the Affidavit of Arrest of the arresting officers.

The last witness for the prosecution was P/Insp. Cruto of the PNP Crime Laboratory. P/Insp. Cruto was the forensic chemist who conducted the physical, chemical, and confirmatory examinations of the contents of the 10 heat-sealed plastic sachets submitted by the RMG-NCRPO on February 18, 2000.

P/Insp. Cruto conducted the physical examination by weighing the contents of each sachet, revealing that two sachets weighed 99.6 grams each; two sachets, 99.5 grams each; one sachet, 99.2 grams; two sachets, 98.4 grams each; one sachet, 98.3 grams; one sachet, 94.9 grams; and one sachet, 93.5 grams. P/Insp. Cruto then took a representative sample from each plastic sachet and proceeded with his chemical and confirmatory examinations. The contents of the 10 heatsealed plastic sachets all tested positive for methamphetamine hydrochloride, otherwise known as shabu.P/Insp. Cruto recorded the result of the examinations in his Physical Sciences Report No. D-097-2000.[14]

The prosecution submitted the following object and documentary evidence: the Joint Affidavit of Arrest[15] dated February 18, 2000 signed by SPO1 Lectura, PO3 Santiago, PO3 Corbe, PO3 Arcancia, PO3 Dela Cruz and PO3 Javier; the Sketch prepared in open court by SPO1 Lectura;[16] the 10 heat-sealed plastic sachets recovered from the possession of accused-appellants;[17] the PNP-RMG Request for Laboratory Examination of the contents of the 10 heat-sealed plastic sachets;[18] the PNP Crime Laboratory Physical Sciences Report No. D-097-2000 dated February 18, 2000 which revealed that the contents of the 10 heat-sealed plastic sachets positively tested for methamphetamine hydrochloride;[19] and the Letter (Referral of the case to the Office of the City Prosecutor)[20] dated February 18, 2000. The RTC admitted all the aforementioned evidence for the prosecution in its Order[21] dated March 1, 2001.

The defense, on the other hand, presented the testimonies of Marlon David,[22] accused-appellant Rolando delos Reyes,[23] Emmanuel de [24] [25] [26] Claro, Roberto de Claro, and Mary Jane Lantion-Tom. Accused-appellant Reyes did not testify. Marlon David was 17 years old and a fourth year high school student of Rizal High School in Pasig City. He recalled that on February 17, 2000, at about 1:00 p.m., he accompanied accused-appellant Rolando delos Reyes, whom he referred to as Kuya Botong, to the Buenas Market in Cainta, Rizal, to collect some money.

While accused-appellant Rolando delos Reyes and Marlon David were inside their car at the parking area of said market, another car suddenly arrived, from which an armed male passenger alighted and approached them. Four other armed men followed and poked their guns at accused-appellant Rolando delos Reyes and Marlon David. The armed men, in civilian attire, were carrying an SM plastic shopping bag and questioned accused-appellant Rolando delos Reyes if he knew the owner of said plastic bag. Accused-appellant Rolando delos Reyes denied any knowledge about the plastic bag. Marlon David was also asked and he answered that he knew nothing about the plastic bag.

Thereafter, the armed men, who later introduced themselves as police officers, pulled accused-appellant Rolando delos Reyes from the driver seat of the latters car, transferred him and Marlon David to the back seat of said car, and blindfolded both of them. Two of the armed men sat in the front seats of the car, while one of them sat at the back, beside accused-appellant Rolando delos Reyes and Marlon David. The armed men drove the car around (paikot-ikot). The armed men then separated accused-appellant Rolando delos Reyes from Marlon David. They ordered Marlon David to alight from the car and transfer to another vehicle. While in the other car, the armed men boxed and mauled Marlon David to force him to admit to be the source of the plastic bag. Each question was accompanied with one punch. Marlon David remained blindfolded until they arrived at the police camp in Bicutan, Taguig, where he again saw accusedappellant Rolando delos Reyes. Marlon David was released the following morning, leaving accused-appellant Rolando delos Reyes behind at the police camp. Marlon David went home and told Virginia delos Reyes, the wife of accused-appellant Rolando delos Reyes, about the incident.

Marlon David, during his cross examination, denied knowing any person with the name Mac-Mac. Marlon David additionally relayed that he was told by accused-appellant Rolando delos Reyes that the latter was likewise mauled by the armed men.

Accused-appellant Rolando delos Reyes or Botong gave a similar account of the incident that took place at 1:00 p.m. on February 17, 2000, while he and Marlon David were at the Buenas Market in Cainta, Rizal. Their car was surrounded by four armed men. The armed men poked their guns at him and Marlon David, shouting at them to open the car doors. He lowered the car window and the armed men opened the car door. The armed men forced him and Marlon David to get down from the front seats of the car and to transfer to the back seat, blindfolded them, and asked them who were the owners of the SM plastic bag. After they left Buenas Market, he noticed that they were just driving around. The car stopped only when Marlon David was taken out and transferred to another car. It was already late in the evening when the car finally stopped. He then realized, after his blindfold had been removed, that he was at Camp Bagong Diwa in Bicutan, Taguig.

Accused-appellant Rolando delos Reyes denied the accusation of the police that he was selling or delivering shabu to anyone. He asserted that he was not arrested at Whistle Stop restaurant in Mandaluyong City, rather, he was illegally arrested at Buenas Market in Cainta, Rizal. Accused-appellant Reyes or MacMac was his friend who owed him money. He and accused-appellant Reyes agreed to meet at Buenas Market for the settlement of the latters loan, but the meeting did not take place because the armed men arrived. He further claimed that he only met Emmanuel de Claro at Camp Bagong Diwa in Bicutan, Taguig. He never knew Emmanuel de Claro before that time, and he found out the latters name only when they were already detained at the Mandaluyong City Jail.

Emmanuel de Claro or Cocoy testified that on February 17, 2000 at around 10:00 a.m., he was at the Department of Trade and Industry in Buendia, Makati City, with his common-law wife Mary Jane Lantion-Tom to follow up their application for business permit. At around 1:00 p.m., they had lunch at Glorietta. Emmanuel de Claro was no longer feeling well so he and Lantion-Tom passed by the house of his brother Roberto de Claro to request the latter to drive for them. James, Roberto de Claros friend, also went with them.

The vehicle driven by Emmanuel de Claro was a rented car because his own car was in the auto shop. Emmanuel de Claro, Lantion-Tom, Roberto de Claro, and James first went to Las Pias City to check on Emmanuel de Claros car at the auto shop. From there, they proceeded to Libertad in Pasay City and ate dinner at the Duty Free Philippines. Afterwards, the group made their way to Mandaluyong City where Lantion-Tom had a scheduled appointment with Daisy Milan (Milan), her accountant. Emmanuel de Claro and Lantion-Tom met Milan at Whistle Stop Restaurant located at Shangri-La Plaza in Mandaluyong City. Milan and LantionTom discussed matters pertaining to the business permit. Emmanuel de Claro stepped outside the restaurant for a moment to smoke a cigarette, then, returned inside to wait for the meeting between Lantion-Tom and Milan to finish. After their meeting, Lantion-Tom walked Milan outside the restaurant, while Emmanuel de Claro waited for Lantion-Tom inside.

Three male persons suddenly approached Emmanuel de Claro and introduced themselves as police officers. They warned Emmanuel de Claro not to make a scene and just go with them peacefully. Emmanuel de Claro obeyed. He was brought outside the restaurant and was forced to get into a waiting car. For about three hours inside the car, he was punched, handcuffed, blindfolded, and told to bow down his head. He was likewise being forced to admit something about the shabu, but he denied knowing anything about it. He heard from the radio inside the car that the police officers were waiting for another car. After three hours of traveling, the car finally stopped and when his blindfold was removed, he learned that they were already at Camp Bagong Diwa in Bicutan, Taguig.

Emmanuel de Claro was placed in one room where he stayed for almost an hour, until he was called into another room where he met his co-accused for the first time. He later saw Lantion-Tom at the office of one of the police officers. They were interrogated by the police and being forced to admit that the drugs being shown to them belonged to them. They asked for a lawyer but their plea was ignored. The police told Emmanuel de Claro and Lantion-Tom that

somebody should be held responsible for the shabu so they were made to choose whether both of them or only one of them would be charged. Emmanuel de Claro was compelled to choose the latter option.

Roberto de Claro corroborated Emmanuel de Claros testimony. On February 17, 2000, Roberto de Claro was at home playing video games when his brother Emmanuel de Claro and the latters wife, Lantion-Tom, arrived and requested him to drive their car because Emmanuel was not feeling well. James, Roberto de Claros friend, rode with them. They first went to Las Pias City to check on Emmanuel de Claros car at the auto shop, then they proceeded to Libertad, Pasay City, where they had dinner at Duty Free Philippines. They next drove to Whistle Stop Restaurant at Shangri-La Plaza in Mandaluyong City to meet Ms. Milan. Only Emmanuel de Claro and Lantion-Tom went inside the restaurant. Roberto de Claro and James stayed in the car.

Two hours later, Roberto de Claro saw Lantion-Tom and Ms. Milan walking towards them. As the two women were approaching, armed men suddenly appeared, surrounded their car, and pointed guns at them. Roberto de Claro got terrified. It was as if an armed robbery (hold-up) was taking place. The armed men knocked at the car window. Out of fear, Roberto de Claro opened the window, then the door of the car. Roberto de Claro, James, and Lantion-Tom were made to sit at the back seat of the car. Two of the armed men sat on the front seats of the car, while one sat at the back with Roberto de Claro, James, and Lantion-Tom. The armed men introduced themselves as police officers.

Inside the car, the police officers mauled (siniko, sinuntok sa ulo) Roberto de Claro, James, and Lantion-Tom, all the while ordering them to keep their heads bowed down. The police officers drove the car for two hours, stopping at a gas station for about five minutes. At this moment, Roberto de Claro was able to raise his head but was immediately told to bow down his head again.Roberto de Claro also heard from the police officers radio that they were still waiting for somebody. They travelled again for quite a long time and stopped in a dark

place. The police officers took Roberto de Claros wallet containing P7,000.00 cash. Early in the following morning, they arrived at the police station where Roberto de Claro saw his brother Emmanuel de Claro once more. They stayed in one room until Roberto de Claro and James were released by the police the next day.

When Lantion-Tom was called to testify, the prosecution and the defense agreed to consider her Counter Affidavit dated March 23, 2000 and Supplemental Affidavit dated March 29, 2000 as her direct examination.

On cross-examination, Lantion-Tom confirmed that she was among those arrested on February 17, 2000 at the vicinity of Shangri-La Plaza in Mandaluyong City for her alleged involvement in an illegal drug deal. At the time of the arrest, she was with Emmanuel de Claro, Roberto de Claro, and James. She was also brought to Camp Bagong Diwa in Taguig where she was interrogated without a lawyer. She was shown a box containing shabu which she had never seen before. Lantion-Tom insisted that she was in Mandaluyong City to meet her accountant, Milan, regarding her application for a business permit. Lantion-Tom pointed out that the charge against her was eventually dismissed.

The documentary evidence for the defense consisted of Emmanuel de Claros Counter Affidavit dated March 23, 2000,[27] Lantion-Toms Counter Affidavit dated March 23, 2000,[28]Emmanuel de Claro and Lantion-Toms Supplemental Affidavit dated March 29, 2000,[29] Roberto de Claros Witness Affidavit dated March 29, 2000,[30] Marlon Davids Sinumpaang Salaysaydated March 14, 2000,[31] Virginia delos Reyes Sinumpaang Salaysay dated March 14, 2000,[32] Navarros Sinumpaang Salaysay dated March 14, 2000,[33] accusedappellant Rolando delos ReyesSinumpaang Kontra Salaysay dated March 14, 2000,[34] and a Barangay Blotter dated February 19, 2000 by Virginia delos Reyes.[35] The RTC admitted all these documentary evidence for the defense in its Order[36] dated September 13, 2002.

In its Decision dated September 23, 2003, the RTC found accusedappellants and Emmanuel de Claro guilty beyond reasonable doubt of the crime charged, and decreed: WHEREFORE, the prosecution having successfully proved the guilt of the accused beyond reasonable doubt for unlawfully possessing/selling, delivering, transporting and distributing methamphetamine hydrochloride otherwise known as shabu, a regulated drug, without lawful authority in violation of Sections 15 and 16 of Article III in relation to Section 21 of Article IV of R.A. No. 6425, as amended, they are hereby sentenced to suffer the penalty of LIFE IMPRISONMENT and to pay a fine of P20,000.00 each and the costs of suit. Further, all the methamphetamine hydrochloride (shabu) taken and seized from the accused during the aforesaid operation are forfeited and confiscated in favor of the government shall be turned over to the PDEA pursuant to law for proper disposal without delay.[37]

Emmanuel de Claro filed his notice of appeal[38] on October 23, 2003. Accusedappellants Roberto delos Reyes and Reyes each filed his notice of appeal[39] on October 29, 2003 and December 30, 2003, respectively. Emmanuel de Claro, however, subsequently moved to withdraw his notice of appeal,[40] instead, filing before the RTC an Omnibus Motion for Reconsideration and to Re-Open Proceedings Pursuant [to] Section 24, Rule 119 of the Rules of Court[41] on October 30, 2003, and a Supplemental Motion for Reconsideration[42] on November 3, 2003. Emmanuel de Claro asked the RTC to review its judgment of conviction based on the following grounds: I.

THE HONORABLE COURT GRAVELY ERRED IN RULING THAT THE ACCUSED DEFENSE OF FRAME-UP IS A MERE ALIBI AND HAS THUS ERRED IN ADOPTING THE THEORY OF THE PROSECUTION THAT ALL THE THREE (3) ACCUSED WERE PICKED-UP AT THE VICINITY OF EDSA SHANGRI-LA PLAZA HOTEL.

II.

THAT THE HONORABLE COURT GRAVELY ERRED IN RULING THAT THE WARRANTLESS ARREST WAS LAWFUL SINCE THE ACCUSED WERE CAUGHT IN FLAGRANTE DELICTO.

III.

THE HONORABLE COURT GRAVELY ERRED IN FINDING THAT THERE WAS CONSPIRACY AMONG THE THREE (3) ACCUSED IN THE ALLEGED COMMISSION OF THE CRIME OF UNLAWFUL SALE, DELIVERY AND TRANSPORTATION OF THE PROHIBITED DRUG.

IV.

THE HONORABLE COURT GRAVELY ERRED IN FINDING BOTH ACCUSED GUILTY BEYOND REASONABLE DOUBT OF THE CRIME CHARGED IN THE INFORMATION ON THE BASIS MAINLY OF A DISPUTABLE PRESUMPTION OF LACK OF IMPROPER MOTIVE ON THE PART OF THE POLICE OFFICERS.

V.

THAT THE HONORABLE COURT GRAVELY ERRED IN ITS FAILURE TO CONSIDER THE FACT THAT ACCUSED EMMANUEL DE CLARO WAS NOT AFFORDED HIS CONSTITUTIONAL RIGHTS DURING CUSTODIAL INVESTIGATION.[43]

Emmanuel de Claro principally contended that the accusation that he was engaging in an illegal drug deal, levied against him by prosecution witnesses SPO1 Lectura, PO3 Santiago, and PO3 Yumul was suspicious, if not incredible. Emmanuel de Claro pointed out that although these police officers

testified that Lantion-Tom, from the car, handed to him the plastic bag containing the box with sachets of shabu, the prosecution still dropped the criminal charges against Lantion-Tom. Emmanuel de Claro also strongly argued that the prosecution failed to contradict his well-supported alibi that he, his wife, and his brother went to Shangri-La Plaza in Mandaluyong City to meet his wifes accountant, so they could attend to several documents pertaining to a business permit.Emmanuel de Claro further insisted that the RTC should have highly regarded accused-appellant Rolando delos Reyes testimony which directly contradicted the police officers statements. In its Order[44] dated November 11, 2003, the RTC granted Emmanuel de Claros motion to withdraw his notice of appeal and required the prosecution to comment to his motions for reconsideration. The prosecution filed its Comment/Opposition[45] on December 19, 2003, objecting to Emmanuel de Claros motions for reconsideration and maintaining that its police-witnesses categorical, consistent, and straight-forward testimonies were sufficient to convict Emmanuel de Claro. In a complete turnabout from its previous findings and conclusion, the RTC, in its Order[46] dated January 12, 2004, acquitted Emmanuel de Claro of the crime charged. The RTC explicitly admitted that it erred in giving full faith and credit to the testimonies of prosecution witnesses SPO1 Lectura, PO3 Santiago, and PO3 Yumul, and in entirely rejecting the alibi of the defense. Thus, the RTC disposed: WHEREFORE, the motion of accused-movant Emmanuel De Claro is hereby GRANTED and a new one entered, ACQUITTING him of the crime charged. Consequently, his immediate release from detention is hereby ordered unless he is detained for other cause or causes.[47]

Nevertheless, in view of the pending notices of appeal of accusedappellants, the RTC forwarded the complete records of the case to us on March 29, 2004, and we gave due course to the said appeals in our Resolution[48] dated June 21, 2004. Accused-appellant Rolando delos Reyes filed his Appellants Brief[49] on September 15, 2004, while accused-appellant Reyes filed his Appellants Brief[50] on November 26, 2004. Pursuant to our pronouncement in People v. Mateo,[51] we transferred the case to the Court of Appeals for appropriate action

and disposition.[52] Accordingly, the plaintiff-appellee, represented by the Office of the Solicitor General (OSG), filed before the appellate court its Consolidated Brief[53] on January 21, 2005. The Court of Appeals, in its Decision dated July 12, 2006, sustained the conviction of accused-appellants, and merely modified the penalty imposed upon them, from life imprisonment toreclusion perpetua. According to the appellate court, the police officers testimonies deserve credence than accused-appellants defenses of denial and alibi, there being no evidence to rebut the presumption that the police officers regularly performed their official duties. The case was then elevated to us for final review. In our Resolution[54] dated January 31, 2007, we required the parties to submit their supplemental briefs. Plaintiff-appellee and accused-appellants Rolando delos Reyes and Reyes filed their manifestations[55] on March 14, 2007, April 10, 2007, and April 13, 2007, respectively, opting to stand by the briefs they had already filed before the Court of Appeals. In his Appellants Brief, accused-appellant Rolando delos Reyes assigned the following errors of the RTC: I.

II. III.

THE COURT A QUO ERRED IN FAILING TO RESOLVE THE CONTRADICTORY TESTIMONY AS TO THE PLACE OF THE ARREST IN FAVOR OF THE ACCUSED. THE COURT A QUO ERRED IN FINDING [THE] TESTIMONIES OF PO3 VIRGILIO SANTIAGO CREDIBLE. THE COURT A QUO ERRED IN FAILING TO APPRECIATE THE PROSECUTIONS EVIDENCE WHICH WAS PREVIOUSLY CATEGORIZE[D] AS WEAK WHEN THE COURT A QUO GRANTED BAIL TO THE ACCUSED.

[56]

Accused-appellant Reyes cited these errors in his Appellants Brief:

I.

THE TRIAL COURT ERRED IN NOT FINDING THE WARRANTLESS ARREST OF ACCUSED-APPELLANT RAYMUNDO REYES AS UNLAWFUL.

II.

ASSUMING ARGUENDO THAT THE WARRANTLESS ARREST WAS VALID, ACCUSED-APPELLANT RAYMUNDO REYES CANNOT BE CONVICTED FOR VIOLATION OF R.A. 6425.[57]

Accused-appellants essentially assert that the charge of illegal drug deal lodged against them by the police is a complete fabrication and frameup. Accused-appellants called attention to the material inconsistencies in the prosecutions evidence. PO3 Santiago testified during direct examination that accused-appellant Rolando delos Reyes handed the plastic bag with box inside to accused-appellant Reyes, but he admitted during cross-examination that he did not see such transfer. The prosecution was unable to present any evidence to prove the source of the plastic bag containing the box with sachets of shabu, and the money paid as consideration for the illegal drugs. The prosecution likewise failed to rebut accused-appellant Rolando delos Reyes straightforward, coherent, and truthful narration, corroborated by Marlon David, that he was illegally arrested at Buenas Market in Cainta, Rizal, and not at Shangri-la Plaza in Mandaluyong City.

Accused-appellants additionally argued that even the prosecutions version of the arrests of the suspects and seizure of the shabu shows that the same were effected in violation of accused-appellants fundamental rights. The arrests were executed without any warrant or any of the exceptional circumstances to justify a warrantless arrest. The suspects, including accused-appellants, were arrested without warrants based on a mere tip from a confidential informant and not because of any apparent criminal activity. A tip does not constitute probable cause for a warrantless arrest or search and seizure incidental thereto. Thus, the shabu allegedly seized from accused-appellants is inadmissible in evidence. Plaintiff-appellee, on the other hand, stand by the convictions of accusedappellants, maintaining that:

I.

THE POSITIVE AND CREDIBLE TESTIMONIES OF THE PROSECUTION WITNESSES HAVE ESTABLISHED THE GUILT OF APPELLANTS BEYOND REASONABLE DOUBT.

II.

THE WARRANTLESS ARREST CONDUCTED BY THE POLICE IS VALID SINCE IT FALLS SQUARELY UNDER RULE 113, SECTION 5(A) OF THE REVISED RULES ON CRIMINAL PROCEDURE.

III.

THE EVIDENCE PRESENTED BY THE PROSECUTION MORE THAN SUFFICE TO CONVICT APPELLANTS OF THE CRIME CHARGED.

IV.

CONSPIRACY ATTENDED THE COMMISSION OF THE OFFENSE.

V.

MERE DENIAL AND HULIDAP, WITHOUT MORE, CANNOT EXCULPATE APPELLANTS FROM CRIMINAL LIABILITY.

VI.

THE PRESUMPTION OF REGULARITY IN THE PERFORMANCE OF OFFICIAL DUTY UNDER SECTION 3(M) OF RULE 131 OF THE REVISED RULES OF COURT HAD NOT BEEN OVERCOME BY DEFENSE EVIDENCE.

VII.

CONCLUSION OF THE TRIAL JUDGE REGARDING THE CREDIBILITY OF WITNESSES COMMANDS GREAT RESPECT AND CONSIDERATION.[58]

Plaintiff-appellee avers that the inconsistencies in the police officers statements, as pointed out by accused-appellants, are trivial and do not affect the weight of their testimonies; while accused-appellants defenses of denial and frame-up could be easily concocted and, thus, should be looked upon with disfavor. Moreover, there is no need for proof of consideration for the illegal drug deal, since consideration is not an element of the crime charged.

Plaintiff-appellee avows that accused-appellants were caught while in the commission of a crime or in flagrante delicto, which justifies their warrantless arrests under Section 5(a), Rule 113 of the Rules of Court. Accused-appellants were arrested while in possession and in the act of distributing, without legal authority, a total of 980.9 grams of methamphetamine hydrochloride or shabu, on the night of February 17, 2000 at the parking area of Shangri-La Plaza in Mandaluyong City. In addition, in the absence of satisfactory proof to the contrary, the warrantless arrests executed by the police officers enjoy the presumption that official duty has been regularly performed.

We grant the appeal and reverse the assailed decision of the Court of Appeals.

At the outset, we observe that the prosecutors and the RTC both displayed uncertainty as to the facts surrounding accused-appellants arrest on the night of February 17, 2000.

The Office of the City Prosecutor of Mandaluyong City, after preliminary investigation and reinvestigation, recommended that the RTC drop accusedappellant Rolando delos Reyes and Lantion-Tom from the criminal charge. The RTC only partially adopted the recommendations of the Office of the City Prosecutor: dropping the criminal charge against Lantion-Tom, but still finding probable cause against accused-appellant Rolando delos Reyes.[59] Even after trial, the RTC wavered in its findings and conclusion. In its Decision[60] dated September 23, 2003, the RTC initially convicted accusedappellants and Emmanuel de Claro, but acting on Emmanuel de Claros motions for reconsideration, said trial court, in its Order[61] dated January 12, 2004, totally reversed itself and acquitted Emmanuel de Claro. This time, the RTC gave more weight to the evidence presented by the defense. The Court of Appeals, on appeal, refused to consider the subsequent acquittal of Emmanuel de Claro by the RTC. Instead, the appellate court upheld the earlier ruling of the RTC giving absolute credence to the testimonies of the prosecution witnesses and convicted accused-appellants of the crime charged. Despite the varying judgments of the RTC, the Court of Appeals speciously ratiocinated in its assailed decision that when the issue involves the credibility of a witness, the trial courts assessment is entitled to great weight.[62] Guided by the settled rule that where the inculpatory facts admit of several interpretations, one consistent with accused's innocence and another with his guilt, the evidence thus adduced fail[ed] to meet the test of moral certainty,[63] we find that the findings and conclusion of the RTC in its subsequent Order[64] dated January 12, 2004 (in which it acquitted Emmanuel de Claro) is more in keeping with the evidence on record in this case. It bears to stress that the very same evidence were presented against Emmanuel de Claro and accused-appellants; if the evidence is insufficient to convict the former, then it is also insufficient to convict the latter.

Indeed, the testimonies of prosecution witnesses SPO1 Lectura, PO3 Santiago, and PO3 Yumul are unreliable and suspiciously fabricated. In its Order dated January 12, 2004, the RTC correctly observed that: Viewed vis--vis the peculiar factual milieu of this case, not to say the insistence by the accused-movant [Emmanuel de Claro] that a reevaluation or reassessment of the evidence by the prosecution be considered, this court has decided to revisit the evidence put forward by the prosecution through the crucible of a severe testing by taking a more than casual consideration of every circumstance of the case. It is noted that the testimony given by the witnesses for the prosecution and that of the defense are diametrically opposed to each other. While this court had already made its conclusion that the testimonies of prosecution witnesses PO3 Santiago, SPO1 Lectura and PO3 Yumul are given full faith and credit and reject the frame-up and alibi story of the accused-movant [Emmanuel de Claro], nonetheless, upon reassessment of the same it appears that the court erred. In sum, the conveniently dovetailing accounts of the prosecution eyewitnesses, all of them police officers, with regard to the material facts of how the crime was allegedly committed engenders doubt as to their credibility. Firstly, the court noted that these police officers gave identical testimonies of the events that happened from the moment they arrived at 2 oclock in the afternoon until the arrest of the accused at 10:30 oclock in the evening at the EDSA ShangriLa premises. This uniform account given by these witnesses cannot but generate the suspicion that the material circumstances testified to by them were integral parts of a well thought-out and prefabricated story. Because of the close camaraderie of these witnesses who belong to the same police force it is not difficult for them to make the same story.Furthermore, their testimonies are so general which shows only too clearly that they testified uniformly only as to material facts but have not given the particulars and the details having relation with the principal facts. While they testified that they were at Shangri-La from 2 in the afternoon to 10 in the evening, they were not able to tell the court how their group positioned strategically at the premises without being noticed by their target. They could not also gave (sic) an explanation how their confidential informant was able to obtain information regarding the drug deal that was supposed to take place on that date involving several personalities. Except for their bare allegation that they have that information regarding the drug deal they were not able to present any proof of such report, say, entry in their logbook of such confidential report and a spot report. Even their operation is not recorded as no documentary evidence was presented. Worth remembering in this regard is People v. Alviar, 59 SCRA 136, where it is said that: . . . [i]t often happens with fabricated stories that minute particulars have not been thought of. It has also been said that an honest witness, who has sufficient memory to state one fact, and that fact a material one, cannot be safely relied upon as such weakness of memory not only leaves the case incomplete, but throws doubt upon the accuracy of the statements made. Such a witness may be honest, but his testimony is not reliable.[65] (Emphasis supplied.)

There are also material inconsistencies between the police-witnesses sworn statements following accused-appellants arrest and their testimonies before the RTC. The police officers attested in their Joint Affidavit of Arrest dated February 18, 2000 that upon sensing suspicious transactions being undertaken thereat, team leader thru hand signaled immediately accosted the suspects and introduced themselves as Police Officers and after that, subject persons deliberately admitted that they have in their possession illegal drugs and thereafter showed the same to the herein undersigned arresting officers thus they were placed under arrest.[66] Yet, during trial before the RTC, the police officers uniformly testified that they brought accused-appellants, Emmanuel de

Claro and Lantion-Tom to the police office after arresting the four suspects in flagrante delicto, without mention at all of the suspects purported admission. We also consider the fact that Lantion-Tom was never charged with any criminal involvement even when, according to the prosecutions version of events, she was the first person to deliver theshabu. This seriously dents the prosecutions sequence of events on the night of February 17, 2000. In contrast, accused-appellants presented clear and convincing evidence in support of their defenses, which the prosecution failed to rebut. Specifically, accused-appellant Rolando delos Reyes testified that he was illegally arrested without warrant at Buenas Market, Cainta, Rizal, not at Shangri-La Plaza in Mandaluyong City; and that he and Marlon David were coerced to incriminate themselves for possession of shabu. His claims were corroborated by Marlon Davids testimony and Navarros Sinumpaang Salaysay dated March 14, 2000. Also, Emmanuel de Claro, Lantion-Tom, and Roberto de Claro consistently testified that they were at Shangri-La Plaza to meet Milan, Lantion-Toms accountant, regarding documents for a business permit (photocopies of the said documents were presented during trial); and that they were illegally arrested without warrant and forced to admit criminal liability for possession of shabu. These pieces of evidence are overwhelmingly adequate to overthrow the presumption of regularity in the performance by the arresting police officers of their official duties and raise reasonable doubt in accused-appellants favor.

Furthermore, even assuming that the prosecutions version of the events that took place on the night of February 17, 2000 were true, it still failed to establish probable cause to justify the in flagrante delicto arrests of accusedappellants and search of accused-appellants persons, incidental to their arrests, resulting in the seizure of the shabu in accused-appellants possession.

Section 2, Article III of the Constitution provides:

Section 2. The right of the people to be secure in their persons, houses, papers, and effects against unreasonable searches and seizures of whatever nature and for any purpose shall be inviolable, and no search warrant or warrant of arrest shall issue except upon probable cause to be determined personally by the judge after examination under oath or affirmation of the complainant and the witnesses he may produce, and particularly describing the place to be searched and the persons or things to be seized.

Complementary to the above provision is the exclusionary rule enshrined in Section 3, paragraph 2 of Article III of the Constitution, which solidifies the protection against unreasonable searches and seizures, thus:

Section 3. (1) The privacy of communication and correspondence shall be inviolable except upon lawful order of the court, or when public safety or order requires otherwise as prescribed by law.

(2) Any evidence obtained in violation of this or the preceding section shall be inadmissible for any purpose in any proceeding. (Emphases supplied.)

The foregoing constitutional proscription is not without exceptions. Search and seizure may be made without a warrant and the evidence obtained therefrom may be admissible in the following instances: (1) search incident to a lawful arrest; (2) search of a moving motor vehicle; (3) search in violation of customs laws; (4) seizure of evidence in plain view; (5) when the accused himself

waives his right against unreasonable searches and seizures; and (6) stop and frisk situations.[67]

The first exception (search incidental to a lawful arrest) includes a valid warrantless search and seizure pursuant to an equally valid warrantless arrest which must precede the search. In this instance, the law requires that there be first a lawful arrest before a search can be made the process cannot be reversed. As a rule, an arrest is considered legitimate if effected with a valid warrant of arrest. The Rules of Court, however, recognizes permissible warrantless arrests. Thus, a peace officer or a private person may, without warrant, arrest a person: (a) when, in his presence, the person to be arrested has committed, is actually committing, or is attempting to commit an offense (arrest in flagrante delicto); (b) when an offense has just been committed and he has probable cause to believe based on personal knowledge of facts or circumstances that the person to be arrested has committed it (arrest effected in hot pursuit); and (c) when the person to be arrested is a prisoner who has escaped from a penal establishment or a place where he is serving final judgment or is temporarily confined while his case is pending, or has escaped while being transferred from one confinement to another (arrest of escaped prisoners).[68]

In People v. Molina,[69] we cited several cases involving in flagrante delicto arrests preceding the search and seizure that were held illegal, to wit: In People v. Chua Ho San, the Court held that in cases of in flagrante delicto arrests, a peace officer or a private person may, without a warrant, arrest a person when, in his presence, the person to be arrested has committed, is actually committing, or is attempting to commit an offense. The arresting officer, therefore, must have personal knowledge of such fact or, as recent case law adverts to, personal knowledge of facts or circumstances convincingly indicative or constitutive of probable cause. As discussed in People v. Doria, probable cause means an actual belief or reasonable grounds of suspicion. The grounds of suspicion are reasonable when, in the absence of actual

belief of the arresting officers, the suspicion that the person to be arrested is probably guilty of committing the offense, is based on actual facts, i.e., supported by circumstances sufficiently strong in themselves to create the probable cause of guilt of the person to be arrested. A reasonable suspicion therefore must be founded on probable cause, coupled with good faith on the part of the peace officers making the arrest.

As applied to in flagrante delicto arrests, it is settled that "reliable information" alone, absent any overt act indicative of a felonious enterprise in the presence and within the view of the arresting officers, are not sufficient to constitute probable cause that would justify an in flagrante delicto arrest. Thus, in People v. Aminnudin, it was held that "the accused-appellant was not, at the moment of his arrest, committing a crime nor was it shown that he was about to do so or that he had just done so. What he was doing was descending the gangplank of the M/V Wilcon 9 and there was no outward indication that called for his arrest. To all appearances, he was like any of the other passengers innocently disembarking from the vessel. It was only when the informer pointed to him as the carrier of the marijuana that he suddenly became suspect and so subject to apprehension."

Likewise, in People v. Mengote, the Court did not consider "eyes . . . darting from side to side . . . [while] holding . . . [one's] abdomen," in a crowded street at 11:30 in the morning, as overt acts and circumstances sufficient to arouse suspicion and indicative of probable cause. According to the Court, "[b]y no stretch of the imagination could it have been inferred from these acts that an offense had just been committed, or was actually being committed, or was at least being attempted in [the arresting officers'] presence." So also, in People v.

Encinada, the Court ruled that no probable cause is gleanable from the act of riding a motorela while holding two plastic baby chairs.

Then, too, in Malacat v. Court of Appeals, the trial court concluded that petitioner was attempting to commit a crime as he was "'standing at the corner of Plaza Miranda and Quezon Boulevard' with his eyes 'moving very fast' and 'looking at every person that come (sic) nearer (sic) to them."' In declaring the warrantless arrest therein illegal, the Court said:

Here, there could have been no valid in flagrante delicto ... arrest preceding the search in light of the lack of personal knowledge on the part of Yu, the arresting officer, or an overt physical act, on the part of petitioner, indicating that a crime had just been committed, was being committed or was going to be committed.

It went on to state that

Second, there was nothing in petitioner's behavior or conduct which could have reasonably elicited even mere suspicion other than that his eyes were "moving very fast" an observation which leaves us incredulous since Yu and his teammates were nowhere near petitioner and it was already 6:30 p.m., thus presumably dusk. Petitioner and his companions were merely standing at the corner and were not creating any commotion or trouble . . .

Third, there was at all no ground, probable or otherwise, to believe that petitioner was armed with a deadly weapon. None was visible to Yu, for as he admitted, the alleged grenade was "discovered" "inside the front waistline" of petitioner, and from all indications as to the distance between Yu and petitioner, any telltale bulge, assuming that petitioner was indeed hiding a grenade, could not have been visible to Yu.

Clearly, to constitute a valid in flagrante delicto arrest, two requisites must concur: (1) the person to be arrested must execute an overt act indicating that he has just committed, is actually committing, or is attempting to commit a crime; and (2) such overt act is done in the presence or within the view of the arresting officer.[70] (Emphases supplied.)

Similar to the above-cited cases in Molina, there is a dearth of evidence in this case to justify the in flagrante delicto arrests of accused-appellants and search of their persons incidental to the arrests.

A close examination of the testimonies of SPO1 Lectura, PO3 Santiago, and PO3 Yumul reveal that they simply relied on the information provided by their confidential informant that an illegal drug deal was to take place on the night of February 17, 2000 at Shangri-la Plaza in Mandaluyong City. Without any other independent information, and by simply seeing the suspects pass from one to another a white plastic bag with a box or carton inside, the police team was already able to conclude that the box contained shabu and sensed that an illegal drug deal took place.

SPO1 Lectura testified on direct examination as follows: Q: What was the information gathered by your informant? A: That there will be a drug deal between 6 to 11 in the evening, sir.

Q: You were there as early as 2:00 p.m.? A: Yes, sir.

Q: What did you do after briefing? A: We positioned ourselves strategically, we waited for the arrival of the subject, sir.

xxxx

Q: When you are already positioned in your respective area at the vicinity of Shangri-La Plaza, what happened next, if any? A: At around 10:00 p.m. two (2) cars arrived and they were identified by the informant that they were the personalities involved.

xxxx

Q: When this two (2) cars arrive what happened next? A: They talked for a while after few minutes Botong entered, sir.

xxxx

Q: Do you know this Botong prior this incident? A: No, sir.

Q: How did you come to know that he is Botong? A: Through our informant, sir.

Q: When Botong went to the Whistle Stop, what happened next? A: According to my other companion he talked to another person then after that they went out, sir.

xxxx

Q: How long did Botong stay in Whistle Stop Restaurant? A: One (1) minute, sir.

xxxx

Q: When you say they who is the companion? A: Cocoy, sir.

xxxx

Q: What happened next after they went out to the car? A: They went to another car and Cocoy got something from his car and handed to Botong, sir.

xxxx

Q: Did you see that something that was taken inside that car? A: White plastic bag, sir.

Q: What happened after that? A: Cocoy went inside the Whistle Stop, sir.

Q: With the bag? A: No, it was left with Botong, sir.

Q: What happened next after that? A: Botong proceeded to his car near Mac-Mac, sir.

Q: What happened next after that? A: We already sensed that drug deal has transpired, sir. We accosted him.

xxxx

Q: What did you do? A: I arrested Mac-Mac, sir.

xxxx

Q: Who of your companion apprehended Botong or Rolando delos Reyes? A: Botong was arrested by Yumul and Padpad, sir.

Q: How about De Claro? A: Arrested by Santiago, sir.

xxxx

Q: Then what did you do after apprehending these people? A: We brought them to our office for investigation, sir.[71] (Emphases supplied.)

PO3 Santiagos testimony also did not offer much justification for the warrantless arrest of accused-appellants and search of their persons: Q: When these two (2) persons went out of the restaurant and went to the place where blue Mazda car was parked, what happened next? A: The person inside the Mazda car, from the backseat, handed a white plastic bag with a box inside to Emmanuel de Claro [Cocoy], sir. Then, Emmanuel de Claro [Cocoy] gave it to Rolando Delos Reyes [Botong], sir.

Q: You mentioned about somebody handling box to De Claro [Cocoy] from inside that Mazda car? A: Yes, sir.

Q: Who was this somebody handling that box? A: It was Mary Jane Lantion, sir.

xxxx

Q: When you see De Claro [Cocoy] handling the box to Botong, what happened after that? A: Botong proceeded to the place of Mac-Mac and Emmanuel De Claro [Cocoy] returned back inside the said restaurant, sir.

Q: Where was Mac-Mac then at that time?

A: Near their car, sir. He was waiting for Botong.

Q: After that what happened next? A: When Botong returned to Mac-Mac, he gave white plastic bag with box inside to Mac-Mac, sir.

Q: What happened after that? A: Our

team leader, sensing that the drug consummated, we apprehended them, sir.

deal

have

been

Q: How did you come to know that there was a drug deal at that particular place and time? A: Because of the information given to us by the informant, sir.

Q: Are you aware of the contents of that box at that time? A: No, sir.

Q: How did you come to know that there was a consummation of a drug deal? A: Because of the information given to us by the informant that there will be a drug-deal, sir.

xxxx

Q: Then what did you do? A: We brought them to our office for proper investigation, sir.

Q: At your office, what else did you do? A: We confiscated the evidence, marked them and a request for laboratory examination was made and other pertaining papers regarding the arrest of the accused.

Q: You mentioned about the confiscated evidence. What is that confiscated evidence that you are saying? A: Ten (10) pieces of white plastic transparent plastic bag with white crystalline substance suspected to be methamphetamine hydrochloride, sir.

Q: How were these evidences confiscated by your group? A: They were confiscated from Mac-Mac, sir.

Q: In what condition were they at that time that they were confiscated from Mac-Mac? A: They were placed inside the box, sir.[72] (Emphases supplied.)

PO3 Yumuls narration of events was not any different from those of SPO1 Lectura and PO3 Santiago:

Q: When did you meet the confidential informant? A: At the vicinity of EDSA Shangri-La Plaza, sir.

Q: And what was the information that was relayed to you by the confidential informant? A: The identities of the persons, sir.

Q: What did he particularly tells you in that particular time you meet the confidential informant at the vicinity of EDSA Shangri-La Plaza? A: That there will be a drug-deal and the people involved will arrived together with their car, sir.

xxxx

Q: And what happened after the confidential informant relayed to you the information? A: After we were brief by the confidential informant, we strategically positioned ourselves in the place where the drug-deal will occur, sir.

xxxx

Q: So what did you do after positioning yourselves in that place of EDSA Shangri-La Plaza and Whistle Stop restaurant, what happened next after that? A: At around 10:00, one car arrived, a white Toyota corolla . . .

Q: 10:00 what? In the morning or in the evening? A: In the evening, sir, of February 17, 2000, sir.

Q: And you stated that two vehicles arrived? A: Yes, sir.

xxxx

Q: So what happened when this vehicle arrived? A: The red Toyota corolla follows, sir.

xxxx

Q: Then what happened? What did you do, if any? A: Our confidential informant told us that, that is our subject, sir.

xxxx

Q: What happened next, if any, were they alighted from the car? A: Yes, sir.

xxxx

Q: Then, what happened next, if any? A: They talked after they alighted from their car, sir.

Q: When you say nag-usap sila to whom are you referring? A: To Mac-Mac and Botong, sir.

xxxx

Q: What happened next after you see them talking to each other? A: When they talk Mac-Mac called through cellphone, sir.

Q: By the way, did you hear the conversation of this two? A: No, sir.

xxxx

Q: How about the one calling over the cellphone, did you hear also what was the subject of their conversation?

A: No, sir.

Q: So what happened next after seeing them having a conversation with each other? A: Botong immediately walked and proceeding to the Whistle Stop, sir.

xxxx

Q: Then what happened when Botong went to Whistle Stop? A: He talked to somebody inside, sir.

xxxx

Q: And did you hear what was the subject of their conversation? A: No, sir.

Q: Then what happened next when Botong talked to somebody inside the Whistle Stop? A: The companion stood up and they went outside and both of them went to the side of Whistle Stop in front of the blue car, sir.

xxxx

Q: What did you do then? A: Somebody opened the window in back of the blue car, sir.

Q: And then what happened next, if any? A: A white plastic bag was handed to him with carton inside, sir.

xxxx

Q: And who received that item or article from the car? A: Cocoy, sir.

xxxx

Q: Were you able to know the person inside that car and who handed to Cocoy the white plastic bag? A: Yes, sir.

Q: Who was that person? A: Mary Jane Lantion, sir.

xxxx

Q: And when this white plastic bag with carton placed inside handed to Cocoy, what did you do? A: It was first handed by Cocoy to Botong, the plastic bag and then they walked in different direction, Cocoy went back inside the Whistle Stop and then Botong went back to Mac-Mac, sir.

xxxx

Q: And then what happened next after that? A: I followed Cocoy inside the Whistle Stop, sir.

xxxx

Q: So what did you do then? A: I observed him inside but after a few minutes PO3 Virgilio Santiago went inside and told me that we will going to get them, sir.

Q: Why are you going to get them? A: Because the two were already arrested outside the Whistle Stop, Mac-Mac and Botong, sir.

xxxx

Q: So what did you do when PO3 Santiago told you that?

A: PO3 Santiago approached Cocoy and I am just assisting him, PO3 Santiago to avoid commotion, sir.

Q: Then what did you do next after that? A: We were able to get Cocoy and we went outside, sir.

Q: And then what did you do, if any? A: After arresting them we boarded to the car and we went to the office, sir.[73] (Emphases supplied.)

Evident from the foregoing excerpts that the police officers arrested accused-appellants and searched the latters persons without a warrant after seeing Rolando delos Reyes and Emmanuel de Claro momentarily conversing in the restaurant, and witnessing the white plastic bag with a box or carton inside being passed from Lantion-Tom to Emmanuel de Claro, to accused-appellant Rolando delos Reyes, and finally, to accused-appellant Reyes. These circumstances, however, hardly constitute overt acts indicative of a felonious enterprise. SPO1 Lectura, PO3 Santiago, and PO3 Yumul had no prior knowledge of the suspects identities, and they completely relied on their confidential informant to actually identify the suspects. None of the police officers actually saw what was inside that box. There is also no evidence that the confidential informant himself knew that the box contained shabu. No effort at all was taken to confirm that the arrested suspects actually knew that the box or carton inside the white plastic bag, seized from their possession, contained shabu. The police officers were unable to establish a cogent fact or circumstance that would have reasonably invited their attention, as officers of the law, to suspect that accusedappellants, Emmanuel de Claro, and Lantion-Tom has just committed, is actually committing, or is attempting to commit a crime, particularly, an illegal drug deal.

Finally, from their own account of the events, the police officers had compromised the integrity of the shabu purportedly seized from accusedappellants.

In People v. Sy Chua,[74] we questioned whether the shabu seized from the accused was the same one presented at the trial because of the failure of the police to mark the drugs at the place where it was taken, to wit: Furthermore, we entertain doubts whether the items allegedly seized from accused-appellant were the very same items presented at the trial of this case. The record shows that the initial field test where the items seized were identified as shabu, was only conducted at the PNP headquarters of Angeles City. The items were therefore not marked at the place where they were taken. In People v. Casimiro, we struck down with disbelief the reliability of the identity of the confiscated items since they were not marked at the place where they were seized, thus:

The narcotics field test, which initially identified the seized item as marijuana, was likewise not conducted at the scene of the crime, but only at the narcotics office. There is thus reasonable doubt as to whether the item allegedly seized from accused-appellant is the same brick of marijuana marked by the policemen in their headquarters and given by them to the crime laboratory.[75](Emphases supplied.)

In the instant case, SPO1 Lectura, PO3 Santiago, and PO3 Yumul uniformly testified before the RTC that they brought the arrested suspects to the police office for investigation. SPO1 Lectura and PO3 Santiago were vague as to how they ascertained as shabu the contents of the box inside the white plastic bag, immediately after seizing the same from accused-appellant Reyes and before proceeding to the police office; while PO3 Yumul explicitly testified on crossexamination[76] that he saw the shabu for the first time at the police office. At any rate, all three police officers recounted that the shabu was marked by SPO1 Benjamin David only at the police office.

Without valid justification for the in flagrante delicto arrests of accusedappellants, the search of accused-appellants persons incidental to said arrests, and the eventual seizure of the shabufrom accused-appellants possession, are also considered unlawful and, thus, the seized shabu is excluded in evidence as fruit of a poisonous tree. Without the corpus delicti for the crime charged, then the acquittal of accused-appellants is inevitable.

As we aptly held in People v. Sy Chua[77]:

All told, the absence of ill-motive on the part of the arresting team cannot simply validate, much more cure, the illegality of the arrest and consequent warrantless search of accused-appellant. Neither can the presumption of regularity of performance of function be invoked by an officer in aid of the process when he undertakes to justify an encroachment of rights secured by the Constitution. In People v. Nubla, we clearly stated that:

The presumption of regularity in the performance of official duty cannot be used as basis for affirming accusedappellant's conviction because, first, the presumption is precisely just that a mere presumption. Once challenged

by evidence, as in this case, . . . [it] cannot be regarded as binding truth. Second, the presumption of regularity in the performance of official functions cannot preponderate over the presumption of innocence that prevails if not overthrown by proof beyond reasonable doubt.

xxxx

The government's drive against illegal drugs needs the support of every citizen. But it should not undermine the fundamental rights of every citizen as enshrined in the Constitution. The constitutional guarantee against warrantless arrests and unreasonable searches and seizures cannot be so carelessly disregarded as overzealous police officers are sometimes wont to do. Fealty to the constitution and the rights it guarantees should be paramount in their minds, otherwise their good intentions will remain as such simply because they have blundered. The criminal goes free, if he must, but it is the law that sets him free. Nothing can destroy a government more quickly than its failure to observe its own laws, or worse, its disregard of the charter of its own existence.[78]

WHEREFORE, the Decision dated July 12, 2006 of the Court of Appeals in CA-G.R. CR.-H.C. No. 01733 is hereby REVERSED and SET ASIDE. Accused-appellants Rolando delos Reyes and Raymundo Reyes are ACQUITTED on the ground of reasonable doubt and they are ORDERED forthwith released from custody, unless they are being lawfully held for another crime. SO ORDERED. G.R. No. 197293

April 21, 2014

ALFREDO C. MENDOZA, Petitioner, vs. PEOPLE OF THE PHILIPPINES AND JUNO CARS, INC., Respondents. DECISION LEONEN, J.: While the determination of probable cause to charge a person of a crime is the sole function of the. prosecutor, the trial court may, in the protection of one's fundamental right to liberty, dismiss the case if, upon a personal assessment of the evidence, it finds that the evidence does not establish probable cause. This is a petition for review on certiorari1 assailing the Court of Appeals' decision2 dated January 14, 2011, which reversed the Regional Trial Court's dismissal of the complaint against petitioner Alfredo C. Mendoza for qualified theft and estafa. This case stems from a complaint-affidavit filed by Juno Cars, Inc. through its representative, Raul C. Evangelista, on January 8, 2008 for qualified theft and estafa against Alfredo.3 In the complaint-affidavit, Juno Cars alleged that on June 2, 2007, it hired Alfredo as Trade-In/Used Car Supervisor. On November 19, 2007, its Dealer/Operator, Rolando Garcia, conducted a partial audit of the used cars and discovered that five (5) cars had been sold and released by Alfredo without Rolando’s or the finance manager’s permission.4 The partial audit showed that the buyers of the five cars made payments, but Alfredo failed to remit the payments totalling ₱886,000.00. It was further alleged that while there were 20 cars under Alfredo’s custody, only 18 were accounted for. Further investigation revealed that Alfredo failed to turn over the files of a 2001 Hyundai Starex and a Honda City 1.5 LXI. Juno Cars alleged that taking into account the unremitted amounts and the acquisition cost of the Honda City, Alfredo pilfered a total amount of ₱1,046,000.00 to its prejudice and damage.5 In his counter-affidavit, Alfredo raised, among others, Juno Cars’ supposed failure to prove ownership over the five (5) cars or its right to possess them with the purported unremitted payments. Hence, it could not have suffered damage.6 On March 4, 2008, Provincial Prosecutor Rey F. Delgado issued a Resolution7 finding probable cause and recommending the filing of an information against Alfredo for qualified theft and estafa. Alfredo moved for reconsideration, but the motion was denied.8 He then filed a petition for review with the Department of Justice on May 16, 2008.9 While Alfredo’s motion for reconsideration was still pending before the Office of the City Prosecutor of Mandaluyong, two informations for qualified theft10 and estafa11 were filed before the Regional Trial Court, Branch 212, Mandaluyong City. On March 31, 2008, Alfredo filed a motion for determination of probable cause12 before the trial court. On April 28, 2008, he also filed a motion to defer arraignment. Several clarificatory hearings were scheduled but were not conducted.13 On February 4, 2009, the parties agreed to submit all pending incidents, including the clarificatory hearing, for resolution.14

On March 3, 2009, the trial court, through Presiding Judge Rizalina Capco-Umali, issued an order15 dismissing the complaint, stating that: After conducting an independent assessment of the evidence on record which includes the assailed Resolution dated 04 March 2008, the court holds that the evidence adduced does not support a finding of probable cause for the offenses of qualified theft and estafa. x x x.16 Juno Cars filed a motion for reconsideration, which the trial court denied on July 3, 2009.17 Juno Cars then filed a petition for certiorari with the Court of Appeals, arguing that the trial court acted without or in excess of its jurisdiction and with grave abuse of discretion when it dismissed the complaint. It argued that "the determination of probable cause and the decision whether or not to file a criminal case in court, rightfully belongs to the public prosecutor."18 On January 14, 2011, the Court of Appeals rendered a decision,19 reversed the trial court, and reinstated the case. In its decision, the appellate court ruled that the trial court acted without or in excess of its jurisdiction "in supplanting the public prosecutor’s findings of probable cause with her own findings of insufficiency of evidence and lack of probable cause."20 Aggrieved, Alfredo filed a petition for review under Rule 45 before this court. In essence, he argued that the trial court was correct in finding that there was no probable cause as shown by the evidence on record. He argued that "judicial determination of probable cause is broader than [the] executive determination of probable cause"21and that "[i]t is not correct to say that the determination of probable cause is exclusively vested on the prosecutor x x x."22 In its comment,23 Juno Cars argued that Alfredo presented questions, issues, and arguments that were a mere rehash of those already considered and passed upon by the appellate court. The Office of the Solicitor General, arguing for public respondent, stated in its comment24 that the appellate court correctly sustained the public prosecutor in his findings of probable cause against Alfredo. Since there was no showing of grave abuse of discretion on the part of Prosecutor Rey F. Delgado, the trial court should respect his determination of probable cause. In his reply,25 Alfredo reiterated that "judicial determination of probable cause[,] while not a superior faculty[,] covers a broader encompassing perspective in the disposition of the issue on the existence of probable cause."26He argued that the findings of the trial court should be accorded greater weight than the appellate court’s. It merely reviewed the findings of the trial court. The primordial issue is whether the trial court may dismiss an information filed by the prosecutor on the basis of its own independent finding of lack of probable cause. Time and again, this court has been confronted with the issue of the difference between the determination of probable cause by the prosecutor on one hand and the determination of probable cause by the judge on the other. We examine these two concepts again. Juno Cars filed a complaint against Alfredo for qualified theft27 and estafa under Article 315, fourth paragraph, no. 3(c)28 of the Revised Penal Code. Since qualified theft is punishable by reclusion perpetua, a preliminary investigation must first be conducted "to determine whether there is sufficient ground to engender a well-founded belief that a crime has been committed and the respondent is probably guilty thereof, and should be held for trial," in accordance with Rule 112, Section 1 of the Rules on Criminal Procedure.

At this stage, the conduct of the preliminary investigation and the subsequent determination of the existence of probable cause lie solely within the discretion of the public prosecutor.29 If upon evaluation of the evidence, the prosecutor finds sufficient basis to find probable cause, he or she shall then cause the filing of the information with the court. Once the information has been filed, the judge shall then "personally evaluate the resolution of the prosecutor and its supporting evidence"30 to determine whether there is probable cause to issue a warrant of arrest. At this stage, a judicial determination of probable cause exists. In People v. Castillo and Mejia,31 this court has stated: There are two kinds of determination of probable cause: executive and judicial. The executive determination of probable cause is one made during preliminary investigation. It is a function that properly pertains to the public prosecutor who is given a broad discretion to determine whether probable cause exists and to charge those whom he believes to have committed the crime as defined by law and thus should be held for trial. Otherwise stated, such official has the quasi-judicial authority to determine whether or not a criminal case must be filed in court. Whether or not that function has been correctly discharged by the public prosecutor, i.e., whether or not he has made a correct ascertainment of the existence of probable cause in a case, is a matter that the trial court itself does not and may not be compelled to pass upon. The judicial determination of probable cause, on the other hand, is one made by the judge to ascertain whether a warrant of arrest should be issued against the accused. The judge must satisfy himself that based on the evidence submitted, there is necessity for placing the accused under custody in order not to frustrate the ends of justice. If the judge finds no probable cause, the judge cannot be forced to issue the arrest warrant.32 The difference is clear: The executive determination of probable cause concerns itself with whether there is enough evidence to support an Information being filed. The judicial determination of probable cause, on the other hand, determines whether a warrant of arrest should be issued. In People v. Inting:33 x x x Judges and Prosecutors alike should distinguish the preliminary inquiry which determines probable cause for the issuance of a warrant of arrest from the preliminary investigation proper which ascertains whether the offender should be held for trial or released. Even if the two inquiries are conducted in the course of one and the same proceeding, there should be no confusion about the objectives. The determination of probable cause for the warrant of arrest is made by the Judge. The preliminary investigation proper—whether or not there is reasonable ground to believe that the accused is guilty of the offense charged and, therefore, whether or not he should be subjected to the expense, rigors and embarrassment of trial—is the function of the Prosecutor.34 (Emphasis supplied) While it is within the trial court’s discretion to make an independent assessment of the evidence on hand, it is only for the purpose of determining whether a warrant of arrest should be issued. The judge does not act as an appellate court of the prosecutor and has no capacity to review the prosecutor’s determination of probable cause; rather, the judge makes a determination of probable cause independent of the prosecutor’s finding. People v. Court of Appeals and Jonathan Cerbo35 discussed the rationale. In that case, Jonathan Cerbo allegedly shot Rosalinda Dy in the presence of his father, Billy Cerbo. An information for murder was filed against Jonathan Cerbo. The daughter of Rosalinda Dy, as private complainant, executed a complaint-affidavit charging Billy Cerbo with conspiracy. The prosecutor then filed a motion to amend the information, which was granted by the court. The information was then

amended to include Billy Cerbo as one of the accused, and a warrant of arrest was issued against him. Billy Cerbo filed a motion to quash the warrant arguing that it was issued without probable cause. The trial court granted this motion, recalled the warrant, and dismissed the case against him. The Court of Appeals affirmed this dismissal. This court, however, reversed the Court of Appeals and ordered the reinstatement of the amended information against Billy Cerbo, stating that: In granting this petition, we are not prejudging the criminal case or the guilt or innocence of Private Respondent Billy Cerbo. We are simply saying that, as a general rule, if the information is valid on its face and there is no showing of manifest error, grave abuse of discretion or prejudice on the part of the public prosecutor, courts should not dismiss it for ‘want of evidence,’ because evidentiary matters should be presented and heard during the trial. The functions and duties of both the trial court and the public prosecutor in "the proper scheme of things" in our criminal justice system should be clearly understood. The rights of the people from what could sometimes be an "oppressive" exercise of government prosecutorial powers do need to be protected when circumstances so require. But just as we recognize this need, we also acknowledge that the State must likewise be accorded due process. Thus, when there is no showing of nefarious irregularity or manifest error in the performance of a public prosecutor’s duties, courts ought to refrain from interfering with such lawfully and judicially mandated duties. In any case, if there was palpable error or grave abuse of discretion in the public prosecutor’s finding of probable cause, the accused can appeal such finding to the justice secretary and move for the deferment or suspension of the proceedings until such appeal is resolved.36 (Emphasis supplied) In this case, the resolution dated March 4, 2008 of Prosecutor Rey F. Delgado found that the facts and evidence were "sufficient to warrant the indictment of [petitioner] x x x."37 There was nothing in his resolution which showed that he issued it beyond the discretion granted to him by law and jurisprudence. While the information filed by Prosecutor Delgado was valid, Judge Capco-Umali still had the discretion to make her own finding of whether probable cause existed to order the arrest of the accused and proceed with trial. Jurisdiction over an accused is acquired when the warrant of arrest is served. Absent this, the court cannot hold the accused for arraignment and trial. Article III, Section 2 of the Constitution states: The right of the people to be secure in their persons, houses, papers, and effects against unreasonable searches and seizures of whatever nature and for any purpose shall be inviolable, and no search warrant or warrant of arrest shall issue except upon probable cause to be determined personally by the judge after examination under oath or affirmation of the complainant and the witnesses he may produce, and particularly describing the place to be searched and the persons or things to be seized. The Constitution prohibits the issuance of search warrants or warrants of arrest where the judge has not personally determined the existence of probable cause. The phrase "upon probable cause to be determined personally by the judge after examination under oath or affirmation of the complainant and the witnesses he may produce" allows a determination of probable cause by the judge ex parte.

For this reason, Section 6, paragraph (a) of Rule 112 of the Rules on Criminal Procedure mandates the judge to "immediately dismiss the case if the evidence on record fails to establish probable cause." Section 6, paragraph (a) of Rule 112 reads: Section 6. When warrant of arrest may issue. — (a) By the Regional Trial Court. — Within ten (10) days from the filing of the complaint or information, the judge shall personally evaluate the resolution of the prosecutor and its supporting evidence. He may immediately dismiss the case if the evidence on record clearly fails to establish probable cause. If he finds probable cause, he shall issue a warrant of arrest, or a commitment order if the accused has already been arrested pursuant to a warrant issued by the judge who conducted the preliminary investigation or when the complaint or information was filed pursuant to section 7 of this Rule. In case of doubt on the existence of probable cause, the judge may order the prosecutor to present additional evidence within five (5) days from notice and the issue must be resolved by the court within thirty (30) days from the filing of the complaint of information. In People v. Hon. Yadao:38 Section 6, Rule 112 of the Rules of Court gives the trial court three options upon the filing of the criminal information: (1) dismiss the case if the evidence on record clearly failed to establish probable cause; (2) issue a warrant of arrest if it finds probable cause; and (3) order the prosecutor to present additional evidence within five days from notice in case of doubt as to the existence of probable cause. But the option to order the prosecutor to present additional evidence is not mandatory. The court’s first option under the above is for it to "immediately dismiss the case if the evidence on record clearly fails to establish probable cause." That is the situation here: the evidence on record clearly fails to establish probable cause against the respondents.39 (Emphasis supplied) 1âw phi1

It is also settled that "once a complaint or information is filed in court, any disposition of the case, whether as to its dismissal or the conviction or the acquittal of the accused, rests in the sound discretion of the court."40 In this case, Judge Capco-Umali made an independent assessment of the evidence on record and concluded that "the evidence adduced does not support a finding of probable cause for the offenses of qualified theft and estafa."41 Specifically, she found that Juno Cars "failed to prove by competent evidence"42 that the vehicles alleged to have been pilfered by Alfredo were lawfully possessed or owned by them, or that these vehicles were received by Alfredo, to be able to substantiate the charge of qualified theft. She also found that the complaint "[did] not state with particularity the exact value of the alleged office files or their valuation purportedly have been removed, concealed or destroyed by the accused,"43 which she found crucial to the prosecution of the crime of estafa under Article 315, fourth paragraph, no. 3(c) of the Revised Penal Code. She also noted that: x x x As a matter of fact, this court had even ordered that this case be set for clarificatory hearing to clear out essential matters pertinent to the offense charged and even directed the private complainant to bring documents relative to the same/payment as well as affidavit of witnesses/buyers with the end view of satisfying itself that indeed probable cause exists to commit the present case which private complainant failed to do.44 Accordingly, with the present laws and jurisprudence on the matter, Judge Capco-Umali correctly dismissed the case against Alfredo.

Although jurisprudence and procedural rules allow it, a judge must always proceed with caution in dismissing cases due to lack of probable cause, considering the preliminary nature of the evidence before it. It is only when he or she finds that the evidence on hand absolutely fails to support a finding of probable cause that he or she can dismiss the case. On the other hand, if a judge finds probable cause, he or she must not hesitate to proceed with arraignment and trial in order that justice may be served. WHEREFORE, the petition is GRANTED. The decision dated January 14, 2011 of the Court of Appeals in CA-G.R. SP. No. 110774 is REVERSED and SET ASIDE. Criminal Case Nos. MC0811604-05 against Alfredo C. Mendoza are DISMISSED. G.R. No. 176830

February 11, 2014

SATURNINO C. OCAMPO, Petitioner, vs. HON. EPHREM S. ABANDO, in his capacity as Presiding Judge of the Regional Trial Court of Hilongos, Leyte, Branch 18, CESAR M. MERIN, in. his capacity as Approving Prosecutor and Officer-in-Charge, ROSULO U. VIVERO, in his capacity as Investigating Prosecutor, RAUL M. GONZALEZ, in his capacity as Secretary of the Department of Justice, Respondents. x-----------------------x G.R. No. 185587 RANDALL B. ECHANIS, Petitioner, vs. HON. THELMA BUNYl-MEDINA, in her capacity as Presiding Judge of the Regional Trial Court of Manila, Branch 32, HON. EPHREM S. ABANDO, in his capacity as Presiding Judge of the Regional Trial Court of Hilongos, Leyte, Branch 18, CESAR M. MERIN, in his capacity as Approving Prosecutor and Officer-in-Charge, ROSULO U. VIVERO, in his capacity as Investigating Prosecutor, RAUL M. GONZALEZ, in his capacity as Secretary of the Department of Justice, Respondents. x-----------------------x G.R. No. 185636 RAFAEL G. BAYLOSIS, Petitioner, vs. HON. THELMA BUNYI-MEDINA, in her capacity as Presiding Judge of the Regional Trial Court of Manila, Branch 32, HON. EPHREM S. ABANDO, in his capacity as Presiding Judge of the Regional Trial Court of Hilongos, Leyte, Branch 18, CESAR M. MERIN, in his capacity as Approving Prosecutor and Officer-in-Charge, ROSULO U. VIVERO, in his capacity as Investigating Prosecutor, RAUL M. GONZALEZ, in his capacity as Secretary of the Department of Justice, Respondents. x-----------------------x G.R. No. 190005

VICENTE P. LADLAD, Petitioner, vs. HON. THELMA BUNYI-MEDINA, in her capacity as Presiding Judge of the Regional Trial Court of Manila, Branch 32, and the PEOPLE OF THE PHILIPPINES, Respondents. DECISION SERENO, CJ.: On 26 August 2006, a mass grave was discovered by elements of the 43rd Infantry Brigade of the Philippine Army at Sitio Sapang Daco, Barangay Kaulisihan, Inopacan, Leyte.1 The mass grave contained skeletal remains of individuals believed to be victims of "Operation Venereal Disease" (Operation VD) launched by members of the Communist Party of the Philippines/New People’s Army/National Democratic Front of the Philippines (CPP/NPA/NDFP) to purge their ranks of suspected military informers. While the doctrine of hierarchy of courts normally precludes a direct invocation of this Court’s jurisdiction, we take cognizance of these petitions considering that petitioners have chosen to take recourse directly before us and that the cases are of significant national interest. Petitioners have raised several issues, but most are too insubstantial to require consideration. Accordingly, in the exercise of sound judicial discretion and economy, this Court will pass primarily upon the following: 1. Whether petitioners were denied due process during preliminary investigation and in the issuance of the warrants of arrest. 2. Whether the murder charges against petitioners should be dismissed under the political offense doctrine. ANTECEDENT FACTS These are petitions for certiorari and prohibition2 seeking the annulment of the orders and resolutions of public respondents with regard to the indictment and issuance of warrants of arrest against petitioners for the crime of multiple murder. Police Chief Inspector George L. Almaden (P C/Insp. Almaden) of the Philippine National Police (PNP) Regional Office 8 and Staff Judge Advocate Captain Allan Tiu (Army Captain Tiu) of the 8th Infantry Division of the Philippine Army sent 12 undated letters to the Provincial Prosecutor of Leyte through Assistant Provincial Prosecutor Rosulo U. Vivero (Prosecutor Vivero).3 The letters requested appropriate legal action on 12 complaint-affidavits attached therewith accusing 71 named members of the Communist Party of the Philippines/New People’s Army/National Democratic Front of the Philippines (CPP/NPA/NDFP) of murder, including petitioners herein along with several other unnamed members. The letters narrated that on 26 August 2006, elements of the 43rd Infantry Brigade of the Philippine Army discovered a mass grave site of the CPP/NPA/NDFP at Sitio Sapang Daco, Barangay Kaulisihan, Inopacan, Leyte.4 Recovered from the grave site were 67 severely deteriorated skeletal remains believed to be victims of Operation VD.5

The PNP Scene of the Crime Operation (SOCO) Team based in Regional Office 8 was immediately dispatched to the mass grave site to conduct crime investigation, and to collect, preserve and analyze the skeletal remains.6Also, from 11-17 September 2006, an investigation team composed of intelligence officers, and medico-legal and DNA experts, conducted forensic crime analysis and collected from alleged relatives of the victims DNA samples for matching.7 The Initial Specialist Report8 dated 18 September 2006 issued by the PNP Crime Laboratory in Camp Crame, Quezon City, was inconclusive with regard to the identities of the skeletal remains and even the length of time that they had been buried. The report recommended the conduct of further tests to confirm the identities of the remains and the time window of death.9 However, in a Special Report10 dated 2 October 2006, the Case Secretariat of the Regional and National Inter-Agency Legal Action Group (IALAG) came up with the names of ten (10) possible victims after comparison and examination based on testimonies of relatives and witnesses.11 The 12 complaint-affidavits were from relatives of the alleged victims of Operation VD. All of them swore that their relatives had been abducted or last seen with members of the CPP/NPA/NDFP and were never seen again. They also expressed belief that their relatives’ remains were among those discovered at the mass grave site. Also attached to the letters were the affidavits of Zacarias Piedad,12 Leonardo C. Tanaid, Floro M. Tanaid, Numeriano Beringuel, Glecerio Roluna and Veronica P. Tabara. They narrated that they were former members of the CPP/NPA/NDFP.13 According to them, Operation VD was ordered in 1985 by the CPP/NPA/NDFP Central Committee.14 Allegedly, petitioners Saturnino C. Ocampo (Ocampo),15 Randall B. Echanis (Echanis),16 Rafael G. Baylosis (Baylosis),17 and Vicente P. Ladlad (Ladlad)18 were then members of the Central Committee. According to these former members, four sub-groups were formed to implement Operation VD, namely, (1) the Intel Group responsible for gathering information on suspected military spies and civilians who would not support the movement; (2) the Arresting Group charged with their arrests; (3) the Investigation Group which would subject those arrested to questioning; and (4) the Execution Group or the "cleaners" of those confirmed to be military spies and civilians who would not support the movement.19 From 1985 to 1992, at least 100 people had been abducted, hog-tied, tortured and executed by members of the CPP/NPA/NDF20 pursuant to Operation VD.21 On the basis of the 12 letters and their attachments, Prosecutor Vivero issued a subpoena requiring, among others, petitioners to submit their counter-affidavits and those of their witnesses.22 Petitioner Ocampo submitted his counter-affidavit.23 Petitioners Echanis24 and Baylosis25 did not file counteraffidavits because they were allegedly not served the copy of the complaint and the attached documents or evidence. Counsel of petitioner Ladlad made a formal entry of appearance on 8 December 2006 during the preliminary investigation.26 However, petitioner Ladlad did not file a counter-affidavit because he was allegedly not served a subpoena.27 In a Resolution28 dated 16 February 2007, Prosecutor Vivero recommended the filing of an Information for 15 counts of multiple murder against 54 named members of the CPP/NPA/NDFP, including petitioners herein, for the death of the following: 1) Juanita Aviola, 2) Concepcion Aragon, 3) Gregorio Eras, 4) Teodoro Recones, Jr., 5) Restituto Ejoc, 6) Rolando Vasquez, 7) Junior

Milyapis, 8) Crispin Dalmacio, 9) Zacarias Casil, 10) Pablo Daniel, 11) Romeo Tayabas, 12) Domingo Napoles, 13) Ciriaco Daniel, 14) Crispin Prado, and 15) Ereberto Prado.29 Prosecutor Vivero also recommended that Zacarias Piedad, Leonardo Tanaid, Numeriano Beringuel and Glecerio Roluna be dropped as respondents and utilized as state witnesses, as their testimonies were vital to the success of the prosecution.30 The Resolution was silent with regard to Veronica Tabara. The Information was filed before the Regional Trial Court (RTC) Hilongos, Leyte, Branch 18 (RTC Hilongos, Leyte) presided by Judge Ephrem S. Abando (Judge Abando) on 28 February 2007, and docketed as Criminal Case No. H-1581.31 Petitioner Ocampo filed an Ex Parte Motion to Set Case for Clarificatory Hearing dated 5 March 2007 prior to receiving a copy of the Resolution recommending the filing of the Information.32 On 6 March 2007, Judge Abando issued an Order finding probable cause "in the commission by all mentioned accused of the crime charged."33 He ordered the issuance of warrants of arrest against them with no recommended bail for their temporary liberty.34 On 16 March 2007, petitioner Ocampo filed before us this special civil action for certiorari and prohibition under Rule 65 of the Rules of Court and docketed as G.R. No. 176830 seeking the annulment of the 6 March 2007 Order of Judge Abando and the 16 February 2007 Resolution of Prosecutor Vivero.35 The petition prayed for the unconditional release of petitioner Ocampo from PNP custody, as well as the issuance of a temporary restraining order/ writ of preliminary injunction to restrain the conduct of further proceedings during the pendency of the petition.36 Petitioner Ocampo argued that a case for rebellion against him and 44 others (including petitioners Echanis and Baylosis37 and Ladlad38) docketed as Criminal Case No. 06-944 was then pending before the RTC Makati, Branch 150 (RTC Makati).39 Putting forward the political offense doctrine, petitioner Ocampo argues that common crimes, such as murder in this case, are already absorbed by the crime of rebellion when committed as a necessary means, in connection with and in furtherance of rebellion.40 We required41 the Office of the Solicitor General (OSG) to comment on the petition and the prayer for the issuance of a temporary restraining order/ writ of preliminary injunction, and set42 the case for oral arguments on 30 March 2007. The OSG filed its Comment on 27 March 2007.43 The following were the legal issues discussed by the parties during the oral arguments: 1. Whether the present petition for certiorari and prohibition is the proper remedy of petitioner Ocampo; 2. Assuming it is the proper remedy, whether he was denied due process during preliminary investigation and in the issuance of the warrant of arrest; 3. Whether the murder charges against him are already included in the rebellion charge against him in the RTC.44 Afterwards, the parties were ordered to submit their memoranda within 10 days.45 On 3 April 2007, the Court ordered the provisional release of petitioner Ocampo under a ₱100,000 cash bond.46 Acting on the observation of the Court during the oral arguments that the single Information filed before the RTC Hilongos, Leyte was defective for charging 15 counts of murder, the prosecution

filed a Motion to Admit Amended Information and New Informations on 11 April 2007.47 In an Order dated 27 July 2007, Judge Abando held in abeyance the resolution thereof and effectively suspended the proceedings during the pendency of G.R. No. 176830 before this Court.48 While the proceedings were suspended, petitioner Echanis was arrested on 28 January 2008 by virtue of the warrant of arrest issued by Judge Abando on 6 March 2007.49 On 1 February 2008, petitioners Echanis and Baylosis filed a Motion for Judicial Reinvestigation/ Determination of Probable Cause with Prayer to Dismiss the Case Outright and Alternative Prayer to Recall/ Suspend Service of Warrant.50 On 30 April 2008, Judge Abando issued an Order denying the motion.51 Petitioners Echanis and Baylosis filed a Motion for Reconsideration52 dated 30 May 2008, but before being able to rule thereon, Judge Abando issued an Order dated 12 June 2008 transmitting the records of Criminal Case No. H-1581 to the Office of the Clerk of Court, RTC Manila.53 The Order was issued in compliance with the Resolution dated 23 April 2008 of this Court granting the request of then Secretary of Justice Raul Gonzales to transfer the venue of the case. The case was re-raffled to RTC Manila, Branch 32 (RTC Manila) presided by Judge Thelma BunyiMedina (Judge Medina) and re-docketed as Criminal Case No. 08-262163.54 Petitioner Echanis was transferred to the PNP Custodial Center in Camp Crame, Quezon City. On 12 August 2008, petitioners Echanis and Baylosis filed their Supplemental Arguments to Motion for Reconsideration.55 In an Order56 dated 27 October 2008, Judge Medina suspended the proceedings of the case pending the resolution of G.R. No. 176830 by this Court. On 18 December 2008, petitioner Ladlad filed with the RTC Manila a Motion to Quash and/or Dismiss.57 On 23 December 2008, petitioner Echanis filed before us a special civil action for certiorari and prohibition under Rule 65 of the Rules of Court seeking the annulment of the 30 April 2008 Order of Judge Abando and the 27 October 2008 Order of Judge Medina.58 The petition, docketed as G.R. No. 185587, prayed for the unconditional and immediate release of petitioner Echanis, as well as the issuance of a temporary restraining order/writ of preliminary injunction to restrain his further incarceration.59 On 5 January 2009, petitioner Baylosis filed before us a special civil action for certiorari and prohibition under Rule 65 of the Rules of Court also seeking the annulment of the 30 April 2008 Order of Judge Abando and the 27 October 2008 Order of Judge Medina.60 The petition, docketed as G.R. No. 185636, prayed for the issuance of a temporary restraining order/ writ of preliminary injunction to restrain the implementation of the warrant of arrest against petitioner Baylosis.61 The Court consolidated G.R. Nos. 185587 and 185636 on 12 January 2009.62 On 3 March 2009, the Court ordered the further consolidation of these two cases with G.R. No. 176830.63 We required64 the OSG to comment on the prayer for petitioner Echanis’s immediate release, to which the OSG did not interpose any objection on these conditions: that the temporary release shall only be for the purpose of his attendance and participation in the formal peace negotiations between the Government of the Republic of the Philippines (GRP) and the CPP/NPA/NDFP, set to begin in August 2009; and that his temporary release shall not exceed six (6) months.65 The latter condition was later modified, such that his temporary liberty shall continue for the duration of his actual participation in the peace negotiations.66

On 11 August 2009, the Court ordered the provisional release of petitioner Echanis under a ₱100,000 cash bond, for the purpose of his participation in the formal peace negotiations.67 Meanwhile, the Department of Justice (DOJ) filed its Opposition68 to petitioner Ladlad’s motion to quash before the RTC Manila. The trial court conducted a hearing on the motion on 13 February 2009.69 On 6 May 2009, Judge Medina issued an Order70 denying the motion to quash. The motion for reconsideration filed by petitioner Ladlad was also denied on 27 August 2009.71 On 9 November 2009, petitioner Ladlad filed before us a special civil action for certiorari under Rule 65 of the Rules of Court seeking the annulment of the 6 May 2009 and 27 August 2009 Orders of Judge Medina.72 The petition was docketed as G.R. No. 190005. On 11 January 2010, we ordered the consolidation of G.R. No. 190005 with G.R. Nos. 176830, 185587 and 185636.73 We also required the OSG to file its comment thereon. The OSG submitted its Comment74 on 7 May 2010. On 27 July 2010, we likewise required the OSG to file its Comment in G.R. Nos. 185636 and 185587.75 These Comments were filed by the OSG on 13 December 201076 and on 21 January 2011,77 respectively. Petitioners Echanis and Baylosis filed their Consolidated Reply78 on 7 June 2011. On 2 May 2011, petitioner Ladlad filed an Urgent Motion to Fix Bail.79 On 21 July 2011, petitioner Baylosis filed A Motion to Allow Petitioner to Post Bail.80 The OSG interposed no objection to the grant of a ₱100,000 cash bail to them considering that they were consultants of the NDFP negotiating team, which was then holding negotiations with the GRP peace panel for the signing of a peace accord.81 On 17 January 2012, we granted the motions of petitioners Ladlad and Baylosis and fixed their bail in the amount of ₱100,000, subject to the condition that their temporary release shall be limited to the period of their actual participation in the peace negotiations.82 Petitioner Ladlad filed his Reply83 to the OSG Comment on 18 January 2013. OUR RULING Petitioners were accorded due process during preliminary investigation and in the issuance of the warrants of arrest. A. Preliminary Investigation A preliminary investigation is "not a casual affair."84 It is conducted to protect the innocent from the embarrassment, expense and anxiety of a public trial.85 While the right to have a preliminary investigation before trial is statutory rather than constitutional, it is a substantive right and a component of due process in the administration of criminal justice.86 In the context of a preliminary investigation, the right to due process of law entails the opportunity to be heard.87 It serves to accord an opportunity for the presentation of the respondent’s side with

regard to the accusation. Afterwards, the investigating officer shall decide whether the allegations and defenses lead to a reasonable belief that a crime has been committed, and that it was the respondent who committed it. Otherwise, the investigating officer is bound to dismiss the complaint. "The essence of due process is reasonable opportunity to be heard and submit evidence in support of one's defense."88 What is proscribed is lack of opportunity to be heard.89 Thus, one who has been afforded a chance to present one’s own side of the story cannot claim denial of due process.90 Petitioners Echanis and Baylosis allege that they did not receive a copy of the complaint and the attached documents or evidence.91 Petitioner Ladlad claims that he was not served a subpoena due to the false address indicated in the 12 undated letters of P C/Insp. Almaden and Army Captain Tiu to Prosecutor Vivero.92Furthermore, even though his counsels filed their formal entry of appearance before the Office of the Prosecutor, petitioner Ladlad was still not sent a subpoena through his counsels’ addresses.93 Thus, they were deprived of the right to file counter-affidavits. Petitioner Ocampo claims that Prosecutor Vivero, in collusion with P C/Insp. Almaden and Army Captain Tiu, surreptitiously inserted the Supplemental Affidavit of Zacarias Piedad in the records of the case without furnishing petitioner Ocampo a copy.94 The original affidavit of Zacarias Piedad dated 14 September 2006 stated that a meeting presided by petitioner Ocampo was held in 1984, when the launching of Operation VD was agreed upon.95 Petitioner Ocampo refuted this claim in his Counter-affidavit dated 22 December 2006 stating that he was in military custody from October 1976 until his escape in May 1985.96 Thereafter, the Supplemental Affidavit of Zacarias Piedad dated 12 January 2007 admitted that he made a mistake in his original affidavit, and that the meeting actually took place in June 1985.97 Petitioner Ocampo argues that he was denied the opportunity to reply to the Supplemental Affidavit by not being furnished a copy thereof. Petitioner Ocampo also claims that he was denied the right to file a motion for reconsideration or to appeal the Resolution of Prosecutor Vivero, because the latter deliberately delayed the service of the Resolution by 19 days, effectively denying petitioner Ocampo his right to due process.98 As to the claim of petitioners Echanis and Baylosis, we quote the pertinent portion of Prosecutor Vivero’s Resolution, which states: In connection with the foregoing and pursuant to the Revised Rules of Criminal Procedure[,] the respondents were issued and served with Subpoena at their last known address for them to submit their counter-affidavits and that of their witnesses. Majority of the respondents did not submit their counter-affidavits because they could no longer be found in their last known address, per return of the subpoenas. On the other hand, Saturnino Ocampo @ Satur, Fides Lim, Maureen Palejaro and Ruben Manatad submitted their CounterAffidavits. However, Vicente Ladlad and Jasmin Jerusalem failed to submit the required Counter Affidavits in spite entry of appearance by their respective counsels.99 Section 3(d), Rule 112 of the Rules of Court, allows Prosecutor Vivero to resolve the complaint based on the evidence before him if a respondent could not be subpoenaed. As long as efforts to reach a respondent were made, and he was given an opportunity to present countervailing evidence, the preliminary investigation remains valid.100 The rule was put in place in order to foil underhanded attempts of a respondent to delay the prosecution of offenses.101 In this case, the Resolution stated that efforts were undertaken to serve subpoenas on the named respondents at their last known addresses. This is sufficient for due process. It was only because a

majority of them could no longer be found at their last known addresses that they were not served copies of the complaint and the attached documents or evidence. Petitioner Ladlad claims that his subpoena was sent to the nonexistent address "53 Sct. Rallos St., QC,"102 which had never been his address at any time.103 In connection with this claim, we take note of the fact that the subpoena to Fides Lim, petitioner Ladlad’s wife,104 was sent to the same address, and that she was among those mentioned in the Resolution as having timely submitted their counteraffidavits. Despite supposedly never receiving a subpoena, petitioner Ladlad’s counsel filed a formal entry of appearance on 8 December 2006.105 Prosecutor Vivero had a reason to believe that petitioner Ladlad had received the subpoena and accordingly instructed his counsel to prepare his defense. Petitioner Ladlad, through his counsel, had every opportunity to secure copies of the complaint after his counsel’s formal entry of appearance and, thereafter, to participate fully in the preliminary investigation. Instead, he refused to participate. We have previously cautioned that "litigants represented by counsel should not expect that all they need to do is sit back, relax and await the outcome of their case."106 Having opted to remain passive during the preliminary investigation, petitioner Ladlad and his counsel cannot now claim a denial of due process, since their failure to file a counter-affidavit was of their own doing. Neither do we find any merit in petitioner Ocampo’s allegation of collusion to surreptitiously insert the Supplemental Affidavit of Zacarias Piedad in the records. There was nothing surreptitious about the Supplemental Affidavit since it clearly alludes to an earlier affidavit and admits the mistake committed regarding the date of the alleged meeting. The date of the execution of the Supplemental Affidavit was also clearly stated. Thus, it was clear that it was executed after petitioner Ocampo had submitted his counter-affidavit. Should the case go to trial, that will provide petitioner Ocampo with the opportunity to question the execution of Zacarias Piedad’s Supplemental Affidavit. Neither can we uphold petitioner Ocampo’s contention that he was denied the right to be heard. For him to claim that he was denied due process by not being furnished a copy of the Supplemental Affidavit of Zacarias Piedad would imply that the entire case of the prosecution rested on the Supplemental Affidavit. The OSG has asserted that the indictment of petitioner Ocampo was based on the collective affidavits of several other witnesses107attesting to the allegation that he was a member of the CPP/NPA/NDFP Central Committee, which had ordered the launch of Operation VD. As to his claim that he was denied the right to file a motion for reconsideration or to appeal the Resolution of Prosecutor Vivero due to the 19-day delay in the service of the Resolution, it must be pointed out that the period for filing a motion for reconsideration or an appeal to the Secretary of Justice is reckoned from the date of receipt of the resolution of the prosecutor, not from the date of the resolution. This is clear from Section 3 of the 2000 National Prosecution Service Rule on Appeal: Sec. 3. Period to appeal. – The appeal shall be taken within fifteen (15) days from receipt of the resolution, or of the denial of the motion for reconsideration/ reinvestigation if one has been filed within fifteen (15) days from receipt of the assailed resolution. Only one motion for reconsideration shall be allowed. (Emphasis supplied) Thus, when petitioner Ocampo received the Resolution of Prosecutor Vivero on 12 March 2007,108 the former had until 27 March 2007 within which to file either a motion for reconsideration before the latter or an appeal before the Secretary of Justice. Instead, petitioner Ocampo chose to file the instant petition for certiorari directly before this Court on 16 March 2007.

B. Issuance of the Warrants of Arrest Article III, Section 2 of the Constitution provides that "no search warrant or warrant of arrest shall issue except upon probable cause to be determined personally by the judge after examination under oath or affirmation of the complainant and the witnesses he may produce." Petitioner Ocampo alleges that Judge Abando did not comply with the requirements of the Constitution in finding the existence of probable cause for the issuance of warrants of arrest against petitioners.109 Probable cause for the issuance of a warrant of arrest has been defined as "such facts and circumstances which would lead a reasonably discreet and prudent man to believe that an offense has been committed by the person sought to be arrested."110 Although the Constitution provides that probable cause shall be determined by the judge after an examination under oath or an affirmation of the complainant and the witnesses, we have ruled that a hearing is not necessary for the determination thereof.111 In fact, the judge’s personal examination of the complainant and the witnesses is not mandatory and indispensable for determining the aptness of issuing a warrant of arrest.112 It is enough that the judge personally evaluates the prosecutor’s report and supporting documents showing the existence of probable cause for the indictment and, on the basis thereof, issue a warrant of arrest; or if, on the basis of his evaluation, he finds no probable cause, to disregard the prosecutor's resolution and require the submission of additional affidavits of witnesses to aid him in determining its existence.113 Petitioners Echanis and Baylosis claim that, had Judge Abando painstakingly examined the records submitted by Prosecutor Vivero, the judge would have inevitably dismissed the charge against them.114 Additionally, petitioner Ocampo alleges that Judge Abando did not point out facts and evidence in the record that were used as bases for his finding of probable cause to issue a warrant of arrest.115 The determination of probable cause for the issuance of warrants of arrest against petitioners is addressed to the sound discretion of Judge Abando as the trial judge.116 Further elucidating on the wide latitude given to trial judges in the issuance of warrants of arrest, this Court stated in Sarigumba v. Sandiganbayan117 as follows: x x x. The trial court's exercise of its judicial discretion should not, as a general rule, be interfered with in the absence of grave abuse of discretion. Indeed, certiorari will not lie to cure errors in the trial court's appreciation of the evidence of the parties, the conclusion of facts it reached based on the said findings, as well as the conclusions of law. x x x. Whether or not there is probable cause for the issuance of warrants for the arrest of the accused is a question of fact based on the allegations in the Informations, the Resolution of the Investigating Prosecutor, including other documents and/or evidence appended to the Information. Here, the allegations of petitioners point to factual matters indicated in the affidavits of the complainants and witnesses as bases for the contention that there was no probable cause for petitioners’ indictment for multiple murder or for the issuance of warrants for their arrest. As stated above, the trial judge’s appreciation of the evidence and conclusion of facts based thereon are not interfered with in the absence of grave abuse of discretion. Again, "he sufficiently complies with the requirement of personal determination if he reviews the [I]nformation and the documents attached

thereto, and on the basis thereof forms a belief that the accused is probably guilty of the crime with which he is being charged."118 Judge Abando’s review of the Information and the supporting documents is shown by the following portion of the judge’s 6 March 2007 Order: On the evaluation of the Resolution and its Information as submitted and filed by the Provincial Prosecution of Leyte Province supported by the following documents: Affidavits of Complainants, Sworn Statements of Witnesses and other pertinent documents issued by the Regional Crime Laboratory Office, PNP, Region VIII and Camp Crame, Quezon City, pictures of the grave site and skeletal remains, this court has the findings [sic] of probable cause in the commission by all mentioned accused of the crime charged.119 At bottom, issues involving the finding of probable cause for an indictment and issuance of a warrant of arrest, as petitioners are doubtless aware, are primarily questions of fact that are normally not within the purview of a petition for certiorari,120 such as the petitions filed in the instant consolidated cases. The political offense doctrine is not a ground to dismiss the charge against petitioners prior to a determination by the trial court that the murders were committed in furtherance of rebellion. Under the political offense doctrine, "common crimes, perpetrated in furtherance of a political offense, are divested of their character as "common" offenses and assume the political complexion of the main crime of which they are mere ingredients, and, consequently, cannot be punished separately from the principal offense, or complexed with the same, to justify the imposition of a graver penalty."121 Any ordinary act assumes a different nature by being absorbed in the crime of rebellion.122 Thus, when a killing is committed in furtherance of rebellion, the killing is not homicide or murder. Rather, the killing assumes the political complexion of rebellion as its mere ingredient and must be prosecuted and punished as rebellion alone. However, this is not to say that public prosecutors are obliged to consistently charge respondents with simple rebellion instead of common crimes. No one disputes the well-entrenched principle in criminal procedure that the institution of criminal charges, including whom and what to charge, is addressed to the sound discretion of the public prosecutor.123 But when the political offense doctrine is asserted as a defense in the trial court, it becomes crucial for the court to determine whether the act of killing was done in furtherance of a political end, and for the political motive of the act to be conclusively demonstrated.124 Petitioners aver that the records show that the alleged murders were committed in furtherance of the CPP/NPA/NDFP rebellion, and that the political motivation behind the alleged murders can be clearly seen from the charge against the alleged top leaders of the CPP/NPA/NDFP as coconspirators. We had already ruled that the burden of demonstrating political motivation must be discharged by the defense, since motive is a state of mind which only the accused knows.125 The proof showing

political motivation is adduced during trial where the accused is assured an opportunity to present evidence supporting his defense. It is not for this Court to determine this factual matter in the instant petitions. As held in the case of Office of the Provincial Prosecutor of Zamboanga Del Norte v. CA,126 if during trial, petitioners are able to show that the alleged murders were indeed committed in furtherance of rebellion, Section 14, Rule 110 of the Rules of Court provides the remedy, to wit: SECTION 14. Amendment or substitution. — A complaint or information may be amended, in form or in substance, without leave of court, at any time before the accused enters his plea. After the plea and during the trial, a formal amendment may only be made with leave of court and when it can be done without causing prejudice to the rights of the accused. However, any amendment before plea, which downgrades the nature of the offense charged in or excludes any accused from the complaint or information, can be made only upon motion by the prosecutor, with notice to the offended party and with leave of court. The court shall state its reasons in resolving the motion and copies of its order shall be furnished all parties, especially the offended party. (n) If it appears at any time before judgment that a mistake has been made in charging the proper offense, the court shall dismiss the original complaint or information upon the filing of a new one charging the proper offense in accordance with Section 19, Rule 119, provided the accused shall not be placed in double jeopardy. The court may require the witnesses to give bail for their appearance at the trial. (Emphasis supplied) Thus, if it is shown that the proper charge against petitioners should have been simple rebellion, the trial court shall dismiss the murder charges upon the filing of the Information for simple rebellion, as long as petitioners would not be placed in double jeopardy. Section 7, Rule 117 of the Rules of Court, states: SEC. 7. Former conviction or acquittal; double jeopardy. — When an accused has been convicted or acquitted, or the case against him dismissed or otherwise terminated without his express consent by a court of competent jurisdiction, upon a valid complaint or information or other formal charge sufficient in form and substance to sustain a conviction and after the accused had pleaded to the charge, the conviction or acquittal of the accused or the dismissal of the case shall be a bar to another prosecution for the offense charged, or for any attempt to commit the same or frustration thereof, or for any offense which necessarily includes or is necessarily included in the offense charged in the former complaint or information. Based on the above provision, double jeopardy only applies when: (1) a first jeopardy attached; (2) it has been validly terminated; and (3) a second jeopardy is for the same offense as in the first.127 A first jeopardy attaches only after the accused has been acquitted or convicted, or the case has been dismissed or otherwise terminated without his express consent, by a competent court in a valid indictment for which the accused has entered a valid plea during arraignment.128 To recall, on 12 May 2006, an Information for the crime of rebellion, as defined and penalized under Article 134 in relation to Article 135 of the Revised Penal Code, docketed as Criminal Case No. 06944 was filed before the RTC Makati against petitioners and several others.129

However, petitioners were never arraigned in Criminal Case No. 06-944. Even before the indictment for rebellion was filed before the RTC Makati, petitioners Ocampo, Echanis and Ladlad had already filed a petition before this Court to seek the nullification of the Orders of the DOJ denying their motion for the inhibition of the members of the prosecution panel due to lack of impartiality and independence.130 When the indictment was filed, petitioners Ocampo, Echanis and Ladlad filed supplemental petitions to enjoin the prosecution of Criminal Case No. 06-944.131 We eventually ordered the dismissal of the rebellion case. It is clear then that a first jeopardy never had a chance to attach. 1awp++i1

Petitioner Ocampo shall remain on provisional liberty under the ₱100,000 cash bond posted before the Office of the Clerk of Court. He shall remain on provisional liberty until the termination of the proceedings before the RTC Manila. 1âwphi1

The OSG has given its conformity to the provisional liberty of petitioners Echanis, Baylosis and Ladlad in view of the ongoing peace negotiations. Their provisional release from detention under the cash bond of ₱100,000 each shall continue under the condition that their temporary release shall be limited to the period of their actual participation as CPP-NDF consultants in the peace negotiations with the government or until the termination of the proceedings before the RTC Manila, whichever is sooner. It shall be the duty of the government to inform this Court the moment that peace negotiations are concluded. WHEREFORE, the instant consolidated petitions are DISMISSED. The RTC of Manila, Branch 32, is hereby ORDERED to proceed with dispatch with the hearing of Criminal Case No. 08-262163. Petitioner Saturnino C. Ocampo shall remain on temporary liberty under the same bail granted by this Court until the termination of the proceedings before the RTC Manila. Petitioners Randall B. Echanis, Rafael G. Baylosis and Vicente P. Ladlad shall remain on temporary liberty under the same bail granted by this Court until their actual participation as CPP-NDF consultants in the peace negotiations with the government are concluded or terminated, or until the termination of the proceedings before the RTC Manila, whichever is sooner. .R. No. L-45950

June 20, 1938

LEONA PASION VIUDA DE GARCIA, petitioner, vs. DIEGO LOCSIN, Judge of First Instance of Tarlac, FELIX IMPERIAL, Provincial Fiscal of Tarlac, and the ANTI-USURY BOARD, respondents. Benigo S. Aquino and Marcial P. Lichauco for petitioner Adolfo N. Feliciano for the respondent Anti-Usury Board. Office of the Solicitor-General Tuason for other respondents. LAUREL, J.: This is a petition for mandamus presented to secure the annulment of a search warrant and two orders of the respondent judge, and the restoration of certain documents alleged to have been illegally seized by an agent of the Anti-Usuary Board. It appears that on November 10, 1934, Mariano G. Almeda, an agent of the Anti-Usuary Board, obtained from the justice of the peace of Tarlac, Tarlac, a search warrant(Exhibit B) commanding any officer of the law to search the person, house or store of the petitioner at Victoria, Tarlac, for "certain books, lists, chits, receipts, documents and other papers relating to her activities as usurer." The search warrant was issued upon an affidavit given by the said Almeda "that he has and there

(is) just and probable cause to believe and he does believe that Leona Pasion de Garcia keeps and conceals in her house and store at Victoria, Tarlac, certain books, lists, chits, receipts, documents, and other papers relating to her activities as usurer, all of which is contrary to the statute in such cases made and provided." On the same date, the said Mariano G. Almeda, accompanied by a captain of the Philippine Constabulary, went to the office of the petitioner in Victoria, Tarlac and, after showing the search warrant to the petitioner's bookkeeper, Alfredo Salas, and, without the presence of the petitioner who was ill and confined at the time, proceeded with the execution thereof. Two packages of records and a locked filing cabinet containing several Papers and documents were seized by Almeda and a receipt therefor issued by him to Salas. The papers and documents seized were kept for a considerable length of time by the Anti-Usury Board and thereafter were turned over by it to the respondent fiscal who subsequently filed, in the Court of First Instance of Tarlac, six separate criminal cases against the herein petitioner for violation of the AntiUsury Law. On several occasions, after seizure, the petitioner, through counsel, demanded from the respondent Anti-Usury Board the return of the documents seized. On January 7. and, by motion, on June 4, 1937, the legality of the search warrant was challenged by counsel for the petitioner in the six criminal cases and the devolution of the documents demanded. By resolution of October 5, 1937, the respondent Judge of First Instance denied the petitioner's motion of June 4 for the reason that though the search warrant was illegal, there was a waiver on the part of the petitioner. "En el caso presente," declared the respondent judge, "teniendo en cuenta que la acusada Por si o por medio de su representante, no presento protests alguna contra el registro de autos, at verificarse el mismo, o despues de un tiempo rezonable, el juzgado declare que la citada con su silencio y conducta, ha renunciado implicitanmente a su derecho a no ser sometido a un registro irrazonable, por lo que no le es pemitido quejarse despues, puesto que cualquier defecto queha adolecido lo expedicion de la orden de registro y su ejecucion, ha quidado implilcitamente subsanado." A motion for reconsideration was presented but was denied by order of January 3, 1938. Petitioner registered her exception. The resolution of October 5, 1937 and the order of January 3, 1938 are sought, together with the search warrant, Exhibit B, to be nullified in these proceedings. Paragraph 3, section 1 of the bill of right of our Constitution provides as follows: The right of the people to be secure in their persons, houses, papers, and effects against unreasonable searches and seizures shall not be violated, and no warrants shall issue but upon probable cause, to be judge after examination under oath or affirmation of the complainant and the witnesses he may produce, and particularly describing the place to be searched, and the persons or things to be seized. Freedom from unreasonable searches and seizures is declared a popular right and for a search warrant to be valid, (1) it must be issued upon probable cause; (2) the probable cause must be determined by the judge himself and not by the applicant or any other person; (3) in the determination of probable cause, the judge must examine, under oath or affirmation, the complainant and such witnesses as the latter may produce; and (4) the warrant issued must particularly describe the place to be searched and persons or things to be seized. These requirements are complemented by the Code of Criminal Procedure (G. O. No. 58), particularly with reference to the duration of the validity of the search warrant and the obligation of the officer seizing the property to deliver the same to the corresponding court (secs. 102-104). On more than one occasion, since the approval of the Constitution, we had emphasized the necessity of adherence to the constitutional requirements on this subject (Alvarez vs. Court of First Instance of Tayabas and Anti-Usury Board [1937], 35 Off. Gaz., 1183; People vs. Sy Juco [1937], G.R. No. 41957; Rodriguez vs. Villamiel [1937], G.R. No. 44328; and Molo vs. Yatco [1936], 35 Off. Gaz., 1935) and we do not deem it necessary to reiterate what has been said or observed in these cases. In the instant case the existence of probable cause was determined not by the judge himself but by the applicant. All that the judge did was to accept as true the affidavit made by agent Almeda. He did

not decide for himself. It does not appear that he examined the applicant and his witnesses, if any. Even accepting the description of the properties to be seized to be sufficient and on the assumption that the receipt issued is sufficiently detailed within the meaning of the law, the properties seized were not delivered to the court which issued the warrant, as required by law. (See, secs. 95 and 104, G. O. No. 58.) instead, they were turned over to the respondent provincial fiscal and used by him in building up cases against the petitioner. Considering that at the time the warrant was issued there was no case pending against the petitioner, the averment that the warrant was issued primarily for exploration purposes is not without basis. The lower court is, therefore, correct in reaching the conclusion that the search warrant (Exhibit B) was illegally issued by the justice of the peace of Tarlac, Tarlac. The important question presented is whether upon the facts and under the circumstances of the present case, there has been a waiver by the petitioner of her constitutional immunity against unreasonable searches and seizures. While the Solicitor-General admits that, in the light of decisions of this court, the search warrant was illegally issued, he maintains "(1) that the petitioner had waived her constitutional right by her acquiescence after the search and seizure, and (2) that the application for the return of the documents illegally seized was made after an unreasonable length of time after the date of seizure." Doubtless, the constitutional immunity against unreasonable searches and seizures is a personal right which may be waived. (People vs. Kagui Malasugui, 34 Off. Gaz., pp. 2163, 2164; 56 C.J., pp. 1178, 1179; Cf. Rodriguez vs. Villamiel, supra.) The waiver may be either express or implied (67 C.J., p. 304). No express waiver has been made in the case before us. It is urged, however, that there has been a waiver by implication. It is well-settled that to constitute a waiver of a constitutional right, it must appear, first, that the right exists; secondly, that the persons involved had knowledge, either actual or constructive, of the existence of such right; and, lastly, that said person had an actual intention to relinquish the right. (67 C. J., 299.) It is true that the petitioner did not object to the legality of the search when it was made. She could not have objected because she was sick and was not present when the warrant was served upon Alfredo Salas. Certainly, the constitutional immunity from unreasonable searches and seizures, being a personal one, cannot be waived by anyone except the person whose rights are invaded or one who is expressly authorized to do so in his or her behalf. (56 C. J., p. 1183.) Of course, the petitioner came to know later of the seizure of some of her papers and documents. But this was precisely the reason why she sent her attorneys to the office of the Anti-Usuary Board to demand the return of the documents seized. In any event, the failure on the part of the petitioner and her bookkeeper to resist or object to the execution of the warrant does not constitute an implied waiver of constitutional right. It is, as Judge Cooley observes, but a submission to the authority of the law. (Const. Lim., 8th ed., Vol., I, p. 630.) As the constitutional guaranty is not dependent upon any affirmative act of the citizen, the courts do not place the citizen in the position of either contesting an officer's authority by force, or waiving his constitutional rights; but instead they hold that a peaceful submission to a search or seizure is not a consent or an invitation thereto, but is merely a demonstration of regard for the supremacy of the law. (56 C.J., pp. 1180, 1181.) As a general proposition, it may be admitted that waiver may be the result of a failure to object within a reasonable time to a search and seizure illegally made. It must be observed, however, that the petitioner, on several occasions, and prior to the filing of criminal actions against her, had demanded verbally, through counsel, the return by the Anti-Usuary Board of the properties seized. This is admitted by Adolfo N. Feliciano, acting chief of the board, who said that the demand was refused simply because no habiamos terminado con nuestra investigacion. (T.s.n., pp. 24-25.) On July 7, 1936, counsel for the petitioner wrote a letter to the Anti-Usuary Board demanding again the return of the documents withheld. And in connection with the criminal cases pending against the petitioner, similar demands were made on January 7, 1937 and on June 4, 1937. In the light of these circumstances, we find that the petitioner did not waive her constitutional right. The delay in making demand for the return of the documents seized is not such as to result in waiver by implication.

In view of the foregoing, the writ prayed for is granted. The search warrant, Exhibit B, is hereby declared void and of no effect; the orders of October 5, 1937 and January 3, 1938 of the respondent judge are set aside; and the respondents Anti-Usuary Board and the provincial fiscal of Tarlac or those acting in their behalf, are hereby ordered to return and restore to the petitioner all the properties, documents, papers and effects illegally seized from her, within forty-eight (48) hours from the time this decision becomes final. Without costs. So ordered.

1. Requirements for Search Warrants

Yee Sue Kuy v. Almeda, 70 Phil. 141 (1940) F:

By virtue of the sworn application of Almeda, the Chief agent of the Anti-Usury Board, a SW was issued to search the store and premises of the petitioner, accused of violating the Anti-Usury Law. Receipt books, PNs and other articles were seized and retained in the possession of the Anti-Usury Board.

ISSUE: W/n the requirements for the issuance of valid SW were complied with. RULING: YES. The applicant, Almeda, in his application, swore that "he made his own personal investigation and ascertained that petitioner is lending money without a license, charging usurious rates." The witness he presentted testified before the judge and swore that he knew that YEE was lending without a license because he personally investigated the victims who secured loans from the petitioner. Their affidavits were sufficient for, thereunder, they could be held liable for perjury. The existence of probable cause has been determined by the justice of the peace before issuing the warrant complained of, as shown in the warrant itself. Charo. [G.R. No. 50720. March 26, 1984.] SORIANO MATA, Petitioner, v. HON. JOSEPHINE K. BAYONA, in her capacity as Presiding Judge of the City Court of Ormoc, BERNARDO GOLES and REYNALDO MAYOTE, Respondents. Valeriano R. Ocubillo for Petitioner. The Solicitor General for Respondents.

SYLLABUS

1. CONSTITUTIONAL LAW; BILL OF RIGHTS; RIGHT AGAINST UNLAWFUL SEARCH AND SEIZURE; REQUISITES FOR ISSUANCE OF SEARCH WARRANT. — Under the Constitution "no search warrant shall issue but upon probable cause to be determined by the Judge or such other responsible officer as may be authorized by law after examination under oath or affirmation of the complainant and the witnesses he may produce." More emphatic and detailed is the implementing rule of the constitutional injunction, Section 4 of Rule 126 which provides that the judge must before issuing the warrant personally examine on oath or affirmation the complainant and any witnesses he may produce and take their depositions in writing, and attach them to the record, in addition to any affidavits presented to him. 2. ID.; ID.; ID.; ID.; INSUFFICIENCY OF AFFIDAVITS OF COMPLAINANT AND HIS WITNESSES IN THE CASE AT BAR. — Before issuing a search warrant, the examining Judge has to take depositions in writing of the complainant and the witnesses he may produce and to attach them to the record. Such written deposition is necessary in order that the Judge may be able to properly determine the existence or non-existence of the probable cause, and to hold liable for perjury the person giving it if it will be found later that his declarations are false. Mere affidavits of the complainant and his witnesses are thus not sufficient. 3. ID.; ID.; ID.; ID.; NO "DEPOSITION IN WRITING" ATTACHED TO RECORDS OF CASE IN CASE AT BAR. —

The judge’s insistence that she examined the complainants under oath has become dubious by petitioner’s claim that at the particular time when he examined all the relevant papers connected with the issuance of the questioned search warrant, after he demanded the same from the lower court since they were not attached to the records, he did not find any certification at the back of the joint affidavit of the complainants. Before he filed his motion to quash the search warrant and for the return of the articles seized, he was furnished, upon his request, certified true copies of the said affidavits by the Clerk of Court but which certified true copies do not bear any certification at the back. Petitioner likewise claims that his xerox copy of the said joint affidavit obtained at the outset of this case does not show also the certification of respondent judge. This doubt becomes more confirmed by respondent Judge’s own admission, while insisting that she did examine thoroughly the applicants, that "she did not take the deposition of Mayote and Goles because to have done so would be to hold a judicial proceeding which will be open and public", such that, according to her, the persons subject of the intended raid will just disappear and move his illegal operations somewhere else. Could it be that the certification was made belatedly to cure the defect of the warrant? Be that as it may, there was no "deposition in writing" attached to the records of the case in palpable disregard of the statutory prohibition heretofore quoted. 4. ID.; ID.; ID.; ID.; DEPOSITIONS, HOW TAKEN. — The searching questions propounded to the applicants of the search warrant and his witnesses must depend to a large extent upon the discretion of the Judge just as long as the answers establish a reasonable ground to believe the commission of a specific offense and that the applicant is one authorized by law, and said answers particularly describe with certainty the place to be searched and the persons or things to be seized. The examination or investigation which must be under oath may not be in public. It may even be held in the secrecy of his chambers. Far more important is that the examination or investigation is not merely routinary but one that is thorough and elicit the required information. To repeat, it must be under oath and must be in writing. 5. ID.; ID.; ID.; ID.; MUST BE STRICTLY COMPLIED WITH; CASE AT BAR. — Nothing can justify the issuance of the search warrant but the fulfillment of the legal requisites. Thus, in issuing a search warrant the Judge must strictly comply with the requirements of the Constitution and the statutory provisions. In the case at bar, the search warrant is tainted with illegality by the failure of the Judge to conform with essential requisites of taking the depositions in writing and attaching them to record, rendering the search warrant invalid. 6. ID.; ID.; ID.; ALTHOUGH ILLEGAL, THINGS SEIZED CANNOT BE RETURNED; CASE AT BAR. — While the search warrant is illegal, the return of the things seized cannot be ordered. In Castro v. Pabalan (70 SCRA 478), it was held that the illegality of the search warrant does not call for the return of the things seized, the possession of which is prohibited.

DECISION

DE CASTRO, J.:

The validity of the search warrant issued by respondent Judge (not reappointed) is challenged by petitioner for its alleged failure to comply with the requisites of the Constitution and the Rules of Court. Specifically, the contention is that the search warrant issued by respondent Judge was based merely on the application for search warrant and a joint affidavit of private respondents which were wrongfully it is alleged subscribed, and sworn to before the Clerk of Court of respondent Judge. Furthermore, there was allegedly a failure on the part of respondent Judge to attach the necessary papers pertinent to the issuance of the search warrant to the records of Criminal Case No. 4298-CC wherein petitioner is accused under PD 810, as amended by PD 1306, the information against him alleging that Soriano Mata offered, took and arranged bets on the Jai Alai game by "selling illegal tickets known as ‘Masiao tickets’ without any authority from the Philippine Jai Alai & Amusement Corporation or from the government authorities concerned." 1 Petitioner claims that during the hearing of the case, he discovered that nowhere from the records of the said case could be found the search warrant and other pertinent papers connected to the issuance of the same, so that he had to inquire from the City Fiscal its whereabouts, and to which inquiry respondent Judge replied, "it is with the court." The Judge then handed the records to the Fiscal who attached them to the records. chan roble s.com : vi rtual law lib rary

This led petitioner to file a motion to quash and annul the search warrant and for the return of the articles seized, citing and invoking, among others, Section 4 of Rule 126 of the Revised Rules of Court. The motion was denied by respondent Judge on March 1, 1979, stating that the court has made a thorough investigation and examination under oath of Bernardo U. Goles and Reynaldo T. Mayote, members of the Intelligence Section of 352nd PC Co./Police District II INP; that in fact the court made a certification to that effect; and that the fact that documents relating to the search warrant were not attached immediately to the record of the criminal case is of no moment, considering that the rule does not specify when these documents are to be attached to the records. 2 Petitioner’s motion for reconsideration of the aforesaid order having been denied, he came to this Court, with the instant petition, praying, among others, that this Court declare the search warrant to be invalid and all the articles confiscated under such warrant as inadmissible as evidence in the case, or in any proceedings on the matter. We hold that the search warrant is tainted with illegality for being violative of the Constitution and the Rules of Court. Under the Constitution "no search warrant shall issue but upon probable cause to be determined by the Judge or such other responsible officer as may be authorized by law after examination under oath or affirmation of the complainant and the witnesses he may produce." More emphatic and detailed is the implementing rule of the constitutional injunction, Section 4 of Rule 126 which provides that the judge must before issuing the warrant personally examine on oath or affirmation the complainant and any witnesses he may produce and take their depositions in writing, and attach them to the record, in addition to any affidavits presented to him. Mere affidavits of the complainant and his witnesses are thus not sufficient. The examining Judge has to take depositions in writing of the complainant and the witnesses he may produce and to attach them to the record. Such written deposition is necessary in order that the Judge may be able to properly determine the existence or non-existence of the probable cause, to hold liable for perjury the person giving it if it will be found later that his declarations are false. We, therefore, hold that the search warrant is tainted with illegality by the failure of the Judge to conform with the essential requisites of taking the depositions in writing and attaching them to the record, rendering the search warrant invalid. chan roblesv irt u alawlibra ry

The judge’s insistence that she examined the complainants under oath has become dubious by petitioner’s claim that at the particular time when he examined all the relevant papers connected with the issuance of the questioned search warrant, after he demanded the same from the lower court since they were not attached to the records, he did not find any certification at the back of the joint affidavit of the complainants. As stated earlier, before he filed his motion to quash the search warrant and for the return of the articles seized, he was furnished, upon his request, certified true copies of the said affidavits by the Clerk of Court but which certified true copies do not bear any certification at the back. Petitioner likewise claims that his xerox copy of the said joint affidavit obtained at the outset of this case does not show also the certification of respondent judge. This doubt becomes more confirmed by respondent Judge’s own admission, while insisting that she did examine thoroughly the applicants, that "she did not take the deposition of Mayote and Goles because to have done so would be to hold a judicial proceeding which will be open and public", 3 such that, according to her, the persons subject of the intended raid will just disappear and move his illegal operations somewhere else. Could it be that the certification was made belatedly to cure the defect of the warrant? Be that as it may, there was no "deposition in writing" attached to the records of the case in palpable disregard of the statutory prohibition heretofore quoted. Respondent Judge impresses this Court that the urgency to stop the illegal gambling that lures every man, woman and child, and even the lowliest laborer who could hardly make both ends meet justifies her action. She claims that in order to abate the proliferation of this illegal "masiao" lottery, she thought it more prudent not to conduct the taking of deposition which is done usually and publicly in the court room. Two points must be made clear. The term "depositions" is sometimes used in a broad sense to describe any written statement verified by oath; but in its more technical and appropriate sense the meaning of the word is limited to written testimony of a witness given in the course of a judicial proceeding in advance of the trial or hearing upon oral examination. 4 A deposition is the testimony of a witness, put or taken in writing, under oath or affirmation before a commissioner, examiner or other judicial officer, in answer to

interlocutory and cross interlocutory, and usually subscribed by the witnesses. 5 The searching questions propounded to the applicants of the search warrant and his witnesses must depend to a large extent upon the discretion of the Judge just as long as the answers establish a reasonable ground to believe the commission of a specific offense and that the applicant is one authorized by law, and said answers particularly describe with certainty the place to be searched and the persons or things to be seized. The examination or investigation which must be under oath may not be in public. It may even be held in the secrecy of his chambers. Far more important is that the examination or investigation is not merely routinary but one that is thorough and elicit the required information. To repeat, it must be under oath and must be in writing. c ralawnad

The other point is that nothing can justify the issuance of the search warrant but the fulfillment of the legal requisites. It might be well to point out what has been said in Asian Surety & Insurance Co., Inc. v. Herrera: jgc: chan roble s.com.p h

"It has been said that of all the rights of a citizen, few are of greater importance or more essential to his peace and happiness than the right of personal security, and that involves the exemption of his private affairs, books, and papers from inspection and scrutiny of others. While the power to search and seize is necessary to the public welfare, still it must be exercised and the law enforced without transgressing the constitutional rights of the citizens, for the enforcement of no statute is of sufficient importance to justify indifference to the basic principles of government." 6 Thus, in issuing a search warrant the Judge must strictly comply with the requirements of the Constitution and the statutory provisions. A liberal construction should be given in favor of the individual to prevent stealthy encroachment upon, or gradual depreciation of the rights secured by the Constitution. 7 No presumption of regularity are to be invoked in aid of the process when an officer undertakes to justify it. 8 While We hold that the search warrant is illegal, the return of the things seized cannot be ordered. In Castro v. Pabalan, 9 it was held that the illegality of the search warrant does not call for the return of the things seized, the possession of which is prohibited. WHEREFORE, the writ of certiorari is granted and the order of March 1, 1979 denying the motion to annul the search warrant as well as the order of March 21, 1979 denying the motion for reconsideration are hereby reversed, the search warrant, being declared herein as illegal. Notwithstanding such illegality, the things seized under such warrant, such as stock of "masiao" tickets; "masiao" issue tickets; bet money; control pad or "masiao" numbers; stamping pad with rubber stamp marked Ormoc City Jai-Alai," cannot be returned as sought by petitioner. No costs.

G.R. Nos. 78347-49 November 9, 1987 ADOLFO OLAES and LINDA M. CRUZ, petitioners, vs. PEOPLE OF THE PHILIPPINES and HON. JUDGE ALICIA L. SANTOS (In her capacity as Presiding Judge of the Regional Trial Court of Olongapo City, Branch 73), respondents.

CRUZ, J: In this petition for certiorari and prohibition with preliminary injunction, the petitioners challenge the admission by the respondent judge of evidence seized by virtue of an allegedly invalid March warrant and of an extrajudicial confession taken from them without according them the right to assistance of counsel. 1 They seek to restrain further proceedings in the criminal case against them for violation of the Dangerous Drugs Act (which we have suspended) 2 and ask that they be acquitted with the setting aside of the questioned orders.

The Solicitor General, in his Comment, suggests that the petition should be dismissed as it is not alleged therein that the respondent judge has committed grave abuse of discretion or acted without or in excess of jurisdiction. He adds that if any reversible error has been committed, it may be

corrected not in this petition but in an ordinary appeal, which may not even be necessary if the petitioners are exonerated. 3 The petitioners, in their Reply, do not meet these arguments head-on, thus impliedly admitting the formal defect in their petition, but subject that technicalities should yield to substantial questions in the interest of justice and to avoid unnecessarilyor protracted litigation. Their contention is that since there are important constitutional issues involved, these questions should disposition of their case 4 be decided in this petition instead of having them debated and resolved first in the lower court in acconce with the usual procedure, to the prejudice of the speedy We are not usually persuaded by this kind of argument, since procedural rules are intended precisely to insure an orderly administration of justice. Rights are best established in accordance with the procedure laid down by the adjective law, which is as binding on the parties as the substantive law since they are supposed to complement each other. The Solicitor General is obviously correct in faulting the petition and in contending that, besides being defective, it is not the proper remedy at this time. There is no disputing this stand. Worthy of note in this connection is The separate opinion of the present Chief Justice in Joseph v. Vilialuz, 5 where he declared that: . . . the Court adheres to the settled rule that it will not overrule in a special civil action the trial court's interlocutory order denying a motion to dismiss for failure or insufficiency of the prosecution's evidence since it cannot review in such special civil action the prosution's evidence and decide here and now in advance that it has or has not established beyond reasonable doubt the guilt of the petitioners-accused. The orderly procedure prescribed by the Rules of Court is for the accused to present their evidence after which the trial court will on the basis of the evidence presented before it by both the prosecution and the defense render its judgment of conviction or acquittal. If the verdict be one of acquittal, the case ends there. If it be a verdict of conviction, then appeal is the proper remedy - and such appeal in order to have a review of the trial court's findings of fact hes within the exclusive appellate jurisdiction of the Court of Appeals. We reiterate the rule here. Even so, the Court has decided, without detracting from the validity of the above-cited observations., to deviate from the established procedure on this matter and to categorically resolve the issues presented iii the case before us. The challenged orders are, indeed, interlocutory. Nevertheless, a restatement of the principles governing such issues wilt it is expected, simplify the proceedings in the court. below and speed up the disposition of the criminal case against the petitioners. The petitioners claim that the search warrant issued by the respondent judge is unconstitutional because it does not indicate the specific offense they are supposed to have committed. There is, therefore, according to them, no valid finding of probable cause as a justification for the issuance of the said warrant in conformity with the Bill of Rights. In support of this argument, they cite Stonehill v. Diokno, 6 where Chief Justice Concepcion struck down the search warrants issued therein for being based on the general allegation that the petitioners had committed violations of "Central Bank Laws, Tariff and Customs Laws, Internal Revenue Code and Revised Penal Code." He declared: In other words, no specific offense had been alleged in said applications. The averments thereof with respect to the offense committed were abstract. As a consequence, it was impossible for the judges who issued the warrants to have

found the existence of probable cause, for the same presupposes the introduction of competent proof that the party against whom it is sought has performed particular acts, or committed specific omissions, violating a given provision of our criminal law. We have examined the search warrant issued in the instant case and find it does not come under the structures of the Stonehill doctrine. In the case cited, there was a bare reference to the laws in general, without any specification of the particular sections thereof that were alleged to have been violated out of the hundreds of prohibitions contained in such modifications. There is no similar ambiguity in the instant case. While it is true that the caption of the search warrant states that it is in connection with "Violation of RA 6425, otherwise known as the Dangerous Drugs Acts of 1972," it is clearly recited in the text thereof that "There is probable cause to believe that Adolfo Olaes alias "Debie" and alias "Baby" of No. 628 Comia St., Filtration, Sta. Rita, Olongapo City, has in their possession and control and custody of marijuana dried stalks/leaves/seeds/cigarettes and other regulated/prohibited and exempt narcotics preparations which is the subject of the offense stated above." 7 Although the specific section of the Dangerous Drugs Act is not pinpointed, there is no question at all of the specific offense alleged to have been committed as a basis for the finding of probable cause. The search warrant also satisfies the requirement in the Bill of Rights of the particularity of the description to be made of the "place to be searched and the persons or things to be seized." The petitioners also fault the admission of the extrajudicial confessions which they had given without the assistance or advice of counsel and cite Section 20 of the Bill of Rights of the 1973 Constitution providing that "any confession obtained in violation of this section shall be inadmissible in evidence." In the separate sworn statements taken from Adolfo Olaes and Linda Cruz on September 24, 1982, 8 it appears that both petitioners were, before being examined, specifically informed of their right to the assistance of counsel, which would be provided them by the investigating office at their request. Asked if they understood, they said "Opo" and affixed their signatures opposite their answer. This was followed by a statement entitled "Pagpapatunay" or Verification in which they said inter alia that they did not need the assistance of counsel ("Hindi ko na kailangan and tulong ng isang manananggol.") which they also signed. It was only after these preliminary precautions had been taken that the interrogation began and was recorded in the sworn statement later introduced against them at their trial. There is no claim that any force, violence, intimidation or threat or any means vitiating the free wig was employed against them. Their only objection to the extrajudicial confessions is that they were obtained without the assistance of counsel. They do not aver in their petition that they were not apprised of their right to counsel or that they were denied the assistance of counsel when they asked for it, or, indeed, that they had asked for it. Even so, their investigation did not conform to the requirements laid down in People v. Galit, 9 where we declared: At the time a person is arrested, it shall be the duty of the arresting officer to inform him of the reason for the arrest and he must be shown the warrant of arrest, if any, He shall be informed of his constitutional rights to remain silent and to counsel, and that any statement he might make could be used against him. The person arrested shall have the right to communicate with his lawyer, a relative, or anyone he chooses by the most expedient means by telephone if possible — or by letter or messenger. It shall be the responsibility of the arresting officer to see to it that this is accomplislied. No custodial investigation shall be conducted unless it be in the presence of coursel engaged by the person arrested, by any person on his behalf, or appointed by the court upon petition either of the detainee himself or by anyone on his behalf. The

right to counsel may be waived but the waiver shall not be valid unless made with the assistance of counsel. Any statement obtained in violation of the procedure herein laid down, whether exculpatory or inculpatory, in whole or in part, shall be inadmissible in evidence. These requirements were made even stricter under Article III, Section 12 of the 1987 Constitution, providing as follows: Sec. 12. (1) Any person under investigation for the commission offense shall have the right to be informed of his right to remain silent and to have competent and independent counsel preferally of his own choice. If the person cannot afford the services of counsel, he must be provided with one. These rights cannot be waived except in writing and in the presence of counsel. (3) Any confession or admission obtained in violation of this or Section 17 hereof shall be inadmissible in evidence against him. Applying the above rules, we reach the conclusion that the extrajudicial confessions should be declared inadmissible as evidence against the herein petitioners. WHEREFORE, the petition is partly granted. The extrajudicial confessions are excluded but the articles seized under the challenged search warrant may be admitted in evidence. Our temporary restraining order of May 25, 1987, is lifted. No costs. G.R. No. 82870 December 14, 1989 DR. NEMESIO E. PRUDENTE, petitioner, vs. THE HON. EXECUTIVE JUDGE ABELARDO M. DAYRIT, RTC Manila, Branch 33 and PEOPLE OF THE PHILIPPINES, respondents. Francisco SB Acejas III, Oscar S. Atencio, Rodolfo M. Capocyan, Ernesto P. Fernandez, Romulo B. Macalintal, Rodrigo H. Melchor, Rudegelio D. Tacorda Virgilio L. Valle and Luciano D. Valencia for petitioner.

PADILLA, J.: This is a petition for certiorari to annul and set aside the order of respondent Judge dated 9 March 1988 which denied the petitioner's motion to quash Search Warrant No. 87-14, as well as his order dated 20 April 1988 denying petitioner's motion for reconsideration of the earlier order. It appears that on 31 October 1987, P/Major Alladin Dimagmaliw, Chief of the Intelligence Special Action Division (ISAD) of the Western Police District (WPD) filed with the Regional Trial Court (RTC) of Manila, Branch 33, presided over by respondent Judge Abelardo Dayrit, now Associate Justice of the Court of Appeals. an application1 for the issuance of a search warrant, docketed therein as SEARCH WARRANT NO. 87-14, for VIOLATION OF PD NO. 1866 (Illegal Possession of Firearms, etc.) entitled "People of the Philippines, Plaintiff, versus Nemesis E. Prudente, Defendant." In his application for search warrant, P/Major Alladin Dimagmaliw alleged, among others, as follows:

1. That he has been informed and has good and sufficient reasons to believe that NEMESIO PRUDENTE who may be found at the Polytechnic University of the Philippines, Anonas St. Sta. Mesa, Sampaloc, Manila, has in his control or possession firearms, explosives handgrenades and ammunition which are illegally possessed or intended to be used as the means of committing an offense which the said NEMESIO PRUDENTE is keeping and concealing at the following premises of the Polytechnic University of the Philippines, to wit: a. Offices of the Department of Military Science and Tactics at the ground floor and other rooms at the ground floor; b. Office of the President, Dr. Nemesio Prudente at PUP, Second Floor and other rooms at the second floor; 2. That the undersigned has verified the report and found it to be a fact, and therefore, believes that a Search Warrant should be issued to enable the undersigned or any agent of the law to take possession and bring to this Honorable Court the following described properties: a. M 16 Armalites with ammunitions; b. .38 and .45 Caliber handguns and pistols; c. explosives and handgrenades; and, d. assorted weapons with ammunitions. In support of the application for issuance of search warrant, P/Lt. Florenio C. Angeles, OIC of the Intelligence Section of (ISAD) executed a "Deposition of Witness" dated 31 October 1987, subscribed and sworn to before respondent Judge. In his deposition, P/Lt. Florenio Angeles declared, inter alia, as follows: Q: Do you know P/Major Alladin Dimagmaliw, the applicant for a Search Warrant? A: Yes, sir, he is the Chief, Intelligence and Special Action Division, Western Police District. Q: Do you know the premises of Polytechnic University of the Philippines at Anonas St., Sta. Mesa, Sampaloc, Manila A: Yes, sir, the said place has been the subject of our surveillance and observation during the past few days. Q: Do you have personal knowledge that in the said premises is kept the following properties subject of the offense of violation of PD No. 1866 or intended to be used as a means of committing an offense: a. M 16 Armalites with ammunitions; b. .38 and 45 Caliber handguns and pistols;

c. explosives and handgrenades; and d. Assorted weapons with ammunitions? A: Yes sir. Q: Do you know who is or who are the person or persons who has or have control of the above-described premises? A: Yes sir, it is Dr. Nemesio Prudente, President of the Polytechnic University of the Philippines. Q: How do you know that said property is subject of the offense of violation of Pres. Decree No. 1866 or intended to be used as the means of committing an offense? A: Sir, as a result of our continuous surveillance conducted for several days, we gathered information from verified sources that the holder of said firearms and explosives as well as ammunitions aren't licensed to possess said firearms and ammunition. Further, the premises is a school and the holders of these firearms are not students who were not supposed to possess firearms, explosives and ammunition. On the same day, 31 October 1987, respondent Judge issued Search Warrant No. 87-14, 3 the pertinent portions of which read as follows: It appearing to the satisfaction of the undersigned, after examining under oath applicant ALLADIN M. DIMAGMALIW and his witness FLORENIO C. ANGELES that there are good and sufficient reasons to believe (probable cause) that NEMESIO PRUDENTE has in his control in the premises of Polytechnic University of the Philippines, Anonas St., Sta. Mesa, Sampaloc, Manila, properties which are subject of the above offense or intended to be used as the means of committing the said offense. You are hereby commanded to make an immediate search at any time in the day or night of the premises of Polytechnic University of the Philippines, more particularly (a) offices of the Department of Military Science and Tactics at the ground floor and other rooms at the ground floor; (b) office of the President, Dr. Nemesio Prudente at PUP, Second Floor and other rooms at the second floor, and forthwith seize and take possession of the following personal properties, to wit: a. M 16 Armalites with ammunition; b. .38 and .45 Caliber handguns and pistols; c. explosives and hand grenades; and d. assorted weapons with ammunitions. and bring the above described properties to the undersigned to be dealt with as the law directs.

On 1 November 1987, a Sunday and All Saints Day, the search warrant was enforced by some 200 WPD operatives led by P/Col. Edgar Dula Torre, Deputy Superintendent, WPD, and P/Major Romeo Maganto, Precinct 8 Commander. In his affidavit, 4 dated 2 November 1987, Ricardo Abando y Yusay, a member of the searching team, alleged that he found in the drawer of a cabinet inside the wash room of Dr. Prudente's office a bulging brown envelope with three (3) live fragmentation hand grenades separately wrapped with old newspapers, classified by P/Sgt. J.L. Cruz as follows (a) one (1) pc.—M33 Fragmentation hand grenade (live); (b) one (11) pc.—M26 Fragmentation hand grenade (live); and (c) one (1) pc.— PRB—423 Fragmentation hand grenade (live). On 6 November 1987, petitioner moved to quash the search warrant. He claimed that (1) the complainant's lone witness, Lt. Florenio C. Angeles, had no personal knowledge of the facts which formed the basis for the issuance of the search warrant; (2) the examination of the said witness was not in the form of searching questions and answers; (3) the search warrant was a general warrant, for the reason that it did not particularly describe the place to be searched and that it failed to charge one specific offense; and (4) the search warrant was issued in violation of Circular No. 19 of the Supreme Court in that the complainant failed to allege under oath that the issuance of the search warrant on a Saturday was urgent. 5 The applicant, P/Major Alladin Dimagmaliw thru the Chief, Inspectorate and Legal Affairs Division, WPD, opposed the motion. 6 After petitioner had filed his reply 7 to the opposition, he filed a supplemental motion to quash. 8 Thereafter, on 9 March 1988, respondent Judge issued an order, 9denying the petitioner's motion and supplemental motion to quash. Petitioner's motion for reconsideration 10 was likewise denied in the order 11 dated 20 April 1988. Hence, the present recourse, petitioner alleging that respondent Judge has decided a question of substance in a manner not in accord with law or applicable decisions of the Supreme Court, or that the respondent Judge gravely abused his discretion tantamount to excess of jurisdiction, in issuing the disputed orders. For a valid search warrant to issue, there must be probable cause, which is to be determined personally by the judge, after examination under oath or affirmation of the complainant and the witnesses he may produce, and particularly describing the place to be searched and the persons or things to be seized.12 The probable cause must be in connection with one specific offense 13 and the judge must, before issuing the warrant, personally examine in the form of searching questions and answers, in writing and under oath, the complainant and any witness he may produce, on facts personally known to them and attach to the record their sworn statements together with any affidavits submitted. 14 The "probable cause" for a valid search warrant, has been defined "as such facts and circumstances which would lead a reasonably discreet arid prudent man to believe that an offense has been committed, and that objects sought in connection with the offense are in the place sought to be searched." 15 This probable cause must be shown to be within the personal knowledge of the complainant or the witnesses he may produce and not based on mere hearsay. 16 Petitioner assails the validity of Search Warrant No. 87-14 on the ground that it was issued on the basis of facts and circumstances which were not within the personal knowledge of the applicant and his witness but based on hearsay evidence. In his application for search warrant, P/Major Alladin Dimagmaliw stated that "he has been informed" that Nemesio Prudente "has in his control and

possession" the firearms and explosives described therein, and that he "has verified the report and found it to be a fact." On the other hand, in his supporting deposition, P/Lt. Florenio C. Angeles declared that, as a result of their continuous surveillance for several days, they "gathered informations from verified sources" that the holders of the said fire arms and explosives are not licensed to possess them. In other words, the applicant and his witness had no personal knowledge of the facts and circumstances which became the basis for issuing the questioned search warrant, but acquired knowledge thereof only through information from other sources or persons. While it is true that in his application for search warrant, applicant P/Major Dimagmaliw stated that he verified the information he had earlier received that petitioner had in his possession and custody the t there is nothing in the record to show or indicate how and when said applicant verified the earlier information acquired by him as to justify his conclusion that he found such information to be a fact. He might have clarified this point if there had been searching questions and answers, but there were none. In fact, the records yield no questions and answers, whether searching or not, visa-vis the said applicant. What the records show is the deposition of witness, P/Lt. Angeles, as the only support to P/Major Dimagmaliw's application, and the said deposition is based on hearsay. For, it avers that they (presumably, the police authorities) had conducted continuous surveillance for several days of the suspected premises and, as a result thereof, they "gathered information from verified sources" that the holders of the subject firearms and explosives are not licensed to possess them. In Alvarez vs. Court of First Instance, 17 this Court laid the following test in determining whether the allegations in an application for search warrant or in a supporting deposition, are based on personal knowledge or not— The true test of sufficiency of a deposition or affidavit to warrant issuance of a search warrant is whether it has been drawn in a manner that perjury could be charged thereon and the affiant be held liable for damage caused. The oath required must refer to the truth of the facts within the personal knowledge of the applicant for search warrant, and/or his witnesses, not of the facts merely reported by a person whom one considers to be reliable. Tested by the above standard, the allegations of the witness, P/Lt. Angeles, in his deposition, do not come up to the level of facts of his personal knowledge so much so that he cannot be held liable for perjury for such allegations in causing the issuance of the questioned search warrant. In the same Alvarez case, 18 the applicant stated that his purpose for applying for a search warrant was that: "It had been reported to me by a person whom I consider to be reliable that there are being kept in said premises books, documents, receipts, lists, chits and other papers used by him in connection with his activities as a money lender, challenging usurious rate of interests, in violation of law." The Court held that this was insufficient for the purpose of issuing a search warrant. In People vs. Sy Juco, 19 where the affidavit contained an allegation that there had been a report to the affiant by a person whom lie considered reliable that in said premises were "fraudulent books, correspondence and records," this was likewise held as not sufficient for the purpose of issuing a search warrant. Evidently, the allegations contained in the application of P/ Major Alladin Dimagmaliw and the declaration of P/Lt. Florenio C. Angeles in his deposition were insufficient basis for the issuance of a valid search warrant. As held in the Alvarezcase: The oath required must refer to the truth of the facts within the personal knowledge of the petitioner or his witnesses, because the purpose thereof is to convince the

committing magistrate, not the individual making the affidavit and seeking the issuance of the warrant, of the existence of probable cause. Besides, respondent Judge did not take the deposition of the applicant as required by the Rules of Court. As held in Roan v. Gonzales, 20 "(m)ere affidavits of the complainant and his witnesses are thus not sufficient. The examining Judge has to take depositions in writing of the complainant and the witnesses he may produce and attach them to the record." Moreover, a perusal of the deposition of P/Lt. Florenio Angeles shows that it was too brief and short. Respondent Judge did not examine him "in the form of searching questions and answers." On the contrary, the questions asked were leading as they called for a simple "yes" or "no" answer. As held in Quintero vs. NBI," 21 the questions propounded by respondent Executive Judge to the applicant's witness are not sufficiently searching to establish probable cause. Asking of leading questions to the deponent in an application for search warrant, and conducting of examination in a general manner, would not satisfy the requirements for issuance of a valid search warrant." Manifestly, in the case at bar, the evidence failed to show the existence of probable cause to justify the issuance of the search warrant. The Court also notes post facto that the search in question yielded, no armalites, handguns, pistols, assorted weapons or ammunitions as stated in the application for search warrant, the supporting deposition, and the search warrant the supporting hand grenades were itself Only three (3) live fragmentation found in the searched premises of the PUP, according to the affidavit of an alleged member of the searching party. The Court avails of this decision to reiterate the strict requirements for determination of "probable cause" in the valid issuance of a search warrant, as enunciated in earlier cases. True, these requirements are stringent but the purpose is to assure that the constitutional right of the individual against unreasonable search and seizure shall remain both meaningful and effective. Petitioner also assails the validity of the search warrant on the ground that it failed to particularly describe the place to be searched, contending that there were several rooms at the ground floor and the second floor of the PUP. The rule is, that a description of a place to be searched is sufficient if the officer with the warrant can, with reasonable effort, ascertain and Identify the place intended .22 In the case at bar, the application for search warrant and the search warrant itself described the place to be searched as the premises of the Polytechnic University of the Philippines, located at Anonas St., Sta. Mesa, Sampaloc, Manila more particularly, the offices of the Department of Military Science and Tactics at the ground floor, and the Office of the President, Dr. Nemesio Prudente, at PUP, Second Floor and other rooms at the second floor. The designation of the places to be searched sufficiently complied with the constitutional injunction that a search warrant must particularly describe the place to be searched, even if there were several rooms at the ground floor and second floor of the PUP. Petitioner next attacks the validity of the questioned warrant, on the ground that it was issued in violation of the rule that a search warrant can be issued only in connection with one specific offense. The search warrant issued by respondent judge, according to petitioner, was issued without any reference to any particular provision of PD No. 1866 that was violated when allegedly P.D. No. 1866 punishes several offenses. In Stonehill vs. Diokno, 23 Where the warrants involved were issued upon applications stating that the natural and juridical persons therein named had committed a "violation of Central Bank Laws, Tariff and Customs Laws, Internal Revenue Code and Revised Penal Code," the Court held that no specific offense had been alleged in the applications for a search warrant, and that it would be a

legal hearsay of the highest order to convict anybody of a "Violation of Central Bank Laws, Tariff and Customs Laws, Internal Revenue Code and Revised Penal Code" without reference to any determinate provision of said laws and codes. In the present case, however, the application for search warrant was captioned: "For Violation of PD No. 1866 (Illegal Possession of Firearms, etc.) While the said decree punishes several offenses, the alleged violation in this case was, qualified by the phrase "illegal possession of firearms, etc." As explained by respondent Judge, the term "etc." referred to ammunitions and explosives. In other words, the search warrant was issued for the specific offense of illegal possession of firearms and explosives. Hence, the failure of the search warrant to mention the particular provision of PD No. 1866 that was violated is not of such a gravity as to call for its invalidation on this score. Besides, while illegal possession of firearms is penalized under Section 1 of PD No. 1866 and illegal possession of explosives is penalized under Section 3 thereof, it cannot be overlooked that said decree is a codification of the various laws on illegal possession of firearms, ammunitions and explosives; such illegal possession of items destructive of life and property are related offenses or belong to the same species, as to be subsumed within the category of illegal possession of firearms, etc. under P.D. No. 1866. As observed by respondent Judge: 24 The grammatical syntax of the phraseology comparative with the title of PD 1866 can only mean that illegal possession of firearms, ammunitions and explosives, have been codified under Section 1 of said Presidential Decree so much so that the second and third are forthrightly species of illegal possession of firearms under Section (1) thereof It has long been a practice in the investigative and prosecution arm of the government, to designate the crime of illegal possession of firearms, ammunitions and explosives as 'illegal possession of firearms, etc.' The Constitution as well as the Rules of Criminal Procedure does not recognize the issuance of one search warrant for illegal possession of firearms, one warrant for illegal possession of ammunitions, and another for illegal possession of explosives. Neither is the filing of three different informations for each of the above offenses sanctioned by the Rules of Court. The usual practice adopted by the courts is to file a single information for illegal possession of firearms and ammunitions. This practice is considered to be in accordance with Section 13, Rule 110 of the 1985 Rules on Criminal Procedure which provides that: 'A complaint or information must charge but one offense, except only in those cases in which existing laws prescribe a single punishment for various offenses. Describably, the servers did not search for articles other than firearms, ammunitions and explosives. The issuance of Search Warrant No. 87-14 is deemed profoundly consistent with said rule and is therefore valid and enforceable. (Emphasis supplied) Finally, in connection with the petitioner's contention that the failure of the applicant to state, under oath, the urgent need for the issuance of the search warrant, his application having been filed on a Saturday, rendered the questioned warrant invalid for being violative of this Court's Circular No. 19, dated 14 August 1987, which reads: 3. Applications filed after office hours, during Saturdays, Sundays and holidays shall likewise be taken cognizance of and acted upon by any judge of the court having jurisdiction of the place to be searched, but in such cases the applicant shall certify and state the facts under oath, to the satisfaction of the judge, that the issuance is urgent. it would suffice to state that the above section of the circular merely provides for a guideline, departure from which would not necessarily affect the validity of an otherwise valid search warrant.

WHEREFORE, all the foregoing considered, the petition is GRANTED. The questioned orders dated 9 March 1988 and 20 April 1988 as well as Search Warrant No. 87-14 are hereby ANNULLED and SET ASIDE. The three (3) live fragmentation hand grenades which, according to Ricardo Y. Abando, a member of the searching team, were seized in the washroom of petitioner's office at the PUP, are ordered delivered to the Chief, Philippine Constabulary for proper disposition. G.R. No. L-43810 September 26, 1989 TOMAS CHIA, Owner-Manager of the Sony Merchandising (Phil.) of No. 691 Calle Raon, Quiapo, Manila, and TOM'S ELECTRONICS of No. 690 Calle Raon, Quiapo, Manila, petitioner, vs. THE ACTING COLLECTOR OF CUSTOMS, HON ALFREDO T. FRANCISCO, Port Area, Manila, and, GENER SULA ASAC, Camp Emilio Aguinaldo, Quezon City, respondents. Eliseo P. Legaspi for petitioner.

GRIÑO-AQUINO, J.: This petition for certiorari, prohibition, mandamus and injunction seeks: (1) to nullify the warrants of seizure and detention issued and signed by the Collector of Customs; and (2) to recover the confiscated goods seized under these general warrants, as well as damages. Acting on a verified report of a confidential informant that assorted electronic and electrical equipment and other articles illegally imported into the Philippines by a syndicate engaged in unlawful "shipside" activities (foreign goods are unloaded from foreign ships in transit through Philippine waters into motorized bancas and landed on Philippine soil without passing through the Bureau of Customs, thereby evading payment of the corresponding customs duties and taxes thereon) were found inside "Tom's Electronics" and "Sony Merchandising (Philippines)" stores located at 690 and 691 Gonzalo Puyat corner Evangelista Street, Quiapo, Manila, a letter- request dated April 23, 1976 was addressed to the Collector of Customs by the Deputy Director of the Regional Anti-Smuggling Action Center, Manila Bay Area (RASAC-MBA) for the issuance of warrants of seizure and detention. After evaluation, the Collector of Customs issued Warrants of Seizure and Detention Nos: 14925 and 14925-A, directing the Anti-Smuggling Action Center to seize the goods mentioned therein, which read as follows: Republic of the Philippines, _ versus Various electronic equipments like cassette tape recorders, car stereos, phonograph needles (diamond), portable TV sets, imported long playing records, spare parts of TVs and radios and other electrical appliances.

TOM'S ELECTRONICS Claimant Seizure Identification No. 14925-A SONY MERCHANDISING (PHIL.) Claimant Seizure Identification No. 14925 To: The Director or his duly-authorized representative ASAC Camp Aguinaldo, Quezon City GREETINGS: WHEREAS, the above-described articles are liable for forfeiture for having been imported in violation of Section 2536 of the Tariff and Customs Code as amended in relation to Section 2530 (m)-l of the same Code; WHEREAS, the said articles are at present in the custody of Tom's Electronics/Sony Merchandising (Phil.); WHEREFORE, by virtue of the authority vested in me by law and in compliance with Finance Department Order No. 96-67 as published in Customs Memorandum Circular No. 133-67 dated July 25, 1967, you are hereby ordered to forthwith seize the aforementioned articles and turn them over to the custody of the Auction and Cargo Disposal Division of this Bureau. (Annexes A & A-1, pp. 10-11, Rollo.) A RASAC team was formed and given a mission order to enforce the warrants, which it implemented with the assistance of: (1) the National Customs Police (augmenting the team with two members), (2) the Detective Bureau of the Manila Western Police District Headquarters (with three detectives), as well as, (3) Precinct 3 of the Manila Western Police District which exercised jurisdictional control over the place to be raided. The intended raid was entered in the respective police blotters of the police detective bureaus. On the strength of the warrants of seizure and detention, the raid was conducted in the afternoon of April 25,1976 at the two stores of the petitioner. ASAC team leader Gener Sula, together with his agents Badron Dobli, Arturo Manuel, Rodolfo Molina and Servillano Florentin of Camp Aguinaldo, Quezon City, assisted by two customs policemen, Val Martinez and Renato Sorima, and Manila policemen Rogelio Vinas and John Peralta, recovered from the stores, assorted electronic equipment and other articles, listed in Annex B of the petition, the customs duties on which allegedly had not been paid (p. 12, Rollo). They were turned over to the Customs Auction ana Cargo Disposal Unit of the Bureau of Customs. On May 17, 1976, in the afternoon, the hearing officer of Acting Collector of Customs Alfredo Francisco conducted a hearing on the confiscation of the goods taken by Gener Sula and his agents. Two days later, petitioner Tomas Chia filed this petition for certiorari, prohibition and mandamus to enjoin the Collector of Customs and/or his agents from further proceeding with the forfeiture healing and prayed that the search warrants be declared null and void, that the respondents be ordered to return the confiscated articles to the petitioner, and to pay damages. Upon filing a Pl,000-bond, the Court issued a writ of preliminary injunction to stop the forfeiture proceedings.

The pivotal issue raised in the petition is whether the warrants of seizure and detention (or Seizure Identifications Nos.14925 and 14925-A) are general warrants issued in violation of Rule 126, Section 3, of the Rules of Court which provides that: A search warrant shall not issue but upon probable cause in connection with one specific offense to be determined by the judge or justice of the peace after examination under oath or affirmation of the complainant and the witnesses he may produce, and particularly describing the place to be searched and the persons or things to be seized. No search warrant shall issue for more than one specific offense. and under Section 3 of the Bill of Rights of the 1973 Constitution which provided that: The right of the people to be secured in their persons, houses, papers and effects against unreasonable searches and seizures of whatever nature and for any purpose shall not be violated, and no search warrant or warrant of arrest shall issue except upon probable cause to be determined by the judge or such other responsible officer as may be authorized by law after examination under oath or affirmation of the complainant and the witnesses he may produce, and particularly describing the place to be searched, and the persons or things to be seized (Emphasis supplied.) On the other hand, the respondents contend that the goods seized from petitioner's stores by the RASAC-MBA team were only those subject to customs duties and taxes but which were not supported by any evidence of payment of those duties and taxes. Those goods are subject to forfeiture for having been imported in violation of Section 2536 of the Tariff and Customs Code, as amended, in relation to Section 2530 (m)-l, which provides: SEC. 2536. SEIZURES OF OTHER ARTICLES-The Commissioner of Customs and Collector of Customs and/or any other customs officer, with the prior authorization in writing by the Commissioner, may demand evidence of payment of duties and taxes on foreign articles openly offered for sale or kept in storage, and if no such evidence can be produced, such articles may be seized and subjected to forfeiture proceedings: Provided, however, that during such proceedings the person or entity from whom such articles have been seized shall be given the opportunity to prove or show the source of such articles and the payment of duties and taxes thereon. The petition is devoid of merit. Not only may goods be seized without a search and seizure warrant under Section 2536 of the Customs and Tariff Code, when they (the goods) are openly offered for sale or kept in storage in a store as in this case, but the fact is that petitioner's stores — Tom's Electronics" and "Sony Merchandising (Phil.)" — were searched upon warrants of search and detention issued by the Collector of Customs, who, under the 1973 Constitution, was "a responsible officer authorized by law" to issue them. Sections 2208 and 2209 of the Tariff and Customs Code provide when a search may be made without a warrant and when a warrant is necessary: SEC. 2208. RIGHT OF POLICE OFFICER TO ENTER INCLOSURE — For the more effective discharge of his official duties, any person exercising the powers herein conferred, may at any time enter, pass through or search any land or inclosure or any warehouse, store or other building, not being a dwelling house. A warehouse, store or other building or inclosure used for the keeping or storage of articles does not become a dwelling house within the meaning hereof merely by

reason of the fact that a person employed as watchman lives in the place, nor will the fact that his family stays there with him alter the case. SEC. 2209.- SEARCH OF A DWELLING HOUSE. — A dwelling house may be entered and searched only upon warrant issued by a Judge of the court or such other responsible officers as may be authorized by law, upon sworn application showing probable cause and particularly describing the place to be searched and the person or thing to be seized. The warrants issued by the Collector of Customs in this case were not general warrants, as erroneously alleged by the petitioner for they identified the stores to be searched, described the articles to be seized and specified the provision of the Tariff and Customs Code violated. Upon effecting the seizure of the goods, the Bureau of Customs acquired exclusive jurisdiction not only over the case but also over the goods seized for the purpose of enforcing the tariff and customs laws. A party dissatisfied with the decision of the Collector may appeal to the Commissioner of Customs, whose decision is appealable to the Court of Tax Appeals in the manner and within the period prescribed by law and regulations. The decision of the Court of Tax Appeals may be elevated to the Supreme Court for review (Secs. 2309-2316; 2401 & 2402 of the Tariff and Customs Code; Collector of Customs vs. Torres, et al., 45 SCRA 272). Since petitioner did not exhaust his administrative remedies, his recourse to this Court is premature (Acting Collector of Customs of the Port of Manila vs. Caluag, 20 SCRA 204; Laganapan vs. Asedillo, 154 SCRA 377; National Development Co. vs. Hervilla, 151 SCRA 520). If for no other reason, the petition is dismissible on that score. WHEREFORE, the petition is dismissed. The writ of preliminary injunction which we issued on May 28, 1976 is hereby lifted and set aside. Costs against petitioner. G.R. Nos. 76649-51 August 19, 1988 20TH CENTURY FOX FILM CORPORATION, petitioner, vs. COURT OF APPEALS, EDUARDO M. BARRETO, RAUL SAGULLO and FORTUNE LEDESMA, respondents. Siguion Reyna, Montecillo & Ongsiako Law Office for petitioner. B.C. Salazar & Associates for respondents.

GUTIERREZ, JR., J.: The petitioner questions the application of the constitutional provision against illegal searches and seizures to raids conducted in connection with the government's anti-film piracy campaign. The main issue hinges on whether or not the judge properly lifted the search warrants he issued earlier upon the application of the National Bureau of Investigation on the basis of the complaint filed by the petitioner.

In a letter-complaint dated August 26, 1985, petitioner 20th Century Fox Film Corporation through counsel sought the National Bureau of Investigation's (NBI) assistance in the conduct of searches

and seizures in connection with the latter's anti-film piracy campaign. Specifically, the lettercomplaint alleged that certain videotape outlets all over Metro Manila are engaged in the unauthorized sale and renting out of copyrighted films in videotape form which constitute a flagrant violation of Presidential Decree No. 49 (otherwise known as the Decree on the Protection of Intellectual Property). Acting on the letter-complaint, the NBI conducted surveillance and investigation of the outlets pinpointed by the petitioner and subsequently filed three (3) applications for search warrants against the video outlets owned by the private respondents. The applications were consolidated and heard by the Regional Trial Court of Makati, Branch 132. On September 4, 1985, the lower court issued the desired search warrants. Armed with the search warrants, the NBI accompanied by the petitioner's agents, raided the video outlets and seized the items described therein. An inventory of the items seized was made and left with the private respondents. Acting on a motion to lift search warrants and release seized properties filed by the private respondents, the lower court issued an order dated October 8, 1985, lifting the three (3) search warrants issued earlier against the private respondents by the court. The dispositive portion of the order reads: WHEREFORE, the Court hereby orders that Search Warrants Nos. SW- 85-024; issued against Eduardo M. Barreto of the Junction Video, etc., Paranaque, Metro Manila; SW No. 85-025, issued against Raul M. Sagullo of South Video Bug Center, Inc., etc., also of No. 5355 Pres. Avenue BF Homes, Parañaque, Metro Manila; and SW No. 85-026, issued against Fortune A. Ledesma of Sonix Video Services of San Antonio Plaza, Forbes Park, Makati, Metro Manila, be lifted. Consequently, the articles listed in the returns of the three search warrants which could not be a basis of any criminal prosecution, now in the possession of the National Bureau of Investigation which under the law must be delivered to this Court, but which the NBI failed to do, are hereby ordered to be returned to their owners through their lawyer, Atty. Benito Salazar or his agents or representatives, against proper receipt, to be forwarded to this Court for record purposes, as proof that said properties have been returned to the possession of the rightful owners." (p. 34, Rollo) The lower court denied a motion for reconsideration filed by the petitioner in its order dated January 2, 1986. The petitioner filed a petition for certiorari with the Court of Appeals to annul the October 8, 1985 and January 2, 1986 orders of the lower court. The petition was dismissed. Hence, this petition. The main issue hinges on the meaning of "probable cause" within the context of the constitutional provision against illegal searches and seizures (Section 3, Article IV, 1973 Constitution, now, Section 2, Article Ill, 1987 Constitution. The petitioner maintains that the lower court issued the questioned search warrants after finding the existence of a probable cause justifying their issuance. According to the petitioner, the lower court

arrived at this conclusion on the basis of the depositions of applicant NBI's two witnesses which were taken through searching questions and answers by the lower court. Section 2, Article III of the present Constitution which substantially reproduces Section 3, Article IV of the 1973 Constitution on illegal searches and seizures provides: The right of the people to be secure in their persons, houses, papers, and effects against unreasonable searches and seizures of whatever nature and for any purpose shall be inviolable, and no search warrant or warrant of arrest shall issue except upon probable cause to be determined personally by the judge after examination under oath or affirmation of the complainant and the witnesses he may produce, and particularly describing the place to be searched and the persons or things to be seized. This constitutional right protects a citizen against wanton and unreasonable invasion of his privacy and liberty as to his person, papers and effects. We have explained in the case of People v. Burgos (144 SCRA 1) citingVillanueva v. Querubin (48 SCRA 345) why the right is so important: It is deference to one's personality that lies at the core of this right, but it could be also looked upon as a recognition of a constitutionally protected area, primarily one's home, but not necessarily thereto confined. (Cf. Hoffa v. United States, 385 US 293 119661) What is sought to be guarded is a man's prerogative to choose who is allowed entry to his residence. In that haven of refuge, his individuality can assert itself not only in the choice of who shall be welcome but likewise in the kind of objects he wants around him. There the state, however powerful, does not as such have access except under the circumstances above noted, for in the traditional formulation, his house, however humble, is his castle. Thus is outlawed any unwarranted intrusion by government, which is called upon to refrain from any invasion of his dwelling and to respect the privacies of his life. (Cf Schmerber v. California, 384 US 757 [1966], Brennan, J. and Boyd v. United States, 116 630 [1886]). In the same vein, Landynski in his authoritative work (Search and Seizure and the Supreme Court [1966]), could fitly characterize constitutional right as the embodiment of a "spiritual concept: the belief that to value the privacy of home and person and to afford its constitutional protection against the long reach of government is no less than to value human dignity, and that his privacy must not be disturbed except in case of overriding social need, and then only under stringent procedural safeguards."(ibid, p. 74). The government's right to issue search warrants against a citizen's papers and effects is circumscribed by the requirements mandated in the searches and seizures provision of the Constitution. In the case of Burgos, Sr. v. Chief of Staff, AFP (133 SCRA 800), we defined probable cause for a valid search "as such facts and circumstances which would lead a reasonably discreet and prudent man to believe that an offense has been committed and that the objects sought in connection with the offense are in the place sought to be searched." This constitutional provision also demands "no less than personal knowledge by the complainant or his witnesses of the facts upon which the issuance of a search warrant may be justified" in order to convince the judge, not the individual making the affidavit and seeking the issuance of the warrant, of the existence of a probable cause. (Alvarez v. Court of First Instance, 64 Phil. 33; Burgos, Sr. v. Chief of Staff, AFP, supra).

In the instant case, the lower court lifted the three questioned search warrants against the private respondents on the ground that it acted on the application for the issuance of the said search warrants and granted it on the misrepresentations of applicant NBI and its witnesses that infringement of copyright or a piracy of a particular film have been committed. Thus the lower court stated in its questioned order dated January 2,1986: According to the movant, all three witnesses during the proceedings in the application for the three search warrants testified of their own personal knowledge. Yet, Atty. Albino Reyes of the NBI stated that the counsel or representative of the Twentieth Century Fox Corporation will testify on the video cassettes that were pirated, so that he did not have personal knowledge of the alleged piracy. The witness Bacani also said that the video cassettes were pirated without stating the manner it was pirated and that it was Atty. Domingo that has knowledge of that fact. On the part of Atty. Domingo, he said that the re-taping of the allegedly pirated tapes was from master tapes allegedly belonging to the Twentieth Century Fox, because, according to him, it is of his personal knowledge. At the hearing of the Motion for Reconsideration, Senior NBI Agent Atty. Albino Reyes testified that when the complaint for infringement was brought to the NBI, the master tapes of the allegedly pirated tapes were shown to him and he made comparisons of the tapes with those purchased by their man Bacani. Why the master tapes or at least the film reels of the allegedly pirated tapes were not shown to the Court during the application gives some misgivings as to the truth of that bare statement of the NBI agent on the witness stand. " Again as the application and search proceedings is a prelude to the filing of criminal cases under PD 49, the copyright infringement law, and although what is required for the issuance thereof is merely the presence of probable cause, that probable cause must be satisfactory to the Court, for it is a time- honored precept that proceedings to put a man to task as an offender under our laws should be interpreted in strictissimi juris against the government and liberally in favor of the alleged offender. xxx xxx xxx This doctrine has never been overturned, and as a matter of fact it had been enshrined in the Bill of Rights in our 1973 Constitution. So that lacking in persuasive effect, the allegation that master tapes were viewed by the NBI and were compared to the purchased and seized video tapes from the respondents' establishments, it should be dismissed as not supported by competent evidence and for that matter the probable cause hovers in that grey debatable twilight zone between black and white resolvable in favor of respondents herein. But the glaring fact is that 'Cocoon,' the first video tape mentioned in the search warrant, was not even duly registered or copyrighted in the Philippines. (Annex C of Opposition p. 152 record). So, that lacking in the requisite presentation to the Court of an alleged master tape for purposes of comparison with the purchased evidence of the video tapes allegedly pirated and those seized from respondents, there was no way to determine whether there really was piracy, or copying of the film of the complainant Twentieth Century Fox." (pp. 37-39, Rollo)

xxx xxx xxx The lower court, therefore, lifted the three (3) questioned search warrants in the absence of probable cause that the private respondents violated P.D. 49. As found out by the court, the NBI agents who acted as witnesses did not have personal knowledge of the subject matter of their testimony which was the alleged commission of the offense by the private respondents. Only the petitioner's counsel who was also a witness during the application for the issuance of the search warrants stated that he had personal knowledge that the confiscated tapes owned by the private respondents were pirated tapes taken from master tapes belonging to the petitioner. However, the lower court did not give much credence to his testimony in view of the fact that the master tapes of the allegedly pirated tapes were not shown to the court during the application. All these factors were taken into consideration by the lower court when it lifted the three questioned search warrants. There is no truth, therefore, to the petitioner's allegation that the lower court based its January 2, 1986 order only "on the fact that the original or master copies of the copyrighted films were not presented during the application for search warrants, thus leading it to conclude that it had been "misled by the applicant and his witnesses." (p. 17, Rollo) The presentation of the master tapes of the copyrighted films from which the pirated films were allegedly copied, was necessary for the validity of search warrants against those who have in their possession the pirated films. The petitioner's argument to the effect that the presentation of the master tapes at the time of application may not be necessary as these would be merely evidentiary in nature and not determinative of whether or not a probable cause exists to justify the issuance of the search warrants is not meritorious. The court cannot presume that duplicate or copied tapes were necessarily reproduced from master tapes that it owns. The application for search warrants was directed against video tape outlets which allegedly were engaged in the unauthorized sale and renting out of copyrighted films belonging to the petitioner pursuant to P.D. 49. The essence of a copyright infringement is the similarity or at least substantial similarity of the purported pirated works to the copyrighted work. Hence, the applicant must present to the court the copyrighted films to compare them with the purchased evidence of the video tapes allegedly pirated to determine whether the latter is an unauthorized reproduction of the former. This linkage of the copyrighted films to the pirated films must be established to satisfy the requirements of probable cause. Mere allegations as to the existence of the copyrighted films cannot serve as basis for the issuance of a search warrant. Furthermore, we note that the search warrants described the articles sought to be seized as follows: xxx xxx xxx xxx xxx xxx c) Television sets, Video Cassettes Recorders, rewinders, tape head cleaners, accessories, equipments and other machines used or intended to be used in the unlawful reproduction, sale, rental/lease distribution of the above-mentioned video tapes which she is keeping and concealing in the premises above-described." (p. 26, Rollo) In the case of Burgos v. Chief of Staff, AFP supra, we stated:

xxx xxx xxx Another factor which makes the search warrants under consideration constitutionally objectionable is that they are in the nature of general warrants. The search warrants describe the articles sought to be seized in this wise: l] All printing equipment, paraphernalia, paper, ink, photo equipment, typewriters, cabinets, tables communications/recording equipment, tape recorders, dictaphone and the like used and/or connected in the printing of the 'WE FORUM' newspaper and any and all document/communications, letters and facsimile of prints related to "WE FORUM" newspaper. 2] Subversive documents, pamphlets, leaflets, books, and other publications to promote the objectives and purposes of the subversive organizations known as Movement for Free Philippines, Light-a-Fire Movement and April 6 Movement; and 3] Motor vehicles used in the distribution/circulation of the 'WE FORUM and other subversive materials and propaganda, more particularly, 1] Toyota-Corolla, colored yellow with Plate No. NKA 892; 2] DATSUN pick-up colored white with Plate No. NKV 969; 3] A delivery truck with Plate No. NBS 542; 4] TOYOTA-TAMARAW, colored white with Plate No. PBP 665;and, 5] TOYOTA Hi-Lux, pick-up truck with Plate No. NGV 472 with marking "Bagong Silang." In Stanford v. State of Texas (379 U.S. 476,13 L ed 2nd 431), the search warrant which authorized the search for 'books, records, pamphlets, cards, receipts, lists, memoranda, pictures, recordings and other written instruments concerning the Communist Parties of Texas, and the operations of the Community Party in Texas," was declared void by the U.S. Supreme Court for being too general. In like manner, directions to "seize any evidence in connection with the violation of SDC 13-3703 or otherwise' have been held too general, and that portion of a search warrant which authorized the seizure of any "paraphernalia which could be used to violate Sec. 54197 of the Connecticut General Statutes [the statute dealing with the crime of conspiracy]"' was held to be a general warrant, and therefore invalid (68 Am. Jur. 2d., pp. 736-737). The description of the articles sought to be seized under the search warrants in question cannot be characterized differently. (at pp. 814-815) Undoubtedly, a similar conclusion can be deduced from the description of the articles sought to be confiscated under the questioned search warrants. Television sets, video cassette recorders, reminders and tape cleaners are articles which can be found in a video tape store engaged in the legitimate business of lending or renting out betamax tapes. In short, these articles and appliances are generally connected with, or related to a legitimate business not necessarily involving piracy of intellectual property or infringement of copyright laws. Hence, including these articles without specification and/or particularity that they were really

instruments in violating an Anti-Piracy law makes The search warrant too general which could result in the confiscation of all items found in any video store. In fact, this actually happened in the instant case. Thus, the lower court, in its questioned order dated October 8, 1985 said: Although the applications and warrants themselves covered certain articles of property usually found in a video store, the Court believes that the search party should have confined themselves to articles that are according to them, evidence constitutive of infringement of copyright laws or the piracy of intellectual property, but not to other articles that are usually connected with, or related to, a legitimate business, not involving piracy of intellectual property, or infringement of copyright laws. So that a television set, a rewinder, and a whiteboard listing Betamax tapes, video cassette cleaners video cassette recorders as reflected in the Returns of Search Warrants, are items of legitimate business engaged in the video tape industry, and which could not be the subject of seizure, The applicant and his agents therefore exceeded their authority in seizing perfectly legitimate personal property usually found in a video cassette store or business establishment." (p. 33, Rollo) All in all, we find no grave abuse of discretion on the part of the lower court when it lifted the search warrants it earlier issued against the private respondents. We agree with the appellate court's findings to the effect that: An assiduous examination of the assailed orders reveal that the main ground upon which the respondent Court anchored said orders was its subsequent findings that it was misled by the applicant (NBI) and its witnesses 'that infringement of copyright or a piracy of a particular film have been committed when it issued the questioned warrants.' Stated differently, the respondent Court merely corrected its erroneous findings as to the existence of probable cause and declared the search and seizure to be unreasonable. Certainly, such action is within the power and authority of the respondent Court to perform, provided that it is not exercised in an oppressive or arbitrary manner. Indeed, the order of the respondent Court declaring the existence of probable cause is not final and does not constitute res judicata. A careful review of the record of the case shows that the respondent Court did not commit a grave abuse of discretion when it issued the questioned orders. Grave abuse of discretion' implies such capricious and whimsical exercise of judgment as is equivalent to lack of jurisdiction, or, in other words, where the power is exercised in an arbitrary or despotic manner by reason of passion or personal hostility, and it must be so patent and gross as to amount to an evasion of positive duty or to a virtual refusal to perform the duty enjoined or to act at all in contemplation of law.' But far from being despotic or arbitrary, the assailed orders were motivated by a noble desire of rectifying an error, much so when the erroneous findings collided with the constitutional rights of the private respondents. In fact, the petitioner did not even contest the righteousness and legality of the questioned orders but instead concentrated on the alleged denial of due process of law." (pp. 44-45, Rollo) The proliferation of pirated tapes of films not only deprives the government of much needed revenues but is also an indication of the widespread breakdown of national order and discipline. Courts should not impose any unnecessary roadblocks in the way of the anti-film piracy campaign. However, the campaign cannot ignore or violate constitutional safeguards. To say that the problem of pirated films can be solved only by the use of unconstitutional shortcuts is to denigrate the long history and experience behind the searches and seizures clause of the Bill of Rights. The trial court did not commit reversible error.

WHEREFORE, the instant petition is DISMISSED. The questioned decision and resolution of the Court of Appeals are AFFIRMED. G.R. No. 199032

November 19, 2014

RETIRED SP04 BIENVENIDO LAUD, Petitioner, vs. PEOPLE OF THE PHILIPPINES, Respondent. DECISION PER CURIAM: Assailed in this petition for review on certiorari1 are the Decision2 dated April 25, 2011 and the Resolution3 dated October 17, 2011 of the Court of Appeals (CA) in CA-G.R. SP. No. 113017 upholding the validity of Search Warrant No. 09-14407.4 The Facts On July 10, 2009, the Philippine National Police (PNP), through Police Senior Superintendent Roberto B. Fajardo, applied with the Regional Trial Court (RTC) of Manila, Branch50 (Manila-RTC) for a warrant to search three (3) caves located inside the Laud Compound in Purok 3, Barangay Maa, Davao City, where the alleged remains of the victims summarily executed by the so-called "Davao Death Squad" may be found.5 In support of the application, a certain Ernesto Avasola (Avasola) was presented to the RTC and there testified that he personally witnessed the killing of six (6) persons in December 2005, and was, in fact, part of the group that buried the victims.6 Judge William Simon P. Peralta (Judge Peralta), acting as Vice Executive Judge of the Manila-RTC, found probable cause for the issuance of a search warrant, and thus, issued Search Warrant No. 09144077 which was later enforced by the elements ofthe PNP-Criminal Investigation and Detection Group, in coordination withthe members of the Scene of the Crime Operatives on July 15, 2009.The search of the Laud Compound caves yielded positive results for the presence of human remains.8 On July 20, 2009, herein petitioner, retired SPO4 Bienvenido Laud (Laud), filed an Urgent Motion to Quash and to Suppress Illegally Seized Evidence9 premised on the following grounds: (a) Judge Peralta had no authority to act on the application for a search warrant since he had been automatically divested of his position asVice Executive Judge when several administrative penalties were imposed against him by the Court;10 (b) the Manila-RTC had no jurisdiction to issue Search Warrant No. 09-14407 which was to be enforced in Davao City;11 (c) the human remains sought to be seized are not a proper subject of a search warrant;12 (d) the police officers are mandated to follow the prescribed procedure for exhumation of human remains;13 (e) the search warrant was issued despite lack of probable cause;14 (f) the rule against forum shopping was violated;15 and (g) there was a violation of the rule requiring one specific offense and the proper specification of the place to be searched and the articles to be seized.16 The Manila-RTC Ruling In an Order17 dated July 23, 2009, the Manila-RTC granted the motion of Laud "after a careful consideration [of] the grounds alleged [therein]." Aside from this general statement, the said Order contained no discussion on the particular reasons from which the Manila-RTC derived its conclusion.

Respondent, the People of the Philippines (the People), filed a Motion for Reconsideration18 which was, however, denied in an Order19 dated December 8, 2009, wherein the Manila-RTC, this time, articulated its reasons for the warrant’s quashal, namely: (a) the People failed to show any compelling reason to justify the issuanceof a search warrant by the Manila RTC which was to be implemented in Davao City where the offense was allegedly committed, in violation of Section 2, Rule 126 of the Rules of Court;20 (b) the fact that the alleged offense happened almost four (4) years before the search warrant application was filed rendered doubtful the existence of probable cause;21 and (c) the applicant, i.e., the PNP, violated the rule against forum shopping as the subject matter of the present search warrant application is exactly the sameas the one contained in a previous application22 before the RTC of Davao City, Branch 15 (Davao-RTC) which had been denied.23 Unconvinced, the People filed a petition for certioraribefore the CA, docketed as CA-G.R. SP. No. 113017. The CA Ruling In a Decision24 dated April 25, 2011, the CA granted the People’s petition and thereby annulled and set aside the Orders of the Manila-RTC for having been tainted with grave abuse of discretion. It held that the requirements for the issuance of a search warrant were satisfied, pointing out that an application therefor involving a heinous crime, such as Murder, is an exception to the compelling reasons requirement under Section 2, Rule 126 of the Rules of Court as explicitly recognized in A.M. No. 99-20-09-SC25 and reiterated in A.M. No. 03-8-02-SC,26 provided that the application is filed by the PNP, the National Bureau of Investigation (NBI), the Presidential Anti-Organized Crime Task Force (PAOC-TF) or the Reaction Against Crime Task Force (REACT-TF),27 with the endorsement of its head, before the RTC of Manila or Quezon City, and the warrant be consequently issued by the Executive Judge or Vice-Executive Judge of either of the said courts, as in this case.28 Also, the CA found that probable cause was established since, among others, witness Avasola deposed and testified that he personally witnessed the murder of six (6) persons in December 2005 and was actually part of the group that buried the victims – two bodies in each of the three (3) caves.29 Further, it observed that the Manila-RTC failed to consider the fear of reprisal and natural reluctance of a witness to get involved in a criminal case, stating that these are sufficient reasons to justify the delay attending the application of a search warrant.30Accordingly, it deemed that the physical evidence of a protruding human bone in plain view in one of the caves, and Avasola’s firsthand eye witness account both concur and point to the only reasonable conclusion that the crime ofMurder had been committed and that the human remains of the victims were located in the Laud Compound.31 Finally, the CA debunked the claim of forum shopping, finding that the prior application for a search warrant filed before the Davao-RTC was based on facts and circumstances different from those in the application filed before the Manila-RTC.32 Dissatisfied, Laud moved for reconsideration which was, however, denied in a Resolution33 dated October 17, 2011,hence, this petition. The Issues Before the Court The issues for the Court’s resolution are as follows: (a) whether the administrative penalties imposed on Judge Peralta invalidated Search Warrant No. 09-14407; (b) whether the Manila-RTC had jurisdiction to issue the said warrant despite non-compliance with the compelling reasons

requirement under Section 2, Rule126 of the Rules of Court; (c) whether the requirements of probable cause and particular description were complied with and the one-specific-offense rule under Section 4, Rule 126 of the Rules of Court was violated; and (d) whether the applicant for the search warrant,i.e., the PNP, violated the rule against forum shopping. 1âwphi 1

The Court's Ruling The petition has no merit. A. Effect of Judge Peralta’s Administrative Penalties.

Citing Section 5, Chapter III of A.M. No. 03-8-02-SC which provides that "[t]he imposition upon an Executive Judge or Vice-Executive Judge of an administrative penalty of at least a reprimand shall automatically operate to divest him of his position as such,"Laud claims that Judge Peralta had no authority to act as Vice-Executive Judge and accordingly issue Search Warrant No. 09-14407 in view of the Court’s Resolution in Dee C. Chuan & Sons, Inc. v. Judge Peralta34 wherein he was administratively penalized with fines of ₱15,000.00 and ₱5,000.00.35 While the Court does agree that the imposition of said administrative penalties did operate to divest Judge Peralta’s authority to act as ViceExecutive Judge, it must be qualified thatthe abstraction of such authority would not, by and of itself, result in the invalidity of Search Warrant No. 09-14407 considering that Judge Peralta may be considered to have made the issuance as a de facto officer whose acts would, nonetheless, remain valid. Funa v. Agra36 defines who a de factoofficer is and explains that his acts are just as valid for all purposes as those of a de jureofficer, in so far as the public or third persons who are interested therein are concerned, viz.: A de facto officer is one who derives his appointment from one having colorable authority to appoint, if the office is an appointive office, and whose appointment is valid on its face. He may also be one who is in possession of an office, and is discharging [his] duties under color of authority, by which is meant authority derived from an appointment, however irregular or informal, so that the incumbent is not a mere volunteer. Consequently, the acts of the de factoofficer are just as valid for all purposes as those of a de jure officer, in so far as the public or third persons who are interested therein are concerned.37 The treatment of a de factoofficer’s acts is premised on the reality that third persons cannot always investigate the right of one assuming to hold an important office and, as such, have a right to assume that officials apparently qualified and in office are legally such.38 Public interest demands that acts of persons holding, under color of title, an office created by a valid statute be, likewise, deemed valid insofar as the public – as distinguished from the officer in question – is concerned.39 Indeed, it is far more cogently acknowledged that the de factodoctrine has been formulated, not for the protection of the de facto officer principally, but rather for the protection of the public and individuals who get involved in the official acts of persons discharging the duties of an office without being lawful officers.40 In order for the de facto doctrine to apply, all of the following elements must concur: (a) there must be a de jureoffice; (b) there must be color of right or general acquiescence by the public; and (c) there must be actual physical possession of the office in good faith.41

The existence of the foregoing elements is rather clear in this case. Undoubtedly, there is a de jureoffice of a 2nd Vice-Executive Judge. Judge Peralta also had a colorable right to the said office as he was duly appointed to such position and was only divested of the same by virtue of a supervening legal technicality – that is, the operation of Section 5, Chapter III of A.M. No. 03-8-02SC as above-explained; also, it may be said that there was general acquiescence by the public since the search warrant application was regularly endorsed to the sala of Judge Peralta by the Office of the Clerk of Court of the Manila-RTC under his apparent authority as 2nd Vice Executive Judge.42 Finally, Judge Peralta’s actual physical possession of the said office is presumed to bein good faith, as the contrary was not established.43 Accordingly, Judge Peralta can be considered to have acted as a de factoofficer when he issued Search Warrant No. 09-14407, hence, treated as valid as if it was issued by a de jureofficer suffering no administrative impediment. B. Jurisdiction of the Manila-RTC to Issue Search Warrant No. 09- 14407; Exception to the Compelling Reasons Requirement Under Section 2, Rule 126 of the Rules of Court.

Section 12, Chapter V of A.M.No. 03-8-02-SC states the requirements for the issuance of search warrants in special criminal cases by the RTCs of Manilaand Quezon City. These special criminal cases pertain to those "involving heinous crimes, illegal gambling, illegal possession of firearms and ammunitions, as well as violations of the Comprehensive Dangerous Drugs Act of 2002, the Intellectual Property Code, the Anti-Money Laundering Act of 2001, the Tariff and Customs Code, as amended, and other relevant laws that may hereafter be enacted by Congress, and included herein by the Supreme Court." Search warrant applications for such cases may befiled by "the National Bureau of Investigation (NBI), the Philippine National Police(PNP) and the AntiCrime Task Force (ACTAF)," and "personally endorsed by the heads of such agencies." As in ordinary search warrant applications, they "shall particularly describe therein the places to be searched and/or the property or things to be seized as prescribed in the Rules of Court." "The Executive Judges [of these RTCs] and,whenever they are on official leave of absence or are not physically present in the station, the Vice-Executive Judges" are authorized to act on such applications and "shall issue the warrants, if justified, which may be served in places outside the territorial jurisdiction of the said courts." The Court observes that all the above-stated requirements were complied with in this case. As the records would show, the search warrant application was filed before the Manila-RTC by the PNP and was endorsed by its head, PNP Chief Jesus Ame Versosa,44 particularly describing the place to be searched and the things to be seized (as will be elaborated later on) in connection with the heinous crime of Murder.45 Finding probable cause therefor, Judge Peralta, in his capacity as 2nd Vice-Executive Judge, issued Search Warrant No. 09-14407 which, as the rules state, may be served in places outside the territorial jurisdiction of the said RTC. Notably, the fact that a search warrant application involves a "special criminal case" excludes it from the compelling reason requirement under Section 2, Rule 126 of the Rules of Court which provides: SEC. 2. Court where application for search warrant shall be filed. — An application for search warrant shall be filed with the following: a) Any court within whose territorial jurisdiction a crime was committed. b) For compelling reasons stated in the application, any court within the judicial region where the crime was committed if the place of the commission of the crime isknown, or any court within the judicial region where the warrant shall be enforced.

However, if the criminal action has already been filed, the application shall only be made in the court where the criminal action is pending. (Emphasis supplied) As explicitly mentioned in Section 12, Chapter V of A.M. No. 03-8- 02-SC, the rule on search warrant applications before the Manila and Quezon City RTCs for the above-mentioned special criminal cases "shall be an exception to Section 2 of Rule 126 of the Rules of Court." Perceptibly, the fact that a search warrant is being applied for in connection with a special criminal case as aboveclassified already presumes the existence of a compelling reason; hence, any statement to this effect would be super fluous and therefore should be dispensed with. By all indications, Section 12, Chapter V of A.M. No. 03-8-02-SC allows the Manila and Quezon City RTCs to issue warrants to be servedin places outside their territorial jurisdiction for as long as the parameters under the said section have been complied with, as in this case. Thus, on these grounds, the Court finds nothing defective in the preliminary issuance of Search Warrant No. 09-14407. Perforce, the RTC-Manila should not have overturned it. C. Compliance with the Constitutional Requirements for the Issuance of Search Warrant No. 0914407 and the One-SpecificOffense Rule Under Section 4, Rule 126 of the Rules of Court.

In order to protect the people’s right against unreasonable searches and seizures, Section 2, Article III of the 1987 Philippine Constitution (Constitution) provides that no search warrant shall issue except upon probable causeto be determined personally by the judgeafter examination under oath or affirmation of the complainant and the witnesses he may produce, and particularly describing the place to be searched and the persons or things to be seized: SEC. 2. The right of the people to be secure in their persons, houses, papers, and effects against unreasonable searches and seizures of whatever nature and for any purpose shall be inviolable, and no search warrant or warrant of arrest shall issue except upon probable cause to be determined personally by the judge after examination under oath or affirmation of the complainant and the witnesses he may produce, and particularly describing the place to besearched and the persons or things to be seized. Complementarily, Section 4, Rule 126 of the Rules of Court states that a search warrant shall not be issued except upon probable cause in connection with one specific offense: SEC. 4. Requisites for issuing search warrant. - A search warrant shall not issue except upon probable cause in connection with one specific offenseto be determined personally by the judge after examination under oath or affirmation of the complainant and the witnesses he may produce, and particularly describing the place to be searched and the things to be seized which may be anywhere in the Philippines. (Emphasis supplied) In this case, the existence of probable cause for the issuance of Search Warrant No. 09-14407 is evident from the first-hand account of Avasola who, in his deposition, stated that he personally witnessed the commission of the afore-stated crime and was, in fact, part of the group that buried the victims: Q9-Who are these six (6) male victims who were killed and buried in the caves in December 2005 at around 9:00 p.m.?

A9-I heard Tatay Laud calling the names of the two victims when they were still alive as Pedro and Mario. I don’t know the names of the other four victims. Q10-What happened after Pedro, Mario and the other four victims were killed? A10-Tatay Laud ordered me and the six (6) killers to bring and bury equally the bodies inthe three caves. We buried Pedro and Mario altogether in the first cave, located more or less 13 meters from the makeshift house of Tatay Laud, the other two victims in the second cave and the remaining two in the third cave. Q11-How did you get there at Laud Compound in the evening of December 2005? A11-I was ordered by Tatay Laud to go [to] the place. I ran errands [for] him.46 Avasola’s statements in his deposition were confirmed during the hearing on July 10, 2009, where Judge Peralta conducted the following examination: Court: x x x Anong panandaan mo? Nandoon ka ba noong naghukay, nakatago o kasama ka? Mr. Avasola: Kasama po ako sa pagbuhat ng mga tao, sir. Court: Mga ilang katao? Mr. Avasola: Anim (6) po. Court: May mass grave ba na nahukay? Mr. Avasola: May tatlong kweba po na maliliit yung isa malaki. x x x.47 Verily, the facts and circumstancesestablished from the testimony of Avasola, who was personally examined by Judge Peralta, sufficiently show that more likely than not the crime of Murder of six (6) persons had been perpetrated and that the human remains in connection with the same are in the place sought to be searched. In Santos v. Pryce Gases, Inc.,48 the Court explained the quantum of evidence necessary to establish probable cause for a search warrant, as follows: Probable cause for a search warrant is defined as such facts and circumstances which would lead a reasonably discrete and prudent man to believe that an offense has been committed and that the objects sought in connection with the offense are in the place sought to be searched. A finding of probable cause needs only torest on evidence showing that, more likely than not, a crime has been committed and that it was committed by the accused. Probable cause demands more than bare suspicion; it requires less than evidence which would justify conviction. The existence depends to a large degree upon the finding or opinion of the judge conducting the examination. However, the findings of the judge should not disregard the facts before him nor run counter to the clear dictates of reason.49 In light of the foregoing, the Court finds that the quantum of proof to establish the existence of probable cause had been met. That a "considerable length of time" attended the search warrant’s application from the crime’s commission does not, by and of itself, negate the veracity of the applicant’s claims or the testimony of the witness presented. As the CA correctly observed, the delay may be accounted for by a witness’s fear of reprisal and natural reluctance to get involved in a criminal case.50 Ultimately, in determining the existence of probable cause, the facts and

circumstances must be personally examined by the judge in their totality, together with a judicious recognition of the variable complications and sensibilities attending a criminal case. To the Court’s mind, the supposed delay in the search warrant’s application does not dilute the probable cause finding made herein. In fine, the probable cause requirement has been sufficiently met. The Court similarly concludes that there was compliance with the constitutional requirement that there be a particular description of "the place to be searched and the persons or things to be seized." "[A] description of a place to be searched is sufficient if the officer with the warrant can, with reasonable effort, ascertain and identify the place intended and distinguish it from other places in the community. Any designation or description known to the locality that points out the place to the exclusion of all others, and on inquiry leads the officers unerringly to it, satisfies the constitutional requirement."51 Search Warrant No. 09-14407 evidently complies with the foregoing standard since it particularly describes the place to be searched, namely, the three (3) caves located inside the Laud Compound in Purok 3, Barangay Maa, Davao City: You are hereby commanded to makean immediate search at any time [of] the day of the premises above describe[d] particularly the three (3) caves (as sketched) inside the said Laud Compound, Purok 3, Brgy. Ma-a, Davao Cityand forthwith seize and take possession of the remains of six (6) victims who were killed and buried in the just said premises. x x x x52 (Emphases supplied) For further guidance in its enforcement, the search warrant even made explicit reference to the sketch53contained in the application. These, in the Court’s view, are sufficient enough for the officers to, with reasonable effort, ascertain and identify the place to be searched, which they in fact did. The things to be seized were also particularly described, namely, the remains of six (6) victims who were killed and buried in the aforesaid premises. Laud’s posturing that human remains are not "personal property" and, hence, could not be the subject of a search warrant deserves scant consideration. Section 3, Rule 126 of the Rules of Court states: SEC. 3.Personal property to be seized. – A search warrant may be issued for the search and seizure of personal property: (a) Subject of the offense; (b) Stolen or embezzled and other proceeds, or fruits of the offense; or (c) Used or intended to be used as the means of committing an offense. (Emphases supplied) "Personal property" in the foregoing context actually refers to the thing’s mobility, and not to its capacity to be owned or alienated by a particular person. Article416 of the Civil Code,54 which Laud himself cites,55 states that in general, all things which can be transported from place to place are deemed to be personal property. Considering that human remains can generally be transported from place toplace, and considering further that they qualify under the phrase "subject of the offense" given that they prove the crime’s corpus delicti,56it follows that they may be valid subjects of a search warrant under the above-cited criminal procedure provision. Neither does the Court agree with Laud’s contention that the term "human remains" is too all-embracing so as to subvert the particular description requirement.

Asthe Court sees it, the description points to no other than the things that bear a direct relation to the offense committed, i.e., of Murder. It is also perceived that the description is already specific as the circumstances would ordinarily allow given that the buried bodies would have naturally decomposed over time. These observations on the description’s sufficient particularity square with the Court’s pronouncement in Bache and Co., (Phil.), Inc. v. Judge Ruiz,57wherein it was held: A search warrant may be said to particularly describe the things to be seized when the description therein is as specific as the circumstances will ordinarily allow(People v. Rubio, 57 Phil. 384 [1932]); or when the description expresses a conclusion of fact — not of law — by which the warrant officer may be guided in making the search and seizure (idem., dissent of Abad Santos, J.); or when the things described are limited to those which bear direct relation to the offense for which the warrant is being issued(Sec. 2, Rule 126, Revised Rules of Court) x x x If the articles desired to be seized have any direct relation to an offense committed, the applicant must necessarily have some evidence, other than those articles, to prove the said offense; and the articles subject of search and seizure should come in handy merely to strengthen such evidence. (Emphases supplied)58 Consequently, the Court finds that the particular description requirement – both as to the place to be searched and the things to be seized – had been complied with. Finally, the Court finds no violation of the one-specific-offense rule under Section 4, Rule 126 of the Rules of Court as above-cited which, to note, was intended to prevent the issuance of scattershot warrants, or those which are issued for more than one specific offense. The defective nature of scatter-shot warrants was discussed in the case of People v. CA59 as follows: There is no question that the search warrant did not relate to a specific offense, in violation of the doctrine announced in Stonehill v. Diokno and of Section 3 [now, Section 4] of Rule 126 providing as follows: SEC. 3. Requisites for issuing search warrant.— A search warrant shall not issue but upon probable cause in connection with one specific offense to be determined personally by the judge after examination under oath or affirmation of the complainant and the witnesses he may produce, and particularly describing the place to be searched and the things to be seized. Significantly, the petitioner has not denied this defect in the search warrant and has merely said that there was probable cause, omitting to continue that it was in connection withone specific offense. He could not, of course, for the warrant was a scatter-shot warrant that could refer, in Judge Dayrit’s own words, "to robbery, theft, qualified theft or estafa." On this score alone, the search warrantwas totally null and void and was correctly declared to be so by the very judge who had issued it.60 In Columbia Pictures, Inc. v. CA,61 the Court, however, settled that a search warrant that covers several counts of a certain specific offense does not violate the one-specific-offense rule, viz.: That there were several counts of the offenseof copyright infringement and the search warrant uncovered several contraband items in the form of pirated video tapes is not to be confused with the number of offenses charged. The search warrant herein issued does not violate the one-specificoffense rule. (Emphasis supplied)62 Hence, given that Search Warrant No. 09-14407 was issued only for one specific offense – that is, of Murder, albeit for six (6) counts – it cannot be said that Section 4, Rule 126 of the Rules of Court had been violated. That being said, the Court now resolves the last issue on forum shopping. D. Forum Shopping.

There is forum shopping when a litigant repetitively avails of several judicial remedies in different courts, simultaneously or successively, all substantially founded on the same transactions and the same essential facts and circumstances, and all raising substantially the same issues either pending in or already resolved adversely by some other court to increase his chances of obtaining a favorable decision if not in one court, then in another.63 Forum shopping cannot be said to have been committed in this case considering the various points of divergence attending the search warrant application before the Manila-RTC and that before the Davao-RTC. For one, the witnesses presented in each application were different. Likewise, the application filed in Manila was in connection with Murder, while the one in Davao did not specify any crime. Finally, and more importantly, the places to be searched were different – that inManila sought the search of the Laud Compound caves, while that in Davao was for a particular area in the Laud Gold Cup Firing Range. There being no identity of facts and circumstances between the two applications, the ruleagainst forum shopping was therefore not violated. Thus, for all the above-discussed reasons, the Court affirms the CA Ruling which upheld the validity of Search Warrant No. 09-14407. WHEREFORE, the petition is DENIED. The Decision dated April 25, 2011 and the Resolution dated October 17, 2011 of the Court of Appeals in CA-G.R. SP. No. 113017 are hereby AFFIRMED.

SECOND DIVISION

PEOPLE OF THE PHILIPPINES, G.R. No. 152950 HON. LOURDES F. GATBALITE, Presiding Judge, Branch 56, Regional Trial Court, Angeles City and ATTY. BENNIE NICDAO, Special Prosecutor, Special Operative Group, Economic Intelligence & Investigation Bureau, Petitioners,

Present:

PUNO, J., Chairperson, - v e r s u s - SANDOVAL-GUTIERREZ, CORONA, AZCUNA and GARCIA, JJ.

CHRISTOPHER CHOI, Respondent. Promulgated: August 3, 2006

x------------------------------------------x

DECISION

CORONA, J.:

This petition for review on certiorari[1] seeks the reversal of the decision[2] of the Court of Appeals (CA) dated April 10, 2002 in CAG.R. SP No. 59587, the dispositive portion of which read:

WHEREFORE, the petition for certiorari and prohibition is GRANTED. Search Warrant No. 99-17 is deemed NULL and VOID and SET ASIDE. Respondent ATTY. BENNY NICDAO is prohibited from using in evidence the articles seized by virtue of Search Warrant No. 99-17 in Crim. Case No. I.S. No. 99-8116.

SO ORDERED.[3]

The factual antecedents follow.

On April 27, 1999, Mario P. Nieto, Intelligence Operative of the Economic Intelligence and Investigation Bureau, Department of Finance, applied for a search warrant with the Regional Trial Court (RTC) of Angeles City, Pampanga, Branch 56,[4] against respondent Christopher Choi for violation of Section 168, paragraphs 2 and 3 (a) and (c), in relation to Section 169 of RA 8293,[5] also known as the Intellectual Property Code.[6] After examination of the applicant and his witnesses, namely, Max Cavalera and David Lee Sealey, Judge Lourdes F. Gatbalite issued Search Warrant No. 99-17 dated April 27, 1999 worded as follows:

TO ANY PEACE OFFICER:

G r e e t i n g s:

It appearing to the satisfaction of the undersigned, after examining under oath in the form of searching and probing questions, the applicant, MARIO P. NIETO, Intelligence Operative, Economic Intelligence Investigation Bureau, Department of Finance, and his witnesses Max Cavalera and David Lee Sealey that there are good and sufficient reasons to believe that Christopher Choi of No. 25-13 Columbia Street, Carmenville Subd., Angeles City has in his possession, control and custody [r]eams and packs of fake Marlboro Red Cigarettes, as well as cardboard cases of fake Marlboro Red Cigarettes (each cardboard case contains two (2) [m]aster [c]ases of Marlboro and each [m]aster case contains fifty (50) reams) being distributed, kept and sold thereat in violation of Section 168, par. 2 and 3 (a) and (c) in relation to Section 169 of R.A. 8293;

You are hereby commanded to make an immediate search at anytime of the day or night of the above-premises and forthwith seize and take possession of the aforedescribed items found at the residence/warehouse of Christopher Choi at No. 25-13 Columbia Street, Carmenville Subd., Angeles City.

THEREFORE, seize and bring the said articles to the undersigned to be dealt with in accordance with law.

You are hereby further directed to submit a return within ten (10) days from today.

Given under my hand this 27th day of April, 1999 at Angeles City, Philippines.[7]

The search was conducted on the same date.[8] On May 12, 1999, respondent filed a motion to quash search warrant[9] and a supplemental motion to quash[10] on June 22, 1999. Both were denied by Judge Gatbalite in an order dated November 29, 1999.[11] Reconsideration was likewise denied.[12] On June 19, 2000, respondent filed a petition for certiorari and prohibition[13] before the CA. He alleged that Judge Gatbalite committed grave abuse of discretion in refusing to quash the search warrant,

arguing

that

probable

cause

was

not

sufficiently

established as the examination conducted was not probing and exhaustive and the warrant did not particularly describe the place to

be

searched. Respondent

also

prayed

that

Atty.

Bennie

Nicdao[14] be prohibited from using as evidence the articles seized by virtue of the search warrant. This was granted by the CA in a decision dated April 10, 2002. According to the CA, in determining whether there was probable cause to believe that the cigarettes purchased by Nieto were fake and in violation of RA 8293,[15] Judge Gatbalite failed to ask searching and probing questions of witness David Lee Sealey.[16] The examination of Sealey went this way:

Court:

Q There was testimony here given by Mr. Mario Nieto and Max Cavalera, that fake Marlboro cigarettes bought by them from Michael Chua, Christopher Choi and Johnny Chang were turned over to you for examination, is that correct? A Yes, your Honor.

Q After the same had been turned over to you, what did you do with the said merchandise, if you did anything? A I examined the sample of cigarettes and their packaging bearing the Marlboro Trade Marks which were suspected to be produc[ed] and manufactured by La Suerte or [with] the permission of Philip Morris.

Q What was the result of your examination? A Based on the packaging of the packs, the color of the box and the printing on the front side of the packs and the cigarettes themselves, I concluded that they are counterfeit or unauthorized product[s].

Q Do you have any knowledge of this person named Christopher Choi? A None, your Honor.

Q There is an affidavit here marked as exhibit, executed by one David Lee Sealey, do you know this David Lee Sealey? A Yes, your Honor, I am the one.

Q Whose signature is this appearing on the printed name David Lee Sealey? A This is my signature, your Honor.

Q Do you affirm and confirm other contents of this affidavit? A Yes, your Honor.

Court:

Thats all.[17]

In addition, the CA ruled that Judge Gatbalite committed grave abuse of discretion when she merely relied on the conclusion of Sealey that the cigarettes he received from Nieto were fake. She should have at least required Sealey to present the alleged fake Marlboro cigarettes and the genuine ones for comparison, instead of relying on his testimony alone. The CA reasoned that this was an absolute

requirement

under

the

Supreme

Court

in 20th Century Fox Film Corporation v. Court of Appeals.[18]

ruling

Hence, this petition. The People of the Philippines aver that the CA erred in finding that Judge Gatbalite committed grave abuse of discretion in issuing the search warrant allegedly because she failed to determine probable cause pursuant to Sections 4 and 5 of Rule 126 of the Rules of Court.[19] The People assail the finding of the CA that, in issuing the search warrant, Judge Gatbalite purportedly did not comply strictly with the requirement to determine the existence of probable cause by personally examining the applicant and his witnesses through searching questions and answers. The People also

assert

that

the

CA

erred

in

applying

the

doctrine

in 20th Century Fox Film Corporation[20] since it had already been superseded by Columbia Pictures, Inc. v. Court of Appeals.[21] We rule for the People of the Philippines.

Sections 4 and 5 of Rule 126 state: Sec. 4. Requisites for issuing search warrant. A search warrant shall not issue except upon probable cause in connection with one specific offense to be determined personally by the judge after examination under oath or affirmation of the complainant and the witnesses he may produce, and particularly describing the place to be searched and the things to be seized which may be anywhere in the Philippines.

Sec. 5. Examination of complainant; record. The judge must, before issuing the warrant, personally examine in the form of searching questions and answers, in writing and under oath, the complainant and the witnesses he may produce on facts personally known to them and attach to the record their sworn statements, together with the affidavits submitted.

According to the foregoing provisions, a search warrant can be issued only upon a finding of probable cause. Probable cause means such facts and circumstances which would lead a reasonably discreet and prudent man to believe that an offense has been committed and that the objects sought in connection with the offense are in the place sought to be searched.[22] The determination of the existence of probable cause requires the following: (1)

the judge must examine the complainant and his witnesses personally;

(2)

the examination must be under oath and

(3)

the examination must be reduced in writing in the form of searching questions and answers.[23]

The searching questions propounded to the applicant and the witnesses depend largely on the discretion of the judge. Although there is no hard-and-fast rule governing how a judge should

conduct his examination, it is axiomatic that the examination must be

probing

and

exhaustive,

not

merely

routinary,

general,

peripheral, perfunctory or pro-forma.[24] The judge must not simply rehash the contents of the affidavit but must make his own inquiry on the intent and justification of the application.[25] The questions should not merely be repetitious of the averments stated in the affidavits or depositions of the applicant and the witnesses.[26] If the judge fails to determine probable cause by personally examining the applicant and his witnesses in the form of searching questions before issuing a search warrant, grave abuse of discretion is committed.[27]

The determination of probable cause does not call for the application of rules and standards of proof that a judgment of conviction requires after trial on the merits. As the term implies, probable cause is concerned with probability, not absolute or even moral certainty.

The standards of judgment are those of a

reasonably prudent man, not the exacting calibrations of a judge after a full-blown trial.[28] No law or rule states that probable cause requires a specific kind of evidence. No formula or fixed rule for its determination exists.[29] Probable cause is determined in the light of conditions obtaining in a given situation.[30] The entirety of the questions propounded by the court and the answers thereto must be considered by the judge.[31]

In this case, aside from the testimony of Sealey, petitioner judge also heard the testimony of applicant Nieto: Q: In connection with Search Warrant 99-17, are you the same Mario Nieto who is the applicant in this application for search warrant filed today April 27, 1999? A: Yes, your Honor.

Q: Do you know this Christopher Choi referred to herein? A: Yes, your Honor.

Q: Why do you know him? A: He was introduced to us by Michael Chua, your Honor.

Q: As what? A: As the supplier for the goods.

Q: Subject of the application? A: Yes, your Honor, in violation of Section 169 of R.A. 8293.

Q: How did you know him? A: When I was conducting a test-buy operation against Mr. Michael Chua, Mr. Michael Chua told me that the bulk of supply if we need more supply we can get from the source,

a certain Christopher Choi, who lives in the same village and who is actually the supplier for the entire region.

Q: Where did you see him. This Christopher Choi? A: I went to his house, your Honor.

Q: Where? A: At No. 25-13 Columbia St., Carmenville Subd., Angeles City, Pampanga.

Q: Upon arriving at the place what did you do? A: Upon arriving at the place, your Honor, I introduced myself as the one who was referred by a certain Michael Chua who is interested in buying the Marlboro cigarettes from him and he accommodated me and showed me the sample that he has and I was able to procure the samples from him, the samples that like what we did to the others were inspected by certain Mr. David Lee Sealey, the representative and authority from the Philip Morris.

Q: Did you actually buy those samples? A: Yes, your Honor, I got the samples form Mr. Christopher Choi and I submitted them to Mr. David Lee Sealey.

Q: How many Marlboro cigarettes did you buy? A: We bought only one ream, P17.00 per pack.

Q: Do you know from what particular place the house of Christopher Choi did he got (sic) those samples? A: The volume stocks were found inside the house, they are almost everywhere in the house of Christopher Choi.

Q: There is a sketch here attached to your application, can you point it out here? A: Yes, your Honor, at the warehouse, in the storage room as shown in the lay out of the house, it is adjacent to the residential house as shown in the sketch.

Q: You went to the warehouse? A: We were shown [the] entire area by the supplier, Christopher Choi. As a matter of fact he was trying to show us how much volume he has and his capacity to supply.[32]

Max Cavalera, a witness who accompanied Nieto during the test-buy operation,[33] also testified: Q How about this Christopher Choi? A As Ive said earlier, he was one of those identified by the informant storing and selling counterfeit Marlboro cigarettes, so on April 22, 1999 we conducted a surveillance and we were able to confirm that the said cigarettes are being stored at the subject place.

Q At what place? A At 25-13 Columbia St., Carmenville Subd., Angeles City. On April 23, 1999 at about 8:30 p.m., Mario Nieto and I again went to the subject place to conduct a test-buy operation. [A]fter Mr. Choi had been convinced of our intention to buy cigarettes from him, he brought us to his warehouse where he showed to us several cardboard cases of Marlboro cigarettes.[34]

Given the foregoing testimonies and applying the established standards in determining probable cause, we cannot say that Judge Gatbalite committed grave abuse of discretion in issuing the search warrant. Her questions

were sufficiently probing, not at all

superficial and perfunctory. The testimonies were consistent with each other and the narration of facts was credible. The testimonies and other evidence on record constituted adequate bases to establish probable cause that the alleged offense had been committed. Since probable cause is dependent largely on the opinion and findings of the judge who conducted the examination and who had the opportunity to question the applicant and his witnesses,[35] the findings of the judge deserve great weight. The reviewing court can overturn such findings only upon proof that the judge disregarded

the facts before him or ignored the clear dictates of reason.[36] We thus find no reason to disturb Judge Gatbalites findings. Furthermore,

as

correctly

pointed

out

by

petitioners, 20th Century Fox Film Corporation, insofar as it required the presentation of the master tapes for comparison with the pirated copies for a search warrant to issue, had already been superseded by Columbia Pictures, Inc. v. Court of Appeals:

More to the point, it is felt that the reasonableness of the added requirement in 20th Century Fox calling for the production of the master tapes of the copyrighted films for determination of probable cause in copyright infringement cases needs revisiting and clarification.

xxx xxx xxx

In fine, the supposed pronunciamento in said case regarding the necessity for the presentation of the master tapes of the copyrighted films for the validity of search warrants should at most be understood to merely serve as a guidepost in determining the existence of probable cause in copyright infringement cases where there is doubt as to the true nexus between the master tape and the pirated copies. An objective and careful reading of the decision in said case could lead to no other conclusion than that said directive was hardly intended to be a sweeping and inflexible requirement in all or similar copyright infringement cases. Judicial dicta should always be construed within the factual matrix of their parturition, otherwise a careless interpretation thereof could unfairly fault the writer with

the vice of overstatement and the reader with the fallacy of undue generalization. xxx xxx xxx

It is evidently incorrect to suggest, as the ruling in 20th Century Fox may appear to do, that in copyright infringement cases, the presentation of master tapes of the copyrighted films is always necessary to meet the requirement of probable cause and that, in the absence thereof, there can be no finding of probable cause for the issuance of a search warrant. It is true that such master tapes are object evidence, with the merit that in this class of evidence the ascertainment of the controverted fact is made through demonstrations involving the direct use of the senses of the presiding magistrate. Such auxiliary procedure, however, does not rule out the use of testimonial or documentary evidence, depositions, admissions or other classes of evidence tending to prove the factum probandum, especially where the production in court of object evidence would result in delay, inconvenience or expenses out of proportion to its evidentiary value.

xxx xxx xxx

Accordingly, to restrict the exercise of discretion by a judge by adding a particular requirement (the presentation of master tapes, as intimated by 20th Century Fox) not provided nor implied in the law for a finding of probable cause is beyond the realm of judicial competence or statesmanship. It serves no purpose but to stultify and constrict the judicious exercise of a courts prerogatives and to denigrate the judicial duty of determining the existence of probable cause to a mere ministerial or mechanical

function. There is, to repeat, no law or rule which requires that the existence of probable cause is or should be determined solely by a specific kind of evidence. Surely, this could not have been contemplated by the framers of the Constitution, and we do not believe that the Court intended the statement in 20th Century Fox regarding master tapes as the dictum for all seasons and reasons in infringement cases.[37] (emphasis supplied)

It is obvious that 20th Century Fox Film Corporation should not be applied to the present case since this involves the offense of unfair competition and not copyright infringement. More importantly,

as

pronounced

by

the

Court

in Columbia

Pictures, Inc., the judges exercise of discretion should not be unduly restricted by adding a requirement that is not sanctioned by law.

WHEREFORE, the petition is hereby GRANTED. The assailed decision of the Court of Appeals dated April 10, 2002 in CA-G.R. SP No. 59587 is REVERSEDand SET ASIDE. Judgment is hereby rendered declaring Search Warrant No. 99-17 as VALID. G.R. No. L-69803 October 8, 1985 CYNTHIA D. NOLASCO, MILA AGUILAR-ROQUE and WILLIE C. TOLENTINO, petitioners, vs. HON. ERNANI CRUZ PAÑO, Executive Judge, Regional Trial Court of Quezon City; HON. ANTONIO P. SANTOS, Presiding Judge, Branch XLII, Metropolitan Trial Court of Quezon City: HON. SERGIO F. APOSTOL, City Fiscal, Quezon City; HON. JUAN PONCE ENRILE, LT. GEN. FIDEL RAMOS and COL. JESUS ALTUNA, respondents. Jose W .Diokno, Joker P. Arroyo, Rene A. V. Sarmiento, Dan Malabonga and Cesar Maravilla for petitioners.

MELENCIO-HERRERA, J.: The facts before the Court in these Certiorari, Prohibition, and mandamus proceedings will be briefly stated. The three petitioners will be referred to through their surnames of NOLASCO, AGUILARROQUE and TOLENTINO. 1. Prior to August 6, 1984 (hereinafter to be referred to without the year), AGUILAR-ROQUE was one of the accused of Rebellion in Criminal Case No. MC-25-113 of Military Commission No. 25, both cases being entitled "People of the Philippines vs. Jose Ma. Sison, et al." She was then still at large. 2. At 11:30 A.M. on August 6th, AGUILAR-ROQUE and NOLASCO were arrested by a Constabulary Security Group (CSG) at the intersection of Mayon Street and P. Margall Street, Quezon City. The stated time is an allegation of petitioners, not denied by respondents. The record does not disclose that a warrant of arrest had previously beeen issued against NOLASCO. 3. At 12:00 N. on August 6th, elements of the CSG searched the premises at 239-B Mayon Street, Quezon City. The stated time is an allegation of petitioners, not specifically denied by respondents. In their COMMENT, however, respondents have alleged that the search was conducted "late on the same day"; that is late on august 6th. 4. On August 6th, at around 9:00 A.M., Lt. Col. Virgilio G. Saldajeno of the CSG, applied for a Search Warrant from respondent Hon. Ernani Cruz Paño, Executive Judge of the Regional Trial Court in Quezon City, to be served at No. 239-B Mayon Street, Quezon City, determined tyo be the leased residence of AGUILAR-ROQUE, after almost a month of "round the clock surveillance" of the premises as a "suspected underground house of the CPP/NPA." AGUILAR-ROQUE has been long wanted by the military for being a high ranking officer of the Communist Party of the Philippines, particularly connected with the MV Karagatan/Doña Andrea cases. In connection with the Search Warrant issued, the following may be stated: (a) The Search Warrant was issued in proceedings entitled "People of the Philippines vs. Mila Aguilar-Roque, Accused, Search Warrant No. 80- 84 for rebellion" (the SEARCH WARRANT CASE). Judge Panos Court was Branch 88. (b) It does not appear from the records before us that an application in writing was submitted by Lt. Col. Saldajeno to Judge Paño. (c) According to the record, Lt. Col. Saldajeno and his witness S/A Dionicio A. Lapus, were examined under oath by Judge Paño but only the deposition of S/A Lapus has been submitted to us. The latter deposed that to his personal knowledge, there were kept in the premises to be searched records, documents and other papers of the CPP/NPA and the National Democratic Front, including support money from foreign and local sources intended to be used for rebellion. 1 5. In connection with the search made at 12:00 N. of August 6th the following may be stated: (a) TOLENTINO was a person then in charge of the premises. He was arrested by the searching party presumably without a warrant of arrest.

(b) The searching party seized 428 documents and written materials, 2 and additionally a portable typewriter, and 2 wooden boxes, making 431 items in all. 3 (c) According to the Return, submitted in the SEARCH WARRANT CASE on August 10th, 4 the search was made in the presence of Dra. Marciana Galang, owner of the premises, and of two (2) Barangay Tanods. No mention was made that TOLENTINO was present. The list of the 428 articles and documents attached to the Return was signed by the two Barangay Tanods, but not by Dra. Galang. 6. (a) On August 10th, the three petitioners, AGUILAR-ROQUE, NOLASCO and TOLENTINO, were charged before the Quezon City Fiscal's Office (the CITY FISCAL, for short) upon complaint filed by the CSG against petitioners for "Subversion/Rebellion and/or Conspiracy to Commit Rebellion/Subversion." (b) On August 13th, the CITY FISCAL filed an Information for Violation of Presidential Decree No. 33 (Illegal Possession of Subversive Documents) against petitioners before Branch 42 of the Metropolitan Trial Court of Quezon City (the SUBVERSIVE DOCUMENTS CASE), respondent Judge Antonio P. Santos, presiding. (c) On August 16th, CSG filed a Motion for Reconsideration with the CITY FISCAL, praying that AGUILAR-ROQUE and NOLASCO be charged with Subversion. The Motion was denied on November 16th. 7. (a) On September 10th, the CSG submitted an Amended Return in the SEARCH WARRANT CASE praying, inter alia, that the CSG be allowed to retain the seized 431 documents and articles, in connection with cases that are presently pending against Mila Aguilar Roque before the Quezon City Fiscal's Office and the court. 5 (b) On September 28th, petitioners were required by Judge Pano to comment on the Amended Return, which AGUILAR-ROQUE did on October 18th, raising the issue of the inadmissibility of any evidence obtained pursuant to the Search Warrant. (c) On December 13, 1984, Judge Paño admitted the Amended Return and ruled that the seized documents "shall be subject to disposition of the tribunal trying the case against respondent." 8. (a) On December 12th, petitioners filed a Motion to Suppress in the SUBVERSIVE DOCUMENTS CASE, praying that such of the 431 items belonging to them be returned to them. It was claimed that the proceedings under the Search Warrant were unlawful. Judge Santos denied the Motion on January 7, 1985 on the ground that the validity of the Search Warrant has to be litigated in the SEARCH WARRANT CASE. He was apparently not aware of the Order of Judge Paño of December 13th issued in the SEARCH WARRANT CASE. Hence, this Petition for Certiorari, Prohibition and mandamus to annul and set aside the (1) Search Warrant issued by respondent RTC Judge Paño; (2) his Order admitting the Amended Return and granting the Motion to Retain Seized Items; and (3) Order of respondent MTC Judge Santos denying petitioners' Motion to Suppress. This Court, on February 12, 1985, issued a Temporary Restraining Order enjoining the respondents or their duly authorized representatives from introducing evidence obtained under the Search Warrant.

The PETITIONERS principally assert that the Search Warrant is void because it is a general warrant since it does not sufficiently describe with particularity the things subject of the search and seizure, and that probable cause has not been properly established for lack of searching questions propounded to the applicant's witness. The respondents, represented by the Solicitor General, contend otherwise, adding that the questions raised cannot be entertained in this present petition without petitioners first moving for the quashal of the disputed Search Warrant with the issuing Judge. We find merit in the Petition. Section 3, Article IV of the Constitution, guarantees the right of the people to be secure in their persons, houses, papers and effects against unreasonable searches and seizures of whatever nature and for any purpose. It also specifically provides that no Search Warrant shall issue except upon probable cause to be determined by the Judge or such other responsible officer as may be authorized by law, after examination under oath or affirmation of the complainant and the witnesses he may produce, and particularly describing the place to be searched and the things to be seized. The disputed Search Warrant (No. 80-84) describes the personalities to be seized as follows: Documents, papers and other records of the Communist Party of the Phihppines/New Peoples Army and/or the National Democratic Front, such as Minutes of the Party Meetings, Plans of these groups, Programs, List of possible supporters, subversive books and instructions, manuals not otherwise available to the public, and support money from foreign or local sources. It is at once evident that the foregoing Search Warrant authorizes the seizure of personal properties vaguely described and not particularized. It is an all- embracing description which includes everything conceivable regarding the Communist Party of the Philippines and the National Democratic Front. It does not specify what the subversive books and instructions are; what the manuals not otherwise available to the public contain to make them subversive or to enable them to be used for the crime of rebellion. There is absent a definite guideline to the searching team as to what items might be lawfully seized thus giving the officers of the law discretion regarding what articles they should seize as, in fact, taken also were a portable typewriter and 2 wooden boxes. It is thus in the nature of a general warrant and infringes on the constitutional mandate requiring particular description of the things to be seized. In the recent rulings of this Court, search warrants of similar description were considered null and void for being too general. Thus: Subversive documents, pamphlets, leaflets, books, and other publications to promote the objectives and purposes of the subversive organizations known as Movement for Free Philippines. Light-a-Fire Movement and April 6 Movement. 6 The things to be seized under the warrant issued by respondent judge were described as 'subversive documents, propaganda materials, FAs, printing paraphernalia and all other subversive materials Such description hardly provided a definite guideline to the search team as to what articles might be lawfully seized thereunder. Said description is no different from if not worse than, the description found in the search warrants in "Burgos, et al. v. the Chief of Staff"which this Court declared null and void for being too general. 7 In the case at bar, the search warrant issued by respondent judge allowed the seizure of printed copies of the Philippine Times, manuscripts/drafts of articles for publication, newspaper dummies subversive documents, articles, etc., and even

typewriters, duplicating machines, mimeographing and tape recording machines. Thus, the language used is so all embracing as to include all conceivable records and equipment of petitioner regardless of whether they are legal or illegal. The search warrant under consideration was in the nature of a general warrant which is constitutionally objectionable. 8 The lack of particularization is also evident in the examination of the witness presented by the applicant for Search Warrant. Q Mr. Dionicio Lapus, there is an application for search warrant filed by Lt. Col. Virgilio Saldajeno and the Court would like to know if you affirm the truth of your answer in this deposition? (The deposition instead)— A Yes, sir, Q How long did it take you for the surveillance? A Almost a month, sir. Q Are you a lawyer, Mr. Lapus? A No, Your Honor, but I was a student of law. Q So, you are more or less familiar with the requisites of the application for search warrant? A Yes, Your Honor. Q How did you come to know of the person of Mila Aguilar-Roque? A Because of our day and night surveillance, Your Honor, there were so many suspicious persons with documents. Q What kind of documents do you refer to? A Documents related to the Communist Party of Philippines and New People's Army. Q What else? A Conferences of the top ranking officials from the National Democratic Front, Organization of the Communist Party of the Philippines ... Q And may include what else? A Other papers and documents like Minutes of the Party Meetings, Plans of these groups, Programs, List of possible supporters,

subversive books and instructions, manuals not otherwise available to the public and support money from foreign and local sources. 9 The foregoing questions propounded by respondent Executive Judge to the applicant's witness are not sufficiently searching to establish probable cause. The "probable cause" required to justify the issuance of a search warrant comprehends such facts and circumstances as will induce a cautious man to rely upon them and act in pursuant thereof. 10 Of the 8 questions asked, the 1st, 2nd and 4th pertain to Identity. The 3rd and 5th are leading not searching questions. The 6th, 7th and 8th refer to the description of the personalities to be seized, which is Identical to that in the Search Warrant and suffers from the same lack of particularity. The examination conducted was general in nature and merely repetitious of the deposition of said witness. Mere generalization will not suffice and does not satisfy the requirements of probable cause upon which a warrant may issue. 11 Respondents claim, however, that the proper forum for questioning the illegality of a Search Warrant is with the Court that issued it instead of this original, independent action to quash. The records show, however, that petitioners did raise that issue in the SEARCH WARRANT CASE in their Comment, dated October 18, 1984. In fact, they already questioned the admissibility of the evidence obtained under the Search Warrant, even during the inquest investigation on August 10, 1984. And in the SUBVERSIVE DOCUMENTS CASE, they filed a Motion to Suppress on December 12, 1984 claiming that the proceedings under the Search Warrant were unlawful. Substantially, therefore, while not denominated as a motion to quash, petitioners had questioned the legality of the Search Warrant. Parenthetically, it strikes the Court that the pendency of the SEARCH WARRANT CASE and of the SUBVERSIVE DOCUMENTS CASE before two different Courts is not conducive to an orderly administration of justice. It should be advisable that, whenever a Search Warrant has been issued by one Court, or Branch, and a criminal prosecution is initiated in another Court, or Branch, as a result of the service of the Search Warrant, the SEARCH WARRANT CASE should be consolidated with the criminal case for orderly procedure. The later criminal case is more substantial than the Search Warrant proceeding, and the Presiding Judge in the criminal case should have the right to act on petitions to exclude evidence unlawfully obtained. Notwithstanding the irregular issuance of the Search Warrant and although, ordinarily, the articles seized under an invalid search warrant should be returned, they cannot be ordered returned in the case at bar to AGUILAR-ROQUE. Some searches may be made without a warrant. Thus, Section 12, Rule 126, Rules of Court, explicitly provides: Section 12. Search without warrant of person arrested.—A person charged with an offense may be searched for dangerous weapons or anything which may be used as proof of the commission of the offense. The provision is declaratory in the sense that it is confined to the search, without a search warrant, of a person who had been arrested. It is also a general rule that, as an incident of an arrest, the place or premises where the arrest was made can also be search without a search warrant. In this latter case, "the extent and reasonableness of the search must be decided on its own facts and circumstances, and it has been stated that, in the application of general rules, there is some confusion in the decisions as to what constitutes the extent of the place or premises which may be searched. 12 "What must be considered is the balancing of the individual's right to privacy and the public's interest in the prevention of crime and the apprehension of criminals." 13 Considering that AGUILAR-ROQUE has been charged with Rebellion, which is a crime against public order; that the warrant for her arrest has not been served for a considerable period of time;

that she was arrested within the general vicinity of her dwelling; and that the search of her dwelling was made within a half hour of her arrest, we are of the opinion that in her respect, the search at No. 239-B Mayon Street, Quezon City, did not need a search warrant; this, for possible effective results in the interest of public order. Such being the case, the personalities seized may be retained. by CSG, for possible introduction as evidence in the Rebellion Case, leaving it to AGUILAR-ROQUE to object to their relevance and to ask Special Military Commission No.1 to return to her any and all irrelevant documents and articles. WHEREFORE, while Search Warrant No. 80-84 issued on August 6, 1984 by respondent Executive Judge Ernani Cruz Paño is hereby annulled and set aside, and the Temporary Restraining Order enjoining respondent from introducing evidence obtained pursuant to the Search Warrant in the Subversive Documents case hereby made permanent, the, personalities seized may be retained by the Constabulary Security Group for possible introduction as evidence in Criminal Case No. SMC-11, pending before Special Military commission No. 1, without prejudice to petitioner Mila AguilarRoque objecting to their relevance and asking said Commission to return to her any and all irrelevant documents and articles.

G.R. No. 122092. May 19, 1999]

PAPER INDUSTRIES CORPORATION OF THE PHILIPPINES, EVARISTO M. NARVAEZ JR., RICARDO G. SANTIAGO, ROBERTO A. DORMENDO, REYDANDE D. AZUCENA, NICEFORO V. AVILA, FLORENTINO M. MULA, FELIX O. BAITO, HAROLD B. CELESTIAL, ELMEDENCIO C. CALIXTRO, CARLITO S. LEGACION, ALBINO T. LUBANG, JEREMIAS I. ABAD and HERMINIO V. VILLAMIL, petitioners, vs. JUDGE MAXIMIANO C. ASUNCION, Presiding Judge, Branch 104, Regional Trial Court of Quezon City; STATE PROSECUTOR LEO B. DACERA III; and the SPECIAL OPERATIONS UNIT OF THE PNP TRAFFIC MANAGEMENT COMMAND,respondents. DECISION PANGANIBAN, J.:

To preserve and to uphold the constitutional right against unreasonable searches and seizures, the requisites for the issuance of a search warrant must be followed strictly. Where the judge fails to personally examine the applicant for a search warrant and the latters witnesses, or where the witnesses testify on matters not of their own personal knowledge, the search warrant must be struck down.

The Case

Before us is a Petition for Certiorari and Prohibition[1] praying for (1) the nullification of Search Warrant No. 799 (95) and the Orders dated March 23, 1993 and August 3, 1995, issued by the Regional Trial Court (RTC), Branch 104, of Quezon City;[2] and (2) the issuance of a temporary restraining order (TRO) or an injunction against State Prosecutor Leo B. Dacera III, ordering him to desist from proceeding with IS No. 95-167. In its October 23, 1995 Resolution,[3] this Court issued the TRO prayed for and required the respondents to comment on the said Petition. On December 20, 1995, Respondent PNP Traffic Management Command filed its 31-page Opposition[4] to the Petition, together with 90 pages of annexes.[5] On February 22, 1996, the Office of the Solicitor General filed its Comment[6] agreeing with petitioners that the writs prayed for must be granted. After petitioners filed a Reply to the Opposition, the Court gave due course to the Petition and required the parties to submit their respective memoranda. In view of the contrary opinion of the Office of the Solicitor General, the Court, in its February 5, 1997 Resolution,[7] required State Prosecutor Leo B. Dacera to prepare the memorandum for the public respondents. After issuing a show-cause order to Dacera on June 23, 1997,[8] the Court in its September 24, 1997 Resolution gave him a non-extendible period ending on October 31, 1997 within which to file the required memorandum. In view of Daceras manifestation that he was only a nominal party and that he had yet to receive the records of the case from the PNP, the Court, in its December 8, 1999 Resolution, ordered the Special Operations Unit (SOU) of the PNP Traffic Management Command to file its memorandum within thirty days from notice; otherwise, the petition will be deemed submitted for decision.[9] Even after the expiration of the said period, the required pleading was not yet received by this Court. Hence, this Court considered Respondent SOUs refusal/failure to submit its memorandum as a waiver of its privilege to do so.

The Facts

On January 25, 1995, Police Chief Inspector Napoleon B. Pascua applied for a search warrant before the said RTC of Quezon City, stating:[10]

1. That the management of Paper Industries Corporation of the Philippines, located at PICOP compound, Barangay Tabon, Bislig, Surigao del Sur, represented by its Sr. Vice President Ricardo G[.] Santiago, is in possession or ha[s] in [its] control high powered firearms, ammunitions, explosives, which are the subject of the offense, or used or intended to be used in committing the offense, and which xxx are [being kept] and conceal[ed] in the premises herein described. 2. That a Search Warrant should be issued to enable any agent of the law to take possession and bring to this Honorable Court the following described properties:

'Seventy (70) M16 Armalite rifles cal. 5.56, ten (10) M16 US rifles, two (2) AK-47 rifle[s], two (2) UZI submachinegun[s], two (2) M203 Grenade Launcher[s] cal.40mm, ten (10) cal.45 pistol[s], ten (10) cal.38 revolver[s], two (2) ammunition reloading machine[s], assorted ammunitions for said calibers of firearms and ten (10) handgrenades.' Attached to the application[11] were the joint Deposition of SPO3 Cicero S. Bacolod and SPO2 Cecilio T. Morito,[12] as well as a summary of the information and the supplementary statements of Mario Enad and Felipe Moreno. After propounding several questions to Bacolod, Judge Maximiano C. Asuncion issued the contested search warrant,[13] the pertinent portion of which reads:

It appearing to the satisfaction of the undersigned, after examining under oath, SPO3 Cicero S. Bacolod, that there is probable cause to believe that the management of Paper Industries Corporation of the Philippines, located at PICOP Compound, Barangay Tabon, Bislig, Surigao del Sur, represented by its Sr. Vice President Ricardo G. Santiago, has in its possession or control the following: Seventy (70) M16 Armalite rifles cal. 5.56 Ten (10) M14 US rifles Two (2) AK-47 rifle[s] Two (2) UZI submachinegun[s] Two (2) M203 Grenade Launcher[s] cal. 40mm. Ten (10) cal 45 pistol[s] Ten (10) cal 38 revolver[s] Two (2) ammunition reloading machine[s] Assorted ammunitions for said calibers of firearms Ten (10) handgrenades in violation of the Provisions of PD 1866 (Illegal Possession of Firearms, Ammunition and Explosives), and the same should be seized and brought before this Court. NOW, THEREFORE, you are hereby authorized to make an immediate search daytime between 8:00 a.m. [and] 4:00 p.m. of the aforementioned premises and to seize and bring the articles above-described and make an immediate return there[of][14] On February 4, 1995, the police enforced the search warrant at the PICOP compound and seized the following:[15]

MAKE/TYPE CALIBER SERIAL NUMBER BRAND

01 M16 Rifle 5.56 RP 175636 Elisco 02 M16 Rifle 5.56 RP 175636 (Tampered) Elisco 03 M16 Rifle 5.56 RP 171702 Elisco 04 M16 Rifle 5.56 Defaced Elisco 05 M16 Rifle 5.56 RP174253 (Tampered) Elisco 06 M16 Rifle 5.56 RP173627 (Tampered) Elisco 07 M16 Rifle 5.56 RP171337 Elisco 08 M16 Rifle 5.56 RP171114 Elisco 09 M16 Rifle 5.56 RP171114 (Tampered) Elisco 10 M16 Rifle 5.56 RP171167 (Tampered) Elisco 11 M16 Rifle 5.56 170881 (Tampered) Elisco 12 M16 Rifle 5.56 RP170897 Elisco 13 M16 Rifle 5.56 RP171509 Elisco (With pending case-Casaway Case) 14 M16 Rifle 5.56 RP 171754 Elisco 15 M16 Rifle 5.56 RP170881 (Tampered) Elisco 16 M16 Rifle 5.56 RP174637 Elisco 17 M16 Rifle 5.56 RP171366 Elisco 18 M16 Rifle 5.56 RP174637 (Tampered) Elisco 19 M16 Rifle 5.56 RP174610 Elisco 20 M16 Rifle 5.56 RP171367 (Tampered) Elisco 01 M14Rifle 7.62 1499694 Elisco 02 M14Rifle 7.62 889163 Elisco 01 BAR Cal. 30 865975 Royal 01 Carbine M1 Cal. 30 384181 US Carbin 02 Carbine M1 Cal. 30 998201 US Carbin 01 Garand M1 Cal. 30 1194008 Springfield 02 Garand M1 Cal. 30 3123784 Springfield 01 Shotgun 12 Gauge H359704 Omega 02 Shotgun 12 Gauge 9211 Homemade (Paltik) MAGAZINE ASSEMBLY QTY. 01 M16 (long) 29 pcs. 02 M16 (short) 48 pcs. 03 Carbine M1 171 pcs. 04 BAR 19 pcs.

LIVE AMMUNITION QTY. 01 M16 2,023 rounds 03 Carbine M1 276 rounds 04 M-60 Cal. 7.62 1,800 rounds 05 M1 Garand 1,278 rounds 06 Rifle Grenade 11 rounds 07 Hand Grenade 4 pcs. AMMO DAM POST NO. 24 MAKE/TYPE CALIBER SERIAL NUMBER BRAND 01. M16 Rifle 5.56 171425 (Tampered) Gyno Corp. 02. Machine Pistol .22 651 (Tampered) Landmann MAGAZINE ASSEMBLY QTY. 01. M16 (short) 3 pcs. 02. M16 (long) 1 pc. 03. M14 8 pcs. 04. Clip M1 Garand 3 pcs. 05. Mag Assy. Cal .22 1 pc. LIVE AMMUNITION QTY. 01. M16 73 rounds 02. M14 160 rounds 03. M1 Garand Cal .30 30 rounds 04. Rifle Grenade 1 round MANAGEMENT INTEL/INVEST UNIT MAKE/TYPE CALIBER SERIAL NUMBER BRAND 01. M16 Rifle 5.56 RP 171725 Elisco 02. M16 Rifle 5.56 RP 170799 (Tampered) Elisco 03. M16 Rifle 5.56 RP 132320 Elisco 04. Machine 9 MM 54887 Intratec Pistol 05. Three (3) 12 Gauge Surit-Surit (H) Shotguns

MAGAZINE ASSEMBLY QTY. 01. M16 (long) 3 pcs. 02. M16 (short) 4 pcs. 03. Intratec 1 pc. 04. US Carbine (defective) 2 pcs. LIVE AMMUNITION QTY. 01. M16 147 rds. 02. Cal. 30 5 rounds 03. 12 gauge Shotgun 7 rounds 04. Carbine 5 rounds 05. Rifle grenade (AVA-0051-84/0056-84) 2 rounds 06. 9MM 30 rounds NEW ARMORY POST NO. 16 MAKE/TYPE CALIBER SERIAL NUMBER BRAND 01. Shotgun 12 Gauge A359910 Armscor 02. Shotgun 12 Gauge A359716 Armscor 03. Shotgun 12 Gauge A359706 Armscor 04. Shotgun 12 Gauge A359707 Armscor 05. Shotgun 12 Gauge 1036847 Armscor 06. Shotgun 12 Gauge A359702 Armscor 07. Shotgun 12 Gauge A359732 Armscor 08. Shotgun 12 Gauge A359728 Armscor 09. Shotgun 12 Gauge A359708 Armscor 10. Shotgun 12 Gauge A359711 Armscor 11. Shotgun 12 Gauge A359723 Armscor 12. Shotgun 12 Gauge A359713 Armscor 13. Shotgun 12 Gauge 1031271 Armscor 14. Shotgun 12 Gauge A262338 SB 15. Shotgun 12 Gauge A261619 SB 16. Shotgun 12 Gauge Defaced Not Indicated LIVE AMMUNITION QTY. 01. 12 GAUGE shotgun 306 rds. 02. M16 2,349 rds.

MAGAZINE ASSEMBLY QTY. 01. Carbine (defective) 76 pcs. 02. Cal. 22 -do- 16 pcs. 03. M16 (long-defective) 2 pcs. 04. M16 (short-defective) 2 pcs. 05. Thompson (defective) 8 pcs. 06. Shotgun 12 Gauge (defective) 17 pcs. 07. BAR (defective) 2 pcs. Believing that the warrant was invalid and the search unreasonable, the petitioners filed a Motion to Quash[16] before the trial court. Subsequently, they also filed a Supplemental Pleading to the Motion to Quash and a Motion to Suppress Evidence.[17] On March 23, 1995, the RTC issued the first contested Order which denied petitioners motions.[18] On August 3, 1995, the trial court rendered its second contested Order[19] denying petitioners Motion for Reconsideration.[20] Hence, this recourse to this Court on pure questions of law.

Issues

In their Memorandum, petitioners submit the following grounds in support of their cause:[21]

I Petitioners respectfully submit that Judge Asuncion has committed grave abuse of discretion or has exceeded his jurisdiction in refusing to quash Search Warrant No. 799(95). Probable cause [has] not xxx been sufficiently established and partaking as it does of the nature of a general warrant. II Petitioners respectfully submit that Judge Asuncion has committed grave abuse of discretion or has exceeded his jurisdiction in refusing to quash Search Warrant No. 799(95) on the ground that it was unlawfully served or implemented. III Petitioners respectfully submit that State Prosecutor Dacera is acting with grave abuse of discretion or exceeding his jurisdiction in continuing with the proceedings in IS No. 95-167 on the basis of illegally seized evidence.

In the main, petitioners question the validity of the search warrant. As a preliminary matter, we shall also discuss respondents argument that the Petition should be dismissed for raising factual questions.

This Courts Ruling

The petition is meritorious.

Preliminary Issue: Alleged Factual Questions

In their Opposition, respondents argue that the Petition should be dismissed for raising questions of fact, which are not proper in a petition for certiorari under Rule 65. They maintain that the Petition merely assails the factual basis for the issuance of the warrant and the regularity of its implementation.[22] This argument is not convincing. It is settled that there is a question of fact when the doubt arises as to the truth or the falsity of alleged facts.[23] In the present case, petitioners do not question the truth of the facts as found by the judge; rather, they are assailing the way in which those findings were arrived at, a procedure which they contend was violative of the Constitution and the Rules of Court. We agree that the Petition raises only questions of law, which may be resolved in the present case.

Main Issue: Validity of the Search Warrant

The fundamental right against unreasonable searches and seizures and the basic conditions for the issuance of a search warrant are laid down in Section 2, Article III of the 1987 Constitution, which reads:

The right of the people to be secure in their persons, houses, papers and effects against unreasonable searches and seizures of whatever nature and for any purpose shall be inviolable, and no search warrant or warrant of arrest shall issue except upon probable cause to be determined personally by the judge after examination under oath or affirmation of the complainant and the witnesses he may produce, and particularly describing the place to be searched and the persons or things to be seized. (Emphasis supplied) Consistent with the foregoing constitutional provision, Sections 3 and 4, Rule 126 of the Rules of Court,[24] detail the requisites for the issuance of a valid search warrant as follows:

SEC. 3. Requisite for issuing search warrant. -- A search warrant shall not issue but upon probable cause in connection with one specific offense to be determined personally by the judge after examination under oath or affirmation of the complainant and the witnesses he may produce, and particularly describing the place to be searched and the things to be seized. SEC. 4. Examination of complainant; record. -- The judge must, before issuing the warrant, personally examine in the form of searching questions and answers, in writing and under oath the complainant and any witnesses he may produce on facts personally known to them and attach to the record their sworn statements together with any affidavits submitted. More simply stated, the requisites of a valid search warrant are: (1) probable cause is present; (2) such presence is determined personally by the judge; (3) the complainant and the witnesses he or she may produce are personally examined by the judge, in writing and under oath or affirmation; (4) the applicant and the witnesses testify on facts personally known to them; and (5) the warrant specifically describes the place to be searched and the things to be seized. [25] In the present case, the search warrant is invalid because (1) the trial court failed to examine personally the complainant and the other deponents; (2) SPO3 Cicero Bacolod, who appeared during the hearing for the issuance of the search warrant, had no personal knowledge that petitioners were not licensed to possess the subject firearms; and (3) the place to be searched was not described with particularity.

No Personal Examination of the Witnesses

In his Order dated March 23, 1995, the trial judge insisted that the search warrant was valid, stating that before issuing the subject warrant, the court propounded searching questions to the applicant and the witnesses in order to determine whether there was probable cause x x x.[26] (Emphasis supplied.) This was supported by the Opposition to the Motion to Quash, which argued that it is erroneous for PICOP to allege that the Honorable Court did not propound searching questions upon applicant P/Chief Inspector Napoleon Pascua and the witnesses he produced.[27] The records, however, proclaim otherwise. As earlier stated, Chief Inspector Pascuas application for a search warrant was supported by (1) the joint Deposition of SPO3 Cicero S. Bacolod and SPO2 Cecilio T. Morito, (2) a summary of information and (3) supplementary statements of Mario Enad and Felipe Moreno. Except for Pascua and Bacolod, however, none of the aforementioned witnesses and policemen appeared before the trial court. Moreover, the applicants participation in the hearing for the issuance of the search warrant consisted only of introducing Witness Bacolod:[28] COURT: Where is the witness for this application for search warrant? P/Chief Insp. NAPOLEON PASCUA:

SPO3 CICERO S. BACOLOD, Your Honor. COURT: Swear the witness. STENOGRAPHER: (To the witness) Please raise your right hand, sir. Do you swear to tell the truth, the whole truth and nothing but the truth before this Court? WITNESS: Yes Maam. STENOGRAPHER: Please state your name, age, civil status, occupation, address and other personal circumstances. WITNESS: SPO3 Cicero S. Bacolod, 42 years old, married, policeman, c/o Camp Crame, Quezon City, SOU, TMC.

xxxxxxxxx Chief Inspector Pascua was asked nothing else, and he said nothing more. In fact, he failed even to affirm his application. Contrary to his statement, the trial judge failed to propound questions, let alone probing questions, to the applicant and to his witnesses other than Bacolod (whose testimony, as will later be shown, is also improper). Obviously, His Honor relied mainly on their affidavits. This Court has frowned on this practice in this language:

Mere affidavits of the complainant and his witnesses are thus not sufficient. The examining Judge has to take depositions in writing of the complainant and the witnesses he may produce and attach them to the record. Such written deposition is necessary in order that the Judge may be able to properly determine the existence or non-existence of the probable cause, to hold liable for perjury the person giving it if it will be found later that his declarations are false. xxxxxxxxx It is axiomatic that the examination must be probing and exhaustive, not merely routinary or proforma, if the claimed probable cause is to be established. The examining magistrate must not simply rehash the contents of the affidavit but must make his own inquiry on the intent and justification of the application.[29]

Bacolods Testimony Pertained Not to Facts Personally Known to Him

Bacolod appeared during the hearing and was extensively examined by the judge. But his testimony showed that he did not have personal knowledge that the petitioners, in violation of

PD 1866, were not licensed to possess firearms, ammunitions or explosives. In his Deposition, he stated: Q How do you know that said properties were subject of the offense? A Sir, as a result of our intensified surveillance and case build up for several days, we gathered informations from reliable sources that subject properties [which] are in their possession and control [are] the herein described properties subject of the offense. (Summary of Information dtd Oct 94, SSs of Mario Enad and Felipe Moreno both dtd 30 Nov 94 are hereto attached).[30]

When questioned by the judge, Bacolod stated merely that he believed that the PICOP security guards had no license to possess the subject firearms. This, however, does not meet the requirement that a witness must testify on his personal knowledge, not belief. He declared: Q This is an application for Search Warrant against Paper Industries Corporation located at PICOP Compound, Barangay Tabon, Bislig, Surigao del Sur. How come that you have knowledge that there are illegal firearms in that place? A At Camp Crame, Quezon City, I was dispatched by our Commander to investigate the alleged assassination plot of Congressman Amante. Q In the course of your investigation, what happened? A We found out that some of the suspects in the alleged assassination plot are employees of PICOP. Q Know[ing] that the suspects are employees of PICOP, what did you do? A We conducted the surveillance in that area inside the compound of PICOP in Tabon. Q What did you find xxx? A I found xxx several high-powered firearms. Q How were you able to investigate the compound of PICOP? A I exerted effort to enter the said compound. Q By what means? A By pretending to have some official business with the company. Q So, in that aspect, you were able to investigate the compound of PICOP? A Yes, sir. Q What did you f[i]nd xxxt? A I found xxx several high-powered firearms being kept in the compound of PICOP. Q Where are those located? A Sir, there are firearms kept inside the ammo dam. Q Inside the compound? A Located inside the compound. Q Then what? A Others, sir, were kept in the security headquarters or office.

Q You mean to say that this Paper Industries Corporation has its own security guards? A Yes, they call it Blue Guards. Q You mean to say that their own security guards guarded the PICOP? A Yes, sir. Q So, it is possible that the firearms used by the security guards are illegally obtained? A I believe they have no license to possess high-powered firearms. As far as the verification at FEU, Camp Crame, [is concerned,] they have no license. (Emphasis supplied.) Q Have you investigated the Blue Guards Security Agency? A I conducted the inquiry. Q What did you find out? A They are using firearms owned by PICOP. Q Using firearms owned by PICOP? A Yes, sir. Q You mean to say that this Blue Guard Security Agency has no firearms of their own? A No high-powered firearms. Q By the way, Mr. Witness, what kind of firearms have you seen inside the compound of PICOP? A There are M-16 armalite rifles. Q What else? A AK-47, armalites, M-203 Grenade Launcher, M-14 US rifles, .38 caliber revolvers, .45 caliber pistols, several handgrenades and ammos.[31] (Emphasis supplied)

Moreover, Bacolod failed to affirm that none of the firearms seen inside the PICOP compound was licensed. Bacolod merely declared that the security agency and its guards were not licensed. He also said that some of the firearms were owned by PICOP. Yet, he made no statement before the trial court that PICOP, aside from the security agency, had no license to possess those firearms. Worse, the applicant and his witnesses inexplicably failed to attach to the application a copy of the aforementioned no license certification from the Firearms and Explosives Office (FEO) of the PNP, or to present it during the hearing. Such certification could have been easily obtained, considering that the FEO was located in Camp Crame where the unit of Bacolod was also based. In People v. Judge Estrada,[32] the Court held:

The facts and circumstances that would show probable cause must be the best evidence that could be obtained under the circumstances. The introduction of such evidence is necessary in cases where the issue is the existence of the negative ingredient of the offense charged for instance, the absence of a license required by law, as in the present case and such evidence is within the knowledge and control of the applicant who could easily produce the same. But if the best evidence could not be secured at the time of the application, the applicant must show a justifiable reason therefor during the examination by the judge.

Particularity of the Place to Be Searched

In view of the manifest objective of the constitutional safeguard against unreasonable search, the Constitution and the Rules limit the place to be searched only to those described in the warrant.[33] Thus, this Court has held that this constitutional right [i]s the embodiment of a spiritual concept: the belief that to value the privacy of home and person and to afford its constitutional protection against the long reach of government is no less than to value human dignity, and that his privacy must not be disturbed except in case of overriding social need, and then only under stringent procedural safeguards.[34] Additionally, the requisite of particularity is related to the probable cause requirement in that, at least under some circumstances, the lack of a more specific description will make it apparent that there has not been a sufficient showing to the magistrate that the described items are to be found in a particular place.[35] In the present case, the assailed search warrant failed to describe the place with particularity. It simply authorizes a search of the aforementioned premises, but it did not specify such premises. The warrant identifies only one place, and that is the Paper Industries Corporation of the Philippines, located at PICOP Compound, Barangay Tabon, Bislig[,] Surigao del Sur. The PICOP compound, however, is made up of 200 offices/buildings, 15 plants, 84 staff houses, 1 airstrip, 3 piers/wharves, 23 warehouses, 6 POL depots/quick service outlets and some 800 miscellaneous structures, all of which are spread out over some one hundred fifty-five hectares.[36] Obviously, the warrant gives the police officers unbridled and thus illegal authority to search all the structures found inside the PICOP compound.[37] In their Opposition, the police state that they complied with the constitutional requirement, because they submitted sketches of the premises to be searched when they applied for the warrant. They add that not one of the PICOP Compound housing units was searched, because they were not among those identified during the hearing.[38] These arguments are not convincing. The sketches allegedly submitted by the police were not made integral parts of the search warrant issued by Judge Asuncion. Moreover, the fact that the raiding police team knew which of the buildings or structures in the PICOP Compound housed firearms and ammunitions did not justify the lack of particulars of the place to be searched.[39] Otherwise, confusion would arise regarding the subject of the warrant the place indicated in the warrant or the place identified by the police. Such conflict invites uncalled for mischief or abuse of discretion on the part of law enforcers. Thus, in People v. Court of Appeals,[40] this Court ruled that the police had no authority to search the apartment behind the store, which was the place indicated in the warrant, even if they really intended it to be the subject of their application. Indeed, the place to be searched cannot be changed, enlarged or amplified by the police, viz.:

x x x. In the instant case, there is no ambiguity at all in the warrant. The ambiguity lies outside the instrument, arising from the absence of a meeting of the minds as to the place to be searched between the applicants for the warrant and the Judge issuing the same; and what was done was to substitute for the place that the Judge had written down in the warrant, the premises that the executing officers had in their mind. This should not have been done. It [was] neither fair nor licit to allow police officers to

search a place different from that stated in the warrant on the claim that the place actually searched although not that specified in the warrant [was] exactly what they had in view when they applied for the warrant and had demarcated in their supporting evidence. What is material in determining the validity of a search is the place stated in the warrant itself, not what the applicants had in their thoughts, or had represented in the proofs they submitted to the court issuing the warrant. Indeed, following the officers theory, in the context of the facts of this case, all four (4) apartment units at the rear of Abigail's Variety Store would have been fair game for a search. The place to be searched, as set out in the warrant, cannot be amplified or modified by the officers own personal knowledge of the premises, or the evidence they adduced in support of their application for the warrant. Such a change is proscribed by the Constitution which requires inter alia the search warrant to particularly describe the place to be searched as well as the persons or things to be seized. It would concede to police officers the power of choosing the place to be searched, even if it not be that delineated in the warrant. It would open wide the door to abuse of the search process, and grant to officers executing a search warrant that discretion which the Constitution has precisely removed from them. The particularization of the description of the place to be searched may properly be done only by the Judge, and only in the warrant itself; it cannot be left to the discretion of the police officers conducting the search. (Emphasis supplied.) Seized Firearms and Explosives Inadmissible in Evidence

As a result of the seizure of the firearms, effected pursuant to Search Warrant No. 799 (95) issued by the respondent judge, the PNP filed with the Department of Justice a complaint docketed as IS No. 95-167 against herein petitioners for illegal possession of firearms. State Prosecutor Dacera, to whom the Complaint was assigned for preliminary investigation, issued a subpoena requiring petitioners to file their counter-affidavits. Instead of complying with the subpoena, petitioners asked for the suspension of the preliminary investigation, pending the resolution of their motion to quash the search warrant. They argued, as they do now, that the illegally obtained firearms could not be the basis of the criminal Complaint. Their motion was denied. A subsequent Motion for Reconsideration met the same fate. In the present Petition for Certiorari and Prohibition, petitioners assert that State Prosecutor Dacera cannot have any tenable basis for continuing with the proceedings in IS No. 95-167.[41] Because the search warrant was procured in violation of the Constitution and the Rules of Court, all the firearms, explosives and other materials seized were inadmissible for any purpose in any proceeding.[42] As the Court noted in an earlier case, the exclusion of unlawfully seized evidence was the only practical means of enforcing the constitutional injunction against

unreasonable searches and seizures.[43] Verily, they are the fruits of the poisonous tree. Without this exclusionary rule, the constitutional right would be so ephemeral and so neatly severed from its conceptual nexus with the freedom from all brutish means of coercing evidence xxx.[44] In the present case, the complaint for illegal possession of firearms is based on the firearms and other materials seized pursuant to Search Warrant No. 799 (95). Since these illegally obtained pieces of evidence are inadmissible, the Complaint and the proceedings before State Prosecutor Dacera have no more leg to stand on. This Court sympathizes with the police effort to stamp out criminality and to maintain peace and order in the country; however, it reminds the law enforcement authorities that they must do so only upon strict observance of the constitutional and statutory rights of our people. Indeed, there is a right way to do the right thing at the right time for the right reason.[45] WHEREFORE, the instant petition for certiorari and prohibition is hereby GRANTED and Search Warrant No. 799 (95) accordingly declared NULL and VOID. The temporary restraining order issued by this Court on October 23, 1995 is hereby MADE PERMANENT. No pronouncement as to costs.

[G.R. No. 126859. September 4, 2001]

YOUSEF AL-GHOUL, ISAM MOHAMMAD ABDULHADI, WAIL RASHID AL-KHATIB, NABEEL NASSER AL-RIYAMI, ASHRAF HASSAM AL-YAZORI, AND MOHAMMAD ABUSHENDI, petitioners, vs. COURT OF APPEALS AND THE PEOPLE OF THE PHILIPPINES, respondents. DECISION QUISUMBING, J.:

Petitioners assail the decision[1] dated September 30, 1996, of the Court of Appeals, which affirmed the orders of the Regional Trial Court of Kalookan City, Branch 123, thereby dismissing petitioners special civil action for certiorari.[2] The facts leading to the present petition under Rule 65 are as follows: On March 31, 1995, Judge Geronimo S. Mangay, presiding judge of the Regional Trial Court, National Capital Judicial Region, Branch 125, Kalookan City, issued search warrants 5495[3] and 55-95[4] for the search and seizure of certain items in Apartment No. 2 at 154 Obiniana Compound, Deparo Road, Kalookan City. On April 1, 1995, the police searched Apartment No. 8, in the same compound and found one (1) .45 caliber pistol. Found in Apartment No. 2 were:

2 M-16 rifles with 2 magazines and 20 live M-16 ammunitions

1 Bar of demolition charge 1 Caliber Pistol with no. 634 and other nos. were placed with magazine of Caliber .45 and 3 live 45 ammunitions 1 22 Caliber handgun with 5 live ammunitions in its cylinder 1 Box containing 40 pieces of .25 caliber ammunitions 2 pieces of fragmentation grenade 1 roll of detonating cord color yellow 2 big bags of ammonium nitrate suspected to be explosives substance 22 detonating cords with blasting caps and pound of high explosives TNT 1 timer alarm clock 2 bags of suspected gun powder 2 small plastic bag of suspected explosive substance 1 small box of plastic bag of suspected dynamites One weighing scale Two (2) batteries 9 volts with blasting caps and detonating cord.

[5]

The firearms, ammunitions, explosives and other incendiary devices seized at the apartments were acknowledged in the receipt signed by SPO2 Melanio de la Cruz. Petitioners were charged before the Regional Trial Court of Kalookan City, Branch 123, in informations docketed as Criminal Cases Nos. C-48666-67, accusing them with illegal possession of firearms, ammunitions and explosives, pursuant to Presidential Decree No. 1866.[6] Thereafter, petitioners were arrested and detained. Petitioners filed a motion for bail on May 24, 1995, the resolution of which was held in abeyance by the RTC pending the presentation of evidence from the prosecution to determine whether or not the evidence presented is strong.[7] On February 7, 1996, at the hearing for bail, the RTC admitted all exhibits being offered for whatever purpose that they maybe worth after the prosecution had finished adducing its evidence despite the objection by the petitioners on the admissibility of said evidence.

On February 19, 1996, the RTC denied petitioners motion for bail earlier filed, giving as reasons the following:

To begin with, the accused are being charged of two criminal offenses and both offenses under Presidential Decree 1866, Sections 1 and 3 thereof prescribe the penalty of Reclusion Temporal in its maximum period to Reclusion Perpetua. Under Rule 114 of the Rules on Criminal Procedure as amended by Supreme Court Administrative Circular No. 12-94, particularly Section 7 thereof, no person charged with a capital offense or an offense punishable by reclusion perpetua or life imprisonment, when evidence of guilt is strong shall be admitted to bail regardless of the stage of the criminal prosecution. [8]

As petitioners action before respondent appellate court also proved futile, petitioners filed the instant petition on the ground that it had acted with grave abuse of discretion tantamount to lack or in excess of jurisdiction. They present for our consideration the following issues: I. WHETHER OR NOT THE EVIDENCE OFFERED BY THE PROSECUTION ARE ADMISSIBLE; II. WHETHER OR NOT ACCUSED HAVE THE RIGHT TO BAIL.[9]

The issue on bail has been resolved in our resolution dated November 24, 1998, where this Court ruled:

Consequent to the enactment of RA 8294, the penalty prescribed in Section 1 and 3 of P.D. 1866 for illegal possession of firearms, ammunitions and explosives under which petitioners were charged, has now been reduced to prision mayor in its minimum period and prision mayor in its maximum period to reclusion temporal, respectively. Evidently, petitioners are now entitled to bail as a matter of right prior to their conviction by the trial court pursuant to Section 4 of SC Administrative Circular No. 12-94 [10]

xxx

WHEREFORE, the petitioners motion is hereby GRANTED. The Temporary Restraining Order issued by this Court in the Resolution of November 20, 1996 is hereby PARTIALLY LIFTED in so far as petitioners pending motion for bail before the RTC of Kalookan City, Branch 123 is concerned. The trial court is hereby ordered to proceed with the hearing of the motion for bail and resolve the same with dispatch. [11]

The issue that remains is whether the respondent court erred and gravely abused its discretion when it ruled that the search and seizure orders in question are valid and the objects seized admissible in evidence.

Petitioners contend that the search and seizure orders violated Sections 2 and 3 of the Bill of Rights[12] as well as Section 3 of Rule 126 of the Rules of Court on Criminal Procedure[13] because the place searched and articles seized were not described with particularity. They argue that the two-witness requirement under Section 10 of Rule 126[14] was ignored when only one witness signed the receipt for the properties seized during the search, and said witness was not presented at the trial. Petitioners also aver that the presumption of regularity of the implementation of the search warrant was rebutted by the defense during cross-examination of prosecution witnesses. According to petitioners, respondent court failed to appreciate the fact that the items seized were not turned over to the police evidence custodian as required under Section 18 of the Department of Justice Circular No. 61 dated September 21, 1993.Finally, they fault the lower courts finding that petitioners were in possession of the items allegedly confiscated from them.[15] For the State, the Office of the Solicitor General avers that the search of Apartment 2 was legal, and the items seized therein are admissible in evidence. However, the OSG agrees with petitioners that the search warrants issued by the RTC, Branch 125, Kalookan City on March 31, 1995, namely search warrant 54-95[16] and search warrant 55-95,[17] specified the place to be searched, namely Apartment No. 2, 154 Obiniana Compound, Deparo Road, Kalookan City. There was no mention of Apartment No. 8. Thus, we find that the search conducted at Apartment No. 8 clearly violated Sections 2 and 3 (2) of the Bill of Rights, in relation to Section 3 of Rule 126 of the Rules of Court. As held in PICOP v. Asuncion,[18] the place to be searched cannot be changed, enlarged nor amplified by the police. Policemen may not be restrained from pursuing their task with vigor, but in doing so, care must be taken that constitutional and legal safeguards are not disregarded. Exclusion of unlawfully seized evidence is the only practical means of enforcing the constitutional injunction against unreasonable searches and seizures. Hence, we are constrained to declare that the search made at Apartment No. 8 is illegal and the .45 caliber pistol taken thereat is inadmissible in evidence against petitioners. Now, in contrast, the search conducted at Apartment No. 2 could not be similarly faulted. The search warrants in question specifically mentioned Apartment No. 2. The search was done in the presence of its occupants, herein petitioners,[19] in accordance with Section 7 of Rule 126, Revised Rules of Court.[20] Petitioners allege lack of particularity in the description of objects to be seized pursuant to the warrants. Hence, they also question the seizure of the following articles from Apartment No. 2, namely:

One M16 rifles, Colt AR-15 with 2 magazines and 20 rds ammo live One (1) bar demolition charge One (1) .45 caliber pistol numbers were defaced with magazine and with three (3) live .45 cal ammos One (1) .22 caliber handgun with live ammos in its cylinder One (1) box containing (40) forty pieces of .22 cal. live ammos (magnum)

Two (2) pieces fragmentation grenade Two (2) magazines of M16 rifles with live ammos.

[21]

To appreciate them fully, we quote the search warrants in question: Search Warrant 54-95

It appearing to the satisfaction of the undersigned, after examining under oath P/Sr Insp Joel D. Pagdilao, Chief, DPIU, OADDI NPDC, Applicant and his witness SPO1 Cesar R. Rivera of District Police Intelligence Unit, Northern Police District Command with postal address c/o NPIU, NPDC, PNP, Langaray St., Kaunlaran Village, Kalookan City that a.k.a. AL, a.k.a. GHUL a.k.a. NADI, a.k.a. NABIL and several John Does of Apartment Nr. 2, Nr. 154 Obiniana Compound, Deparo Road, Kalookan City have in their possession and control the following: 1. One (1) 45 Caliber Pistol You are hereby commanded to make an immediate search anytime of the DAY and NIGHT of the premises above-mentioned and forthwith, seize and take possession of the foregoing property, to wit: 1. One (1) .45 Caliber Pistol and bring to this Court to be dealt with as the law may direct.

[22]

Search Warrant 55-95

It appearing to the satisfaction of the undersigned after examining under oath P/Sr. Insp. Joel D. Pagdilao, Chief, DPIU, OADDI NPDC, Applicant and his witness SPO1 Cesar R. Rivera of District Police Intelligence Unit, Northern Police District Command with postal address c/o NPIU, NPDC, PNP, Langaray St., Kaunlaran Village, Kalookan City that a.k.a. AL, a.k.a. GHUL a.k.a. NADI, a.k.a. NABIL and several John Does of Apartment Nr. 2, Nr. 154 Obiniana Compound, Deparo Road, Kalookan City have in their possession and control the following: 1. One (1) 5.56 M16 Rifle with corresponding ammunitions 2. One (1) 9MM Pistol with corresponding ammunitions 3. Three (3) boxes of explosives 4. More or less ten (10) sticks of dymanites (sic)

5. More or less thirty (30) pieces of blasting caps pieces of detonating cords

You are hereby commanded to make an immediate search anytime of the DAY or NIGHT of the premises above-mentioned and forthwith seize and take possession of the foregoing properties, to wit: 1. One (1) 5.56 M16 Rifle with corresponding ammunitions 2. One (1) 9MM Pistol with corresponding ammunitions 3. Three (3) boxes of explosives 4. More or less ten (10) sticks of dymanites (sic) 5. More or less thirty (30) pieces of blasting caps pieces of detonating cords

and bring to this Court to be dealt with as the law may direct.

[23]

That the articles seized during the search of Apartment No. 2 are of the same kind and nature as those items enumerated in the search warrant above-quoted appears to us beyond cavil. The items seized from Apartment No. 2 were described with specificity in the warrants in question. The nature of the items ordered to be seized did not require, in our view, a technical description. Moreover, the law does not require that the things to be seized must be described in precise and minute details as to leave no room for doubt on the part of the searching authorities, otherwise, it would be virtually impossible for the applicants to obtain a search warrant as they would not know exactly what kind of things they are looking for.[24] Once described, however, the articles subject of the search and seizure need not be so invariant as to require absolute concordance, in our view, between those seized and those described in the warrant. Substantial similarity of those articles described as a class or species would suffice. In People v. Rubio, 57 Phil. 384, 389 (1932), this Court said, While it is true that the property to be seized under a warrant must be particularly described therein and no other property can be taken thereunder, yet the description is required to be specific only in so far as the circumstances will ordinarily allow. Where by the nature of the goods to be seized, their description must be rather general, it is not required that a technical description be given, as this would mean that no warrant could issue. As a corollary, however, we could not logically conclude that where the description of those goods to be seized have been expressed technically, all others of a similar nature but not bearing the exact technical descriptions could not be lawfully subject to seizure. Otherwise, the reasonable purpose of the warrant issued would be defeated by mere technicalities. The case of Bache and Co. (Phil.), Inc. v. Ruiz, 37 SCRA 823, 835 (1971), pointed out that one of the tests to determine the particularity in the description of objects to be seized under a search warrant is when the things described are limited to those which bear direct relation to the offense for which the warrant is being issued. A careful examination of Search Warrant Nos. 5495[25] and 55-95[26] shows that they were worded in such a manner that the enumerated items to be seized could bear a direct relation to the offense of violation of Section 1[27] and 3[28] of Presidential Decree No. 1866, as amended, penalizing illegal possession of firearms,

ammunitions and explosives. What the warrants authorized was the seizure of articles proscribed by that decree, and no other. Lastly, on this score, we note that the Court of Appeals ruled that petitioners waived their right to raise any attack on the validity of the search warrants at issue by their failure to file a motion to quash.[29] But, in conducting the search at Apartment No. 8, not just Apartment No. 2 as ordered specifically in the search warrants, the police committed a gross violation we cannot condone. Thus, we conclude that the gun seized in Apartment No. 8 cannot be used in evidence, but those articles including guns, ammunitions, and explosives seized in Apartment No. 2 are admissible in evidence. Coming now to the two-witness requirement under Section 10, Rule 126 of the Revised Rules of Court, petitioners claim the rule was violated because only one witness signed the receipt for the properties seized. For clarity, let us reproduce the pertinent section:

SEC. 10. Receipt for the property seized.The officer seizing property under the warrant must give a detailed receipt for the same to the lawful occupant of the premises in whose presence the search and seizure were made, or in the absence of such occupant, must, in the presence of at least two witnesses of sufficient age and discretion residing in the same locality, leave a receipt in the place in which he found the seized property. Clearly, the two-witness rule applies only in the absence of the lawful occupants of the premises searched. In the case at bar, petitioners were present when the search and seizure operation was conducted by the police at Apartment No. 2. More importantly, petitioner Nabeel Al-Riyami y Nasser admitted being an actual occupant/resident of Apartment No. 2.[30] Hence, we find here no violation of Section 10, Rule 126 of the Revised Rules of Court. Petitioners contend that they could not be charged with violation of P.D. 1866 because the seized items were not taken actually from their possession. This contention, however, cannot prosper in the light of the settled rule that actual possession of firearms and ammunitions is not an indispensable element for prosecution under P.D. No. 1866. In People v. Dela Rosa, 284 SCRA 158, 168-169 (1998), we clarified that the kind of possession punishable under P.D. 1866 is one where the accused possessed a firearm either physically or constructively with animus possidendi or intent to possess said firearm. Whether or not the evidence would show all the elements of P.D. 1866 in this case is a different matter altogether. We shall not preempt issues properly still within the cognizance of courts below. Likewise, whether or not the articles seized were planted by the police, as claimed by the petitioners, is a matter that must be brought before the trial court. In the same vein, petitioners claim that the properties seized were not turned over to the proper police custodian is a question of fact best ventilated during trial. WHEREFORE, the petition is PARTIALLY GRANTED. The search conducted at Apartment No. 8 is hereby declared illegal and the item (.45 caliber pistol) seized therein inadmissible in evidence. However, the search at Apartment No. 2 pursuant to Search Warrant 55-95 is hereby declared valid and legal, and the articles seized from Apartment No. 2 are found

admissible in evidence. Let this case be remanded to the Regional Trial Court of Kalookan City, Branch 123, for trial on the merits of Criminal Cases Nos. C-48666-67 with dispatch.

[G.R. No. 142295. May 31, 2001]

VICENTE DEL ROSARIO y NICOLAS, petitioner, vs. PEOPLE OF THE PHILIPPINES, respondent. DECISION PARDO, J.:

Petitioner Vicente del Rosario y Nicolas appeals via certiorari from a decision of the Court of Appeals[1] affirming with modification the decision of the Regional Trial Court, Bulacan, Branch 20, Malolos, and finding him guilty beyond reasonable doubt of violation of P. D. No. 1866, as amended by Republic Act No. 8294 (illegal possession of firearms), sentencing him to four (4) years, nine (9) months and eleven (11) days of prision correccional, as minimum, to six (6) years, eight (8) months and one (1) day of prision mayor, as maximum, and to pay a fine of P30,000.00. On June 17, 1996, Assistant Provincial Prosecutor Eufracio S. Marquez of Bulacan filed with the Regional Trial Court, Bulacan, Malolos an Information charging petitioner Vicente del Rosario y Nicolas with violation of P. D. No. 1866, as follows: That on or about the 15th day of June 1996, in the municipality of Norzagaray, Province of Bulacan, Philippines, and within the jurisdiction of this Honorable Court, the above-named accused, did then and there wilfully, unlawfully and feloniously have in his possession under his custody and control, the following, to wit: a) One (1) pc. Pistol Cal. 45 SN:70G23792 (w/o license) b) One (1) pc. Revolver Cal. 22 SN:48673 (w/o license) c) Twenty Seven (27) rds live ammos. For cal. .45 d) Five (5) pcs. Magazines for cal. .45 e) Eight (8) rds live ammunitions for cal. 22 f) Five (5) pcs. Magazines short for cal. 5.56 (M16)

g) Twenty (20) rds live ammunitions for cal 5.56 without first having obtained a proper license therefor. Contrary to law.[2] On June 25, 1996, the trial court arraigned the petitioner. He pleaded not guilty.[3] Trial ensued. The facts, as found by the Court of Appeals, are as follows: Sometime in May 1996, the police received a report that accused-appellant Vicente del Rosario was in possession of certain firearms without the necessary licenses. Acting upon the report, P/Sr. Insp. Jerito Adique of the PNP Criminal Investigation Group at Camp Olivas, Pampanga inquired from the PNP Firearms and Explosive Division whether or not the report was true. On May 10, 1996, P/Sr. Insp. Edwin C. Roque of the PNP Firearms and Explosives Division issued a certification (Exhibit L) stating that per records in his office, the appellant is not a licensed/registered firearm holder of any kind and caliber. Armed with the said certification, P/Sr. Insp. Adique applied for a search warrant to enable his team to search the house of appellant. On June 13, 1996, a search warrant (Exhibit A) was issued by Judge Gil Fernandez, Sr. of the Regional Trial Court of Quezon City, Branch 217, authorizing the search of the residence of appellant at Barangay Tigbe, Norzagaray, Bulacan.[4] On June 15, 1996, at about 7:00 oclock in the morning, a team led by P/Sr. Insp. Adique went to Norzagaray to serve the warrant. Before proceeding to the residence of the appellant, the police officers requested Barangay Chairman Rogelio de Silva and Barangay Councilman Aurelio Panteleon to accompany them in the implementation of the warrant. Upon arrival at the house of appellant, the police officers introduced themselves to the wife of appellant. When the appellant came out, P/Sr. Insp. Adique informed him that they had a search warrant and that they were authorized to search his house. After appellant gave his permission, the police officers conducted a search of the house. The search yielded the following items: (a) a caliber .45 pistol with Serial No. 703792 with five magazines of caliber .45 (Exhibits B and H) found at the masters bedroom; (b) five magazines of 5.56 M-16 rifle and two radios (Exhibits C to C-4) found in the room of appellants daughter; and (c) a caliber .22 revolver with Serial No. 48673 (Exhibit F) containing 8 pieces of live ammunition (Exhibit M) found in the kitchen of the house. When asked about his license to possess the firearms, the appellant failed to produce any. This prompted the police officers to seize the subject firearms.

SPO2 Marion Montezon, one of the searching officers, prepared three separate inventories of the seized items (Exhibits H, M and N). The inventories were signed by P/Sr. Insp. Adique, the appellant and the barangay officials who witnessed the search. Thereafter SPO2 Montezon prepared a certification of orderly search (Exhibit I) which was signed by the appellant and the barangay officials attesting to the orderly conduct of the search. For his defense, appellant contends that he had a license for the caliber .45 pistol recovered in his bedroom and that the other items seized during the search including the caliber .22 revolver, were merely planted by the police officers.Appellant likewise assails the manner in which the search was carried out, claiming that the police officers just barged into his house without asking permission. Furthermore, he claimed that the barangay officials arrived only after the police already had finished the search. After trial and on July 2, 1998, the trial court rendered a judgment of conviction, the dispositive portion of which reads: WHEREFORE, premises considered, the Court finds the accused VICENTE DEL ROSARIO y NICOLAS guilty beyond reasonable doubt of violation of P. D. No. 1866 as charged under the Information dated June 17, 1996. Conformably with the provisions of said law, as amended by Republic Act No. 8294, and pursuant to the provisions of the Indeterminate Sentence Law, the Court hereby sentences the accused to suffer imprisonment of six (6) months of arresto mayor, as minimum, to six (6) years of prision correctional, as maximum, and to pay a fine of Fifteen Thousand Pesos (P15,000.00).[5] On July 20, 1998, petitioner appealed to the Court of Appeals, assailing the decision for being contrary to facts and the law.[6] On July 9, 1999, the Court of Appeals promulgated its decision affirming with modification the decision of the trial court as set out in the opening paragraph of this decision.[7] On August 10, 1999, petitioner filed with the Court of Appeals a motion for reconsideration and/or new trial.[8] He contended that the certification issued by the Chief, Firearms and Explosives Division, Philippine National Police stating that the person named therein had not been issued a firearm license referred to a certain Vicente Vic del Rosario of barangay Bigte, Norzagaray, Bulacan, not to him. He comes from barangay Tigbe, Norzagaray, Bulacan, and that he has a valid firearm license.

On February 22, 2000, the Court of Appeals denied the motion for reconsideration for lack of merit.[9] Hence, this appeal.[10] Petitioner submits that the search conducted at his residence was illegal as the search warrant was issued in violation of the Constitution [11] and consequently, the evidence seized was inadmissible. He also submits that he had a license for the .45 caliber firearm and ammunition seized in his bedroom. The other firearm, a .22 caliber revolver seized in a drawer at the kitchen of his house, a magazine for 5.56 mm. cal. Armalite rifle, and two 2-way radios found in his daughters bedroom, were either planted by the police or illegally seized, as they were not mentioned in the search warrant. We find the petition impressed with merit. We define the issues as follows: First: whether petitioner had a license for the .45 caliber Colt pistol and ammunition seized in his bedroom; and Second: whether the .22 caliber revolver seized in a drawer at the kitchen of his house, a magazine for 5.56 mm. cal. Armalite rifle and two 2-way radios found in his daughters bedroom, were planted by the police or were illegally seized. We shall resolve the issues in seriatim. First: The .45 cal. Colt pistol in question was duly licensed. Normally, we do not review the factual findings of the Court of Appeals and the trial courts.[12] However, this case comes within the exceptions.[13] The findings of fact by the Court of Appeals will not be disturbed by the Court unless these findings are not supported by evidence.[14] In this case, the findings of the lower courts even directly contradict the evidence. Hence, we review the evidence. The trial court held that the copy of the license presented was blurred, and that in any event, the court could rely on the certification dated May 10, 1996, of P/Sr. Inspector Edwin C. Roque, Chief, Records Branch, Firearms and Explosives Division, Philippine National Police stating that Vicente Vic del Rosario of Barangay Bigte, Norzagaray, Bulacan is not a licensed/registered firearm holder of any kind and caliber. [15] As against this, petitioner submitted that he was not the person referred to in the said certification because he is Vicente del Rosario y Nicolas from Barangay Tigbe, Norzagaray, Bulacan. The Court takes judicial notice of the existence of both barangay Tigbe and barangay Bigte, in Norzagaray, Bulacan.[16] In fact, the trial court erred grievously in not taking judicial notice of the barangays within its territorial jurisdiction, believing the prosecutions submission that there was only barangay Tigbe, and that barangay Bigte in the certification was a typographical error.[17] Petitioner presented to the head

of the raiding team, Police Senior Inspector Jerito A. Adique, Chief, Operations Branch, PNP Criminal Investigation Command, a valid firearm license. The court is duty bound to examine the evidence assiduously to determine the guilt or innocence of the accused. It is true that the court may rely on the certification of the Chief, Firearms and Explosives Division, PNP on the absence of a firearm license.[18] However, such certification referred to another individual and thus, cannot prevail over a valid firearm license duly issued to petitioner. In this case, petitioner presented the printed computerized copy of License No. RCL 1614021915 issued to him on July 13, 1993, expiring in January 1995, by the Chief, Firearms and Explosives Division, PNP under the signature of Reynaldo V. Velasco, Sr. Supt. (GSC) PNP, Chief, FEO.[19] On the dorsal side of the printed computerized license, there is stamped the words Validity of computerized license is extended until renewed license is printed dated January 17, 1995, signed by Police Chief Inspector Franklin S. Alfabeto, Chief, Licence Branch, FEO.[20] Coupled with this indefinite extension, petitioner paid the license fees for the extension of the license for the next two-year period.[21] Consequently, we find that petitioner was the holder of a valid firearm license for the .45 caliber Colt pistol seized in the bedroom of his house on June 15, 1996.[22] As required, petitioner presented the license to the head of the raiding team, Police Senior Inspector Jerito A. Adique of the Criminal Investigation Division Group, PNP.[23] As a senior police officer, Senior Inspector Adique could easily determine the genuineness and authenticity of the computerized printed license presented. He must know the computerized license printed form. The stamp is clearly visible. He could decipher the words and the signature of the authorized signing official of the Firearms and Explosives Division, PNP. He belonged to the same national police organization. Nevertheless, Senior Insp. Adique rejected the license presented because, according to him, it was expired. However, assuming that the license presented was expired during the period January 1995 to January 1997, still, possession of the firearm in question, a .45 caliber Colt pistol with serial No. 70G23792, during that period was not illegal. The firearm was kept at home, not carried outside residence. On June 15, 1996, at the time of the seizure of the firearm in question, possession of firearm with an expired license was not considered unlawful, provided that the license had not been cancelled or revoked. Republic Act No. 8294, providing that possession of a firearm with an expired license was unlawful took effect only on July 7, 1997.[24] It could not be given retroactive effect.[25] According to firearm licensing regulations, the renewal of a firearm license was automatically applied for upon payment of the license fees for the renewal period. The expired license was not cancelled or revoked. It served as temporary authority to possess the firearm until the renewed license was issued. Meantime, the applicant may keep the gun at home pending renewal of the firearm license and issuance of a printed

computerized license. He was not obliged to surrender the weapon. Printed at the dorsal side of the computerized license is a notice reading: IMPORTANT 1. This firearm license is valid for two (2) years. Exhibit this license whenever demanded by proper authority. 2. Surrender your firearm/s to the nearest PNP Unit upon revocation or termination of this license. Under any of the following instances, your license shall be revoked for which reason your firearm/s is/are subject to confiscation and its/their forfeiture in favor of the government.

a. Failure to notify the Chief of PNP in writing of your change of address, and/or qualification. b. Failure to renew this license by paying annual license, fees, within six (6) months from your birth month. Renewal of your license can be made within your birth month or month preceding your birth month. Late renewal shall be penalized with 50% surcharge for the first month (from the first day to the last day of this month) followed by an additional 25% surcharge for all of the succeeding five (5) months compounded monthly. c. Loss of firearm/s through negligence. d. Carrying of firearm/s outside of residence without appropriate permit and/or carrying firearm/s in prohibited places. e. Conviction by competent court for a crime involving moral turpitude or for any offense where the penalty carries an imprisonment of more than six (6) months or fine of at least P1,000.00. f. Dismissal for cause from the service. g. Failure to sign license, or sign ID picture or affix right thumbmark. 3. Unauthorized loan of firearm/s to another person is punishable by permanent disqualification and forfeiture of the firearm in favor of the government. 4. If termination is due to death, your next of kin should surrender your firearm/s to the nearest PNP Unit. For those within Metro Manila, surrender should be made with FEO, Camp Crame. 5. When firearms become permanently unserviceable, they should be deposited with the nearest PNP Unit and ownership should be relinquished in writing so that firearms may be disposed of in accordance with law.

6. Application for the purchase of ammunition should be made in case of a resident of Metro Manila direct to the Chief, FEO and for residents of a Province to secure recommendation letter to the nearest PNP Provincial Command who will thereafter endorse same to CHIEF, FEO for issuance of the permit. License must be presented before an authority to purchase ammo could be obtained.[26]

Indeed, as heretofore stated, petitioner duly paid the license fees for the automatic renewal of the firearm license for the next two years upon expiration of the license in January 1995, as evidenced by official receipt No. 7615186, dated January 17, 1995.[27] The license would be renewed, as it was, because petitioner still possessed the required qualifications. Meantime, the validity of the license was extended until the renewed computerized license was printed. In fact, a renewed license was issued on January 17, 1997, for the succeeding two-year period.[28] Aside from the clearly valid and subsisting license issued to petitioner, on January 25, 1995, the Chief, Philippine National Police issued to him a permit to carry firearm outside residence valid until January 25, 1996, for the firearm in question. [29] The Chief, Philippine National Police would not issue a permit to carry firearm outside residence unless petitioner had a valid and subsisting firearm license. Although the permit to carry firearm outside residence was valid for only one year, and expired on January 25, 1996, such permit is proof that the regular firearm license was renewed and subsisting within the two-year term up to January 1997. A Permit to Carry Firearm Outside Residence presupposes that the party to whom it is issued is duly licensed to possess the firearm in question.[30] Unquestionably, on January 17, 1997, the Chief, Firearms and Explosives Division, PNP renewed petitioners license for the .45 cal. Colt pistol in question.[31] Clearly then, petitioner had a valid firearm license during the interregnum between January 17, 1995, to the issuance of his renewed license on January 17, 1997. Finally, there is no rhyme or reason why the Court of Appeals and the trial court did not accept with alacrity the certification dated June 25, 1996, of P/Sr. Inspector Edwin C. Roque,[32] Chief, Records Branch, Firearms and Explosives Division, PNP that Vicente N. del Rosario of Barangay Tigbe, Norzagaray, Bulacan is a licensed/registered holder of Pistol, Colt caliber .45 with serial number 70G23792, covered by computerized license issued dated June 15, 1995, with an expiry date January 1997.[33] Reinforcing the aforementioned certification, petitioner submitted another certification dated August 27, 1999, stating that Vicente N. del Rosario of Barangay Tigbe, Norzagaray, Bulacan, was issued firearm license No. RLC1614021915, for caliber .45 Pistol with Serial Number 70G23792, for the years covering the period from July 13, 1993 to January 1995, and the extension appearing at the back thereof for the years 1995 to 1997. [34] Had the lower courts given full probative value to these official issuances, petitioner would have been correctly acquitted, thus sparing this Court of valuable time and effort.

In crimes involving illegal possession of firearm, the prosecution has the burden of proving the elements thereof, viz.: (a) the existence of the subject firearm and (b) the fact that the accused who owned or possessed it does not have the license or permit to possess the same.[35] The essence of the crime of illegal possession is the possession, whether actual or constructive, of the subject firearm, without which there can be no conviction for illegal possession. After possession is established by the prosecution, it would only be a matter of course to determine whether the accused has a license to possess the firearm.[36] Possession of any firearm becomes unlawful only if the necessary permit or license therefor is not first obtained. The absence of license and legal authority constitutes an essential ingredient of the offense of illegal possession of firearm and every ingredient or essential element of an offense must be shown by the prosecution by proof beyond reasonable doubt. Stated otherwise, the negative fact of lack or absence of license constitutes an essential ingredient of the offense which the prosecution has the duty not only to allege but also to prove beyond reasonable doubt.[37] To convict an accused for illegal possession of firearms and explosives under P. D. 1866, as amended, two (2) essential elements must be indubitably established, viz.: (a)the existence of the subject firearm or explosive which may be proved by the presentation of the subject firearm or explosive or by the testimony of witnesses who saw accused in possession of the same, and (b) the negative fact that the accused had no license or permit to own or possess the firearm or explosive which fact may be established by the testimony or certification of a representative of the PNP Firearms and Explosives Unit that the accused has no license or permit to possess the subject firearm or explosive. x x x We stress that the essence of the crime penalized under P. D. 1866 is primarily the accuseds lack of license or permit to carry or possess the firearm, ammunition or explosive as possession by itself is not prohibited by law.[38] Illegal possession of firearm is a crime punished by special law, a malum prohibitum, and no malice or intent to commit a crime need be proved.[39] To support a conviction, however, there must be possession coupled with intent to possess (animus possidendi) the firearm.[40] In upholding the prosecution and giving credence to the testimony of police officer Jerito A. Adigue, the trial court relied on the presumption of regularity in the performance of official duties by the police officers.[41] This is a flagrant error because his testimony is directly contradictory to the official records of the Firearms and Explosives Division, PNP, which must prevail. Morever, the presumption of regularity can not prevail over the Constitutional presumption of innocence. [42] Right from the start, P/Sr. Insp. Jerito A. Adigue was aware that petitioner possessed a valid license for the caliber .45 Colt pistol in question. Despite this fact, P/Sr. Insp. Adigue proceeded to detain petitioner and charged him with illegal possession of firearms. We quote pertinent portions of the testimony of petitioner: Q: What else did Adigue tell you after showing to him the license of your cal. .45 pistol and the alleged cal. .22 found in a drawer in your kitchen?

A: He told me that since my firearm is licensed, he will return my firearm, give him ten thousand pesos (P10,000.00) and for me to tell who among the people in our barangay have unlicensed firearm, sir. Q: How did he say about the ten thousand pesos? A: He said palit kalabaw na lang tayo sir. Q: And what did you answer him? A: I told him my firearm is licensed and I do not have money, if I have, I will not give him, sir, because he was just trying to squeeze something from me. Q: How about the unlicensed firearms in your barangay which he asked from you? A: I said I do not know any unlicensed firearm in our barangay, sir. Q: About the .22 cal. pistol, what was your answer to him? A: I told him that it was not mine, they planted it, sir. Q: What did he say next? A: He said that it is your word against mine, the Court will believe me because I am a police officer, sir. Q: What was your comment to what he said? A: I said my firearm is licensed and we have Courts of law who do not conform with officials like you and then he laughed and laughed, sir.[43]

The trial court was obviously misguided when it held that it is a matter of judicial notice that a caliber .45 firearm can not be licensed to a private individual.[44] This ruling has no basis either in law or in jurisprudence.[45] Second issue. The seizure of items not mentioned in the search warrant was illegal. With respect to the .22 caliber revolver with Serial No. 48673, that the police raiding team found in a drawer at the kitchen of petitioners house, suffice it to say that the firearm was not mentioned in the search warrant applied for and issued for the search of petitioners house. Section 2, Article III of the Constitution lays down the general rule that a search and seizure must be carried out through or on the strength of a judicial warrant, absent which such search and seizure becomes unreasonable within the meaning of said constitutional provision.[46] Supporting jurisprudence thus outlined the following requisites for a search warrants validity, the absence of even one will cause its downright nullification: (1) it must be issued upon probable cause; (2) the probable cause must be determined by the judge himself and not by the applicant or any other person; (3) in the determination of probable cause, the judge must examine, under oath or affirmation, the complainant and such witnesses as the latter may produce; and (4) the warrant issued must particularly describe the place to be searched and persons or things to be seized.[47] Seizure is limited to those items particularly described in a valid search warrant. Searching officers are without discretion

regarding what articles they shall seize.[48] Evidence seized on the occasion of such an unreasonable search and seizure is tainted and excluded for being the proverbial fruit of a poisonous tree. In the language of the fundamental law, it shall be inadmissible in evidence for any purpose in any proceeding.[49] In this case, the firearm was not found inadvertently and in plain view. It was found as a result of a meticulous search in the kitchen of petitioners house. This firearm, to emphasize, was not mentioned in the search warrant. Hence, the seizure was illegal.[50] The seizure without the requisite search warrant was in plain violation of the law and the Constitution.[51] True that as an exception, the police may seize without warrant illegally possessed firearm or any contraband for that matter, inadvertently found in plain view. However, [t]he seizure of evidence in plain view applies only where the police officer is not searching for evidence against the accused, but inadvertently comes across an incriminating object.[52] Specifically, seizure of evidence in plain view is justified when there is: (a) a prior valid intrusion based on the valid warrantless arrest in which the police are legally present in the pursuit of their official duties; (b) the evidence was inadvertently discovered by the police who had the right to be where they are; (c) the evidence must be immediately apparent, and (d) plain view justified mere seizure of evidence without further search.[53]

Hence, the petitioner rightly rejected the firearm as planted and not belonging to him. The prosecution was not able to prove that the firearm was in the effective possession or control of the petitioner without a license. In illegal possession of firearms, the possessor must know of the existence of the subject firearm in his possession or control. In People v. de Gracia,[54] we clarified the meaning of possession for the purpose of convicting a person under P. D. No. 1866, thus: x x x In the present case, a distinction should be made between criminal intent and intent to possess. While mere possession without criminal intent is sufficient to convict a person for illegal possession of a firearm, it must still be shown that there was animus possidendi or an intent to possess on the part of the accused. x x x x Hence, the kind of possession punishable under P. D. No. 1866 is one where the accused possessed a firearm either physically or constructively with animus possidendi or intention to possess the same.[55] That is the meaning of animus possidendi. In the absence of animus possidendi, the possessor of a firearm incurs no criminal liability. The same is true with respect to the 5.56 cal. magazine found in the bedroom of petitioners daughter. The seizure was invalid and the seized items were inadmissible in evidence. As explained in People v. Doria,[56] the plain view doctrine applies when the following requisites concur: (1) the law enforcement officer is in a position where he has a clear view of a particular area or has prior justification for an intrusion; (2)

said officer inadvertently comes across (or sees in plain view) a piece of incriminating evidence; and (3) it is immediately apparent to such officer that the item he sees may be evidence of a crime or a contraband or is otherwise subject to seizure. With particular reference to the two 2-way radios that the raiding policemen also seized in the bedroom of petitioners daughter, there was absolutely no reason for the seizure. The radios were not contraband per se. The National Telecommunications Commission may license two-way radios at its discretion.[57] The burden is on the prosecution to show that the two-way radios were not licensed. The National Telecommunication Commission is the sole agency authorized to seize unlicensed two-way radios. More importantly, admittedly, the two-way radios were not mentioned in the search warrant. We condemn the seizure as illegal and a plain violation of a citizens right. Worse, the petitioner was not charged with illegal possession of the two-way radios. Consequently, the confiscation of the two 2-way radios was clearly illegal. The possession of such radios is not even included in the charge of illegal possession of firearms (violation of P. D. No. 1866, as amended) alleged in the Information. WHEREFORE, the Court hereby REVERSES the decision of the Court of Appeals in CA-G. R. CR No. 22255, promulgated on July 09, 1999. The Court ACQUITS petitioner Vicente del Rosario y Nicolas of the charge of violation of P. D. No. 1866, as amended by R. A. No. 8294 (illegal possession of firearms and ammunition), in Criminal Case No. 800-M-96, Regional Trial Court, Bulacan, Branch 20, Malolos. Costs de oficio. The Chief, Firearms and Explosives Division, PNP shall return to petitioner his caliber .45 Colt pistol, with Serial Number No. 70G23792, the five (5) extra magazines and twenty seven (27) rounds of live ammunition, and the two 2-way radios confiscated from him. The Chief, Philippine National Police, or his duly authorized representative shall show to this Court proof of compliance herewith within fifteen (15) days from notice. The .22 caliber revolver with Serial No. 48673, and eight (8) live ammunition and the magazine for 5.56 mm. caliber Armalite rifle are confiscated in favor of the government.

[G.R. No. 129035. August 22, 2002]

PEOPLE OF THE PHILIPPINES, plaintiff-appellee, vs. ANNABELLE FRANCISCO y DAVID, @ ANNABELLE TABLAN, accusedappellant. DECISION YNARES-SANTIAGO, J.:

Federico Verona and his live-in girlfriend, accused-appellant Annabelle Francisco, were placed under surveillance after the police confirmed, through a test-buy operation, that they were engaged in selling shabu or methamphetamine hydrochloride. SPO2 Teneros and SPO4 Alberto San Juan of OADDI-WPDC, U.N. Avenue, Manila applied for a search warrant before Branch 23 of the Regional Trial Court of Manila to authorize them to search the premises at 122 M. Hizon St., Caloocan City. Attached to the application was the After-Surveillance Report[1] of SPO2 Teneros. It stated that Dante Baradilla, of 1726 Lallana St., corner Sta. Catalina St., Tondo, Manila, who claimed to be one of Federico Veronas runners in the illegal drugs operations, allegedly sought the assistance of SPO2 Teneros for the arrest of Verona. [2] The search warrant[3] was subsequently issued by Judge Bayhon authorizing the search of shabu and paraphernalia at No. 122 M. Hizon Street, Caloocan City. Accused-appellant Annabelle Francisco, who was then nine months pregnant, was resting inside the second floor masters bedroom of their two- storey apartment at No. 120 M. Hizon Street, Caloocan City, when she heard a loud bang downstairs as if somebody forcibly opened the front door. Eight policemen suddenly entered her bedroom and conducted a search for about an hour. Accused-appellant inquired about their identities but they refused to answer. It was only at the police station where she found out that the team of searchers was led by SPO2 Teneros. The police team, along with Barangay Chairwoman Miguelita Limpo and Kagawad Bernie de Jesus, both of Barangay 64, Zone-6, District 2, Caloocan City, enforced the warrant and seized the following:[4] 1. One (1) Salad Set marked Pyrex wrapped in a plastic containing white crysthalline (sic) substance or methamphetamine hydrochloride or shabu with markings by the undersigned inside the house of subjects residence weighing (230) two hundred thirty (sic) grams of methampetamine hydrochloride or shabu by Aida AbearPascual of NBI Forensic Chemist; 2. Several plastics in different sizes; 3. Two (2) roll of strip aluminum foil; 4. Five (5) tooter water pipe and improvised and two burner improvised; 5. Two (2) pantakal or measuring weight in shabu; 6. Two (2) cellular phone motorola with markings; 7. One (1) monitoring device with cord and with markings; 8. Several pcs. with strip aluminum foil; 9. Two (2) masking tip (sic) with markings; 10. Twentee (sic) two thousand nine hundred ninetee (sic) pesos.

The police team also allegedly seized the amount of P180,000.00, a Fiat car, jewelry, set of keys, an ATM card, bank books and car documents. Consequently, accused-appellant was charged with violation of Section 16, Article III, Republic Act No. 6425, otherwise known as the Dangerous Drugs Act of 1972, in an information[5] which reads:

That about 10:30 oclock in the morning of 30 March 1996 at No. 122 SCL M. Hizon St., Kalookan City and within the jurisdiction of this Honorable court, the abovenamed accused grouping herself together with some other persons whose liabilities are still being determined in a preliminary investigation, conspiring, confederating and mutually helping one another, did then and there, wilfully, unlawfully and feloniously have in their possession, custody and/or control, methamphetamine hydrochloride popularly known as shabu, a regulated drug, with a total weight of 230 grams, without the corresponding license and/or prescription to possess, have custody and/or control over the aforesaid regulated drug. CONTRARY TO LAW. Accused-appellant filed a motion to quash the search warrant [6] asserting that she and her live-in partner Federico Verona had been leasing an apartment unit at No. 120 M. Hizon Street, District 2, Caloocan City, Metro Manila, since 1995 up to the present as certified by the owner of the apartment unit. On arraignment, accused-appellant pleaded not guilty,[7] after which, trial on the merits ensued. The trial court denied the motion to quash and upheld the validity of the search warrant. It rendered a decision finding accused-appellant guilty as charged, the dispositive portion of which reads:

WHEREFORE premises considered, and the prosecution having established beyond an iota of doubt the guilt of the Accused for Violation of Sec. 16, Art. III, RA 6425 as amended by RA 7659 and considering that the total net weight of subject drugs consists of 230 grams, this Court in the absence of any modifying circumstance hereby imposes upon the Accused the penalty of reclusion perpetua and a fine of P1,000,000.00 pesos, and to pay the costs. The period of Accuseds preventive imprisonment shall be credited in full in the service of her sentence pursuant to Art. 29 of the Revised Penal Code. Subject drugs and paraphernalia are hereby declared confiscated and forfeited in favor of the government to be dealt with in accordance with law.

The return to the Accused of the two (2) cellular phones, (Motorola Micro Fac) (sic) Nos. S-2968A and S-3123A, which were turned over by the Acting Branch Clerk of Court of Manila RTC, Br. 3 to her counterpart in this sala (Exh. 30) as well as the deposit of cash money in the amount of P22,990.00 with the Manila RTC Clerk of Court JESUS MANINGAS as evidenced by acknowledgment receipt dated 21 May 1996, are hereby ordered. SO ORDERED.

[8]

On appeal, accused-appellant raised the following assignment of errors:[9] I. THE LOWER COURT ERRED IN CONVICTING THE ACCUSED OF THE CRIME OF ILLEGAL POSSESSION OF SHABU; II. THE LOWER COURT ERRED IN ADMITTING THE EVIDENCE AGAINST THE ACCUSED; III. THE LOWER COURT ERRED IN NOT FINDING THAT THE SEARCH CONDUCTED WAS ILLEGAL AND VIOLATIVE OF ACCUSEDS CONSTITUTIONAL RIGHTS; IV. THE LOWER COURT ERRED IN NOT ACQUITTING THE ACCUSED AFTER FINDING THAT THE SEARCH WAS INDEED CONDUCTED AT A PLACE DIFFERENT FROM THAT DESCRIBED IN THE SEARCH WARRANT.

The appeal is impressed with merit. Plainly, the basic issue submitted for resolution is the reasonableness of the search conducted by the police officers at accused-appellants residence. The trial court, in upholding the validity of the search, stated that:[10]

Re 3rd argument - the fact that the search warrant in question was served at apartment No. 120 and not at the specific address stated therein which is 122 M. Hizon St., Caloocan City will not by itself render as illegal the search and seizure of subject stuff seized by the operatives pursuant thereto. While it is true that the rationale behind the constitutional and procedural requirements that the search warrant must particularly describe the place to be searched is to the end that no unreasonable search warrant and seizure may not be made (sic) and abuses may not be committed, however, this requirement is not without exception. It is the prevailing rule in our jurisdiction that even a description of the place to be searched is sufficient if the officer with the warrant can with reasonable effort ascertain and identify the place intended (People vs. Veloso, G.R. No. L-23051, Oct. 20, 1925). Significantly in the case at bar the implementing officer SPO2 Teneros was principally the concerned official who conducted an active surveillance on the Accused and subject house (Surveillance Report, Exh. 9) and pursued this case by filing the corresponding application for the issuance of a search warrant. Perforce,

SPO2 TENEROS was thereby placed in a position enabling him to have prior and personal knowledge of particular house intended in the warrant which definitely refer to no other than the very place where the same was accordingly served. Accused-appellant, on the other hand, maintains that the search was grossly infirm as the subject search warrant authorized the police authorities to search only No. 122 M. Hizon St., Caloocan City.However, the actual search was conducted at No. 120 M. Hizon St., Caloocan City. The basic guarantee to the protection of the privacy and sanctity of a person, his home and his possessions against unreasonable intrusions of the State is articulated in Section 2, Article III of the Constitution, which reads:

THE RIGHT OF THE PEOPLE TO BE SECURE IN THEIR PERSONS, HOUSES, PAPERS, AND EFFECTS AGAINST UNREASONABLE SEARCHES AND SEIZURES OF WHATEVER NATURE AND FOR ANY PURPOSE SHALL BE INVIOLABLE, AND NO SEARCH WARRANT OR WARRANT OF ARREST SHALL ISSUE EXCEPT UPON PROBABLE CAUSE TO BE DETERMINED PERSONALLY BY THE JUDGE AFTER EXAMINATION UNDER OATH OR AFFIRMATION OF THE COMPLAINANT AND THE WITNESSES HE MAY PRODUCE, AND PARTICULARLY DESCRIBING THE PLACE TO BE SEARCHED AND THE PERSONS OR THINGS TO BE SEIZED. For the validity of a search warrant, the Constitution requires that there be a particular description of the place to be searched and the persons or things to be seized. The rule is that a description of a place to be searched is sufficient if the officer with the warrant can, with reasonable effort, ascertain and identify the place intended and distinguish it from other places in the community. Any designation or description known to the locality that leads the officer unerringly to it satisfies the constitutional requirement.[11] Specifically, the requisites for the issuance of a valid search warrant are: (1) probable cause is present; (2) such presence is determined personally by the judge; (3) the complainant and the witnesses he or she may produce are personally examined by the judge, in writing and under oath or affirmation; (4) the applicant and the witnesses testify on the facts personally known to them; and (5) the warrant specifically describes the place to be searched and the things to be seized.[12] The absence of any of these requisites will cause the downright nullification of the search warrants. The proceedings upon search warrants must be absolutely legal, for there is not a description of process known to the law, the execution of which is more distressing to the citizen. Perhaps there is none which excites such intense feeling in consequence of its humiliating and degrading effect. The warrants will always be construed strictly without, however, going the full length of requiring technical accuracy. No presumptions of regularity are to be invoked in aid of the process when an officer undertakes to justify it.[13]

The application for search warrant filed by SPO2 Teneros requested for authority to search specifically the premises of No. 122 M. Hizon St., Caloocan City. The application was accompanied by a sketch[14]of the area which bears two parallel lines indicated as 10th Avenue drawn vertically on the left-hand side of the paper. Intersecting these lines are two other parallel lines drawn horizontally and indicated as M. Hizon. Above and on the left-hand side of the upper parallel line of the lines identified as M. Hizon, is a square marked as Basketball Court. A similar drawing placed near the right-hand side of the upper parallel line is another square marked as PNR Compound. Beneath the lower parallel line of the lines marked as M. Hizon and right at the center is also a square enclosing an X sign marked as 122, presumably No. 122 M. Hizon St., Caloocan City. During the hearing for the application of the search warrant, police asset Dante Baradilla described the house to be searched as:

Bale dalawang palapag po, semi concrete, color cream na ang mga bintana ay may rehas na bakal at sliding at sa harap ay may terrace at may sasakyan sila na ginagamit sa pagdeliver ng shabu. [15]

The trial court then conducted an ocular inspection of the area. It turned out that No. 122 M. Hizon St., Caloocan City was a concrete two-storey residential building with steel-barred windows and a terrace. It was owned by a certain Mr. Joseph Ching. The house, however, bore no house number. The house marked No. 122 M. Hizon St., Caloocan City was actually two houses away from accused-appellants house at No. 120 M. Hizon St. On the other hand, No. 120 Hizon St. was a compound consisting of three apartments enclosed by only one gate marked on the outside as No. 120. The different units within No. 120 Hizon St. were not numbered separately. Accused-appellant rented the third unit from the entrance which was supposedly the subject of the search. The entire compound had an area of approximately ninety (90) square meters.The second unit was located at the back of the first unit and the third unit was at the rear end of the compound. Hence, access to the third unit from M. Hizon Street was only through the first two units and the common gate indicated as No. 120. The occupants of the premises stated that they commonly use No. 120 to designate their residence. In People v. Veloso, this Court declared that even a description of the place to be searched is sufficient if the officer with the warrant can with reasonable effort, ascertain and identify the place intended.[16]The description of the building in the application for a search warrant in Veloso as well as in the search warrant itself refer to the building No. 124 Calle Arzobispo, City of Manila, Philippine Islands which was considered sufficient designation of the premises to be searched.[17] The prevailing circumstances in the case at bar are definitely different from those in Veloso. At first glance, the description of the place to be searched in the warrant seems to be sufficient. However, from the application for a search warrant as well as the search warrant itself, the police officer serving the warrant cannot, with reasonable effort, ascertain and identify the place intended precisely because it was wrongly described as No. 122, although it may have been located on the same street as No.

120. Even the description of the house by police asset Baradilla referred to that house located at No. 122 M. Hizon St., not at No. 120 M. Hizon St. The particularity of the place described is essential in the issuance of search warrants to avoid the exercise by the enforcing officers of discretion. Hence, the trial court erred in refusing to nullify the actions of the police officers who were perhaps swayed by their alleged knowledge of the place. The controlling subject of search warrants is the place indicated in the warrant itself and not the place identified by the police.[18] It may well be that the police officer identified No. 120 M. Hizon St. as the subject of the actual search. However, as indicated in the witness affidavit[19] in support of the application for a search warrant,[20] No. 122 M. Hizon St. was unmistakably indicated. Inexplicably, a few days after the search warrant was issued by the court and served at No. 120 M. Hizon St., SPO2 Teneros informed Judge Bayhon in the return of search warrant[21] that the warrant was properly served at 122 M. Hizon St., Caloocan City, Metro Manila as indicated in the search warrant itself. SPO2 Teneros attempted to explain the error by saying that he thought the house to be searched bore the address 122 M. Hizon St., Caloocan City instead of No. 120 M. Hizon St.[22] But as this Court ruled in Paper Industries Corporation of the Philippines v. Asuncion,[23] thus:

The police had no authority to search the apartment behind the store, which was the place indicated in the warrant, even if they really intended it to be the subject of their application. Indeed, the place to be searched cannot be changed, enlarged or amplified by the police, viz.: x x x. In the instant case, there is no ambiguity at all in the warrant. The ambiguity lies outside the instrument, arising from the absence of a meeting of the minds as to the place to be searched between the applicants for the warrant and the Judge issuing the same; and what was done was to substitute for the place that the Judge had written down in the warrant, the premises that the executing officers had in their mind. This should not have been done. It [was] neither fair nor licit to allow police officers to search a place different from that stated in the warrant on the claim that the place actually searchedalthough not that specified in the warrant[was] exactly what they had in view when they applied for the warrant and had demarcated in their supporting evidence. What is material in determining the validity of a search is the place stated in the warrant itself, not what the applicants had in their thoughts, or had represented in the proofs they submitted to the court issuing the warrant. Indeed, following the officers theory, in the context of the facts of the case, all four (4) apartment units at the rear of Abigails Variety Store would have been fair game for a search. The place to be searched, as set out in the warrant, cannot be amplified or modified by the officers own personal knowledge of the premises, or the evidence they adduced in

support of their application for the warrant. Such a change is proscribed by the Constitution which requires inter alia the search warrant to particularly describe the place to be searched as well as the persons or things to be seized. It would concede to police officers the power of choosing the place to be searched, even if it not be delineated in the warrant. It would open wide the door to abuse of the search process, and grant to officers executing a search warrant that discretion which the Constitution has precisely removed from them. The particularization of the description of the place to be searched may properly be done only by the Judge, and only in the warrant itself; it cannot be left to the discretion of the police officers conducting the search. All told, the exclusionary rule necessarily comes into play, to wit:

Art. III, Sec. 3 (2), 1987 Constitution. -- ANY EVIDENCE OBTAINED IN VIOLATION OF THIS OR THE PRECEDING SECTION SHALL BE INADMISSIBLE FOR ANY PURPOSE IN ANY PROCEEDING. Consequently, all the items seized during the illegal search are prohibited from being used in evidence. Absent these items presented by the prosecution, the conviction of accused-appellant for the crime charged loses its basis. As the Court noted in an earlier case, the exclusion of unlawfully seized evidence was the only practical means of enforcing the constitutional injunction against unreasonable searches and seizures. Verily, they are the fruits of the poisonous tree. Without this exclusionary rule, the constitutional right would be so ephemeral and so neatly severed from its conceptual nexus with the freedom from all brutish means of coercing evidence.[24] On another note, we find disturbing the variety of the items seized by the searching team in this case. In the return of search warrant, they admitted the seizure of cellular phones, money and television/monitoring device items that are not within the palest ambit of shabu paraphernalia, which were the only items authorized to be seized. What is more disturbing is the suggestion that some items seized were not reported in the return of search warrant, like the Fiat car, bankbooks, and money. In an attempt to justify the presence of the car in the police station, SPO2 Teneros had to concoct a most incredible story that the accused-appellant, whose pregnancy was already in the third trimester, drove her car to the police station after the intrusion at her house even if the police officers had with them several cars. A search warrant is not a sweeping authority empowering a raiding party to undertake a fishing expedition to seize and confiscate any and all kinds of evidence or articles relating to a crime.[25] It need not be stressed anew that this Court is resolutely committed to the doctrine that this constitutional provision is of a mandatory character and therefore must be strictly complied with. To quote from the landmark American decision of Boyd v. United States: It is the duty of courts to be watchful for the constitutional rights of the citizen,

and against any stealthy encroachments thereon. Their motto should be obsta principiis.[26] Those who are supposed to enforce the law are not justified in disregarding the rights of the individual in the name of order. Order is too high a price to pay for the loss of liberty. As Justice Holmes declared: I think it is less evil that some criminals escape than that the government should play an ignoble part. It is simply not allowed in free society to violate a law to enforce another, especially if the law violated is the Constitution itself.[27] WHEREFORE, in view of the foregoing, the decision of the Regional Trial Court, Branch 127, Caloocan City, convicting accused-appellant of violation of Section 16, Article III, Republic Act No. 6425 and sentencing her to suffer the penalty of reclusion perpetua and to pay a fine of P1,000,000.00 as well as the costs, is REVERSED and SET ASIDE. For lack of evidence to establish guilt beyond reasonable doubt, accusedappellant Annabelle Francisco y David @ Annabelle Tablan is ACQUITTED and is ordered immediately RELEASED from confinement, unless she is lawfully held in custody for another cause.

[G.R. No. 163858. June 28, 2005]

UNITED LABORATORIES, INC., petitioner, vs. ERNESTO ISIP and/or SHALIMAR PHILIPPINES and/or OCCUPANTS, Shalimar Building, No. 1571, Aragon Street, Sta. Cruz, Manila, respondents. DECISION CALLEJO, SR., J.:

Rolando H. Besarra, Special Investigator III of the National Bureau of Investigation (NBI), filed an application, in the Regional Trial Court (RTC) of Manila, for the issuance of a search warrant concerning the first and second floors of the Shalimar Building, located at No. 1571, Aragon Street (formerly No. 1524, Lacson Avenue, Sta. Cruz, Manila) occupied and/or used by Shalimar Philippines, owned/operated by Ernesto Isip; and for the seizure of the following for violation of Section 4(a), in relation to Section 8, of Republic Act (R.A.) No. 8203: a. Finished or unfinished products of UNITED LABORATORIES (UNILAB), particularly REVICON multivitamins;

b. Sundry items such as tags, labels, boxes, packages, wrappers, receptacles, advertisements and other paraphernalia used in the offering for sale, sale and/or distribution of counterfeit REVICON multivitamins; c. Sales invoices, delivery receipts, official receipts, ledgers, journals, purchase orders and all other books of accounts and documents used in recording the manufacture and/or importation, distribution and/or sales of counterfeit REVICON multivitamins.[1]

The application was docketed as People v. Ernesto Isip, et al., Respondents, Search Warrant Case No. 04-4916 and raffled to Branch 24 of the court. Appended thereto were the following: (1) a sketch[2]showing the location of the building to be searched; (2) the affidavit[3] of Charlie Rabe of the Armadillo Protection and Security Agency hired by United Laboratories, Inc. (UNILAB), who allegedly saw the manufacture, production and/or distribution of fake drug products such as Revicon by Shalimar Philippines; (3) the letter-request of UNILAB, the duly licensed and exclusive manufacturer and/or distributor of Revicon and Disudrin, for the monitoring of the unauthorized production/manufacture of the said drugs and, if warranted, for their seizure; (4) the letter-complaint[4] of UNILAB issued through its Director of the Security and Safety Group; and (5) the joint affidavit[5] of NBI Agents Roberto Divinagracia and Rolando Besarra containing the following allegations: 2. When learned that an Asset was already placed by ARMADILLO PROTECTIVE AND SECURITY AGENCY named CHARLIE RABE, who was renting a room since November 2003, at the said premises located at No. 1571 Aragon St., Sta. Cruz, Manila. MR. RABE averred that the owner of the premises is a certain MR. ERNESTO ISIP and that the said premises which is known as SHALIMAR PHILIPPINES, Shalimar Building, are being used to manufacture counterfeit UNILAB products, particularly REVICON multivitamins, which was already patented by UNILAB since 1985; 3. Upon verification of the report, we found out that the said premises is a six-story structure, with an additional floor as a penthouse, and colored red-brown. It has a tight security arrangement wherein non-residents are not allowed to enter or reconnoiter in the premises; 4. We also learned that its old address is No. 1524 Lacson Avenue, Sta. Cruz, Manila, and has a new address as 1571 Aragon St., Sta. Cruz, Manila; and that the area of counterfeiting operations are the first and second floors of Shalimar Building; 5. Since we cannot enter the premises, we instructed the Asset to take pictures of the area especially the places wherein the clandestine manufacturing operations were being held. At a peril to his well-being and security, the Asset was able to take photographs herein incorporated into this Search Warrant Application.[6]

A representative from UNILAB, Michael Tome, testified during the hearing on the application for the search warrant. After conducting the requisite searching questions, the court granted the application and issued Search Warrant No. 04-4916 dated January 27, 2004, directing any police officer of

the law to conduct a search of the first and second floors of the Shalimar Building located at No. 1571, Aragon Street, Sta. Cruz, Manila. The court also directed the police to seize the following items: a. Finished or unfinished products of UNITED LABORATORIES (UNILAB), particularly REVICON multivitamins; b. Sundry items such as tags, labels, boxes, packages, wrappers, receptacles, advertisements and other paraphernalia used in the offering for sale, sale and/or distribution of counterfeit REVICON multivitamins; c. Sales invoices, delivery receipts, official receipts, ledgers, journals, purchase orders and all other books of accounts and documents used in recording the manufacture and/or importation, distribution and/or sales of counterfeit REVICON multivitamins.[7]

The court also ordered the delivery of the seized items before it, together with a true inventory thereof executed under oath. The search warrant was implemented at 4:30 p.m. on January 27, 2004 by NBI agents Besarra and Divinagracia, in coordination with UNILAB employees. No fake Revicon multivitamins were found; instead, there were sealed boxes at the first and second floors of the Shalimar Building which, when opened by the NBI agents in the presence of respondent Isip, contained the following: QUANTITY/UNIT DESCRIPTION 792 Bottles Disudrin 60 ml. 30 Boxes (100 pieces each) Inoflox 200 mg.[8] NBI Special Investigator Divinagracia submitted an inventory of the things seized in which he declared that the search of the first and second floors of the Shalimar Building at No. 1571, Aragon Street, Sta. Cruz, Manila, the premises described in the warrant, was done in an orderly and peaceful manner. He also filed a Return of Search Warrant,[9] alleging that no other articles/items other than those mentioned in the warrant and inventory sheet were seized. The agent prayed that of the items seized, ten boxes of Disudrin 60 ml., and at least one box of Inoflox be turned over to the custody of the Bureau of Food and Drugs (BFAD) for examination.[10] The court issued an order granting the motion, on the condition that the turn over be made before the court, in the presence of a representative from the respondents and the court.[11] The respondents filed an Urgent Motion to Quash the Search Warrant or to Suppress Evidence.[12] They contended that the implementing officers of the NBI conducted their search at the first, second, third and fourth floors of the building at No. 1524-A, Lacson Avenue, Sta. Cruz, Manila, where items in

open display were allegedly found. They pointed out, however, that such premises was different from the address described in the search warrant, the first and second floors of the Shalimar Building located at No. 1571, Aragon Street, Sta. Cruz, Manila. The respondents, likewise, asserted that the NBI officers seized Disudrin and Inoflox products which were not included in the list of properties to be seized in the search warrant. UNILAB, in collaboration with the NBI, opposed the motion, insisting that the search was limited to the first and second floors of the Shalimar building located at the corner of Aragon Street and Lacson Avenue, Sta. Cruz, Manila. They averred that, based on the sketch appended to the search warrant application, Rabes affidavit, as well as the joint affidavit of Besarra and Divinagracia, the building where the search was conducted was located at No. 1571, Aragon Street corner Lacson Avenue, Sta. Cruz, Manila. They pointed out that No. 1524 Lacson Avenue, Sta. Cruz, Manila was the old address, and the new address was No. 1571, Aragon Street, Sta. Cruz, Manila. They maintained that the warrant was not implemented in any other place.[13] In reply, the respondents insisted that the items seized were different from those listed in the search warrant. They also claimed that the seizure took place in the building located at No. 1524-A which was not depicted in the sketch of the premises which the applicant submitted to the trial court.[14] In accordance with the ruling of this Court in People v. Court of Appeals,[15] the respondents served a copy of their pleading on UNILAB.[16] On March 11, 2004, the trial court issued an Order[17] granting the motion of the respondents, on the ground that the things seized, namely, Disudrin and Inoflox, were not those described in the search warrant. On March 16, 2004, the trial court issued an advisory[18] that the seized articles could no longer be admitted in evidence against the respondents in any proceedings, as the search warrant had already been quashed. UNILAB, through the Ureta Law Office, filed a motion, in collaboration with the NBI agents, for the reconsideration of the order, contending that the ground used by the court in quashing the warrant was not that invoked by the respondents, and that the seizure of the items was justified by the plain view doctrine. The respondents objected to the appearance of the counsel of UNILAB, contending that the latter could not appear for the People of the Philippines. The respondents moved that the motion for reconsideration of UNILAB be stricken off the record. Disputing the claims of UNILAB, they insisted that the items seized were contained in boxes at the time of the seizure at No. 1524-A, Lacson Avenue corner Aragon Street, Sta. Cruz, Manila, and were not apparently incriminating on plain view. Moreover, the

seized items were not those described and itemized in the search warrant application, as well as the warrant issued by the court itself. The respondents emphasized that the Shalimar Laboratories is authorized to manufacture galenical preparations of the following products: Products: - Povidone Iodine - Chamomile Oil - Salicylic Acid 10 g. - Hydrogen Peroxide 3% Topical Solution - Aceite de Alcamforado - Aceite de Manzanilla[19] In a manifestation and opposition, the respondents assailed the appearance of the counsel of UNILAB, and insisted that it was not authorized to appear before the court under the Rules of Court, and to file pleadings. They averred that the BFAD was the authorized government agency to file an application for a search warrant. In its counter-manifestation, UNILAB averred that it had the personality to file the motion for reconsideration because it was the one which sought the filing of the application for a search warrant; besides, it was not proscribed by Rule 126 of the Revised Rules of Criminal Procedure from participating in the proceedings and filing pleadings. The only parties to the case were the NBI and UNILAB and not the State or public prosecutor. UNILAB also argued that the offended party, or the holder of a license to operate, may intervene through counsel under Section 16 of Rule 110, in relation to Section 7(e), of the Rules of Criminal Procedure. UNILAB prayed that an ocular inspection be conducted of the place searched by the NBI officers.[20] In their rejoinder, the respondents manifested that an ocular inspection was the option to look forward to.[21] However, no such ocular inspection of the said premises was conducted. In the meantime, the BFAD submitted to the court the result of its examination of the Disudrin and Inoflox samples which the NBI officers seized from the Shalimar Building. On its examination of the actual component of Inoflox, the BFAD declared that the substance failed the test.[22] The BFAD, likewise, declared that the examined Disudrin syrup failed the test.[23] The BFAD had earlier issued the following report: PRODUCT NAME Manufacturer 1.Phenylpropanolamine Unilab (Disudrin)

L.N. 21021552

E.D. 3-06

FINDINGS -Registered, however, label/physical

12.5 mg./5mL Syrup

2.Ofloxacin (Inoflox) 200 mg. tablet.

Unilab

99017407

3-05

appearance does not conform with the BFAD approved label/ registered specifications. -Registered, however, label/physical appearance does not conform with the BFAD approved label/ registered specifications.[24]

On May 28, 2004, the trial court issued an Order[25] denying the motion for reconsideration filed by UNILAB. The court declared that: The Search Warrant is crystal clear: The seizing officers were only authorized to take possession of finished or unfinished products of United Laboratories (UNILAB), particularly REVICON Multivitamins, and documents evidencing the counterfeit nature of said products. The Receipt/Inventory of Property Seized pursuant to the warrant does not, however, include REVICON but other products. And whether or not these seized products are imitations of UNILAB items is beside the point. No evidence was shown nor any was given during the proceedings on the application for search warrant relative to the seized products. On this score alone, the search suffered from a fatal infirmity and, hence, cannot be sustained.[26] UNILAB, thus, filed the present petition for review on certiorari under Rule 45 of the Rules of Court, where the following issues are raised: Whether or not the seized 792 bottles of Disudrin 60 ml. and 30 boxes of Inoflox 200 mg. are INADMISSIBLE as evidence against the respondents because they constitute the fruit of the poisonous tree or, CONVERSELY, whether or not the seizure of the same counterfeit drugs is justified and lawful under the plain view doctrine and, hence, the same are legally admissible as evidence against the respondents in any and all actions?[27] The petitioner avers that it was deprived of its right to a day in court when the trial court quashed the search warrant for a ground which was not raised by the respondents herein in their motion to quash the warrant. As such, it argues that the trial court ignored the issue raised by the respondents. The petitioner insists that by so doing, the RTC deprived it of its right to due

process. The petitioner asserts that the description in the search warrant of the products to be seized finished or unfinished products of UNILAB is sufficient to include counterfeit drugs within the premises of the respondents not covered by any license to operate from the BFAD, and/or not authorized or licensed to manufacture, or repackage drugs produced or manufactured by UNILAB. Citing the ruling of this Court in Padilla v. Court of Appeals,[28] the petitioner asserts that the products seized were in plain view of the officers; hence, may be seized by them. The petitioner posits that the respondents themselves admitted that the seized articles were in open display; hence, the said articles were in plain view of the implementing officers. In their comment on the petition, the respondents aver that the petition should have been filed before the Court of Appeals (CA) because factual questions are raised. They also assert that the petitioner has no locus standi to file the petition involving the validity and the implementation of the search warrant. They argue that the petitioner merely assisted the NBI, the BFAD and the Department of Justice; hence, it should have impleaded the said government agencies as parties-petitioners. The petition should have been filed by the Office of the Solicitor General (OSG) in behalf of the NBI and/or the BFAD, because under the 1987 Revised Administrative Code, the OSG is mandated to represent the government and its officers charged in their official capacity in cases before the Supreme Court. The respondents further assert that the trial court may consider issues not raised by the parties if such consideration would aid the court in the just determination of the case. The respondents, likewise, maintain that the raiding team slashed the sealed boxes so fast even before respondent Isip could object. They argue that the seizure took place at No. 1524-A, Lacson Avenue, Sta. Cruz, Manila covered by Transfer Certificate of Title (TCT) No. 220778, and not at No. 1571, Aragon Street, Sta. Cruz, Manila covered by TCT No. 174412 as stated in the search warrant. They assert that the ruling of the Court in People v. Court of Appeals[29] is applicable in this case. They conclude that the petitioner failed to prove the factual basis for the application of the plain view doctrine.[30] In reply, the petitioner asserts that it has standing and is, in fact, the real party-in-interest to defend the validity of the search warrant issued by the RTC; after all, it was upon its instance that the application for a search warrant was filed by the NBI, which the RTC granted. It asserts that it is not proscribed under R.A. No. 8203 from filing a criminal complaint against the respondents and requesting the NBI to file an application for a search warrant. The petitioner points out that the Rules of Criminal Procedure does not specifically prohibit a private complainant from defending the validity of a search warrant. Neither is the participation of a state prosecutor provided in Rule 126 of the

said Rules. After all, the petitioner insists, the proceedings for the application and issuance of a search warrant is not a criminal action. The petitioner asserts that the place sought to be searched was sufficiently described in the warrant for, after all, there is only one building on the two parcels of land described in two titles where Shalimar Philippines is located, the place searched by the NBI officers.[31] It also asserts that the building is located at the corner of Aragon Street and Lacson Avenue, Sta. Cruz, Manila.[32] The petitioner avers that the plain view doctrine is applicable in this case because the boxes were found outside the door of the respondents laboratory on the garage floor. The boxes aroused the suspicion of the members of the raiding team precisely because these were marked with the distinctive UNILAB logos. The boxes in which the items were contained were themselves so designated to replicate true and original UNILAB boxes for the same medicine. Thus, on the left hand corner of one side of some of the boxes[33] the letters ABR under the words 60 ml, appeared to describe the condition/quality of the bottles inside (as it is with genuine UNILAB box of the true medicine of the same brand). The petitioner pointed out that ABR is the acronym for amber bottle round describing the bottles in which the true and original Disudrin (for children) is contained. The petitioner points out that the same boxes also had their own license plates which were instituted as among its internal control/countermeasures. The license plates indicate that the items within are, supposedly, Disudrin. The NBI officers had reasonable ground to believe that all the boxes have one and the same data appearing on their supposedly distinctive license plates. The petitioner insists that although some of the boxes marked with the distinctive UNILAB logo were, indeed, sealed, the tape or seal was also a copy of the original because these, too, were marked with the distinctive UNILAB logo. The petitioner appended to its pleading pictures of the Shalimar building and the rooms searched showing respondent Isip;[34] the boxes seized by the police officers containing Disudrin syrup;[35] and the boxes containing Inoflox and its contents.[36] The issues for resolution are the following: (1) whether the petitioner is the proper party to file the petition at bench; (2) whether it was proper for the petitioner to file the present petition in this Court under Rule 45 of the Rules of Court; and (3) whether the search conducted by the NBI officers of the first and second floors of the Shalimar building and the seizure of the sealed boxes which, when opened, contained Disudrin syrup and Inoflox, were valid. On the first issue, we agree with the petitioners contention that a search warrant proceeding is, in no sense, a criminal action[37] or the commencement

of a prosecution.[38] The proceeding is not one against any person, but is solely for the discovery and to get possession of personal property. It is a special and peculiar remedy, drastic in nature, and made necessary because of public necessity. It resembles in some respect with what is commonly known as John Doe proceedings.[39] While an application for a search warrant is entitled like a criminal action, it does not make it such an action. A search warrant is a legal process which has been likened to a writ of discovery employed by the State to procure relevant evidence of crime.[40] It is in the nature of a criminal process, restricted to cases of public prosecutions.[41] A search warrant is a police weapon, issued under the police power. A search warrant must issue in the name of the State, namely, the People of the Philippines.[42] A search warrant has no relation to a civil process. It is not a process for adjudicating civil rights or maintaining mere private rights.[43] It concerns the public at large as distinguished from the ordinary civil action involving the rights of private persons.[44] It may only be applied for in the furtherance of public prosecution.[45] However, a private individual or a private corporation complaining to the NBI or to a government agency charged with the enforcement of special penal laws, such as the BFAD, may appear, participate and file pleadings in the search warrant proceedings to maintain, inter alia, the validity of the search warrant issued by the court and the admissibility of the properties seized in anticipation of a criminal case to be filed; such private party may do so in collaboration with the NBI or such government agency. The party may file an opposition to a motion to quash the search warrant issued by the court, or a motion for the reconsideration of the court order granting such motion to quash.[46] In this case, UNILAB, in collaboration with the NBI, opposed the respondents motion to quash the search warrant. The respondents served copies of their reply and opposition/comment to UNILAB, through Modesto Alejandro, Jr.[47] The court a quo allowed the appearance of UNILAB and accepted the pleadings filed by it and its counsel. The general rule is that the proper party to file a petition in the CA or Supreme Court to assail any adverse order of the RTC in the search warrant proceedings is the People of the Philippines, through the OSG. However, in Columbia Pictures Entertainment, Inc. v. Court of Appeals,[48] the Court allowed a private corporation (the complainant in the RTC) to file a petition for certiorari, and considered the petition as one filed by the OSG. The Court

in the said case even held that the petitioners therein could argue its case in lieu of the OSG: From the records, it is clear that, as complainants, petitioners were involved in the proceedings which led to the issuance of Search Warrant No. 23. In People v. Nano, the Court declared that while the general rule is that it is only the Solicitor General who is authorized to bring or defend actions on behalf of the People or the Republic of the Philippines once the case is brought before this Court or the Court of Appeals, if there appears to be grave error committed by the judge or a lack of due process, the petition will be deemed filed by the private complainants therein as if it were filed by the Solicitor General. In line with this ruling, the Court gives this petition due course and will allow petitioners to argue their case against the questioned order in lieu of the Solicitor General.[49] The general rule is that a party is mandated to follow the hierarchy of courts. However, in exceptional cases, the Court, for compelling reasons or if warranted by the nature of the issues raised, may take cognizance of petitions filed directly before it.[50] In this case, the Court has opted to take cognizance of the petition, considering the nature of the issues raised by the parties. The Court does not agree with the petitioners contention that the issue of whether the Disudrin and Inoflox products were lawfully seized was never raised in the pleadings of the respondents in the court a quo. Truly, the respondents failed to raise the issue in their motion to quash the search warrant; in their reply, however, they averred that the seized items were not included in the subject warrant and, therefore, were not lawfully seized by the raiding team. They also averred that the said articles were not illegal per se, like explosives and shabu, as to justify their seizure in the course of unlawful search.[51] In their Opposition/Comment filed on March 15, 2004, the respondents even alleged the following: The jurisdiction of this Honorable Court is limited to the determination of whether there is a legal basis to quash the search warrant and/or to suppress the seized articles in evidence. Since the articles allegedly seized during the implementation of the search warrant Disudrin and Inoflux products were not included in the search warrant, they were, therefore, not lawfully seized by the raiding team; they are not illegal per se, as it were, like an arms cache, subversive materials or shabu as to justify their seizure in the course of a lawful search, or being in plain view or some such. No need whatever for some public assay. The NBI manifestation is a glaring admission that it cannot tell without proper examination or assay that the Disudrin and Inoflox samples allegedly seized from

respondents place were counterfeit. All the relevant presumptions are in favor of legality.[52] The Court, therefore, finds no factual basis for the contention of the petitioner that the respondents never raised in the court a quo the issue of whether the seizure of the Disudrin and Inoflox products was valid. In any event, the petitioner filed a motion for the reconsideration of the March 11, 2004 Order of the court a quo on the following claims: 2.01 The Honorable Court ERRED in ruling on a non-issue or the issue as to the alleged failure to particularly describe in the search warrant the items to be seized but upon which NO challenge was then existing and/orNO controversy is raised; 2.02 The Honorable Court ERRED in its ruling that finished or unfinished products of UNILAB cannot stand the test of a particular description for which it then reasons that the search is, supposedly unreasonable; and, 2.03 The Honorable Court ERRED in finding that the evidence seized is lawfully inadmissible against respondents.[53]

The court a quo considered the motion of the petitioner and the issue raised by it before finally resolving to deny the same. It cannot thus be gainsaid that the petitioner was denied its right to due process. On the validity of the seizure of the sealed boxes and its contents of Disudrin and Inoflox, the Court, likewise, rejects the contention of the petitioner. A search warrant, to be valid, must particularly describe the place to be searched and the things to be seized. The officers of the law are to seize only those things particularly described in the search warrant. A search warrant is not a sweeping authority empowering a raiding party to undertake a fishing expedition to seize and confiscate any and all kinds of evidence or articles relating to a crime. The search is limited in scope so as not to be general or explanatory. Nothing is left to the discretion of the officer executing the warrant.[54] Objects, articles or papers not described in the warrant but on plain view of the executing officer may be seized by him. However, the seizure by the officer of objects/articles/papers not described in the warrant cannot be presumed as plain view. The State must adduce evidence, testimonial or documentary, to prove the confluence of the essential requirements for the doctrine to apply, namely: (a) the executing law enforcement officer has a prior justification for an initial intrusion or otherwise properly in a position from which he can view a particular order; (b) the officer must discover

incriminating evidence inadvertently; and (c) it must be immediately apparent to the police that the items they observe may be evidence of a crime, contraband, or otherwise subject to seizure.[55] The doctrine is not an exception to the warrant. It merely serves to supplement the prior justification whether it be a warrant for another object, hot pursuit, search as an incident to a lawful arrest or some other legitimate reason for being present, unconnected with a search directed against the accused. The doctrine may not be used to extend a general exploratory search from one object to another until something incriminating at last emerges. It is a recognition of the fact that when executing police officers comes across immediately incriminating evidence not covered by the warrant, they should not be required to close their eyes to it, regardless of whether it is evidence of the crime they are investigating or evidence of some other crime. It would be needless to require the police to obtain another warrant. [56] Under the doctrine, there is no invasion of a legitimate expectation of privacy and there is no search within the meaning of the Constitution. The immediate requirement means that the executing officer can, at the time of discovery of the object or the facts therein available to him, determine probable cause of the objects incriminating evidence.[57] In other words, to be immediate, probable cause must be the direct result of the officers instantaneous sensory perception of the object.[58] The object is apparent if the executing officer had probable cause to connect the object to criminal activity. The incriminating nature of the evidence becomes apparent in the course of the search, without the benefit of any unlawful search or seizure. It must be apparent at the moment of seizure.[59] The requirement of inadvertence, on the other hand, means that the officer must not have known in advance of the location of the evidence and intend to seize it.[60] Discovery is not anticipated.[61] The immediately apparent test does not require an unduly high degree of certainty as to the incriminating character of evidence. It requires merely that the seizure be presumptively reasonable assuming that there is probable cause to associate the property with criminal activity; that a nexus exists between a viewed object and criminal activity.[62] Incriminating means the furnishing of evidence as proof of circumstances tending to prove the guilt of a person.[63] Indeed, probable cause is a flexible, common sense standard. It merely requires that the facts available to the officer would warrant a man of reasonable caution and belief that certain items may be contrabanded or

stolen property or useful as evidence of a crime. It does not require proof that such belief be correct or more likely than true. A practical, non-traditional probability that incriminating evidence is involved is all that is required. The evidence thus collected must be seen and verified as understood by those experienced in the field of law enforcement.[64] In this case, Disudrin and/or Inoflox were not listed in the search warrant issued by the court a quo as among the properties to be seized by the NBI agents. The warrant specifically authorized the officers only to seize counterfeit Revicon multivitamins, finished or unfinished, and the documents used in recording, manufacture and/or importation, distribution and/or sale, or the offering for sale, sale and/or distribution of the said vitamins. The implementing officers failed to find any counterfeit Revicon multivitamins, and instead seized sealed boxes which, when opened at the place where they were found, turned out to contain Inoflox and Disudrin. It was thus incumbent on the NBI agents and the petitioner to prove their claim that the items were seized based on the plain view doctrine. It is not enough to prove that the sealed boxes were in the plain view of the NBI agents; evidence should have been adduced to prove the existence of all the essential requirements for the application of the doctrine during the hearing of the respondents motion to quash, or at the very least, during the hearing of the NBI and the petitioners motion for reconsideration on April 16, 2004. The immediately apparent aspect, after all, is central to the plain view exception relied upon by the petitioner and the NBI. There is no showing that the NBI and the petitioner even attempted to adduce such evidence. In fact, the petitioner and the NBI failed to present any of the NBI agents who executed the warrant, or any of the petitioners representative who was present at the time of the enforcement of the warrant to prove that the enforcing officers discovered the sealed boxes inadvertently, and that such boxes and their contents were incriminating and immediately apparent. It must be stressed that only the NBI agent/agents who enforced the warrant had personal knowledge whether the sealed boxes and their contents thereof were incriminating and that they were immediately apparent.[65] There is even no showing that the NBI agents knew the contents of the sealed boxes before they were opened. In sum then, the Court finds and so hold that the petitioner and the NBI failed to prove the essential requirements for the application of the plain view doctrine. IN LIGHT OF ALL THE FOREGOING, the petition is DENIED for lack of merit. The assailed orders of the Regional Trial Court are AFFIRMED.

SO ORDERED. U.S. Supreme Court Katz v. United States, 389 U.S. 347 (1967)

Katz v. United States No. 35 Argued October 17, 1967 Decided December 18, 1967 389 U.S. 347 Syllabus Petitioner was convicted under an indictment charging him with transmitting wagering information by telephone across state lines in violation of 18 U.S.C. § 1084. Evidence of petitioner's end of the conversations, overheard by FBI agents who had attached an electronic listening and recording device to the outside of the telephone booth from which the calls were made, was introduced at the trial. The Court of Appeals affirmed the conviction, finding that there was no Fourth Amendment violation, since there was "no physical entrance into the area occupied by" petitioner. Held: 1. The Government's eavesdropping activities violated the privacy upon which petitioner justifiably relied while using the telephone booth, and thus constituted a "search and seizure" within the meaning of the Fourth Amendment. Pp. 389 U. S. 350-353. (a) The Fourth Amendment governs not only the seizure of tangible items, but extends as well to the recording of oral statements. Silverman v. United States, 365 U. S. 505, 365 U. S. 511. P. 389 U. S. 353. (b) Because the Fourth Amendment protects people, rather than places, its reach cannot turn on the presence or absence of a physical intrusion into any given enclosure.

The "trespass" doctrine of Olmstead v. United States, 277 U. S. 438, and Goldman v. United States, 316 U. S. 129, is no longer controlling. Pp. 389 U. S. 351, 389 U. S. 353. 2. Although the surveillance in this case may have been so narrowly circumscribed that it could constitutionally have been authorized in advance, it was not in fact conducted pursuant to the warrant procedure which is a constitutional precondition of such electronic surveillance. Pp. 389 U. S. 354-359. 369 F.2d 130, reversed. Page 389 U. S. 348 Disclaimer: Official Supreme Court case law is only found in the print version of the United States Reports. Justia case law is provided for general informational purposes only, and may not reflect current legal developments, verdicts or settlements. We make no warranties or guarantees about the accuracy, completeness, or adequacy of the information contained on this site or information linked to from this site. Please check official sources.

Justia Annotations is a forum for attorneys to summarize, comment on, and analyze case law published on our site. Justia makes no guarantees or warranties that the annotations are accurate or reflect the current state of law, and no annotation is intended to be, nor should it be construed as, legal advice. Contacting Justia or any attorney through this site, via web form, email, or otherwise, does not create an attorney-client relationship.

Related Documents

Search Search
October 2019 35
Search Search
October 2019 41
Search
October 2019 81
Search And Warrant.docx
November 2019 13
Select And Search Prompts
November 2019 21

More Documents from ""

Search And Warrant.docx
November 2019 13
Part6 (14-17) 2.docx
November 2019 6
Pubcorp 2.docx
November 2019 12
Duc Le Chi Cv
July 2020 2